23.08.2023 Views

test bank for health assessment for nursing practice 7th edition by wilson

You also want an ePaper? Increase the reach of your titles

YUMPU automatically turns print PDFs into web optimized ePapers that Google loves.

Test Bank for Health Assessment for

Nursing Practice 7th Edition by Wilson

This is a bank of tests (study questions) to help you

prepare for the tests.

To clarify, this is a test bank, not a textbook.

You have immediate access to download your test

bank. No

delays, loading is fast and instant immediately after

Purchase!

You will receive a full bank of tests; in other words, all

chapters will be there.

Test banks are presented in PDF format; therefore,

no special software is required to open them


Test Bank for Health Assessment for

Nursing Practice 7th Edition by Wilson.

Chapter 01: Introduction to Health Assessment

MULTIPLE CHOICE

1. A patient comes to the emergency department and tells the triage nurse that he

is “having a heart attack.” What is the nurse’s top priority at this time?

a. Determine the patient’s personal data and

insurance coverage.

b. Ask the patient to take a seat in the waiting

room until his name is called.

c. Request that a nurse collect data for a

comprehensive history.

d. Ask a nurse to start a focused assessment

of this patient now.

ANSWER: D

The nurse needs to begin an assessment as soon as possible that is focused on this patient’s

cardiovascular system. The type of health assessment performed by the nurse is also driven

by patient need. Personal data and insurance information will be obtained, but in this

situation, these data can wait until after the patient is assessed. Based also on Maslow’s

hierarchy of needs, physiologic needs take precedence. Rather than asking the patient to

wait, the nurse needs to begin data collection, such as vital signs, immediately to determine

the patient’s health status. Complications can be prevented if an immediate assessment is

made to analyze the patient’s symptoms. A comprehensive history is not indicated in this

situation at this time. Some subjective data will be collected, such as allergies and medical

history related to cardiovascular disease. Eyes, ears, or a complete musculoskeletal or

mental health assessment is not a priority at this time.

DIF: Cognitive Level: Apply REF: Box 1-3 | p. 3

TOP: Nursing Process: Assessment

MSC: NCLEX Patient Needs: Safe and Effective Care Environment: Management of Care:

Establishing Priorities

2. Which situation illustrates a screening assessment?

a. A patient visits an obstetric clinic for the

first time and the nurse conducts a detailed

history and physical examination.

b. A hospital sponsors a health fair at a local

mall and provides cholesterol and blood

pressure checks to mall patrons.

c. The nurse in an urgent care center checks

the vital signs of a patient who is

complaining of leg pain.


d. A patient newly diagnosed with diabetes

mellitus comes to test his fasting blood

glucose level.

ANSWER: B

A health fair at a local mall that provides cholesterol and blood pressure checks is an

example of a screening assessment focused on disease detection. A detailed history and

physical examination conducted during a first-time visit to an obstetric clinic is an example

of a comprehensive assessment. Assessing a patient complaining of leg pain in the triage

area of an urgent care center is an example of a problem-based/focused assessment. A

patient’s return appointment 1 month after today’s office visit to report fasting blood

glucose levels is an example of an episodic or follow-up assessment.

DIF: Cognitive Level: Understand REF: Box 1-3 | p. 3

TOP: Nursing Process: Assessment

MSC: NCLEX Patient Needs: Health Promotion and Maintenance: Health Screening

3. For which person is a screening assessment indicated?

a. The person who had abdominal surgery

yesterday

b. The person who is unaware of his high

serum glucose levels

c. The person who is being admitted to a

long-term care facility

d. The person who is beginning rehabilitation

after a knee replacement

ANSWER: B

A screening assessment is performed for the purpose of disease detection. In this case this

person may have diabetes mellitus. A shift assessment is most appropriate for the person

who is recovering in the hospital from surgery. A comprehensive assessment is performed

during admission to a facility to obtain a detailed history and complete physical

examination. An episodic or follow-up assessment is performed after knee replacement to

evaluate the outcome of the procedure.

DIF: Cognitive Level: Understand REF: Box 1-3 | p. 3

TOP: Nursing Process: Assessment

MSC: NCLEX Patient Needs: Safe and Effective Care Environment: Management of Care:

Establishing Priorities

4. For which person is a shift assessment indicated?

a. The person who had abdominal surgery

yesterday

b. The person who is unaware of his high

serum glucose levels

c. The person who is being admitted to a

long-term care facility

d. The person who is beginning rehabilitation

after a knee replacement

ANSWER: A

A shift assessment is most appropriate for the person who is recovering in the hospital from

surgery. A screening assessment is performed for the purpose of disease detection, in this

case diabetes mellitus. A comprehensive assessment is performed during admission to a


facility to obtain a detailed history and complete physical examination. An episodic or

follow-up assessment is performed after knee replacement to evaluate the outcome of the

procedure.

DIF: Cognitive Level: Understand REF: Box 1-3 | p. 4

TOP: Nursing Process: Assessment

MSC: NCLEX Patient Needs: Safe and Effective Care Environment: Management of Care:

Establishing Priorities

5. For which person is a comprehensive assessment indicated?

a. The person who had abdominal surgery

yesterday

b. The person who is unaware of his high

serum glucose levels

c. The person who is being admitted to a

long-term care facility

d. The person who is beginning rehabilitation

after a knee replacement

ANSWER: C

A comprehensive assessment is performed during admission to a facility to obtain a

detailed history and complete physical examination. A shift assessment is most appropriate

for the person who is recovering in the hospital from surgery. A screening assessment is

performed for the purpose of disease detection, in this case diabetes mellitus. An episodic

or follow-up assessment is performed after knee replacement to evaluate the outcome of

the procedure.

DIF: Cognitive Level: Understand REF: Box 1-3 | p. 3

TOP: Nursing Process: Assessment

MSC: NCLEX Patient Needs: Safe and Effective Care Environment: Management of Care:

Establishing Priorities

6. For which person is an episodic or follow-up assessment indicated?

a. The person who had abdominal surgery

yesterday

b. The person who is unaware of his high

serum glucose levels

c. The person who is being admitted to a

long-term care facility

d. The person who is beginning rehabilitation

after a knee replacement

ANSWER: D

An episodic or follow-up assessment is performed after the knee replacement to evaluate

the outcome of the procedure. A shift assessment is most appropriate for the person who is

recovering in the hospital from surgery. A screening assessment is performed for the

purpose of disease detection, in this case diabetes mellitus. A comprehensive assessment is

performed during admission to a facility to obtain a detailed history and complete physical

examination.

DIF: Cognitive Level: Understand REF: Box 1-3 | p. 3

TOP: Nursing Process: Assessment


MSC: NCLEX Patient Needs: Safe and Effective Care Environment: Management of Care:

Establishing Priorities

7. Which is an example of data a nurse collects during a physical

examination?

a. The patient’s lack of hair and shiny skin

over both shins

b. The patient’s stated concern about lack of

money for prescriptions

c. The patient’s complaints of tingling

sensations in the feet

d. The patient’s mother’s statements that the

patient is very nervous lately

ANSWER: A

The lack of hair and shiny skin over both shins are objective data or signs that are part of

the physical examination. A patient’s concerns about lack of money are subjective data and

are part of the health history. A patient’s complaints of tingling sensations in the feet are

subjective data and are part of the health history. A patient’s family statements are

considered secondary data, are subjective data, and are part of the health history.

DIF: Cognitive Level: Apply REF: Box 1-3 | p. 3

TOP: Nursing Process: Assessment

MSC: NCLEX Patient Needs: Physiologic Integrity: Reduction of Risk Potential: System

Specific Assessments

8. The nurse documents which information in the patient’s history?

a. The patient’s skin feels warm to the touch.

b. The patient is scratching his arm.

c. The patient’s temperature is 100° F.

d. The patient complains of itching.

ANSWER: D

A patient’s complaint of itching is subjective information, which means it is a symptom

and is documented in the history. The patient’s warm skin is objective information

gathered by the nurse through palpation, is also a sign, and is documented in the physical

examination. The patient’s scratching is objective information gathered by the nurse

through observation, is also a sign, and is documented in the physical examination. The

patient’s elevated temperature is objective information gathered by the nurse through

measurement, is also a sign, and is documented in the physical examination.

DIF: Cognitive Level: Apply REF: p. 1 | p. 2 and Box 1-2

TOP: Nursing Process: Assessment

MSC: NCLEX Patient Needs: Safe and Effective Care Environment: Management of Care:

Establishing Priorities

9. Which patient information does the nurse document in the patient’s physical

assessment?

a. Slurred speech

b. Immunizations

c. Smoking habit

d. Allergies


ANSWER: A

Slurred speech should be noticed by the nurse and documented as objective data in the

physical assessment. Data on immunizations are collected from the patient, are subjective,

and documented in the history. A smoking habit is information that comes from the patient,

making it subjective data that is documented in the history. Allergies are information that

come from the patient, making it subjective data that is documented in the history.

DIF: Cognitive Level: Apply REF: p. 1-2 and Box 1-2

TOP: Nursing Process: Assessment

MSC: NCLEX Patient Needs: Safe and Effective Care Environment: Management of Care:

Establishing Priorities

10. After collecting the data, the nurse begins data analysis with which action?

a. Clustering data

b. Documenting subjective data

c. Reporting information to other health team

members

d. Documenting objective information

ANSWER: A

After collecting data, the nurse organizes or clusters the data so that the problems appear

more clearly. To cluster data, the nurse interprets the assessment data collected.

Documenting subjective data is necessary for the medical record, but does not provide

analysis. Before reporting data to health team members, the nurse clusters and interprets

data. Documenting objective data is necessary for the medical record, but does not provide

analysis.

DIF: Cognitive Level: Understand REF: p. 4

TOP: Nursing Process: Assessment

MSC: NCLEX Patient Needs: Safe and Effective Care Environment: Management of Care:

Establishing Priorities

11. Which activity illustrates the concept of primary prevention?

a. Monthly breast self-examination

b. Annual cervical (Papanicolaou test)

examination

c. Education about living with asthma

d. Exercising three times a week

ANSWER: D

Exercising is an example of primary prevention that prevents disease from developing by

maintaining a healthy lifestyle. Monthly breast self-examination is an example of

secondary prevention and screening efforts to promote early detection of disease. Annual

cervical (Papanicolaou test) examination is an example of secondary prevention and

screening efforts to promote early detection of disease. Teaching a patient how to live with

a chronic disease such as asthma is an example of tertiary prevention directed toward

minimizing the disability from chronic disease and helping the patient maximize his or her

health.

DIF: Cognitive Level: Understand REF: Table 1-1 | p. 5-6

TOP: Nursing Process: Assessment

MSC: NCLEX Patient Needs: Health Promotion and Maintenance: Health Promotion Programs


12. A nurse is teaching a patient how to manage chronic obstructive pulmonary

disease (COPD). This intervention is an example of which level of health promotion?

a. Primary prevention

b. Secondary prevention

c. Tertiary prevention

d. Risk factor prevention

ANSWER: C

Teaching a patient how to live with a chronic disease is an example of tertiary prevention

directed toward minimizing the disability from chronic disease and helping the patient

maximize his or her health. The focus of primary prevention is to prevent a disease from

developing by promoting a healthy lifestyle. Secondary prevention consists of efforts to

promote early detection of disease. Risk factor prevention is part of primary prevention that

focuses on preventing disease by managing risk factors.

DIF: Cognitive Level: Understand REF: Table 1-1 | p. 5-6

TOP: Nursing Process: Assessment

MSC: NCLEX Patient Needs: Health Promotion and Maintenance: Health Promotion Programs

13. Which activity illustrates the concept of secondary prevention?

a. Annual mammogram

b. Nutrition classes on low-fat cooking

c. Education on living with diabetes mellitus

d. Cardiac rehabilitation after coronary artery

bypass surgery

ANSWER: A

A mammogram screens for breast cancer and is an example of secondary prevention to

promote early detection of disease. Nutrition classes are an example of primary prevention

to prevent a disease from developing by promoting a healthy lifestyle. Education about

diabetes mellitus is an example of tertiary prevention directed toward minimizing the

disability from chronic disease and helping the patient maximize his or her health. Cardiac

rehabilitation after coronary artery bypass surgery is an example of tertiary prevention

directed toward minimizing the disability from chronic disease and helping the patient

maximize his or her health.

DIF: Cognitive Level: Understand REF: Table 1-1 | p. 5-6

TOP: Nursing Process: Assessment

MSC: NCLEX Patient Needs: Health Promotion and Maintenance: Health Promotion Programs

14. A community organization sponsors a health fair to increase awareness of colon

cancer. At the health fair, colorectal cancer screening kits are distributed, and health care

professionals answer questions, take blood pressure, and distribute literature. What level of

health prevention is being implemented by this community organization?

a. Primary

b. Secondary

c. Tertiary

d. Risk factor

ANSWER: B

Secondary prevention consists of screening efforts to promote early detection of disease —

in this scenario, colorectal cancer and hypertension. Primary prevention is focused on

preventing disease from developing through the promotion of a healthy lifestyle. Tertiary


Powered by TCPDF (www.tcpd f. o rg )

prevention is directed toward minimizing the disability from chronic disease and helping

the patient maximize his or her health. Risk factor prevention is part of primary prevention

that focuses on preventing disease by managing risk factors.

DIF: Cognitive Level: Apply REF: Table 1-1 | p. 5-6

TOP: Nursing Process: Assessment

MSC: NCLEX Patient Needs: Health Promotion and Maintenance: Health Promotion Programs


Chapter 02: Obtaining a Health History

MULTIPLE CHOICE

1. Which statement or question does the nurse use during the introduction

phase of the interview?

a. “I’m here to learn more about the pain

you’re experiencing.”

b. “Can you describe the pain that you’re

experiencing?”

c. “I heard you say that the pain is ‘all over’

your body.”

d. “What relieves the pain you are having?”

ANSWER: A

“I’m here to learn more about the pain you’re experiencing” is an example of the

introduction phase a nurse may use to explain the purpose of the interview to a patient.

“Can you describe the pain that you’re experiencing?” is an example of part of a symptom

analysis that occurs in the discussion phase. “I heard you say that the pain is ‘all over’ your

body” is an example of a summary statement by the nurse that occurs in the summary

phase. “What relieves the pain you are having?” is an example of part of a symptom

analysis that occurs in the discussion phase.

DIF: Cognitive Level: Apply REF: Box 2-1 | p. 8-9

TOP: Nursing Process: Assessment

MSC: NCLEX Patient Needs: Psychosocial Integrity: Therapeutic Communications

2. Which statement is appropriate to use when beginning an interview with a

new patient?

a. “Have you ever been a patient in this clinic

before?”

b. “What is your purpose for coming to the

clinic today?”

c. “Tell me a little about yourself and your

family.”

d. “Did you have any difficulty finding the

clinic?”

ANSWER: B

“What is your purpose for coming to the clinic today?” is an open-ended question that

focuses on the patient’s reason for seeking care. “Have you ever been a patient in this clinic

before?” is a close-ended question that yields a “yes” or “no” response. This question may

be asked on the first visit, but not as an opening question for a health interview. “Tell me a

little about yourself and your family” is an open-ended question, but it is too general, and it

is at least two questions: one about the patient and another about the family. “Did you have


any difficulty finding the clinic?” is a social question and does not focus on the patient’s

purpose for the visit.

DIF: Cognitive Level: Understand REF: p. 8

TOP: Nursing Process: Assessment

MSC: NCLEX Patient Needs: Psychosocial Integrity: Therapeutic Communications

3. Which statement by the nurse demonstrates a patient-centered interview?

a. “I need to complete this questionnaire

about your medical and family history.”

b. “The hospital requires me to complete this

assessment as soon as possible.”

c. “Tell me about the symptoms you’ve been

having.”

d. “I’ve had the same symptoms that you’ve

described.”

ANSWER: C

“Tell me about the symptoms you’ve been having” focuses on the needs of the patient so

that the patient is free to share concerns, beliefs, and values in his or her own words. “I

need to complete this questionnaire about your medical and family history” focuses on the

nurse’s need to complete the assessment rather than the needs of the patient. “The hospital

requires me to complete this assessment as soon as possible” focuses on the nurse’s need to

meet hospital requirements rather than the needs of the patient. “I’ve had the same

symptoms that you’ve described” focuses on the nurse rather than on the patient.

DIF: Cognitive Level: Apply REF: p. 8

TOP: Nursing Process: Assessment

MSC: NCLEX Patient Needs: Psychosocial Integrity: Therapeutic Communications

4. Which question is an example of an open-ended question?

a. “Have you experienced this pain before?”

b. “Do you have someone to help you at

home?”

c. “How many times a day do you use your

inhaler?”

d. “What were you doing when you felt the

pain?”

ANSWER: D

“What were you doing when you felt the pain?” is a broadly stated question that

encourages a free-flowing, open response. “Have you experienced this pain before?” is

closed-ended, which can obtain a “yes” or “no” answer to the question without any

additional data. “Do you have someone to help you at home?” is closed-ended, which can

obtain a “yes” or “no” answer to the question without any additional data. “How many

times a day do you use your inhaler?” is closed-ended, which can obtain an answer of a

specific number without any additional data.


DIF: Cognitive Level: Understand REF: pp. 10-11

TOP: Nursing Process: Assessment

MSC: NCLEX Patient Needs: Psychosocial Integrity: Therapeutic Communications

5. A nurse suspects a female patient is a victim of physical abuse. Which

response is most likely to encourage the patient to confide in the nurse?

a. “You’ve got a huge bruise on your face.

Did your husband hit you?”

b. “That bruise looks tender. I don’t know

how people can do that to one another.”

c. “If your boyfriend hit you, you can get a

restraining order against him.”

d. “I’ve seen women who have been hurt by

boyfriends or husbands. Does anyone hit

you?”

ANSWER: D

“I’ve seen women who have been hurt by boyfriends or husbands” is an example of a

technique referred to as “permission giving” in which the nurse communicates that it is safe

to discuss uncomfortable topics. “You’ve got a huge bruise on your face. Did your husband

hit you?” assumes that domestic violence did occur, and the comment does not encourage

the patient to divulge additional information. “That bruise looks tender. I don’t know how

people can do that to one another” assumes that domestic violence did occur, and the

comment does not encourage the patient to divulge additional information. “If your

boyfriend has hit you, you can get a restraining order against him” assumes that domestic

violence did occur, and the comment does not encourage the patient to divulge additional

information.

DIF: Cognitive Level: Apply REF: p. 10

TOP: Nursing Process: Assessment

MSC: NCLEX Patient Needs: Psychosocial Integrity: Abuse/Neglect

6. Which technique used by the nurse encourages a patient to continue talking

during an interview?

a. Laughing and smiling during conversation

b. Using phrases such as “Go on,” and

“Then?”

c. Repeating what the patient said, but using

different words

d. Asking the patient to clarify a point

ANSWER: B

Using phrases such as “Go on” and “Then?” encourages the patient to continue talking.

Laughing and smiling during conversation may show attentiveness during the interview,

but does not encourage more talking. Rephrasing what the patient has said is restatement.


It confirms your interpretation of what they said, but does not encourage additional talking.

Asking the patient to clarify a point is done when the information is conflicting, vague, or

ambiguous.

DIF: Cognitive Level: Remember REF: p. 11

TOP: Nursing Process: Assessment

MSC: NCLEX Patient Needs: Psychosocial Integrity: Therapeutic Communications

7. During the history, the patient states that she does not use many drugs.

What is the nurse’s appropriate response to this statement?

a. “Tell me about the drugs you are using

currently.”

b. “To some people six or seven is not

many.”

c. “Do you mean prescription drugs or illicit

drugs?”

d. “How often are you using these drugs?”

ANSWER: A

“Tell me about the drugs you are using currently” is an open-ended question that allows

patients to provide further data. “To some people six or seven is not many” is a comment

that does not ask a question or obtain useful data. “Do you mean prescription drugs or

illicit street drugs?” is a closed-ended question that yields data about the type of drugs used

only. “How often are you using these drugs?” asks about frequency of drug use, which is

not useful until the drugs are known.

DIF: Cognitive Level: Apply REF: p. 11

TOP: Nursing Process: Assessment

MSC: NCLEX Patient Needs: Psychosocial Integrity: Therapeutic Communications

8. A nurse is interviewing a patient who was diagnosed with type 2 diabetes

mellitus 6 months ago. Since that time, the patient has gained weight and her blood glucose

levels remain high. The nurse suspects that the patient is noncompliant with her diet.

Which response by the nurse enhances data collection in this situation?

a. “Tell me about what foods you eat and the

frequency of your meals.”

b. “What symptoms do you notice when your

blood sugar levels are high?”

c. “You need to follow what the doctor has

prescribed to manage your disease.”

d. “Tell me what you know about the cause of

type 2 diabetes.”

ANSWER: A

“Tell me about what foods you eat and the frequency of your meals” gathers more data

from the patient to help the nurse confirm if noncompliance is the reason for the weight

gain and high glucose levels. “What symptoms do you notice when your blood sugar levels


are high?” does not help the nurse determine if the patient is noncompliant. It may be

useful later when teaching the patient about her disease. “You need to follow what the

doctor has prescribed to manage your disease” does not provide additional data for the

nurse and may be viewed as authoritarian and paternalistic. “Tell me what you know about

the cause of type 2 diabetes” assumes that the reason for the weight gain and high glucose

levels is a lack of knowledge. A more therapeutic approach is to gather more data from the

patient about how the diabetes has been managed.

DIF: Cognitive Level: Apply REF: p. 11

TOP: Nursing Process: Assessment

MSC: NCLEX Patient Needs: Psychosocial Integrity: Therapeutic Communications

9. A male patient tells the nurse that he rarely sleeps more than 4 hours a night

and has not experienced any problems because of the lack of sleep. Which response by the

nurse is most appropriate?

a. “That is interesting.”

b. “Only 4 hours of sleep? How do you stay

awake during the day?”

c. “Really? Everyone needs more sleep than

that.”

d. “Did I understand that you sleep 4 hours

every night?”

ANSWER: D

“Did I understand that you sleep 4 hours every night?” is a reflection technique that allows

the nurse to confirm and obtain additional information. “That is interesting” does not

provide an opportunity for the patient to explain any reason for the number of hours of

sleep. “Only 4 hours of sleep? How do you stay awake during the day?” questions the

accuracy of the patient’s data and may not encourage the patient to give further details.

“Really? Everyone needs more sleep than that” can be perceived as argumentative, but

does not encourage further data from the patient.

DIF: Cognitive Level: Apply REF: p. 11

TOP: Nursing Process: Assessment

MSC: NCLEX Patient Needs: Psychosocial Integrity: Therapeutic Communications

10. Which technique should the nurse use to obtain more data about a patient’s

vague or ambiguous statement?

a. Laughing and smiling during conversation

b. Using phrases such as “Go on,” and

“Then?”

c. Repeating what the patient has said, but

using different words

d. Asking the patient to explain a point

ANSWER: D


Asking the patient to explain a point is clarification, which is used to obtain more

information about conflicting, vague, or ambiguous statements. Laughing and smiling

during conversation may show attentiveness during the interview, but does not help to

clarify vague information. Using phrases such as “Go on” and “Then?” encourages patients

to continue talking, but does not help clarify. Rephrasing what the patient has said is

restatement. It confirms your interpretation of what they said, but does not encourage

additional talking.

DIF: Cognitive Level: Understand REF: p. 11

TOP: Nursing Process: Assessment

MSC: NCLEX Patient Needs: Psychosocial Integrity: Therapeutic Communications

11. What does the nurse say to obtain more data about a patient’s vague

statement about diet such as, “My diet’s okay”?

a. “Eating a variety of meats, fruits, and

vegetables each day is important.”

b. “Give me an example of the foods you eat

in a typical day.”

c. “Go on.”

d. “Does your diet meet your needs or does it

need improvement?”

ANSWER: B

“Give me an example of the foods you eat in a typical day.” This statement asks the patient

to clarify the vague statement, “My diet is okay.” “Eating a variety of meats, fruits, and

vegetables each day is important.” While this statement is true, it does not obtain data

about what foods the patient consumes. “Go on” encourages patients to continue talking,

but does not help clarify what foods are consumed. “Does your diet meet your needs or

does it need improvement?” This response does not help clarify what foods the patient eats.

Also it contains two questions rather than asking one question at a time.

DIF: Cognitive Level: Apply REF: p. 11

TOP: Nursing Process: Assessment

MSC: NCLEX Patient Needs: Psychosocial Integrity: Therapeutic Communications

12. While giving a history, a male patient describes several events out of order

that occurred in different decades in his life. What technique does the nurse use to

understand the timeline of these events?

a. State the order of events as understood and

ask the patient to verify the order.

b. Draw conclusions about the order of events

from data given.

c. Ask the patient to elaborate about these

events.

d. Ask the patient to repeat what he said

about these events.


ANSWER: A

State the order of events as understood and ask patient to verify the order is correct. This

therapeutic technique is useful when interviewing a patient who rambles or does not

provide sequential data. Drawing conclusions about the order of events is interpretation. In

this example, the sequence of events is more relevant than an interpretation. The nurse may

have difficulty interpreting an unclear sequence of events. Asking the patient to elaborate

about these events will not provide order to the sequence of events. Asking the patient to

repeat what he said about these events will not necessarily provide a sequence of events.

DIF: Cognitive Level: Understand REF: p. 11

TOP: Nursing Process: Assessment

MSC: NCLEX Patient Needs: Psychosocial Integrity: Therapeutic Communications

13. A male patient is very talkative and shares much information that is not

relevant to his history or the reason for his admission. Which action by the nurse improves

data collection in this situation?

a. Terminate the interview.

b. Use closed-ended questions.

c. Ask the patient to stay on the subject.

d. Ask another nurse to complete the

interview.

ANSWER: B

Using closed-ended questions is useful to obtain specific data when open-ended questions

are not obtaining the needed data. Terminating the interview is not beneficial to the patient

and does not allow data collection. Asking the patient to stay on the subject is not

therapeutic and may result in less data collection. Asking another nurse to complete the

interview may not be practical and interrupts the nurse-patient relationship that has been

established.

DIF: Cognitive Level: Understand REF: p. 11 | p. 12

TOP: Nursing Process: Assessment

MSC: NCLEX Patient Needs: Psychosocial Integrity: Therapeutic Communications

14. A patient answers questions quietly and appears sad. While answering

questions about her marriage, she begins to cry. Which response by the nurse is appropriate

in this situation?

a. “Don’t cry! I’ll come back when you’ve

settled down.”

b. “I only have a few more questions to ask,

and then I’ll leave you alone for a while.”

c. “Everyone has ups and downs in their

marriage. What problems are you having?”

d. “I see that you are upset. Is there

something you’d like to discuss?”


ANSWER: D

“I see that you are upset. Is there something you’d like to discuss?” shows that the nurse is

attentive to the patient’s feelings and does not make assumptions about the reason why the

patient is crying. The crying may signify additional data the nurse needs to collect during

this interview. “Don’t cry! I’ll come back when you’ve settled down” is not a therapeutic

response. The nurse needs to support the patient rather than leave her. “I only have a few

more questions to ask, and then I’ll leave you alone for a while” is not a therapeutic

response. The nurse is more concerned about getting the history than the patient’s response.

“Everyone has ups and downs in their marriage. What problems are you having?” is not a

therapeutic response. The nurse is assuming there are problems in the marriage instead of

collecting more data.

DIF: Cognitive Level: Apply REF: pp. 11-12 | pp. 11-13

TOP: Nursing Process: Assessment

MSC: NCLEX Patient Needs: Psychosocial Integrity: Therapeutic Communications

15. During an interview, a patient begins to cry and appears angry. Which

response by the nurse is most therapeutic?

a. “This topic prompted an emotional

response, tell me what you are feeling.”

b. “This topic does not usually cause such an

emotional response.”

c. “Calm down and tell me what is wrong.”

d. “I will leave you alone for a few minutes so

you can pull yourself together.”

ANSWER: A

Acknowledging the patient’s feelings and encouraging their expression communicates

acceptance of the emotion. Crying is a natural behavior and should be permitted. “This

topic does not usually cause such an emotional response” may be perceived by the patient

as judgmental and it does not help the patient meet the current need. Encouraging the

patient to stop crying so that the nurse can help is not supportive of the patient’s current

need. The therapeutic action is to postpone further questioning until the patient is ready to

proceed. Leaving the room so that the patient can be alone is not supportive of the patient.

DIF: Cognitive Level: Apply REF: p. 12

TOP: Nursing Process: Assessment

MSC: NCLEX Patient Needs: Psychosocial Integrity: Therapeutic Communications

16. In which situation is the nurse’s use of closed-ended questions most

appropriate?

a. When clarifying vague or conflicting data

b. When obtaining a history from an overly

talkative patient

c. When encouraging a patient to elaborate

on details of his or her history


d. When collecting data about the current

health problem

ANSWER: B

When obtaining a history from an overly talkative patient, a nurse can resort to closedended

questions to complete the data collection in a timely manner. When clarifying vague and

conflicting data, the nurse needs to use open-ended questions to obtain data. When

encouraging the patient to elaborate on details of his or her history, the nurse needs to use

open-ended questions to obtain the details. When collecting data about the current problem,

the patient needs to describe the symptoms that brought him or her to seek help. These

details are not collected with closed-ended questions.

DIF: Cognitive Level: Understand REF: p. 12

TOP: Nursing Process: Assessment

MSC: NCLEX Patient Needs: Psychosocial Integrity: Therapeutic Communications

17. The nurse is interviewing a woman with her husband present. The husband

answers the questions for the wife most of the time. What is the most appropriate

therapeutic nursing action to hear the patient’s viewpoint?

a. Continue the interview.

b. Ask the husband to step out of the room.

c. Ask another nurse to complete the

interview.

d. Tell the woman to speak up for herself.

ANSWER: B

Asking the husband to step out of the room will allow the patient to answer questions in her

own way. Continuing the interview is not a therapeutic action because the nurse needs to

obtain the patient’s answers to the questions. Asking another nurse to complete the

interview does not solve the problem that the husband is answering questions for his wife.

Telling the woman to speak up for herself does not solve the problem and may interfere

with the therapeutic relationship between the patient and the nurse.

DIF: Cognitive Level: Remember REF: pp. 12-13

TOP: Nursing Process: Assessment

MSC: NCLEX Patient Needs: Psychosocial Integrity: Therapeutic Communications

18. A female Korean patient accompanied by her husband and son comes to the

emergency department (ED) complaining of abdominal pain. The patient speaks and

understands Korean only. Which person is the appropriate choice for the nurse to use to get

a history from this patient?

a. The patient’s husband who speaks Korean

and English

b. The patient’s son who speaks Korean and

English

c. A male technician who works in the ED

who speaks Korean and English


d. A female interpreter who speaks Korean

and English and is available by phone

ANSWER: D

A female interpreter who speaks Korean and English and is available by phone is the best

choice because she can communicate with the patient and is the same gender as the patient.

The patient’s husband who speaks Korean and English is not the best choice because he is

a family member and may alter the meaning of what is said. The patient’s son who speaks

Korean and English is not the best choice because he is a family member and may alter the

meaning of what is said. A male technician working in the ED who speaks Korean and

English is not a good choice because the patient may feel uncomfortable giving a history to

a stranger who is male.

DIF: Cognitive Level: Understand REF: p. 13

TOP: Nursing Process: Assessment

MSC: NCLEX Patient Needs: Psychosocial Integrity: Cultural Diversity | NCLEX Patient

Needs: Psychosocial Integrity: Therapeutic Communications

19. Which nurse demonstrates culturally competent care for a female patient

from Russia?

a. Nurse A who asks the patient about

cultural factors that influence health care

b. Nurse B who interacts with every patient

from Russia in the same manner

c. Nurse C who learns the cultural variables

of every culture, including Russia

d. Nurse D who relies on her previous

experience with patients from Russia

ANSWER: A

Asking the patient about cultural factors that influence health care is demonstrating

culturally competent care, along with interacting with each patient as a unique person who

is a product of past experiences, beliefs, and values. Interacting with every patient from

Russia in the same manner does not allow for the uniqueness of each person within the

same culture. Learning the cultural variables of every group encountered can be valuable

but it is impractical to learn about all cultures because each patient is unique. A better

approach is to ask patients about their beliefs. Relying on previous experience with patients

from Russia does not allow for the uniqueness of each person within the same culture.

DIF: Cognitive Level: Understand REF: p. 13

TOP: Nursing Process: Assessment

MSC: NCLEX Patient Needs: Psychosocial Integrity: Cultural Diversity

20. For which patient is a focused health history most appropriate?

a. A new patient at the health clinic for an

annual examination


b. A patient admitted to the hospital with

vomiting and abdominal pain

c. A patient at the health care provider’s

office for a sport physical

d. A patient discharged 11 months ago who is

being readmitted today

ANSWER: B

A patient admitted to the hospital with vomiting and abdominal pain benefits from a

focused health history that limits data to the immediate problem. A new patient at the

health clinic for an annual examination needs a comprehensive history that includes

biographic data, reason for seeking care, present health status, past medical history, family

history, personal and psychosocial history, and a review of all body. A patient with a

specific need, such as a sport physical, needs a history for an episodic assessment. A

patient discharged months ago who is being readmitted needs a history for a follow-up

assessment that generally focuses on the specific problem or problems that caused the

patient to be readmitted.

DIF: Cognitive Level: Understand REF: pp. 13-14

TOP: Nursing Process: Assessment

MSC: NCLEX Patient Needs: Psychosocial Integrity: Therapeutic Communications

21. A patient tells the nurse at the clinic, “I can never seem to get warm lately

and feel tired all the time.” The nurse records these data under which section of the health

history?

a. Past health history

b. Present health status

c. Reason for seeking care (chief complaint)

d. Subjective assessment data

ANSWER: C

The reason for seeking care (chief complaint) is the patient’s reason for seeking care (also

called the presenting problem). The patient’s reason for seeking care is often recorded as a

direct quote. The past health history includes data about immunizations, surgeries,

accidents, and childhood illnesses. The present health status includes data the nurse obtains

from the patient, often using a symptom analysis in which more data are collected about the

patient’s reason for seeking care. Subjective assessment data include information from the

patient. In this example, the patient expresses the reason for seeking care, which is directly

quoted and placed in quotation marks in the chief complaint section of the data sheet so

that the patient’s reason for seeking care can be easily identified.

DIF: Cognitive Level: Apply REF: p. 14

TOP: Nursing Process: Assessment

MSC: NCLEX Patient Needs: Psychosocial Integrity: Therapeutic Communications

22. A patient comes to the ambulatory surgery center for an elective procedure this

morning. While giving the admission history, the patient states she is allergic to latex.


What is the most appropriate response by the nurse at this time?

a. Removing all latex products from the

patient’s room

b. Using powdered gloves when providing

care to this patient

c. Informing the surgeon that the patient has

type I hypersensitivity to latex

d. Questioning the patient about symptoms

experienced in the past with latex

ANSWER: D

Questioning the patient about symptoms experienced in the past with latex is the

appropriate response. When patients indicate an allergy to a medication or substance, ask

them to describe what happens with exposure to determine whether the reaction is a side

effect or an allergic reaction. Removing all latex products from the patient’s room is

unnecessary at this time because the latex allergy has not been confirmed. Using powdered

gloves when providing care to this patient is unnecessary at this time because the latex

allergy has not been confirmed. Informing the surgeon that the patient has type I

hypersensitivity to latex is unnecessary at this time because the latex allergy has not been

confirmed.

DIF: Cognitive Level: Understand REF: p. 15

TOP: Nursing Process: Assessment

MSC: NCLEX Patient Needs: Safe and Effective Care Environment: Safety and Infection

Control: Injury Prevention

23. A nurse is interviewing a male patient who reports he has not had a tetanus

immunization in about 15 years because he had a “bad reaction” to the last tetanus

immunization. What is the most appropriate response by the nurse in this case?

a. Notify the health care provider that this

immunization cannot be given.

b. Document that the patient is allergic to the

tetanus vaccine.

c. Give the vaccine after explaining that

adverse reactions are rare.

d. Ask the patient to describe the “bad

reaction” to the vaccine in more detail.

ANSWER: D

The nurse needs to collect more data about the reaction from the patient to determine the

type of reaction experienced. The nurse is trying to assess the relationship between the

“reaction” reported by the patient and an allergic reaction. The immunization should not be

eliminated at this time. Additional facts are needed to determine the type of reaction the

patient experienced. Documenting an allergy to the tetanus vaccine may be an error

because there are insufficient data to make that determination at this time. Giving the


vaccine may be an error if the patient is allergic to the vaccine and additional data indicates

that may be the case.

DIF: Cognitive Level: Apply REF: p. 15

TOP: Nursing Process: Assessment

MSC: NCLEX Patient Needs: Health Promotion and Maintenance: Health Promotion and

Disease Prevention

24. A patient admitted with pneumonia reports that she takes insulin for

diabetes mellitus. In which section of the history does the nurse document the insulin and

diabetes?

a. Past health history

b. Present health status

c. Reason for seeking care (chief complaint)

d. History of present illness

ANSWER: B

The present health status documents the current health conditions, which include chronic

diseases and medications taken. In this case, diabetes and taking insulin are not the reason

for seeking care, but need to be managed while the patient’s pneumonia is being treated

because they may affect the patient’s recovery from pneumonia. The past health history

includes categories of childhood illness, surgeries, hospitalizations, accidents or injuries,

immunizations, and obstetric history. The reason for seeking care (chief complaint) is a

brief statement of the patient’s purpose for requesting the services of a health care provider.

History of present illness further investigates the history of the present problem; best

accomplished by conducting a symptom analysis.

DIF: Cognitive Level: Understand REF: p. 15

TOP: Nursing Process: Assessment

MSC: NCLEX Patient Needs: Safe and Effective Care Environment: Collaboration with

Interdisciplinary Team

25. A nurse is getting a history from a patient who is disabled from rheumatoid

arthritis. Which question will provide data about this patient’s functional ability?

a. “When did your arthritis symptoms

begin?”

b. “How has your arthritis affected your daily

life?”

c. “Why did you come to the clinic today?”

d. ”How do you feel about your diagnosis of

rheumatoid arthritis?”

ANSWER: B

“How has your arthritis affected your daily life?” is a question that leads to data about the

patient’s ability to perform self-care activities or functional abilities. “When did your

arthritis symptoms begin?” is a question asked as part of the history, but does not collect

data about functional ability. “Why did you come to the clinic today?” is a question asked


to obtain the chief complaint about a current problem, but does not focus directly on the

functional assessment. “How do you feel about your diagnosis of rheumatoid arthritis?”

is a question to ask in the psychosocial history, but does not focus directly on the functional

assessment.

DIF: Cognitive Level: Apply REF: p. 17

TOP: Nursing Process: Assessment

MSC: NCLEX Patient Needs: Psychosocial Integrity: Therapeutic Communications

26. An example of a health promotion question included in the health

history

is:

a. “Do you have any allergies?”

b. “How often are you exercising?”

c. “What are you doing to relieve your leg

pain?”

d. “What kind of herbs are you using?”

ANSWER: B

“How often are you exercising?” is a question about activities patients regularly perform to

maintain health. “Do you have any allergies?” is a question for the present health status

rather than health promotion. “What are you doing to relieve your leg pain?” is a question

that is part of the symptom analysis. “What kind of herbs are you using?” is a question for

the present health status rather than health promotion.

DIF: Cognitive Level: Remember REF: p. 17

TOP: Nursing Process: Assessment

MSC: NCLEX Patient Needs: Psychosocial Integrity: Therapeutic Communications

27. The patient reports having a persistent cough for the past 2 weeks

and that the cough disrupts sleep and has not been helped by over-the-counter

cough medicines. Which question is most appropriate for the nurse to ask next?

a. “So what do you think is causing this

persistent cough?”

b. “Have you tried taking sleeping pills to

help you sleep?”

c. “Did you think this will just go away on its

own?”

d. “What other symptoms have you noticed

related to this cough?”

ANSWER: D

“What other symptoms have you noticed related to this cough?” is part of a symptom

analysis to provide more data. The answer to the question “So what do you think is causing

this persistent cough?” is a guess by the patient and does not provide useful data. “Have

you tried taking sleeping pills to help you sleep?” does not focus on the cough, which is


what is disturbing the patient’s sleep. “Did you think this will just go away on its own?”

does not provide useful data and criticizes the patient’s lack of action.

DIF: Cognitive Level: Apply REF: p. 18 | Box 2-3

TOP: Nursing Process: Assessment

MSC: NCLEX Patient Needs: Psychosocial Integrity: Therapeutic Communications

MULTIPLE RESPONSE

1. Which data do nurses document under the heading of Past Health History?

(Select all that apply.)

a. Father has Alzheimer disease.

b. Last tetanus in 2009

c. Had chicken pox as a child

d. Drinks three to four beers each day

e. Had a dental examination 6 months ago

ANSWER: B, C, E

Last tetanus is an immunization, chicken pox as a child is a childhood illness, and last

examinations, including dental, are documented under the heading of Past Health History.

Family History documents father’s Alzheimer disease; patient drinking three to four beers

each day refers to alcohol use, which is documented under the heading Personal and

Psychosocial History.

DIF: Cognitive Level: Understand REF: pp. 15-16

TOP: Nursing Process: Assessment

MSC: NCLEX Patient Needs: Psychosocial Integrity: Therapeutic Communications

2. Which data do nurses document under the heading of Personal and

Psychosocial History? (Select all that apply.)

a. Walks for 45 minutes each day

b. Eats meats, vegetables, and fruit at two

meals daily

c. Is allergic to milk and milk products

d. Is married and has two daughters whom he

is close to

e. Smokes marijuana once a week

f. Grandfather died from prostate cancer

ANSWER: A, B, D, E

Walks for 45 minutes each day is documented under health promotion activity in Personal

and Psychosocial History; eats meats, vegetables, and fruit at two meals daily is

documented about diet activity in Personal and Psychosocial History; is married and has

two daughters whom he is close to is documented under family and social relationship

activity in Personal and Psychosocial History; smokes marijuana once a week is

documented under personal habits activity in Personal and Psychosocial History. Allergic


to milk and milk products is an allergy, which is documented under the heading Present

Health Status; Grandfather died from prostate cancer is documented under the heading

Family History.

DIF: Cognitive Level: Understand REF: p. 16

TOP: Nursing Process: Assessment

MSC: NCLEX Patient Needs: Psychosocial Integrity: Therapeutic Communications

3. Which questions are pertinent to ask when obtaining a symptom analysis

from a patient who reports breathing problems? (Select all that apply.)

a. How long have you had this problem with

your breathing?

b. Do you have a family history of breathing

problems?

c. Does this breathing problem come and go

or is it constant?

d. What do you do to make your breathing

better?

e. How does this breathing problem affect

your work or daily activities?

f. How many packs of cigarettes do you

smoke a day?

ANSWER: A, C, D, E

How long have you had this problem with your breathing?, Does this breathing problem

come and go or is it constant?, What do you do to make your breathing better?, and How

does this breathing problem affect your work or daily activities? are questions asked in a

symptom analysis. Use the mnemonic of OLD CARTS (e.g., onset of symptoms, location

and duration of symptoms, characteristics, aggravating factors, related symptoms,

treatment used, and severity of symptoms). Do you have a family history of breathing

problems? This question relates to the patient’s history; How many packs of cigarettes do

you smoke a day? This question relates to the patient’s history.

DIF: Cognitive Level: Apply REF: p. 17 | Box 2-3

TOP: Nursing Process: Assessment

MSC: NCLEX Patient Needs: Physiologic Integrity: Reduction of Risk Potential: System

Specific Assessments

4. Which questions are pertinent to ask when obtaining a symptom analysis

from a patient who reports a headache? (Select all that apply.)

a. Describe what the headache feels like.

b. When was your last eye examination?

c. What makes the headaches worse?


d. How do you rate the headaches on a scale

of 0 (meaning no pain) to 10 (meaning the

worse pain ever)?

e. Do you have any symptoms with the

headaches, such as nausea?

f. When did you first notice the headaches?

ANSWER: A, C, D, E, F

Describe what the headache feels like?, What makes the headaches worse?, How do you

rate the headaches on a scale of 0 (meaning no pain) to 10 (meaning the worse pain ever)?,

Do you have any symptoms with the headaches, such as nausea?, and When did you first

notice the headaches? are questions asked in a symptom analysis. Use the mnemonic of

OLD CARTS (e.g., onset of symptoms, location and duration of symptoms, characteristics,

aggravating factors, related symptoms, treatment used, and severity of symptoms). When

was your last eye examination? assumes that the headaches are related to a vision problem.

Last eye examination is documented in the history under the heading of Past Health

History.

DIF: Cognitive Level: Apply REF: p. 15 | Box 2-3

TOP: Nursing Process: Assessment

MSC: NCLEX Patient Needs: Physiologic Integrity: Reduction of Risk Potential: System

Specific Assessments

5. Which questions are pertinent for a nurse to ask a patient while performing

a review of the cardiovascular system? (Select all that apply.)

a. Do you remember what your last

cholesterol value was?

b. Have you had chest pain or shortness of

breath?

c. Do you have trouble breathing when you

lie down?

d. Are your feet cold, numb, or do they

change color?

e. How much do you weigh?

f. Have you noticed edema in your ankles at

the end of the day?

ANSWER: B, C, D, F

Have you had chest pain or shortness of breath?, Do you have trouble breathing when you

lie down?, Are your feet cold, numb, or do they change color?, and Have you noticed

edema in your ankles at the end of the day? are questions asked to give the patient an

opportunity to report symptoms of the cardiovascular system. Do you remember what your

last cholesterol value was? relates to a lab value, which is objective data not documented in

the history; How much do you weigh? is objective data not documented in the history.

DIF: Cognitive Level: Remember REF: p. 18


TOP: Nursing Process: Assessment

MSC: NCLEX Patient Needs: Physiologic Integrity: Reduction of Risk Potential: System

Specific Assessments


Chapter 3: Techniques and Equipment for Physical Assessment

Test Bank

MULTIPLE CHOICE

1. What is the most important nursing action to reduce transmission of microorganisms during a

physical assessment?

a. Clean the bell and diaphragm of the stethoscope between patients.

b. Perform hand hygiene.

c. Wear gloves when anticipating exposure to body fluids.

d. Wear eye protection when anticipating spatter of body fluids.

ANSWER: B

A

B

C

D

Feedback

Cleaning the bell and diaphragm of the stethoscope between patients is important

to prevent the spread of microorganisms when auscultating only.

Consensus recommendations of the World Health Organization include use of

hand hygiene techniques to prevent spread of microorganisms before palpating,

percussing, or auscultating patients, and during patient care.

Wearing gloves when anticipating exposure to body fluids is important to prevent

the spread of microorganisms from the patient while giving care.

Wearing eye protection when anticipating spatter of body fluids is important to

prevent the spread of microorganisms from the patient while giving care.

DIF: Cognitive Level: Remember REF: 21

TOP: Nursing Process: Assessment

MSC: NCLEX Patient Needs: Safe and Effective Care Environment: Safety and Infection Control:

Standard Precautions/Transmission-Based Precautions/Surgical Asepsis

2. When examining a patient, the nurse remembers to follow which principle of Standard

Precautions?

a. Wear gloves throughout the entire examination of the patient.

b. Wear gloves when in contact with the patient’s mucous membranes.

c. Wear gloves to reduce the need for handwashing.

d. Wear eye protection and a gown during the examination of the patient.

ANSWER: B

A

B

C

D

Feedback

Wearing gloves throughout the examination of the patient is unnecessary.

Referring to the Standard Precautions for the correct answer; nurses use judgment

to determine when contact with body fluids is possible.

Specifically, this applies to contact with blood, body fluids (e.g., urine, feces,

sputum, wound drainage), nonintact skin, and mucous membranes.

Hands must be washed after removal of gloves.

The nurse should wear a mask with eye protection or a face shield during

procedures that may result in splashes or sprays of the patient’s blood, body

fluids, secretions, or excretions.


DIF: Cognitive Level: Understand REF: 22

TOP: Nursing Process: Assessment

MSC: NCLEX Patient Needs: Safe and Effective Care Environment: Safety and Infection Control:

Standard/Transmission-Based/Other Precautions

3. How do nurses prevent a latex allergy?

a. They use nonlatex gloves for all procedures.

b. They protect their hands using oil-based hand lotion applying latex gloves.

c. They use a powder-free, low-allergen latex gloves.

d. They wash their hands with mild soap and dry thoroughly before applying latex

gloves.

ANSWER: C

A

B

C

D

Feedback

Nonlatex gloves may be used only for activities that are not likely to involve

contact with infectious materials.

NIOSH recommends not using oil-based hand lotions when wearing latex gloves.

Use of these types of gloves is recommended by The National Institute for

Occupational Safety and Health (NIOSH).

NIOSH recommends washing hands after removing latex gloves, not before

applying them.

DIF: Cognitive Level: Remember REF: 22, Box 3-2

TOP: Nursing Process: Assessment

MSC: NCLEX Patient Needs: Safe and Effective Care Environment: Safety and Infection Control:

Standard Precautions/Transmission-Based Precautions/Surgical Asepsis

4. Which explanation is most appropriate for a nurse preparing to palpate a patient’s neck?

a. “I need to feel for tumors in your neck.”

b. “I’m going to feel your neck for any abnormalities.”

c. “I need to press deeply on your neck so please hold still.”

d. “Is there any tenderness in your neck?”

ANSWER: B

A

B

C

D

Feedback

I need to feel for tumors in your neck” uses the term “tumors” and may alarm the

patient unnecessarily.

Palpating the neck enters the patient’s personal space and may have cultural

significance. Thus it is important to inform patients of the impending action and

its purpose.

“I need to press deeply on your neck so please hold still” may alarm the patient

and is not accurate. To palpate the neck, light palpation is used to detect

abnormalities such as enlarged nodes. Deep palpation is used on the abdomen.

“Is there any tenderness in your neck?” obtains subjective data, but does not tell

the patient what the nurse is planning to do.

DIF: Cognitive Level: Apply REF: 23

TOP: Nursing Process: Assessment

MSC: NCLEX Patient Needs: Health Promotion and Maintenance: Techniques of Physical


Assessment

5. Which nurse is performing the technique of light palpation appropriately?

a. Nurse A applies the bimanual technique to determine size and location of the

patient’s heart.

b. Nurse B uses the fingertips to feel for temperature differences on the patient’s legs.

c. Nurse C places the ulnar surface of the hands on the patient’s thorax to detect

vibrations.

d. Nurse D depresses the patient’s abdomen approximately 4 cm to assess pulsations.

ANSWER: C

A

B

C

D

Feedback

The bimanual technique is used to entrap an organ or mass (such as the uterus or

a growth) between the fingertips to determine size and location and is not

palpation.

Temperature differences are best detected using the dorsal surface of the hand;

this technique is not palpation.

Nurse C places the ulnar surface of the hands on the patient’s thorax to detect

vibrations. This is considered a light palpation.

Light pulsation is performed by pressing in to a depth of approximately 1 cm,

rather than 4 cm.

DIF: Cognitive Level: Understand REF: 23

TOP: Nursing Process: Assessment

MSC: NCLEX Patient Needs: Health Promotion and Maintenance: Techniques of Physical

Assessment

6. How does the nurse perform the bimanual technique of palpation to assess organs?

a. Using the palmar surface of the dominant hand to press inward to a depth of about

1 cm

b. Holding a light source in one hand while stroking the skin lightly with the

dominant hand

c. Using the ulnar surfaces of both hands to press inward 4 to 5 cm

d. Using both hands, one anterior and one posterior, to entrap an organ between the

fingertips

ANSWER: D

A

B

C

D

Feedback

Using the palmar surface of the dominant hand to press inward to a depth of

about 1 cm describes light palpation, which is different from the bimanual

technique.

Holding a light source in one hand while stroking the skin lightly with the

dominant hand is used when inspecting rather than palpating.

Using the ulnar surfaces of both hands to press inward 4 to 5 cm describes an

incorrect technique.

Using both hands, one anterior and one posterior, to entrap an organ between the

fingertips is the correct technique for bimanual palpation.

DIF: Cognitive Level: Apply REF: 23-24


TOP: Nursing Process: Assessment

MSC: NCLEX Patient Needs: Health Promotion and Maintenance: Techniques of Physical

Assessment

7. While assessing a patient’s lower extremities, the nurse suspects the lower extremities feel

cooler than the upper extremities. To confirm this suspicion, how does the nurse compare the

temperatures of the lower extremities with the upper extremities?

a. Using the backs (dorsum) of the hands to detect differences

b. Using the ulnar surface of the hands to detect differences

c. Using the pads of the fingers to detect differences

d. Using the palmar surface (underside) of the hands to detect differences

ANSWER: A

A

B

C

D

Feedback

The dorsal surfaces of the hands detect temperature best.

The ulnar surfaces of the hands are the most sensitive to vibration.

The pads of the fingers are used in palpation.

The palmar surfaces (underside) of the fingers and finger pads are better for

determining position, texture, size, consistency, masses, fluid, and crepitus.

DIF: Cognitive Level: Apply REF: 23

TOP: Nursing Process: Assessment

MSC: NCLEX Patient Needs: Health Promotion and Maintenance: Techniques of Physical

Assessment

8. How does a nurse assess for fluid in a patient’s abdomen?

a. Placing the nondominant hand (pleximeter) over the area to be percussed, and

striking the index finger of the pleximeter with the pad of the middle finger of the

dominant hand

b. Applying indirect percussion by tapping one finger lightly on the abdominal wall

with the plexor

c. Placing the middle finger of the nondominant hand (pleximeter) over the area to be

percussed, and striking that finger with the tip of the middle finger of the dominant

hand

d. Using direct percussion by placing one hand over the abdomen and striking lightly

with the other hand

ANSWER: C

A

B

C

D

Feedback

Only the finger being struck touches the area to be percussed; the other fingers

are raised off the skin and the middle finger is struck with the tip of the finger of

the other hand.

Percussing the abdomen requires both hands, one as the plexor and the other as

the pleximeter.

Placing the middle finger of the nondominant hand (pleximeter) over the area to

be percussed, and striking that finger with the tip of the middle finger of the

dominant hand describes the correct technique.

Using direct percussion by placing one hand over the abdomen and striking

lightly with the other hand does not describe the correct technique.


DIF: Cognitive Level: Apply REF: 24

TOP: Nursing Process: Assessment

MSC: NCLEX Patient Needs: Health Promotion and Maintenance: Techniques of Physical

Assessment

9. What assessment data do nurses obtain through striking a hand directly against the flank or

costovertebral angle of a patient’s body?

a. Fluid in the lungs

b. Tenderness over the kidneys

c. Air in the abdomen

d. Tenderness over the liver

ANSWER: B

A

B

C

D

Feedback

Fluid in the lungs is detected by indirect percussion.

Tenderness over the kidneys is detected by direct percussion over the

costovertebral angle.

Air in the abdomen is detected by indirect percussion.

Tenderness over the liver is detected by palpation.

DIF: Cognitive Level: Apply REF: 24

TOP: Nursing Process: Assessment

MSC: NCLEX Patient Needs: Health Promotion and Maintenance: Techniques of Physical

Assessment

10. A patient has been complaining of abdominal cramping and gas; the nurse notes that his

abdomen is slightly distended. Which sound does the nurse expect to hear during percussion

of this patient’s abdomen?

a. Flatness

b. Dullness

c. Resonance

d. Tympany

ANSWER: D

A

B

C

D

Feedback

Flatness is heard over bones and muscle.

Dullness is heard over the liver.

Resonance is heard over normal lung tissue.

Tympany is a loud, high-pitched sound heard over the abdomen.

DIF: Cognitive Level: Apply REF: 24-25

TOP: Nursing Process: Assessment

MSC: NCLEX Patient Needs: Health Promotion and Maintenance: Techniques of Physical

Assessment

11. The nurse is unable to hear the patient’s breath sounds. What checks does the nurse make of

the stethoscope to determine the cause of this problem?

a. Ensure the stethoscope tubing is at least 20 inches long.


b. Ensure the valve is open to the diaphragm on the head of the stethoscope.

c. Ensure the earpieces are pointed toward the back of the ears.

d. Ensure the bell is placed firmly against the patient’s skin.

ANSWER: B

Feedback

A Tubing should be no longer than 12 to 18 inches. If the tubing is longer than 18

inches, the sounds may become distorted.

B The diaphragm is used to hear high-pitched sounds, such as breath sounds, bowel

sounds, and normal heart sounds. Its structure screens out low-pitched sounds.

C Earpieces are angled toward the nose so that sound is projected toward the

tympanic membrane.

D The bell of the stethoscope is used to hear soft, low-pitched sounds such as extra

heart sounds or vascular sounds (bruit).

DIF: Cognitive Level: Apply REF: 27

TOP: Nursing Process: Assessment

MSC: NCLEX Patient Needs: Health Promotion and Maintenance: Techniques of Physical

Assessment

12. What part of the stethoscope do nurses use to auscultate the chest?

a. Press the bell firmly against the skin to hear sounds and vibrations.

b. The bell of the stethoscope is used to hear breath sounds.

c. The diaphragm of the stethoscope is used to hear heart sounds.

d. Either the bell or the diaphragm is used to auscultate the chest.

ANSWER: C

A

B

C

D

Feedback

The bell should be pressed lightly on the skin with just enough pressure to ensure

that a complete seal exists around the bell. If the bell is pressed too firmly on the

skin, the concave surface is filled with skin, and the bell functions like a

diaphragm and inhibits vibrations.

The bell is used to hear soft, low-pitched sounds such as extra heart sounds or

vascular sounds (bruit).

The diaphragm is used to hear breath sounds, bowel sounds, and normal heart

sounds (high-pitched sounds).

Either the bell or the diaphragm is used to auscultate the chest. The diaphragm is

used to hear breath sounds, bowel sounds, and normal heart sounds (high-pitched

sounds).

DIF: Cognitive Level: Understand REF: 27

TOP: Nursing Process: Assessment

MSC: NCLEX Patient Needs: Health Promotion and Maintenance: Techniques of Physical

Assessment

13. How does the nurse detect an extra heart sound in an adult?

a. Using the bell of a stethoscope

b. With a pulse oximeter

c. Using the diaphragm of a stethoscope


d. With a Doppler ultrasound probe

ANSWER: A

A

B

C

D

Feedback

The bell of the stethoscope is used to hear soft, low-pitched sounds such as extra

heart sounds or vascular sounds (bruit).

Pulse oximetry is a noninvasive measurement of arterial oxygen saturation in the

blood.

The diaphragm is used to hear high-pitched sounds such as breath sounds, bowel

sounds, and normal heart sounds.

A Doppler ultrasound probe is used to detect difficult-to-hear vascular sounds

such as fetal heart tones or peripheral pulses.

DIF: Cognitive Level: Remember REF: 27

TOP: Nursing Process: Assessment

MSC: NCLEX Patient Needs: Health Promotion and Maintenance: Techniques of Physical

Assessment

14. A nurse is preparing to take a patient’s blood pressure. The blood pressure cuff is 5 inches

wide and the patient’s upper arm circumference is 20 inches. How accurate will this patient’s

blood pressure be using this blood pressure cuff?

a. Accurate, the actual value

b. Higher than the actual value

c. Lower than the actual value

d. Unable to determine accuracy with available data

ANSWER: B

A

B

C

D

Feedback

For an arm circumference that is 20 inches, the proper size cuff is at least 8 inches

(20 0.40 = 8). Therefore the blood pressure measurement will not be accurate.

For an arm circumference that is 20 inches, the proper size cuff is at least 8 inches

(20 0.40 = 8). The cuff is 5 inches, which is too narrow. A cuff that is too

narrow will overestimate the blood pressure and report a falsely high value.

For an arm circumference that is 20 inches, the proper size cuff is at least 8 inches

(20 0.40 = 8). Therefore the blood pressure measurement will be higher than the

actual value.

Sufficient data provided to determine accuracy. For an arm circumference that is

20 inches, the proper size cuff is at least 8 inches (20 0.40 = 8).

DIF: Cognitive Level: Analyze REF: 29

TOP: Nursing Process: Assessment

MSC: NCLEX Patient Needs: Health Promotion and Maintenance: Techniques of Physical

Assessment

15. Where does the nurse attach the sensor probe of the pulse oximeter to measure a patient’s

oxygen saturation?

a. The chest over the patient’s heart

b. Over the patient’s abdominal aorta


c. Over the patient’s radial pulse

d. Around the patient’s index finger nail

ANSWER: D

A

B

C

D

Feedback

The chest over the patient’s heart is an incorrect option because the LED would

not be able to reflect off oxygenated and deoxygenated hemoglobin molecules

circulating in blood.

Over the patient’s abdominal aorta is an incorrect option because the LED would

not be able to reflect off oxygenated and deoxygenated hemoglobin molecules

circulating in blood.

Over a patient’s radial pulse is an incorrect option because the LED would not be

able to reflect off oxygenated and deoxygenated hemoglobin molecules

circulating in blood.

The sensor is taped to a highly vascular area, such as around the index finger nail

that allows the light-emitting diode (LED) to reflect off oxygenated and

deoxygenated hemoglobin molecules circulating in blood.

DIF: Cognitive Level: Remember REF: 29

TOP: Nursing Process: Assessment

MSC: NCLEX Patient Needs: Health Promotion and Maintenance: Techniques of Physical

Assessment

16. The patient asks about the meaning of his visual assessment of 20/40 using a Snellen visual

acuity chart. What is the nurse’s appropriate response?

a. “20/40 means your vision is about two times normal.”

b. “A person with corrected vision can see at 20 feet what you can see at 40 feet.”

c. “A person with normal vision can see at 20 feet what you can see at 40 feet.”

d. “A person with normal vision can see at 40 feet what you can see at 20 feet.”

ANSWER: D

A

B

C

D

Feedback

This is an incorrect interpretation of the data.

This is an incorrect interpretation of the data.

This is an incorrect interpretation of the data.

The top number of the recording indicates the distance between the patient and

the chart, and the bottom number indicates the distance at which a person with

normal vision should be able to read certain letters of the chart.

DIF: Cognitive Level: Apply REF: 30-31

TOP: Nursing Process: Assessment

MSC: NCLEX Patient Needs: Health Promotion and Maintenance: Techniques of Physical

Assessment

17. The nurse is using the Snellen chart to assess a patient’s vision. The patient states that the

green line on the chart is shorter than the red line. What is the interpretation of this finding?

a. This patient has normal color perception and abnormal field perception.

b. This patient is color blind but has normal field perception.

c. This patient’s color perception and field perception are normal.


d. This patient is color blind and has abnormal field perception.

ANSWER: A

A

B

C

D

Feedback

Naming the colors of the horizontal lines is a screening for color perception. The

top line is green, and the bottom line is red. Asking which line is longer is a

screening for field perception measurement. The green line is longer.

This is an incorrect interpretation of the data.

This is an incorrect interpretation of the data.

This is an incorrect interpretation of the data.

DIF: Cognitive Level: Apply REF: 31

TOP: Nursing Process: Assessment

MSC: NCLEX Patient Needs: Health Promotion and Maintenance: Techniques of Physical

Assessment

18. What tool does the nurse use to assess the patient’s near vision?

a. A Snellen eye chart placed about 12 inches from the patient’s face.

b. An ophthalmoscope with the diopter set at 0 (zero).

c. A Jaeger or Rosenbaum chart placed about 2 feet from the patient’s face.

d. A newspaper held about 14 inches from the patient’s face.

ANSWER: D

A

B

C

D

Feedback

A Snellen chart is used to assess distant vision.

An ophthalmoscope is used to assess the internal eye.

This is incorrect because of the distance specified. These charts can be used to

assess near vision when placed at 14 inches from the patient’s face.

This can be an alternative to using a Jaeger or Rosenbaum chart held at 14 inches

from the face.

DIF: Cognitive Level: Remember REF: 31

TOP: Nursing Process: Assessment

MSC: NCLEX Patient Needs: Health Promotion and Maintenance: Techniques of Physical

Assessment

19. Using an ophthalmoscope, how does the nurse bring a patient’s interior eye structures into

focus?

a. Using the red filter

b. Adjusting the diopters

c. Dilating the patient’s pupils

d. Using the wide-beam light

ANSWER: B

A

B

Feedback

The red filter facilitates the identification of pallor of the disc and permits the

recognition of retinal hemorrhages by making the blood appear black.

The lens selector dial (diopter) allows the nurse to adjust a set of lenses that

controls focus.


C

D

When the patient’s pupils are dilated, a larger light may be used for the internal

eye examination.

The wide beam light can be used when the patient’s pupils are dilated for better

visualization of internal structures.

DIF: Cognitive Level: Understand REF: 31

TOP: Nursing Process: Assessment

MSC: NCLEX Patient Needs: Health Promotion and Maintenance: Techniques of Physical

Assessment

20. Which action by the nurse describes the correct technique for using an otoscope on an adult?

a. Using the pneumatic attachment to observe for tympanic fluctuation

b. Striking the otoscope against the hand to engage

c. Instructing the adult to raise one finger when a sound is heard

d. Selecting the largest size speculum that fits into the adult’s ear canal

ANSWER: D

A

B

C

D

Feedback

The pneumatic attachment is used to evaluate the fluctuation of the tympanic

membrane in children.

The otoscope is not struck. The instrument that is struck before hearing

assessment is a tuning fork.

Instructing the patient to raise one finger when a sound is heard is done when

using an audiometer to assess hearing.

Using the largest speculum allows visualization, while using a smaller speculum

limits inspection and using a speculum that is too large is uncomfortable to the

adult.

DIF: Cognitive Level: Understand REF: 31

TOP: Nursing Process: Assessment

MSC: NCLEX Patient Needs: Health Promotion and Maintenance: Techniques of Physical

Assessment

21. A nurse is preparing to assess a patient’s ability to detect vibrations. Which piece of

equipment is appropriate for this assessment?

a. Reflex hammer

b. Tuning fork

c. Goniometer

d. Monofilament

ANSWER: B

A

B

C

D

Feedback

A reflex hammer is used to test for deep tendon reflexes.

The tuning fork is used to assess the patient’s ability to detect vibration.

A goniometer is used to measure the degrees of flexion and extension of a joint.

A monofilament is used to test for sensation on the lower extremities.

DIF: Cognitive Level: Remember REF: 33

TOP: Nursing Process: Assessment


MSC: NCLEX Patient Needs: Health Promotion and Maintenance: Techniques of Physical

Assessment

22. To test deep tendon reflexes, the nurse uses which instrument?

a. Goniometer

b. Calipers

c. Reflex hammer

d. Monofilament

ANSWER: C

A

B

C

D

Feedback

A monofilament is used to test for sensation on the lower extremities.

Calipers are used to measure thickness of subcutaneous tissue to estimate the

amount of body fat.

A reflex hammer is used to test deep tendon reflexes.

A monofilament is used to test for sensation on the lower extremities.

DIF: Cognitive Level: Remember REF: 33

TOP: Nursing Process: Assessment

MSC: NCLEX Patient Needs: Health Promotion and Maintenance: Techniques of Physical

Assessment

23. A nurse is using the finger pads to palpate a patient’s dorsalis pedis pulses and is unable to

feel any pulses. Which action is appropriate for the nurse to perform next?

a. Document that the dorsalis pedis pulses are not palpable.

b. Have the patient stand and try again to palpate the pulses.

c. Use a Doppler to detect the presence of the pulses.

d. Palpate the dorsalis pedis pulses using the ulnar surface of the hand.

ANSWER: C

A

B

C

D

Feedback

Document that the dorsalis pedis pulses are not palpable. Although the pulse may

not be palpable, the nurse always tries a Doppler to determine if the pulse can be

heard, even when it cannot be felt.

Have the patient stand and try again to palpate the pulses. Changing positions will

not facilitate palpation of a pulse.

Use a Doppler to detect the presence of the pulses. The Doppler uses ultrasonic

waves to detect difficult-to-hear vascular sounds, such as peripheral pulses.

Palpate the dorsalis pedis pulses using the ulnar surface of the hand. The ulnar

surface of the hand is used to palpate for vibrations rather than pulsations.

DIF: Cognitive Level: Analyze REF: 33

TOP: Nursing Process: Assessment

MSC: NCLEX Patient Needs: Health Promotion and Maintenance: Techniques of Physical

Assessment

24. How does the nurse detect a pulse when using a Doppler?

a. The pulsation is felt.

b. The pulsation is heard.


c. The pulse wave is seen on a screen.

d. The pulse wave is printed out on special paper.

ANSWER: B

A

B

C

D

Feedback

A Doppler is used when the pulses cannot be palpated.

A Doppler amplifies sounds difficult to hear with an acoustic stethoscope.

A Doppler amplifies the sound of the pulsation.

A Doppler amplifies the sound of the pulsation

DIF: Cognitive Level: Remember REF: 33

TOP: Nursing Process: Assessment

MSC: NCLEX Patient Needs: Health Promotion and Maintenance: Techniques of Physical

Assessment

25. A nurse is assessing joint function of a patient with severe rheumatoid arthritis. Which

instrument/tool does the nurse use to measure the degree of flexion and extension of the

patient’s knee joints?

a. Calipers

b. Ruler or tape measure

c. Goniometer

d. Doppler

ANSWER: C

A

B

C

D

Feedback

Calipers are used to estimate the amount of body fat.

A ruler or tape measure cannot accurately measure the degree of flexion and

extension of joints.

A goniometer is used to measure the degree of flexion and extension of a joint.

Doppler is used to detect the presence of pulses.

DIF: Cognitive Level: Understand REF: 33

TOP: Nursing Process: Assessment

MSC: NCLEX Patient Needs: Health Promotion and Maintenance: Techniques of Physical

Assessment

26. When does a nurse choose to use skinfold calipers when collecting assessment data?

a. Calculating the patient’s body mass index

b. Inspecting the patient’s skin

c. Determining the amount of the patient’s lean body tissue

d. Estimating the amount of the patient’s body fat

ANSWER: D

A

B

C

Feedback

Body mass index is a formula for determining obesity that is calculated by

dividing a person’s weight in kilograms by the height in meters.

Calipers estimate body fat. They are not needed to inspect skin.

There is no specific method to determine the amount of lean body tissue.


D

Estimating the amount of the patient’s body fat is the purpose of using skin

calipers.

DIF: Cognitive Level: Remember REF: 34

TOP: Nursing Process: Assessment

MSC: NCLEX Patient Needs: Health Promotion and Maintenance: Techniques of Physical

Assessment

27. When does a nurse use a Pederson or Graves speculum for examination of a patient?

a. To inspect the external ear

b. To assess the vaginal canal

c. To inspect nasal passages

d. To assess the oropharynx

ANSWER: B

A

B

C

D

Feedback

The external ear is inspected using an otoscope.

The vaginal canal and cervix are inspected using a Pederson or Graves speculum

or a pediatric or virginal speculum.

The nasal passages are inspected using a nasal speculum.

The oropharynx is inspected using a tongue blade and penlight.

DIF: Cognitive Level: Remember REF: 34

TOP: Nursing Process: Assessment

MSC: NCLEX Patient Needs: Health Promotion and Maintenance: Techniques of Physical

Assessment

28. What are characteristics of an audioscope?

a. Screens for hearing ability

b. Allows visualization into the ear canal

c. Must be calibrated before use

d. Uses vibration to estimate hearing loss

ANSWER: A

A

B

C

D

Feedback

An audioscope screens for hearing ability.

The otoscope allows inspection of the ear canal.

Calibration is unnecessary. An audioscope needs batteries that are charged.

The tuning fork is the tool that uses vibration to detect hearing loss.

DIF: Cognitive Level: Remember REF: 35

TOP: Nursing Process: Assessment

MSC: NCLEX Patient Needs: Health Promotion and Maintenance: Techniques of Physical

Assessment

29. A patient with type 2 diabetes mellitus has an infected lesion on his foot. During the history of

his present illness, he reports, “I had a cut on my foot, but I did not even feel it.” What

equipment does the nurse use to gather more data about his foot?

a. A Wood lamp


b. Transilluminator

c. Monofilament

d. Reflex hammer

ANSWER: C

A

B

C

D

Feedback

A Wood lamp is used to detect fungal infection on the skin.

A transilluminator differentiates the characteristics of tissue, fluid, and air within

a specific body cavity.

A monofilament is used to test for sensation on the lower extremities. Because

this patient could not feel the cut on his foot, perhaps he has lost sensation.

A reflex hammer is used to test for deep tendon reflexes.

DIF: Cognitive Level: Apply REF: 35

TOP: Nursing Process: Assessment

MSC: NCLEX Patient Needs: Health Promotion and Maintenance: Techniques of Physical

Assessment

30. A patient is complaining of pain over the maxillary sinuses. Which device does the nurse use

to determine if there is air or fluid in the patient’s sinuses?

a. Magnification device

b. Transilluminator

c. Monofilament

d. Wood lamp

ANSWER: B

A

B

C

D

Feedback

A magnification device helps visualize the tissue, but will not determine if sinuses

are filled with air or fluid.

A transilluminator disseminates its light source under the surface of the skin to

determine if the areas under the surface, such as the sinuses, are filled with air,

fluid, or tissue.

A monofilament is used to test for sensation on the lower extremities.

A Wood lamp is used to detect fungal infections.

DIF: Cognitive Level: Understand REF: 35

TOP: Nursing Process: Assessment

MSC: NCLEX Patient Needs: Health Promotion and Maintenance: Techniques of Physical

Assessment

31. A nurse suspects that a large skin lesion on the patient’s forearm is a fungal infection. Which

device does the nurse use to confirm his suspicion?

a. Pen light

b. Magnification device

c. Transilluminator

d. Wood lamp

ANSWER: D

Feedback


A

B

C

D

A pen light is used to highlight a lesion for inspection, but will not determine if it

is caused by a fungus.

A magnification device helps visualize the lesion, but will not determine if it is

caused by a fungus.

A transilluminator disseminates its light source under the surface of the skin to

determine if the area under the surface is filled with air, fluid, or tissue.

Skin lesions caused by a fungal infection exhibit a fluorescent yellow-green or

blue-green color when examined with a Wood lamp.

DIF: Cognitive Level: Understand REF: 36

TOP: Nursing Process: Assessment

MSC: NCLEX Patient Needs: Health Promotion and Maintenance: Techniques of Physical

Assessment


Chapter 4: General Inspection and Measurement of Vital Signs

Test Bank

MULTIPLE CHOICE

1. Which body system does the nurse assess primarily by inspection?

a. Respiratory

b. Gastrointestinal

c. Skin

d. Cardiovascular

ANSWER: C

A

B

C

D

Feedback

The respiratory system is assessed primarily using auscultation, but also

percussion and inspection when observing pale or cyanotic skin from hypoxia.

The gastrointestinal system is assessed primarily by auscultation and palpation,

but also with inspection when looking at the contour of the abdomen.

Skin is assessed primarily using inspection, but also palpation.

The cardiovascular system is assessed primarily with auscultation and palpation,

but also by inspection when looking at the color of extremities for evidence of

perfusion or edema.

DIF: Cognitive Level: Remember REF: 37

TOP: Nursing Process: Assessment

MSC: NCLEX: Patient Needs: Physiologic Integrity: Reduction of Risk Potential: System Specific

Assessments

2. A patient is sitting slightly forward bracing his arms on his knees in a tripod position. This

position is associated with which symptom?

a. Abdominal pain

b. Spinal deformity

c. Back pain

d. Breathing difficulty

ANSWER: D

A

B

C

D

Feedback

Positions used by patients with abdominal pain vary depending upon what organ

is involved. For example, patients with appendicitis tend to lie very still; those

with acute pancreatitis prefer the fetal position for pain relief.

Spinal deformity usually affects the patient’s gait or causes a slumped posture.

Back pain usually affects the patient’s gait or causes a slumped posture.

Breathing difficulty is associated with the tripod position, which allows maximal

expansion of the muscles of respiration.

DIF: Cognitive Level: Remember REF: 37

TOP: Nursing Process: Assessment

MSC: NCLEX: Patient Needs: Physiologic Integrity: Reduction of Risk Potential: System Specific

Assessments

This study source was downloaded by 100000838401522 from CourseHero.com on 03-03-2022 08:50:15 GMT -06:00

https://www.coursehero.com/file/20908989/c4/


3. The temperature of a patient is measured every 6 hours at 6 AM, 12 PM, 6 PM, and 12 AM.

Which temperature reading is expected to be low due to a normal variation?

a. The measurement at 6 AM

b. The measurement at 12 PM

c. The measurement at 6 PM

d. The measurement at 12 AM

ANSWER: A

A

B

C

D

Feedback

Early in the morning is the time of the lowest temperature of the day due to

circadian rhythms.

A low temperature due to circadian rhythms is not expected at this time.

The highest temperature occurs in the late afternoon and early evening due to

circadian rhythms.

A low temperature due to circadian rhythms is not expected at this time.

DIF: Cognitive Level: Understand REF: 38

TOP: Nursing Process: Assessment

MSC: NCLEX: Patient Needs: Physiologic Integrity: Reduction of Risk Potential: System Specific

Assessments

4. Which statement is correct regarding taking or interpreting axillary temperatures?

a. Axillary temperatures should not be used in patients less than 2 years of age.

b. Readings may be less accurate.

c. The thermometer is left in place for no more than 3 minutes.

d. The thermometer is placed in the axilla with the shoulder abducted.

ANSWER: B

A

B

C

D

Feedback

The axilla is a common site for temperature measurement on infants and children.

Multiple studies have shown temperature measurements at the axillary site are

less accurate compared with alternative sites.

The thermometer is left in place until the audible signal occurs and the

temperature appears on the screen.

Place the probe in the middle of the axilla, with the arm held against the body

(adducted).

DIF: Cognitive Level: Understand REF: 39

TOP: Nursing Process: Assessment

MSC: NCLEX: Patient Needs: Physiologic Integrity: Reduction of Risk Potential: System Specific

Assessments

5. A temperature of 99.8° F taken in the axilla is equivalent to which temperature value taken

orally?

a. 100.8° F

b. 99.8° F

c. 98.8° F

d. 97.8° F

This study source was downloaded by 100000838401522 from CourseHero.com on 03-03-2022 08:50:15 GMT -06:00

https://www.coursehero.com/file/20908989/c4/


ANSWER: A

A

B

C

D

Feedback

Normal temperature readings from the axilla are about 1° F below the normal oral

temperature.

Normal temperature readings from the axilla are about 1° F below the normal oral

temperature.

Normal temperature readings from the axilla are about 1° F below the normal oral

temperature.

Normal temperature readings from the axilla are about 1° F below the normal oral

temperature.

DIF: Cognitive Level: Apply REF: 39

TOP: Nursing Process: Assessment

MSC: NCLEX: Patient Needs: Physiologic Integrity: Reduction of Risk Potential: System Specific

Assessments

6. The nurse suspects an irregularity in the rhythm of the patient’s radial pulse. What is the most

appropriate action for this nurse to take at this time?

a. Document this rhythm as normal for the patient.

b. Use a Doppler to check the brachial pulse.

c. Count the patient’s apical pulse for a full minute.

d. Count the radial pulse again for 15 seconds and multiply by 4.

ANSWER: C

A

B

C

D

Feedback

An irregular rhythm is not a normal finding. The pulsation between each beat

should be the same or regular.

A Doppler is not indicated in this case; it is used when the pulse cannot be

palpated.

When an irregular pulse is palpated, the nurse counts the number of pulsations for

a full minute.

Counting the radial pulse again for 15 seconds and multiplying by 4 may

reconfirm the initial findings, but does not provide additional data for the nurse

on this patient.

DIF: Cognitive Level: Apply REF: 39

TOP: Nursing Process: Assessment

MSC: NCLEX: Patient Needs: Physiologic Integrity: Reduction of Risk Potential: System Specific

Assessments

7. The patient with a respiratory rate that is within normal limits is the whose respiratory

rate is breaths/min.

a. 16-month-old; 36

b. 6-year-old; 20

c. 14-year-old;26

d. 40-year-old; 10

ANSWER: B

This study source was downloaded by 100000838401522 from CourseHero.com on 03-03-2022 08:50:15 GMT -06:00

https://www.coursehero.com/file/20908989/c4/


Feedback

A A toddler’s respiratory rate ranges from 24 to 32.

B A school-age child’s respiratory rate ranges from 18 to 26.

C An adolescent’s respiratory rate ranges from 12 to 16.

D An adult’s respiratory rate ranges from 12 to 20.

DIF: Cognitive Level: Apply REF: 40

TOP: Nursing Process: Assessment

MSC: NCLEX: Patient Needs: Physiologic Integrity: Reduction of Risk Potential: System Specific

Assessments

8. A nurse is taking vital signs of an adult patient whose oxygen saturation is 96%. The patient’s

temperature is 102° F, blood pressure is 130/86, pulse is 100 beats/min, and respiratory rate is

26 breaths/min. Which factor may be contributing to the elevated respiratory rate?

a. The patient’s temperature

b. The patient’s oxygen saturation

c. The patient’s pulse rate

d. The patient’s blood pressure

ANSWER: A

Feedback

A Fever is a factor that may increase respiratory rate, and this patient’s temperature

is 102° F.

B The patient’s oxygen saturation is a measure of the oxygen carried by hemoglobin

and it is within expected limits—above 90%.

C The patient’s pulse rate may be due to the high temperature, but a pulse of 100

does not contribute to an elevated respiratory rate in this case.

D The patient’s blood pressure is higher than normal, but does not contribute to an

elevated respiratory rate in this case.

DIF: Cognitive Level: Apply REF: 40

TOP: Nursing Process: Assessment

MSC: NCLEX: Patient Needs: Physiologic Integrity: Reduction of Risk Potential: System Specific

Assessments

9. Nurses understand that a patient’s diastolic pressure represents which physiologic function?

a. The pressure needed to open the aortic and pulmonic valves

b. The pressure in blood vessels when the ventricles contract

c. The pressure of the blood returning to the heart from the venous system

d. The pressure in blood vessels when the ventricles are relaxed

ANSWER: D

A

B

C

Feedback

The pressure needed to open the aortic and pulmonic valves is called the

afterload.

The pressure in blood vessels when the ventricles contract is the definition of the

systolic pressure.

The pressure of the blood returning to the heart from the venous system is

incorrect.

This study source was downloaded by 100000838401522 from CourseHero.com on 03-03-2022 08:50:15 GMT -06:00

https://www.coursehero.com/file/20908989/c4/


D

The pressure in blood vessels when the ventricles are relaxed is the definition of

the diastolic pressure.

DIF: Cognitive Level: Understand REF: 41

TOP: Nursing Process: Assessment

MSC: NCLEX: Patient Needs: Physiologic Integrity: Reduction of Risk Potential: System Specific

Assessments

10. According to research findings, which site is preferred for measuring blood pressure when the

nurse is unable to use the patient’s upper arms?

a. Ankle

b. Thigh

c. Calf

d. Wrist

ANSWER: A

A

B

C

D

Feedback

A study comparing accuracy among sites recommended the ankle as an

alternative site for blood pressure measurement.

The thigh is an alternative site, but the ankle is the preferred site.

A study comparing accuracy among sites recommended the ankle site in

preference to the calf as an alternative site for blood pressure measurement if the

upper arm is unavailable.

Approaches to measuring blood pressure using the wrist and finger sites have

been developed, but these lack acceptable accuracy and cost efficiency to be

recommended for clinical practice.

DIF: Cognitive Level: Remember REF: 41

TOP: Nursing Process: Assessment

MSC: NCLEX: Patient Needs: Physiologic Integrity: Reduction of Risk Potential: System Specific

Assessments

11. A patient’s blood pressure has been averaging 120/72 when using the upper arms. Today the

nurse uses this patient’s thigh to measure the blood pressure. What is the expected systolic

pressure using the thigh that is equivalent to a systolic pressure of 120?

a. A systolic reading of 110 mm Hg

b. A systolic reading of 120 mm Hg

c. A systolic reading of 140 mm Hg

d. A systolic reading of 170 mm Hg

ANSWER: C

A

B

C

D

Feedback

A systolic reading of 110 mm Hg is too low.

A systolic reading of 120 mm Hg is too low.

Normally the systolic blood pressure is 10 to 40 mm Hg higher in the leg than in

the arm.

A systolic reading of 170 mm Hg is too high.

DIF: Cognitive Level: Apply REF: 41

This study source was downloaded by 100000838401522 from CourseHero.com on 03-03-2022 08:50:15 GMT -06:00

https://www.coursehero.com/file/20908989/c4/


TOP: Nursing Process: Assessment

MSC: NCLEX: Patient Needs: Physiologic Integrity: Reduction of Risk Potential: System Specific

Assessments

12. A nurse notices that the patient has gained 11 lb. If this increase in weight is related to fluid

retention, the patient is retaining approximately how many liters of fluid?

a. 1 L

b. 5 L

c. 11 L

d. 24 L

ANSWER: B

Feedback

A Every kg equals a liter of fluid. Thus, 11 lb ÷ 2.2 = 5 L.

B Every kilogram (kg) equals a liter of fluid. Thus, 11 lb ÷ 2.2 = 5 L.

C Every kg equals a liter of fluid. Thus, 11 lb ÷ 2.2 = 5 L.

D Every kg equals a liter of fluid. Thus, 11 lb ÷ 2.2 = 5 L. This answer is obtained

by multiplying 11 by 2.2 instead of dividing.

DIF: Cognitive Level: Apply REF: 43

TOP: Nursing Process: Assessment

MSC: NCLEX: Patient Needs: Physiologic Integrity: Physiologic Adaptation: Fluid and Electrolyte

Imbalances

MULTIPLE RESPONSE

1. Which method of temperature measurement indirectly reflects inner core temperature?

Select all that apply.

a. Axillary temperature

b. Oral temperature

c. Tympanic temperature

d. Rectal temperature

e. Temporal artery temperature

ANSWER: B, E

Correct: Inner core temperature is measured indirectly because the probe is placed near an

artery. For oral temperature, the probe is placed near the carotid artery and the temporal artery

is used for the temporal artery temperature.

Incorrect: For axillary, tympanic, and rectal temperatures, the probe is not placed close to any

major blood vessels.

DIF: Cognitive Level: Understand REF: 38

TOP: Nursing Process: Assessment

MSC: NCLEX: Patient Needs: Physiologic Integrity: Reduction of Risk Potential: System Specific

Assessments

2. Which method of temperature measurement does a nurse choose when assessing children?

Select all that apply.

a. Axillary temperature

b. Rectal temperature

This study source was downloaded by 100000838401522 from CourseHero.com on 03-03-2022 08:50:15 GMT -06:00

https://www.coursehero.com/file/20908989/c4/


c. Temporal artery temperature

d. Oral temperature

e. Tympanic membrane temperature

ANSWER: A, C, D, E

Correct: Axillary, temporal artery, oral, and tympanic membrane temperatures are appropriate

for children.

Incorrect: Rectal temperature measurement is considered safe and accurate for adults only.

DIF: Cognitive Level: Apply REF: 38-39

TOP: Nursing Process: Assessment

MSC: NCLEX: Patient Needs: Physiologic Integrity: Reduction of Risk Potential: System Specific

Assessments

3. Which action by the nurse results in the patient’s blood pressure measurement being falsely

high? Select all that apply.

a. Using a blood pressure cuff that is too narrow for the patient’s upper arm

b. Deflating the blood pressure cuff too rapidly

c. Wrapping the blood pressure cuff too loosely

d. Reinflating the blood pressure cuff before it completely deflates

e. Positioning the patient’s arm above the level of the heart

ANSWER: A, C, D, E

Correct: Using a blood pressure cuff that is too narrow for the patient’s upper arm, wrapping

the cuff too loosely, reinflating the cuff before it completely deflates, and positioning the

patient’s arm above the level of the heart all result in readings that are falsely high.

Incorrect: Deflating the blood pressure cuff too rapidly causes the blood pressure reading to

be falsely low.

DIF: Cognitive Level: Remember REF: 43

TOP: Nursing Process: Assessment

MSC: NCLEX: Patient Needs: Physiologic Integrity: Reduction of Risk Potential: System Specific

Assessments

4. Which action by the nurse results in the patient’s blood pressure measurement being falsely

low? Select all that apply.

a. Using a blood pressure cuff that is too wide for the patient’s arm

b. Not inflating the blood pressure cuff enough

c. Positioning the patient’s arm above the level of the heart

d. Wrapping the cuff too loosely around the arm

e. Deflating the cuff too rapidly

ANSWER: A, B, E

Correct: Using a blood pressure cuff that is too wide for the patient’s arm, not inflating the

blood pressure cuff enough, and deflating the cuff too rapidly could result in a false low

reading.

Incorrect: Positioning the patient’s arm above the level of the heart and wrapping the cuff too

loosely around the arm causes the blood pressure to be falsely high.

DIF: Cognitive Level: Remember REF: 43

TOP: Nursing Process: Assessment

MSC: NCLEX: Patient Needs: Physiologic Integrity: Reduction of Risk Potential: System Specific

This study source was downloaded by 100000838401522 from CourseHero.com on 03-03-2022 08:50:15 GMT -06:00

https://www.coursehero.com/file/20908989/c4/


Assessments

5. The nurse taking a patient’s blood pressure recognizes that several factors may cause an

increased blood pressure reading. Which factors below can increase blood pressure? Select all

that apply.

a. The patient rates pain at a level of 7 on a scale of 0 to 10.

b. The cuff was reinflated before being completely deflated.

c. The patient drank cold milk just before the reading.

d. The time of day is late afternoon.

e. The cuff is too wide for the extremity.

ANSWER: A, B, D

Correct: Rating pain at a level of 7 on a scale of 0 to 10, reinflating the cuff before being

completely deflated, and taking the reading in late afternoon are all factors that can increase

blood pressure.

Incorrect: Drinking cold milk just before the reading will not affect blood pressure, but

drinking caffeine such as coffee or cola may increase blood pressure. A wide cuff makes the

reading lower than it actually is rather than higher.

DIF: Cognitive Level: Apply REF: 43

TOP: Nursing Process: Assessment

MSC: NCLEX: Patient Needs: Physiologic Integrity: Reduction of Risk Potential: System Specific

Assessments

COMPLETION

1. A female patient admitted with fluid retention has been in diuretic therapy to remove fluid.

She weighed 187 lb on admission. Today she weighs 179 lb. Since admission, this patient has

lost L from fluid loss.

ANSWER:

3.6

1 kg (2.2 lb) = 1 L; 187 – 179 = 8 lb weight loss divided by 2.2 = 3.6 L.

DIF: Cognitive Level: Apply REF: 43

TOP: Nursing Process: Assessment

MSC: NCLEX: Patient Needs: Physiologic Integrity: Physiologic Adaptation: Fluid and Electrolyte

Imbalances

This study source was downloaded by 100000838401522 from CourseHero.com on 03-03-2022 08:50:15 GMT -06:00

https://www.coursehero.com/file/20908989/c4/


Chapter 5: Ethnic, Cultural, and Spiritual Considerations

Test Bank

MULTIPLE CHOICE

1. What are the characteristics of one’s culture?

a. Color of skin and hair

b. System of beliefs and practices

c. Food preferences

d. Language and religion

ANSWER: B

A

B

C

D

Feedback

Skin and hair color are examples of racial characteristics based on genetics.

System of beliefs and practices is part of the definition of culture.

Food preferences are an example of ethnicity.

Language and religion are examples of ethnicity.

DIF: Cognitive Level: Remember REF: 47

TOP: Nursing Process: Assessment

MSC: NCLEX Patient Needs: Psychosocial Integrity: Cultural Diversity

2. Which example below best characterizes a patient’s race?

a. The language spoken in the patient’s home is Tagalog.

b. The patient’s family follows a kosher diet.

c. The patient and his family have blonde hair and fair skin.

d. The patient’s grandparents came to the United States from Germany.

ANSWER: C

A

B

C

D

Feedback

The language spoken at home refers to ethnicity.

A kosher diet refers to ethnicity.

Blonde hair and fair skin indicate genetics and race.

Although the patient and grandparents may share the same race, that the

grandparents came to the United States from Germany does not relate to race.

DIF: Cognitive Level: Understand REF: 47

TOP: Nursing Process: Assessment

MSC: NCLEX Patient Needs: Psychosocial Integrity: Cultural Diversity

3. After the death of a Native American man, the nurse opened a window to allow spirits to

leave. This action is an example of which attribute of the concept of cultural competence?

a. Adapting interventions based on cultural practices (Tailoring)

b. Gaining information about cultural differences (Knowledge)

c. Considering the effects of another’s values and experiences (Understanding)

d. Showing appreciation for cultural differences (Respect)

ANSWER: A

This study source was downloaded by 100000838401522 from CourseHero.com on 03-03-2022 08:49:55 GMT -06:00

https://www.coursehero.com/file/20909004/c5/


A

B

C

D

Feedback

The nurse changed (adapted) the usual procedure after death by opening the

window, which was a practice of this Native American family.

Gaining information is the first step, but is insufficient to adapt an intervention as

described in this example.

Understanding develops after knowledge, but does not involve taking action.

Respect is the step before Adaptation, when nurses comprehend the cultural

differences, but do not adapt nursing care.

DIF: Cognitive Level: Apply REF: 49

TOP: Nursing Process: Assessment

MSC: NCLEX Patient Needs: Psychosocial Integrity: Cultural Diversity

4. A Hispanic patient tells an African American nurse, “You are African American and can’t

possibly understand how a person like me feels.” What is an appropriate response by the nurse

at this time?

a. Find a nurse who is not African American to interview the patient.

b. Ask the patient, “Why do you think that, since we just met?”

c. Note that the patient is very defensive about being racially different.

d. Encourage the patient to describe what he means by his statement.

ANSWER: D

A

B

C

D

Feedback

Finding a nurse who is not African American to interview the patient is neither

necessary nor practical.

Asking the patient “Why do you think that, since we just met?” sounds defensive

and may not facilitate further communication.

Making an assumption “that the patient is very defensive about being racially

different” is not based on any data.

By encouraging the patient to describe what he means by his statement, the nurse

is demonstrating cultural awareness and sensitivity by gathering more data about

the unique beliefs and value systems of this patient.

DIF: Cognitive Level: Apply REF: 50

TOP: Nursing Process: Assessment

MSC: NCLEX Patient Needs: Psychosocial Integrity: Cultural Diversity

5. A male nurse is assigned to the care of a gay male with alcoholism. This sexual orientation is

inconsistent with the beliefs of the nurse. What actions, if any, can the nurse take to provide

patient-centered care to this patient?

a. No action is necessary at this time.

b. Examine his own feelings about alcoholism and homosexuality.

c. Determine the patient’s degree of risk for contracting the human immunodeficiency

virus.

d. Discuss homosexuality and alcoholism with the patient.

ANSWER: B

Feedback

This study source was downloaded by 100000838401522 from CourseHero.com on 03-03-2022 08:49:55 GMT -06:00

https://www.coursehero.com/file/20909004/c5/


A

B

C

D

Not acting will not facilitate patient-centered care. The nurse needs to reflect on

his feelings about this patient.

Examining one’s own feelings about alcoholism and gay men specifically and

identifying one’s own beliefs and values generally is part of the process for

developing cultural competence, which should allow one to recognize the

uniqueness of patients and to demonstrate respect for individuals.

Determining the patient’s degree of risk for contracting the human

immunodeficiency virus is based on an assumption that because the patient is gay,

he has or will have the human immunodeficiency virus.

A discussion with the patient may help the nurse understand him, but will not

necessarily provide insight into the nurse’s beliefs and feelings.

DIF: Cognitive Level: Apply REF: 50

TOP: Nursing Process: Assessment

MSC: NCLEX Patient Needs: Psychosocial Integrity: Cultural Diversity

6. Which nursing behaviors indicate culturally competent care?

a. Recognizing that there are different definitions of health and illness

b. Complying with the stated plan of treatment despite the patient’s differing opinion

c. Understanding that there is diversity even among people of the same cultural group

d. Helping patients of different cultures adopt the beliefs and behaviors of the

dominant culture

ANSWER: C

A

B

C

D

Feedback

Recognizing that there are many different definitions of health and illness is not

sufficient for culturally competent care.

Complying with the stated plan of treatment despite the patient’s differing opinion

is not a culturally competent behavior. The patient needs to understand, support,

and participate in the plan of care.

Understanding that there is diversity even among people of the same cultural

group prevents assumptions and stereotypes that inhibit culturally competent care.

Helping patients of different cultures adopt the beliefs and behaviors of the

dominant culture is an example of the opposite of cultural competence; it assumes

that all persons should adopt certain beliefs and behaviors.

DIF: Cognitive Level: Understand REF: 50

TOP: Nursing Process: Assessment

MSC: NCLEX Patient Needs: Psychosocial Integrity: Cultural Diversity

7. A nurse is conducting an assessment of an American Indian woman who has come to the

clinic complaining of persistent headaches. The patient tells the nurse that the medicines

prescribed by the tribal healer have done “some good.” What is the appropriate response of

the nurse at this time?

a. “I advise you to stop taking those medicines from the tribal healer.”

b. “Perhaps you should increase the frequency of the healer’s medicines.”

c. “Tell me about these medicines and how often you are using them.”

d. “Could your headaches be caused by the healer’s medicines?”

This study source was downloaded by 100000838401522 from CourseHero.com on 03-03-2022 08:49:55 GMT -06:00

https://www.coursehero.com/file/20909004/c5/


ANSWER: C

A

B

C

D

Feedback

Advising the patient to stop taking any nonprescription medicines is inappropriate

until the nurse knows the details about all medicines used by the patient.

Telling the patient to increase the frequency of the healer’s medicines is

inappropriate until the nurse knows details about the medicines.

Asking the patient about the nature of these medicines and how often the patient

uses them allows the nurse to collect data about the medicines and their uses, to

learn more about the practices used by this patient to improve her health, and to

check for potential drug interaction before prescribing other medications or

treatment.

Suggesting the patient’s headaches are caused by the healer’s medicines is

inappropriate until the nurse knows details about the medicines.

DIF: Cognitive Level: Apply REF: 51

TOP: Nursing Process: Assessment

MSC: NCLEX Patient Needs: Psychosocial Integrity: Cultural Diversity

8. Which question is the most appropriate to learn about a patient’s religious practices?

a. “How often do you go to church?”

b. “Where is your church located?”

c. “Do you mind telling me about your religion?”

d. “Do you have any specific religious or spiritual practices or beliefs?”

ANSWER: D

A

B

C

D

Feedback

Asking the patient about his or her church makes the assumption that the patient

worships in a church, which may not be correct.

Asking the patient about the location of his or her church makes the assumption

that the patient worships in a church, which may not be correct.

The question is not an efficient way to learn about religious or spiritual practices.

“Do you have any specific religious or spiritual practices or beliefs?” is a broad,

open-ended question that allows the patient to describe his or her religious

beliefs. The question makes no reference to any particular faith.

DIF: Cognitive Level: Apply REF: 51

TOP: Nursing Process: Assessment

MSC: NCLEX Patient Needs: Psychosocial Integrity: Cultural Diversity

9. A patient tells the nurse that her religion prohibits her from eating food prepared outside of a

special kitchen. What is the nurse’s appropriate action to meet this patient’s needs?

a. Call the dietary department to cancel the patient’s meal tray.

b. Tell the patient that her diet must be carefully monitored and prepared at the

hospital.

c. Tell the patient that because of her illness, a few changes to her religious

requirements will be necessary.

d. Ask the patient to describe the requirements for the special kitchen.

ANSWER: D

This study source was downloaded by 100000838401522 from CourseHero.com on 03-03-2022 08:49:55 GMT -06:00

https://www.coursehero.com/file/20909004/c5/


A

B

C

D

Feedback

Cancelling the patient’s food tray does not meet the patient’s needs.

Telling the patient that her diet must be carefully monitored and prepared at the

hospital is inappropriate because it does not consider the ethnicity of the patient

and is not culturally competent care.

Asking the patient to change her religious requirements does not respect her

ethnicity and is not culturally competent care.

Asking the patient to describe the requirements for the special kitchen allows the

nurse to collect more data about the needs of the patient.

DIF: Cognitive Level: Apply REF: 52

TOP: Nursing Process: Assessment

MSC: NCLEX Patient Needs: Psychosocial Integrity: Religious and Spiritual Influences on Health

10. Which question is most effective in assessing a patient’s personal beliefs about health and

illness?

a. “What or who do you believe controls your health?”

b. “Do you see your health care provider annually?”

c. “Do you have specific beliefs about health and illness?”

d. “Who makes the health decisions in your family?”

ANSWER: A

A

B

C

D

Feedback

What or who do you believe controls your health?” is a broad question that allows

the patient to describe his or her beliefs.

Asking if the patient visits his or her health care provider annually assumes the

patient has a health care provider, and it is a closed-ended question that yields

little data.

“Do you have specific beliefs about health and illness?” will yield a “yes” or “no”

answer, but does not provide information needed.

“Who makes the health decisions in your family?” does not provide information

about the beliefs of the patient

DIF: Cognitive Level: Understand REF: 51

TOP: Nursing Process: Assessment

MSC: NCLEX Patient Needs: Psychosocial Integrity: Religious and Spiritual Influences on Health

11. An Asian woman comes to the clinic with a complaint of back pain. During the history, she

tells the nurse that she usually uses acupuncture for her pain. What is the nurse’s best

response?

a. “When have you used acupuncture, and what effects did it have?”

b. “Acupuncture is good for some problems, but for major illnesses it’s best to use

medications.”

c. “Why did you use acupuncture?”

d. “I have heard that many Asian people use acupuncture.”

ANSWER: A

This study source was downloaded by 100000838401522 from CourseHero.com on 03-03-2022 08:49:55 GMT -06:00

https://www.coursehero.com/file/20909004/c5/


A

B

C

D

Feedback

Asking about the use and effectiveness of acupuncture acknowledges the patient’s

choice of treatment and allows the nurse to gather more data about its

effectiveness for this patient.

Asking “Why did you use acupuncture?” is a nontherapeutic response and

questions the patient’s reason for her health care decisions.

Asking “Why did you use acupuncture?” is a nontherapeutic response and

questions the patient’s reason for her health care decisions.

Commenting that, “I have heard that many Asian people use acupuncture” is an

example of stereotyping.

DIF: Cognitive Level: Apply REF: 51

TOP: Nursing Process: Assessment

MSC: NCLEX Patient Needs: Psychosocial Integrity: Cultural Diversity

12. A nurse is caring for a woman who has given birth to a healthy baby. The woman’s husband

and mother are in the room, and more family members are in the lobby. Which comment by

the nurse demonstrates culturally competent care?

a. “We need to take your baby to the nursery now for a physical examination.”

b. “Are there any ceremonies or other practices that are important to you at this

time?”

c. “We can only allow immediate family in the room with you at this time.”

d. “Because breastfeeding is the best way to feed your baby, we’ll bring your baby to

you when she is hungry.”

ANSWER: B

A

B

C

D

Feedback

Although a physical examination is important, it is not necessary for culturally

competent care and can be performed at another time.

Asking about ceremonies or other practices collects data about culturally-based

behaviors of this patient and family surrounding childbirth.

Limiting visitors based on an arbitrary or subjective definition of “immediate

family” may interfere with culturally competent care.

Breastfeeding is a choice the mother makes, not the nurse.

DIF: Cognitive Level: Apply REF: 51

TOP: Nursing Process: Intervention

MSC: NCLEX Patient Needs: Psychosocial Integrity: Cultural Diversity

13. A nurse can improve cultural awareness with which behavior?

a. Being sensitive to differences between the cultures of the nurse and patient

b. Making generalizations about various ethnic and cultural groups

c. Learning everything about the various cultural groups in the nurse’s city

d. Taking a foreign language class

ANSWER: A

A

Feedback

Being sensitive to differences between the cultures of the nurse and patient. This

allows the nurse to be open and to collect data about each unique patient.

This study source was downloaded by 100000838401522 from CourseHero.com on 03-03-2022 08:49:55 GMT -06:00

https://www.coursehero.com/file/20909004/c5/


B

C

D

Making generalizations about various ethnic and cultural groups leads to

stereotyping.

Learning everything about the various cultural groups in the nurse’s city provides

useful information about a specific city, but does not necessarily improve cultural

awareness of cultures in other cities.

Learning a foreign language may help a nurse learn information about one

culture, but does not improve cultural awareness of other cultures.

DIF: Cognitive Level: Understand REF: 52

TOP: Nursing Process: Assessment

MSC: NCLEX Patient Needs: Psychosocial Integrity: Cultural Diversity

MULTIPLE RESPONSE

1. Which of the components described below represent the Filipino culture? Select all that apply.

a. Tagalog and Cebuano are the primary dialects spoken.

b. Orientation to the past is evident in their respect for elders.

c. The family, rather than the individual, is the unit.

d. Most Filipinos are Catholic.

e. Filipinos like to eat rice with most meals.

f. Sharing is common since interdependence is important.

ANSWER: B, C, F

Correct: Respect for elders is an example of values and beliefs, a part of culture. Value of

family represents beliefs and customs of culture. Interdependence is a value of Filipinos.

Incorrect: Language, religion, and food preferences are part of ethnicity, rather than culture.

DIF: Cognitive Level: Apply REF: 47

TOP: Nursing Process: Assessment

MSC: NCLEX Patient Needs: Psychosocial Integrity: Cultural Diversity

2. During the first prenatal visit for a 20-year-old Hispanic woman, the nurse assesses the

patient’s health beliefs and practices. Which questions are appropriate as part of this

assessment? Select all that apply.

a. “You are Hispanic, do you need me to find an interpreter?”

b. “What is the language that is usually spoken in your home?”

c. “How do you define health and illness?”

d. “Which Catholic church do you attend?”

e. “Do you have specific beliefs or preferences concerning food or food preparation?”

f. “Do you or the members of your family have certain beliefs and practices

surrounding pregnancy and childbirth?”

ANSWER: B, C, E, F

Correct: “What is the language that is usually spoken in your home?” “How do you define

health and illness?” “Do you have specific beliefs or preferences concerning food or food

preparation?” “Do you or the members of your family have certain beliefs and practices

surrounding pregnancy and childbirth?” These four questions support culturally sensitive

nursing care.

This study source was downloaded by 100000838401522 from CourseHero.com on 03-03-2022 08:49:55 GMT -06:00

https://www.coursehero.com/file/20909004/c5/


Incorrect: “You are Hispanic, do you need me to find an interpreter?” assumes that all

Hispanic persons speak Spanish only and is an example of stereotyping. “Which Catholic

church do you attend?” assumes that all Hispanics are Catholics and is an example of

stereotyping.

DIF: Cognitive Level: Apply REF: 51

TOP: Nursing Process: Assessment

MSC: NCLEX Patient Needs: Psychosocial Integrity: Cultural Diversity

This study source was downloaded by 100000838401522 from CourseHero.com on 03-03-2022 08:49:55 GMT -06:00

https://www.coursehero.com/file/20909004/c5/


Chapter 6: Pain Assessment

Test Bank

MULTIPLE CHOICE

1. How do nurses assess a patient’s pain?

a. By assessing physiologic changes of the patient

b. By understanding the sensory experience related to the amount of tissue damage

c. By the patient’s medical diagnosis or surgical procedure

d. By asking the patient to rate the pain being experienced

ANSWER: D

A

B

C

D

Feedback

The pain perceived is unrelated to the physiologic changes of the patient.

Although pain occurs when tissues are damaged, there is no correlation between

the amount of tissue damage and the degree and intensity of pain experienced.

There is no correlation between pain perceived and a medical diagnosis or

surgical procedure.

Pain is whatever the patient says it is. One person cannot judge the perception or

meaning of pain of another person.

DIF: Cognitive Level: Understand REF: 54| 59

TOP: Nursing Process: Assessment

MSC: NCLEX Patient Needs: Physiologic Integrity: Basic Care and Comfort: Assessing Pain

2. The nurse notes in the patient’s history that the patient has persistent, malignant pain. What is

the meaning of this type of pain?

a. The pain has been present for at least 2 weeks.

b. The pain began after recent surgery and is associated with healing incisions.

c. The pain has been present for 6 or more months.

d. The pain has been present since surgery to remove cancer.

ANSWER: C

A

B

C

D

Feedback

This time frame is too short. Chronic pain may be intermittent or continuous pain

lasting more than 6 months.

This is a description of acute pain rather than chronic.

This is the definition of persistent or chronic pain.

Surgery to remove malignant tissue does not necessarily equate to malignant pain.

DIF: Cognitive Level: Remember REF: 55

TOP: Nursing Process: Assessment

MSC: NCLEX Patient Needs: Physiologic Integrity: Basic Care and Comfort: Assessing Pain

3. A patient has had chronic back pain for several years. On assessment, the nurse notes that the

patient sits quietly in a chair, reads a book, talks with a companion, and does not appear to be

in pain. When questioned, the patient rates the pain as a 6 on a scale of 0 to 10. How does the

nurse interpret these data?

This study source was downloaded by 100000838401522 from CourseHero.com on 03-03-2022 08:52:20 GMT -06:00

https://www.coursehero.com/file/20909012/c6/


a. Many patients cannot be believed when they complain of severe pain lasting many

months.

b. Patients may not have the same objective responses to chronic pain because of

compensation over time.

c. The patient probably has already taken a very effective pain medication.

d. This patient is probably not having as much pain as reported initially, and more

assessment is required.

ANSWER: B

A

B

C

D

Feedback

Pain is whatever the patient says it is. Patients with chronic pain adapt to the pain

and have more subtle manifestations than patients with acute pain.

Clinical manifestations of chronic pain are not those of physiologic stress because

the patient adapts to the pain.

Patients with chronic pain adapt to the pain and have more subtle manifestations

than patients with acute pain despite the effects of pain medication.

Pain is whatever the patient says it is. Patients with chronic pain adapt to the pain

and have more subtle manifestations than patients with acute pain.

DIF: Cognitive Level: Apply REF: 55

TOP: Nursing Process: Assessment

MSC: NCLEX Patient Needs: Physiologic Integrity: Basic Care and Comfort: Assessing Pain

4. Which patient would be expected to experience acute pain?

a. A patient who had abdominal surgery 8 hours ago

b. A patient who has cancer and has been receiving treatment for 4 months

c. A patient who states that he or she has lived with severe pain for many years

d. A patient who has been treated unsuccessfully over the past year for back pain

ANSWER: A

A

B

C

D

Feedback

Acute pain has a recent onset and results from tissue damage; is usually selflimiting;

and ends when the tissue heals.

Acute pain has a recent onset and results from tissue damage; is usually selflimiting;

and ends when the tissue heals.

This patient has experienced chronic pain for years. Acute pain has a recent onset

and results from tissue damage; is usually self-limiting; and ends when the tissue

heals.

This patient has experienced chronic pain for one year. Acute pain has a recent

onset and results from tissue damage; is usually self-limiting; and ends when the

tissue heals.

DIF: Cognitive Level: Apply REF: 55

TOP: Nursing Process: Assessment

MSC: NCLEX Patient Needs: Physiologic Integrity: Basic Care and Comfort: Assessing Pain

5. Which patient has pain caused by abnormal processing of sensory input from the peripheral

nervous system?

a. The patient who has aching pain from muscle strain

This study source was downloaded by 100000838401522 from CourseHero.com on 03-03-2022 08:52:20 GMT -06:00

https://www.coursehero.com/file/20909012/c6/


b. The patient who has burning pain along the sciatic nerve

c. The patient who has cramping pain from a tumor in the colon

d. The patient who has throbbing pain from arthritis

ANSWER: B

A

B

C

D

Feedback

The patient who has aching pain from muscle strain has nociceptor, somatic pain.

The patient who has burning pain along the sciatic nerve has neuropathic pain.

The patient who has cramping pain from a tumor in the colon has nociceptor,

visceral pain.

The patient who has throbbing pain from arthritis has nociceptor, somatic pain.

DIF: Cognitive Level: Analyze REF: 55-56

TOP: Nursing Process: Assessment

MSC: NCLEX Patient Needs: Physiologic Integrity: Physiologic Adaptation: Alteration in Body

Systems

6. A patient reports “right shoulder pain that comes and goes” as the chief complaint. During the

physical examination, the patient asks why the upper right abdomen is being examined for

shoulder pain. What is the appropriate response from the nurse?

a. “A comprehensive examination is required to determine the cause of your pain.”

b. “There may be associated problems that have not produced any symptoms yet that

we want to identify.”

c. “Yes, this can be confusing, but if you will be patient I’m sure we can find

something to help you.”

d. “It does seem odd, but the gallbladder doesn’t have pain receptors of its own, so the

pain shows up in the shoulder.”

ANSWER: D

A

B

C

D

Feedback

A focused examination is indicated at this time, not a comprehensive

examination.

This patient’s pain is due to referred pain, not to associated problems that have

not produced any symptoms of pain.

This response reflects concern for the patient’s pain, but does not address the

patient’s questions about examining the abdomen.

Referred pain is pain felt at a site different from that of an injured or diseased

organ. It commonly occurs during visceral pain because many organs have no

pain receptors; thus, when afferent nerves enter the spinal cord, they stimulate

sensory nerves from unaffected organs in the same spinal cord segment as those

neurons in areas where injury or disease is located.

DIF: Cognitive Level: Apply REF: 55

TOP: Nursing Process: Assessment

MSC: NCLEX Patient Needs: Physiologic Integrity: Physiologic Adaptation: Alteration in Body

Systems

This study source was downloaded by 100000838401522 from CourseHero.com on 03-03-2022 08:52:20 GMT -06:00

https://www.coursehero.com/file/20909012/c6/


7. A patient who had an amputation of his lower leg comes to the clinic with a complaint of pain.

He asks, “How I can be feeling pain in my foot—my foot is gone!” What is the appropriate

response from the nurse?

a. “After your amputation, pain perception increases.”

b. “Amputating your leg caused abnormal processing of sensory input by the

peripheral nervous system.”

c. “Stimulation of nerves from your leg sends impulses to the brain so that you feel

pain even though your leg is no longer there.”

d. “When sensory nerves enter the spinal cord, they stimulate nerves from unaffected

organs in the same spinal cord segment as those neurons in areas where injury or

disease is located.”

ANSWER: C

A

B

C

D

Feedback

“After your amputation, pain perception increases” is a definition of pain

threshold.

“Amputating your leg caused abnormal processing of sensory input by the

peripheral nervous system” is a definition of neuropathic pain.

“Stimulation of nerves from your leg sends impulses to the brain so that you feel

pain even though your leg is no longer there” is a definition of phantom pain.

“When sensory nerves enter the spinal cord, they stimulate nerves from

unaffected organs in the same spinal cord segment as those neurons in areas

where injury or disease is located” is a definition of referred pain.

DIF: Cognitive Level: Analyze REF: 55

TOP: Nursing Process: Assessment

MSC: NCLEX Patient Needs: Physiologic Integrity: Physiologic Adaptation: Alteration in Body

Systems

8. A patient who had extensive surgery asks the nurse for pain medication for a pain of 9 on a

scale of 0 to 10. The nurse completes an assessment of this patient’s pain and agrees to give

pain medication. When the nurse returns to the patient with the ordered intravenous pain

medication, she notices the patient’s eyes are closed and he appears to be sleeping. What is the

nurse’s appropriate action at this time?

a. Lock up the medication in a safe location until the patient awakens.

b. Arouse the patient to confirm he still wants the medication.

c. Give the medication as ordered and agreed to.

d. Consult a colleague about what action to take.

ANSWER: C

A

B

C

D

Feedback

The patient needs to receive the ordered pain medication now.

There is no reason to confirm the need for pain medication requested earlier.

Sleep is not synonymous with pain relief. When the patient reports a pain of 9 and

asks for medication for which there is an order, he needs to receive the

medication.

There is no reason to ask a colleague about giving the pain medication requested

earlier.

This study source was downloaded by 100000838401522 from CourseHero.com on 03-03-2022 08:52:20 GMT -06:00

https://www.coursehero.com/file/20909012/c6/


DIF: Cognitive Level: Apply REF: 54| 56

TOP: Nursing Process: Assessment

MSC: NCLEX Patient Needs: Physiologic Integrity: Basic Care and Comfort: Assessing Pain

9. In the labor and delivery department, the nurse notices that two women who are in labor are

responding differently to their contractions. The first woman, who is having her first baby, has

rated her pain as a “7,” seems agitated, and has asked for pain medication. The second

woman, who is having her third baby, has also rated her pain as a “7,” but is calmer and says

she does not need anything for pain at this time. What explains the differences in the outward

responses to pain between these women?

a. Pain tolerance

b. Pain threshold

c. Nociception

d. Physiologic stress

ANSWER: A

A

B

C

D

Feedback

Pain tolerance is the duration or intensity of pain a person will endure before

outwardly responding. A person’s culture, pain experience, expectations, role

behaviors, and physical and emotional health influence pain tolerance. The

second woman had experienced the birth process before and had different

expectations than the first woman, who was having her first baby.

Pain threshold is the point at which a stimulus is perceived as pain. This threshold

does not vary significantly among people or in the same person over time.

Nociception is the process of pain perception and involves transduction,

transmission, perception, and modulation.

Physiologic stress stimulates the sympathetic nervous system causing

tachycardia, increased respiratory rate, and dilated pupils, but does not

necessarily affect pain response.

DIF: Cognitive Level: Analyze REF: 58

TOP: Nursing Process: Assessment

MSC: NCLEX Patient Needs: Physiologic Integrity: Physiologic Adaptation: Alteration in Body

Systems

10. A patient admitted to the emergency department with “excruciating chest pain, above the

rating of 10,” has a heart rate of 55, rapid, irregular respirations, complains of nausea, and is

too weak to move to the stretcher without aid. The nurse recognizes that this response to

severe pain is due to the response of the nervous system.

a. Parasympathetic

b. Sympathetic

c. Central

d. Peripheral

ANSWER: A

A

Feedback

During severe or deep pain the parasympathetic nervous system may cause pallor;

rapid, irregular breathing; nausea; and vomiting.

This study source was downloaded by 100000838401522 from CourseHero.com on 03-03-2022 08:52:20 GMT -06:00

https://www.coursehero.com/file/20909012/c6/


B

C

D

The sympathetic nervous system responds to acute pain by increasing heart rate,

increasing blood pressure, causing diaphoresis, increasing respiratory rate,

increasing muscle tension, dilating pupils, and decreasing gastrointestinal

motility.

The central nervous system includes the brain and spinal cord. The manifestations

described in the case are due to parasympathetic nervous system stimulation.

The manifestations described in the case are due to parasympathetic nervous

system stimulation.

DIF: Cognitive Level: Remember REF: 59| 63

TOP: Nursing Process: Assessment

MSC: NCLEX Patient Needs: Physiologic Integrity: Basic Care and Comfort: Assessing Pain

11. A patient with gout is complaining of severe, throbbing pain in the great toe. What type of

pain is this patient experiencing?

a. Neuropathic pain

b. Somatic pain

c. Referred pain

d. Visceral pain

ANSWER: B

A

B

C

D

Feedback

Neuropathic pain is caused by abnormal processing of sensory input from the

peripheral nervous system.

Somatic pain arises from bone, joint, muscle, skin, or connective tissues and is

usually aching or throbbing in quality and well located.

Referred pain is pain felt at a site different from that of an injured or diseased

organ.

Visceral pain occurs with obstruction of a hollow organ and causes intermittent

cramping pain.

DIF: Cognitive Level: Remember REF: 59

TOP: Nursing Process: Assessment

MSC: NCLEX Patient Needs: Physiologic Integrity: Physiologic Adaptation: Alteration in Body

Systems

12. A patient with a partial small bowel obstruction describes the pain as “cramping, off-and-on

pain that spreads over my stomach.” What type of pain is this patient experiencing?

a. Referred pain

b. Phantom pain

c. Somatic pain

d. Visceral pain

ANSWER: D

A

B

C

Feedback

Referred pain is felt at a site different from that of an injured or diseased organ.

Phantom pain is associated with amputations.

Somatic pain arises from bone, joint, muscle, skin, or connective tissues and is

usually aching or throbbing in quality and well located.

This study source was downloaded by 100000838401522 from CourseHero.com on 03-03-2022 08:52:20 GMT -06:00

https://www.coursehero.com/file/20909012/c6/


D

Visceral pain occurs with obstruction of a hollow organ and causes intermittent

cramping pain.

DIF: Cognitive Level: Remember REF: 59

TOP: Nursing Process: Assessment

MSC: NCLEX Patient Needs: Physiologic Integrity: Physiologic Adaptation: Alteration in Body

Systems

MULTIPLE RESPONSE

1. A nurse is assessing a patient who complains of “awful” abdominal pain and rates it as a 9 on

a scale of 0 to 10. Which of the following physiologic signs may accompany acute pain?

Select all that apply.

a. Tachycardia

b. Irritability

c. Increased blood pressure

d. Depression

e. Insomnia

f. Sweating

ANSWER: A, C, F

Correct: The sympathetic nervous system responds to acute pain by increasing heart rate,

increasing blood pressure, causing diaphoresis, increasing respiratory rate, increasing muscle

tension, dilating pupils, and decreasing gastrointestinal motility.

Incorrect: Irritability, depression, and insomnia are manifestations of chronic rather than

acute pain.

DIF: Cognitive Level: Apply REF: 59| 63

TOP: Nursing Process: Assessment

MSC: NCLEX Patient Needs: Physiologic Integrity: Physiologic Adaptation: Alteration in Body

Systems

2. The nurse is performing a symptom analysis of a patient with pain. Which questions below are

appropriate for a symptom analysis? Select all that apply.

a. “Have you had any other symptoms such as nausea, vomiting, and sweating?”

b. “Where is the pain located?”

c. “Have you had a pain like this before?”

d. “What does the pain feel like?”

e. “What do you do to make your pain better?”

f. “In your culture, how are you encouraged to express your pain?”

ANSWER: A, B, D, E

Correct: “Have you had any other symptoms such as nausea, vomiting, and sweating?”

“Where is the pain located?” “What does the pain feel like?” “What do you do to make your

pain better?” These four questions are asked in a symptom analysis that includes the following

variables: onset of symptoms, location and duration of symptoms, characteristics, aggravating

factors, related symptoms, self-treatment, and severity.

Incorrect: “Have you had a pain like this before?” This question relates to the patient’s health

history. “In your culture, how are you encouraged to express your pain?” This question relates

to the patient’s culture and does not help analyze the patient’s pain experience.

This study source was downloaded by 100000838401522 from CourseHero.com on 03-03-2022 08:52:20 GMT -06:00

https://www.coursehero.com/file/20909012/c6/


DIF: Cognitive Level: Apply REF: 59-60

TOP: Nursing Process: Assessment

MSC: NCLEX Patient Needs: Physiologic Integrity: Physiologic Adaptation: Alteration in Body

Systems

3. How do nurses assess pain of neonates or of adults with dementia or decreased level of

consciousness? Select all that apply.

a. Ask family or caregivers what indicators they think may indicate the patient’s pain.

b. Review results of blood tests for signs of pain.

c. Administer the ordered analgesic to the patient.

d. Identify any physiologic signs of pain.

e. Examine the patient for possible causes of pain.

ANSWER: A, C, D, E

Correct: These four answers are the clinical practice recommendations of Herr and

colleagues.

Incorrect: Pain cannot be detected with laboratory tests.

DIF: Cognitive Level: Understand REF: 62

TOP: Nursing Process: Assessment

MSC: NCLEX Patient Needs: Physiologic Integrity: Basic Care and Comfort: Assessing Pain

This study source was downloaded by 100000838401522 from CourseHero.com on 03-03-2022 08:52:20 GMT -06:00

https://www.coursehero.com/file/20909012/c6/


Chapter 7: Mental Health and Abusive Behavior Assessment

Test Bank

MULTIPLE CHOICE

1. What function do neurotransmitters have in mental health disorders?

a. Dopamine levels are increased in schizophrenia.

b. Increased levels of gamma aminobutyric acid (GABA) contribute to anxiety.

c. Serotonin is decreased in a state of anxiety.

d. Norepinephrine is increased in depression.

ANSWER: A

A

B

C

D

Feedback

Dopamine levels are increased in schizophrenia.

Insufficient GABA may contribute to anxiety. GABA is an inhibitory

neurotransmitter.

Serotonin is increased in anxiety states.

Norepinephrine is decreased in depression.

DIF: Cognitive Level: Remember REF: 66

TOP: Nursing Process: Assessment

MSC: NCLEX Patient Needs: Psychosocial Integrity: Mental Health Concepts

2. A male patient scores 125 on the Holmes Social Readjustment Scale. How does the nurse

interpret this score?

a. He is experiencing a great deal of stress in his life and needs hospitalization.

b. At this time he has no stress in his life and is healthy both mentally and physically.

c. He has relatively low stress in his life and use of daily relaxation can be beneficial.

d. He has a moderate chance of developing a stress-related illness and can reduce this

by practicing stress management.

ANSWER: C

A

B

C

D

Feedback

A score on the Holmes Social Readjustment Scale greater than 300 is needed for

hospitalization.

This does not apply to this person. The lowest score possible on the Holmes

Social Readjustment Scale (less than 150) indicates the amount of stress

experienced is a result of normal changes in life and is manageable.

A score on the Holmes Social Readjustment Scale of below 150 indicates the

amount of stress experienced is a result of normal changes in life and is

manageable.

A moderate chance of developing a stress-related illness and reduction through

stress management applies to a patient who scores between 150 and 300.

DIF: Cognitive Level: Apply REF: 69

TOP: Nursing Process: Assessment

MSC: NCLEX Patient Needs: Psychosocial Integrity: Mental Health Concepts


3. A 24-year-old male patient tells the nurse he has had no energy for 2 weeks. He has no trouble

falling asleep; in fact, he sleeps deeply about 12 hours every night. He states that he has

gained 10 lb in the past 2 months and has no friends. The nurse associates these

manifestations with which mental health disorder?

a. Depression

b. Schizophrenia

c. Bipolar disorder

d. Anxiety disorder

ANSWER: A

A

B

C

D

Feedback

These are symptoms of depression.

Clinical manifestations of schizophrenia include apathy and confusion, delusions

and hallucinations, and rambling or stylized patterns of speech.

Characteristics of the manic phase are excessive emotional displays, excitement,

euphoria, and hyperactivity. In contrast, characteristics of the depressive phase are

marked apathy and feelings of profound sadness, loneliness, guilt, and lowered

self-esteem.

Anxiety is a feeling of uneasiness or discomfort experienced in varying degrees,

from mild anxiety to panic. The energy that anxiety provides may mobilize a

person to take constructive action such as solving a major problem or filling an

unmet need.

DIF: Cognitive Level: Apply REF: 68-69

TOP: Nursing Process: Assessment

MSC: NCLEX Patient Needs: Psychosocial Integrity: Mental Health Concepts

4. A female patient states that she has had problems with depression in the past and thinks she is

depressed again. Which response by the nurse is most appropriate?

a. “What do you think is causing your depression this time?”

b. “What therapies have worked for you in the past?”

c. “Did you stop taking your medication?’”

d. “Do you think this is a situational depression?”

ANSWER: B

A

B

C

D

Feedback

This question provides information but does not direct the patient toward

identifying a treatment.

This question is a therapeutic response to determine if the same or similar therapy

can be used again for this depression. It is an open-ended question to collect more

data. Also treatment is information collected in a symptom analysis that is useful

in this situation.

This question sounds accusatory, and the nurse is guessing the cause of this

episode of depression without collecting data from the patient. This is a closedended

question asking for a “yes” or “no” response.

This is a closed-ended question and does not collect data to determine if the

patient has depression again.


DIF: Cognitive Level: Apply REF: 70

TOP: Nursing Process: Assessment

MSC: NCLEX Patient Needs: Psychosocial Integrity: Mental Health Concepts

5. Which patient may be experiencing severe anxiety?

a. A woman who tells the nurse she is terrified of cats

b. A man who tells the nurse he feels worthless and is always tired

c. A woman who reports that she is sleeping very lightly each night because her child

has an ear infection

d. A man who phones the nurse five times asking for instructions about how to take

his new medication

ANSWER: D

A

B

C

D

Feedback

Being terrified of cats describes fear or a phobia rather than anxiety. Unlike fear,

which is a response to an actual object or event, anxiety is a response to no

specific source or actual object.

Although fatigue is a characteristic of anxiety, in this case, the patient also

verbalizes feelings of worthlessness, which suggests depression rather than

anxiety.

Although problems falling and staying asleep are characteristics of anxiety, in this

example the patient can identify the cause of the sleeplessness—her ill child—

thus anxiety is not the cause of the sleep disturbance.

A man who phones the nurse five times asking for instructions about how to take

his new medication shows characteristics of anxiety, which includes forgetfulness

and difficulty concentrating or making decisions.

DIF: Cognitive Level: Apply REF: 70

TOP: Nursing Process: Assessment

MSC: NCLEX Patient Needs: Psychosocial Integrity: Mental Health Concepts

6. While assessing a man during a physical examination for work, the nurse suspects alcohol

use. Which assessment tool is appropriate in this situation?

a. AUDIT screening tool

b. Rapid eye test

c. Mental status examination

d. Holmes Social Readjustment Rating Scale

ANSWER: A

A

B

C

D

Feedback

AUDIT, an abbreviation for Alcohol Use Disorders Identification Test, is the

correct assessment tool in this situation.

The rapid eye test is used when there is suspicion of drug intoxication.

Mental status examination is not indicated in this case because there are no data

to suggest an alteration in mental status.

The Holmes Social Readjustment Rating Scale is used to assess stress and predict

the occurrence of a serious illness over the next 2 years based on stress alone.

DIF: Cognitive Level: Understand REF: 71-72


TOP: Nursing Process: Assessment

MSC: NCLEX Patient Needs: Psychosocial Integrity: Mental Health Concepts

7. A nurse screens every adult and adolescent patient for alcohol consumption. Which patient

drinks more than recommended?

a. The man who reports drinking 3 beers and one shot of whiskey each day

b. The woman who reports drinking 2 glasses of wine and 2 vodka martinis each day

c. The older adult man who reports drinking one glass of sherry before going to bed

each night

d. The woman who reports drinking one glass of wine with lunch and dinner each day.

ANSWER: B

A

B

C

D

Feedback

The man who reports drinking 3 beers and one shot of whiskey each day. This

amount of alcohol is within the National Institute on Alcohol Abuse and

Alcoholism recommendations for men to drink fewer than 5 standard drinks daily.

The woman who reports drinking 2 glasses of wine and 2 vodka martinis each

day. The National Institute on Alcohol Abuse and Alcoholism recommends

women drink fewer than 4 standard drinks daily.

The older adult man who reports drinking one glass of sherry before going to bed

each night. This amount of alcohol is within the National Institute on Alcohol

Abuse and Alcoholism recommendations for men to drink fewer than 5 standard

drinks daily.

The woman who reports drinking one glass of wine with lunch and dinner each

day. This amount of alcohol is within the National Institute on Alcohol Abuse and

Alcoholism recommendations for women to drink fewer than 4 standard drinks

daily.

DIF: Cognitive Level: Apply REF: 71

TOP: Nursing Process: Assessment

MSC: NCLEX Patient Needs: Psychosocial Integrity: Mental Health Concepts

8. During a sports physical for a 16-year-old girl, the nurse asks which question to collect data

about drug use?

a. “Many teenagers have tried street drugs. Have you tried these drugs? ”

b. “Tell me which street drugs your friends have offered to you?”

c. “Do most of your friends drink alcohol or do street drugs?”

d. “Your high school has a reputation for drug use. Do you use drugs?”

ANSWER: A

A

B

C

D

Feedback

This uses a questioning technique called “permission giving” in which the nurse

“gives permission” to the patient to discuss drug use. Questions like this help

identify a pattern of drug use and screen for drug abuse.

This is not an appropriate question because the nurse does not need this

information to assess the patient.

This is not an appropriate question because the nurse does not need this

information to assess the patient.

This is not an appropriate question because the nurse is associating the school’s


reputation with the patient’s behaviors.

DIF: Cognitive Level: Apply REF: 72

TOP: Nursing Process: Assessment

MSC: NCLEX Patient Needs: Psychosocial Integrity: Mental Health Concepts

9. In contrasting the assessment of mental status from mental health, a nurse recognizes that data

for the mental status examination are obtained using which techniques?

a. Asking them about their relatives who have mental health disorders

b. Having them demonstrate their ability to reason and calculate

c. Asking them to recall how they have coped with daily stress

d. Having them describe their mood and emotions

ANSWER: B

A

B

C

D

Feedback

This obtains information from patients for the histories, but does not ask patients

to demonstrate mental abilities.

The mental status examination asks patients to perform calculations and other

tasks to show their abilities, rather than asking them about their abilities.

This obtains information from patients for the histories, but does not ask patients

to demonstrate mental abilities.

Having them describe their mood and emotions does not ask patients to

demonstrate mental abilities.

DIF: Cognitive Level: Analyze REF: 70

TOP: Nursing Process: Assessment

MSC: NCLEX Patient Needs: Psychosocial Integrity: Mental Health Concepts

10. A nurse is admitting a new patient. Which statement by the patient suggests a bipolar

disorder?

a. “The last time I had blood drawn at the office, I fainted dead away.”

b. “No matter how hard I try, I just can’t get into an elevator of any kind.”

c. “Everyone knows I can control the financial health of this town with a snap of my

fingers.”

d. “I worked for Frank Sinatra’s band for several months when I lived in New Jersey

years ago.”

ANSWER: C

A

B

C

D

Feedback

This statement does not indicate bipolar disorder and may be a true statement.

This statement is an example of a phobia.

Patients in the manic phase of bipolar disorder have delusions of grandeur, which

is described in the statement.

This may be a true statement depending on the age of the patient.

DIF: Cognitive Level: Apply REF: 76

TOP: Nursing Process: Assessment

MSC: NCLEX Patient Needs: Psychosocial Integrity: Mental Health Concepts


11. During conversation, the nurse observes that the patient is talking continuously and excitedly,

and is switching rapidly from one topic to another with seemingly no relationship between

topics. This behavior is often associated with which disorder?

a. Depression

b. Obsessive-compulsive disorder

c. Schizophrenia

d. Bipolar disorder

ANSWER: C

A

B

C

D

Feedback

This behavior is flight of ideas, which occurs in patients with schizophrenia rather

than depression.

This behavior is flight of ideas, which occurs in patients with schizophrenia rather

than obsessive-compulsive disorder.

This behavior is flight of ideas, which occurs in patients with schizophrenia.

This behavior is flight of ideas, which occurs in patients with schizophrenia rather

than bipolar disorder.

DIF: Cognitive Level: Understand REF: 76

TOP: Nursing Process: Assessment

MSC: NCLEX Patient Needs: Psychosocial Integrity: Mental Health Concepts

12. During a visit to the clinic for an annual gynecologic examination, a patient tells the nurse that

she had a bad experience on an airplane, saying, “When I sat down, my heart started racing, I

was short of breath and sweaty, and I felt as if I was going to die.” She stated that her husband

helped her to calm down after a few minutes. The nurse recognizes that the patient was

describing which problem?

a. Bipolar disorder, manic phase

b. Moderate anxiety

c. Panic

d. Delusions

ANSWER: C

A

B

C

D

Feedback

Characteristics of the manic phase are excessive emotional displays, excitement,

euphoria, hyperactivity accompanied by elation, boisterousness, impaired ability

to concentrate, decreased need for sleep, and limitless energy, often accompanied

by delusions of grandeur.

The moderately anxious person has a narrower field of perception and uses

selective inattention to ignore stimuli in the environment to focus on a specific

concern.

Physical manifestations of a panic attack represent sympathetic nervous system

stimulation. The person experiences muscle tension, tachycardia, dyspnea,

hypertension, increased respiration, and profuse perspiration.

Delusions are persistent abnormal beliefs or perceptions held by a person despite

the evidence that refutes it.

DIF: Cognitive Level: Apply REF: 77

TOP: Nursing Process: Assessment


MSC: NCLEX Patient Needs: Psychosocial Integrity: Mental Health Concepts

13. A patient in the waiting room appears anxious and moves around the room cleaning surfaces

with a disinfectant cloth. This behavior is consistent with which disorder?

a. Bipolar disorder

b. Delirium

c. Schizophrenia

d. Obsessive-compulsive disorder

ANSWER: D

A

B

C

D

Feedback

Bipolar disorder is a type of depression characterized by episodes of mania,

depression, or mixed moods.

Delirium has manifestations that include attention deficits, disorganized thinking,

confusion, disorientation, restlessness, incoherence, anxiety, and excitement.

Schizophrenia is characterized by gross distortion of reality, disturbances of

language and communication, withdrawal from social interaction, and the

disorganization and fragmentation of thought perception and emotional reaction.

This patient was concerned about contamination. Compulsions are unwanted,

repetitive behavior patterns or mental acts that are intended to reduce anxiety. The

person recognizes that the behaviors are excessive or unreasonable but continues

them because of the relief from the discomfort of anxiety that they provide.

DIF: Cognitive Level: Understand REF: 77

TOP: Nursing Process: Assessment

MSC: NCLEX Patient Needs: Psychosocial Integrity: Mental Health Concepts

14. An elderly patient was admitted with pneumonia and a fever of 104.5° F. At the time of

admission he was confused, disoriented, restless, and tried to slap the nurse who started an

intravenous line. His daughter stated, “Just yesterday he was perfectly fine, except for a cold. I

can’t believe he is acting this way now.” Within a few days, his erratic behavior subsided and

his daughter was relieved that he was “back to normal.” The nurse recognizes that this patient

was exhibiting signs of which disorder?

a. Dementia

b. Delirium

c. Panic attack

d. Alcohol withdrawal

ANSWER: B

A

B

C

Feedback

Dementia is a syndrome of acquired, progressive, intellectual impairment that

compromises function, such as memory, language, visual-spatial skills, emotion,

personality, and cognition.

Delirium is characterized by a disturbance of consciousness and a change in

cognition that develops rapidly over a short period of time.

A panic attack is characterized by complete disruption of the perceptual field. The

person experiences intense terror and is unable to think logically or make

decisions. The person experiences muscle tension, tachycardia, dyspnea,

hypertension, increased respiration, and profuse perspiration.


D

Early manifestations of alcohol withdrawal include hand tremors, sweating,

nausea and vomiting, anxiety, and agitation.

DIF: Cognitive Level: Understand REF: 77

TOP: Nursing Process: Assessment

MSC: NCLEX Patient Needs: Psychosocial Integrity: Mental Health Concepts

15. During a report, a nurse hears about a patient who was admitted at 8 PM after an automobile

accident. He had a blood alcohol level of 100 mg/dl at the time of admission. During the 8 AM

assessment, the nurse notes that the patient is having hand tremors, is sweaty, is slightly

agitated, and complains of nausea. The nurse recognizes that the patient may be exhibiting

signs of which disorder?

a. Alcohol withdrawal syndrome

b. Delirium tremens

c. Panic

d. Delirium

ANSWER: A

A

B

C

D

Feedback

The history and laboratory data reflect alcohol withdrawal syndrome. Early

manifestations of alcohol withdrawal include hand tremors, sweating, nausea and

vomiting, anxiety, and agitation. These manifestations begin 6 to 24 hours after

the patient’s last drink, peak in 24 to 36 hours, and end after 48 hours of

abstinence.

The time sequence is not consistent with delirium tremens. During delirium

tremens, a patient experiences cardiac dysrhythmias, hypertension, increased

respirations, profuse sweating, delusion, and hallucinations.

The history is not consistent with panic disorder. In the panic level of anxiety, a

person experiences manifestations of anxiety that represent sympathetic nervous

system stimulation, as well as muscle tension, tachycardia, dyspnea,

hypertension, increased respiration, and profuse perspiration.

Manifestations of delirium include attention deficits, disorganized thinking,

confusion, disorientation, restlessness, incoherence, anxiety, excitement, and, at

times, illusions.

DIF: Cognitive Level: Understand REF: 77

TOP: Nursing Process: Assessment

MSC: NCLEX Patient Needs: Psychosocial Integrity: Mental Health Concepts

MULTIPLE RESPONSE

1. Which neurotransmitters are decreased in patients with depression? Select all that apply.

a. Acetylcholine (Ach)

b. Histamine

c. Norepinephrine (NE)

d. Dopamine (DA)

e. Gamma aminobutyric acid (GABA)\

f. Serotonin (5 HT)


ANSWER: B, C, D, F

Correct: Histamine, norepinephrine (NE), dopamine (DA), and serotonin (5 HT) are

neurotransmitters that are decreased in depression. Drugs prescribed for people with

depression may provide therapy by increasing these neurotransmitters.

Incorrect: Acetylcholine (Ach) is increased in depression. Gamma aminobutyric acid

(GABA) is decreased in schizophrenia and anxiety states.

DIF: Cognitive Level: Understand REF: 66

TOP: Nursing Process: Assessment

MSC: NCLEX Patient Needs: Psychosocial Integrity: Mental Health Concepts

2. During a mental health history, the nurse suspects altered mental status for a patient. Which

questions are appropriate to ask when assessing mental status? Select all that apply.

a. “Do you have difficulty making decisions?”

b. “Do you know where you are?”

c. “Are there times when you wanted to escape?”

d. “If you bought a hat for $5.75 and gave the sales person $10.00, how much change

do you expect back?”

e. “What would you do if a fire started in your home?”

f. “What does this phrase ‘A rolling stone gathers no moss’ mean?”

ANSWER: B, D, E, F

Correct: “Do you know where you are?” assesses orientation. “If you bought a hat for $5.75

and gave the sales person $10.00, how much change do you expect back?” assesses

calculation ability. “What would you do if a fire started in your home?” assesses judgment.

“What does this phrase ‘A rolling stone gathers no moss’ mean?” assesses abstract reasoning.

Incorrect: For the nurse to assess mental status, the patient needs to demonstrate abilities

such as calculation, judgment, and abstract reasoning.

DIF: Cognitive Level: Apply REF: 70-71

TOP: Nursing Process: Assessment

MSC: NCLEX Patient Needs: Psychosocial Integrity: Mental Health Concepts

3. While conducting a health history, the nurse asks which questions to assess for risk factors

associated with depression? Select all that apply.

a. “Has anyone in your family ever been diagnosed with depression?”

b. “Have you noticed a change in how much energy you have?”

c. “Do you have crying spells?”

d. “Do your muscles seem tense?”

e. “Do you feel that something bad is about to happen to you?”

f. “Do you have difficulty making decisions?”

ANSWER: A, B, C, F

Correct: These questions are related to risk factors for depression.

Incorrect: Tense muscles are associated with stress and anxiety rather than depression.

Feeling that something bad is about to happen relates to paranoia rather than depression.

DIF: Cognitive Level: Apply REF: 68-70| 76

TOP: Nursing Process: Assessment

MSC: NCLEX Patient Needs: Psychosocial Integrity: Mental Health Concepts


COMPLETION

1. Researchers have found that it is the of a recent life event that determines a person’s

emotional or psychological reaction to it.

ANSWER:

Perceptions

Each culture influences how a stressful event is perceived and the acceptable ways that people

of that culture are expected to respond.

DIF: Cognitive Level: Remember REF: 67

TOP: Nursing Process: Assessment

MSC: NCLEX Patient Needs: Psychosocial Integrity: Mental Health Concepts


Chapter 8: Nutritional Assessment

Test Bank

MULTIPLE CHOICE

1. A patient with mild renal disease has been put on a 2200-calorie per day diet plan with the

lowest recommended amount of protein. During discharge teaching, the nurse explains to this

patient how to use nutrition labels to determine the amount of protein in the product. The

nurse explains, however, that the label is based on 2000 calories. Which is the appropriate

formula to teach this patient the least amount of protein he can eat on his prescribed diet?

a. 2200 calories 0.15 = 330/9 calories/gram = 36.6 g

b. 2200 calories 0.10 = 220/4 calories/gram = 55 g

c. 2200 calories 0.20 = 440/9 calories/gram = 48.8 g

d. 2200 calories 0.12 = 264/4 calories/gram = 66 g

ANSWER: D

Feedback

A Each gram of protein yields 4 calories rather than 9.

B Twelve percent is the least recommended percentage for proteins, rather than

10%.

C Twelve percent is the least recommended percentage for proteins. Each gram of

protein yields 4 calories rather than 9.

D Proteins should account for 12% to 20% of total calories. Each gram of protein

yields 4 calories.

DIF: Cognitive Level: Apply REF: 81

TOP: Nursing Process: Implementation

MSC: NCLEX Patient Needs: Physiologic Integrity: Basic Care and Comfort: Nutrition and

Hydration

2. A patient is put on an 1800-calorie a day diet plan. During discharge teaching, the nurse

explains to this patient how to use nutrition labels to determine the amount of carbohydrates in

the product. The nurse explains, however, that the label is based on 2000 calories. Which is

the appropriate formula to teach this patient of the maximum grams of carbohydrates she can

eat on her prescribed diet?

a. 1800 calories 0.45 = 810/4 calories/gram = 202.5 g

b. 1800 calories 0.60 = 1080/4 calories/gram = 270 g

c. 1800 calories 0.55 = 990/9 calories/gram = 110 g

d. 1800 calories 0.50 = 900/9 calories/gram = 100 g

ANSWER: B

Feedback

A Carbohydrates should account for a maximum of 60% of total calories rather than

45%.

B Carbohydrates should account for 55% to 60% of total calories. Each gram of

carbohydrates yields 4 calories.

C Each gram of carbohydrates yields 4 calories rather than 9.


D

Carbohydrates should account for a maximum of 60% of total calories, rather

than 50%. Each gram of carbohydrates yields 4 calories rather than 9.

DIF: Cognitive Level: Apply REF: 81

TOP: Nursing Process: Implementation

MSC: NCLEX Patient Needs: Physiologic Integrity: Basic Care and Comfort: Nutrition and

Hydration

3. A patient tells the nurse that she tries to keep her fat intake at less than 15% of her total caloric

intake per day. What is the nurse’s most appropriate response to this patient’s comment?

a. “That is admirable; how do you accomplish fat intake that low on a daily basis?”

b. “Eating fat is essential for good health, and you should consume about 40% of your

fats as monounsaturated fat.”

c. “Limiting fat prevents some diseases, but your fat intake is much lower than the

25% recommended.”

d. “If you want to bring your fat intake down further, you might want to eliminate

eating fast foods.”

ANSWER: C

A

B

C

D

Feedback

The patient’s fat intake is too low. The recommended amount of fat is 25% to

30% of total calories.

The recommended amount of fat is 25% to 30% of total calories.

The recommended amount of fat is 25% to 30% of total calories.

The patient’s fat intake is already too low. The recommended amount of fat is

25% to 30% of total calories.

DIF: Cognitive Level: Apply REF: 81

TOP: Nursing Process: Implementation

MSC: NCLEX Patient Needs: Health Promotion and Maintenance: Lifestyle Choices

4. A patient who keeps his fat consumption at 10% of his total caloric intake is at risk for

deficiency of which nutrient(s)?

a. Iron

b. Vitamins A, D, and K

c. Zinc

d. B and C vitamins

ANSWER: B

A

B

C

D

Feedback

Iron absorption is not affected by low fat intake.

Vitamins A, D, and K are fat soluble vitamins. If the patient does not have enough

fat intake, adequate amounts of these vitamins cannot be absorbed. Fat soluble

vitamins should be linked to a lack of body fat.

Zinc absorption is not affected by low fat intake.

Vitamins B and C are water soluble vitamins and their absorption is not affected

by low fat intake.

DIF: Cognitive Level: Apply REF: 81


TOP: Nursing Process: Assessment

MSC: NCLEX Patient Needs: Physiologic Integrity: Basic Care and Comfort: Nutrition and

Hydration

5. A nurse is asking questions about the present health status of a young woman who has lost

weight recently. Which question is most appropriate when inquiring about present health

status?

a. “What concerns have you had in the past regarding your weight?”

b. “Do you have anorexia?”

c. “Describe the recent changes in your weight.”

d. “Do you have a family history of eating disorders?”

ANSWER: C

A

B

C

D

Feedback

This question implies the patient has or should have concerns about weight loss

and is not focused on the present health status.

This is not an appropriate question. More data are needed about this patient

before anorexia can be suspected or determined.

Asking the patient to describe recent changes in weight is an appropriate, openended

question to begin data collection so that the patient can report her current

health status related to weight.

This question is asked in the history rather than in the present health status part of

the health history.

DIF: Cognitive Level: Apply REF: 83

TOP: Nursing Process: Assessment

MSC: NCLEX Patient Needs: Physiologic Integrity: Basic Care and Comfort: Nutrition and

Hydration

6. While assessing a patient’s ability to consume food, the nurse recalls which types of foods are

the easiest to chew and swallow?

a. Thin liquids

b. Soft foods

c. Dry foods

d. Chewy foods

ANSWER: B

A

B

C

D

Feedback

Thin liquids and foods requiring forceful chewing (such as meat) may not be

tolerated well.

Foods that are soft and highly viscous are most easily chewed and swallowed.

Foods that are soft and highly viscous are most easily chewed and swallowed.

Thin liquids and foods requiring forceful chewing (such as meat) may not be

tolerated well.

DIF: Cognitive Level: Understand REF: 84

TOP: Nursing Process: Assessment

MSC: NCLEX Patient Needs: Physiologic Integrity: Basic Care and Comfort: Nutrition and

Hydration


7. Which tool is the best choice for a nurse to use as a quick screening tool to assess a patient’s

dietary intake?

a. Food diary

b. Calorie count

c. Comprehensive diet history

d. 24-hour recall

ANSWER: D

A

B

C

D

Feedback

A food diary provides detailed information, but is not convenient and requires a

follow-up visit.

A calorie count requires several days to collect data and requires a trained

dietitian to analyze the results.

A comprehensive diet history may provide more accurate reflection of nutrient

intake, but is time-consuming to acquire and requires a trained/skilled dietary

interviewer.

Twenty-four-hour recall is useful as a quick screening tool to assess dietary

intake.

DIF: Cognitive Level: Understand REF: 84

TOP: Nursing Process: Assessment

MSC: NCLEX Patient Needs: Physiologic Integrity: Basic Care and Comfort: Nutrition and

Hydration

8. A nurse calculates a patient’s body mass index (BMI) as 33. This measurement indicates

which class of weight?

a. Overweight

b. Obesity class I

c. Obesity class II

d. Obesity class III

ANSWER: B

Feedback

A Overweight is a BMI of 25 to 29.9.

B Obesity class I is a BMI of 30 to 34.9.

C Obesity class II is a BMI of 35 to 39.9.

D Obesity class III is a BMI greater than 40.

DIF: Cognitive Level: Understand REF: 86-87

TOP: Nursing Process: Assessment

MSC: NCLEX Patient Needs: Physiologic Integrity: Basic Care and Comfort: Nutrition and

Hydration

9. Nurses use which measurement as the most highly correlated with risk of morbidity and

mortality?

a. Waist-to-hip ratio

b. Triceps skinfold measure

c. Desirable body weight


d. Body mass index (BMI)

ANSWER: D

A

B

C

D

Feedback

The waist-to-hip ratio is an indication of the risk of unhealthy fat distribution. A

ratio that exceeds the desired ratio is indicative of upper body obesity. This

increases the risk of developing health problems related to obesity (e.g., diabetes,

hypertension, coronary artery disease, gallbladder disease, osteoarthritis, and

sleep apnea).

Triceps skinfold measurement provides an estimate of total body fat, which is

only one measure of risk for obesity, and not, by itself, the most reliable and valid

measurement to determine risk.

Calculating desirable body weight and comparing it to actual body weight does

not consider height in the assessment of obesity.

The BMI considers both weight and height in the calculation. The U.S. Preventive

Services Task Force advocates the BMI assessment as reliable and valid for

identifying adults at risk of morbidity and mortality because of being overweight

or obese. For this reason, calculating BMI is recommended for all individuals on

a periodic basis.

DIF: Cognitive Level: Understand REF: 86-87

TOP: Nursing Process: Assessment

MSC: NCLEX Patient Needs: Physiologic Integrity: Basic Care and Comfort: Nutrition and

Hydration

10. What is the desired body weight for a male who is 7 feet tall?

a. 178 lb

b. 225 lb

c. 250 lb

d. 275 lb

ANSWER: C

A

B

C

D

Feedback

This incorrect result is obtained when 106 lb is used for the first 6 feet rather than

5 feet (84 inches – 72 inches = 12); thus, 106 + (12 6) = 178 lb.

This incorrect result is obtained when the scale for women is used rather than the

scale for men (84 inches – 60 inches = 24); thus, 105 + (24 5) = 225.

The patient is 84 inches tall. Use 106 lb for the first 5 feet (60 inches) and 6 lb/

inch for the remaining 24 inches (84 inches – 60 inches = 24); thus, 106 + (24

6) = 250 lb. Students must perform a calculation to find the answer.

A result of 275 lb uses an incorrect calculation.

DIF: Cognitive Level: Apply REF: 90

TOP: Nursing Process: Assessment

MSC: NCLEX Patient Needs: Health Promotion and Maintenance: Lifestyle Choices

11. During a physical examination, the nurse notes that the patient’s skin is dry and flaking, with

patches of eczema. Which nutritional deficiency might be present?


a. Vitamin C

b. Vitamin B

c. Essential fatty acid

d. Protein

ANSWER: C

A

B

C

D

Feedback

Vitamin C deficiency causes bleeding gums, arthralgia, and petechiae.

Vitamin B deficiency is too large a category to consider. Specific categories of

vitamin B deficiency have been identified, such as pyridoxine and thiamine.

Dry and scaly skin is a manifestation of essential fatty acid deficiency.

Protein deficiency causes decreased pigmentation and lackluster hair.

DIF: Cognitive Level: Understand REF: 86

TOP: Nursing Process: Assessment

MSC: NCLEX Patient Needs: Physiologic Integrity: Basic Care and Comfort: Nutrition and

Hydration

12. Which patient needs to be taught about how diet and exercise can lower lipids to reduce the

risk for coronary artery disease?

a. A woman with a high-density lipoprotein (HDL) level of 53 mg/dl

b. A man with an HDL level of 43 mg/dl

c. A woman with a low-density lipoprotein (LDL) level of 125 mg/dl

d. A man with an LDL level of 200 mg/dl

ANSWER: D

A

B

C

D

Feedback

This patient’s HDL level is in the expected range.

This patient’s HDL level is in the expected range.

This patient’s LDL level is in the expected range.

The healthy range of LDL for men and women is <130 mg/dl.

DIF: Cognitive Level: Apply REF: 91| 93

TOP: Nursing Process: Assessment

MSC: NCLEX Patient Needs: Health Promotion and Maintenance: Lifestyle Choices

13. Which patient may require additional nutritional assessment?

a. A male patient with a blood glucose level of 100 mg/dl

b. A pregnant patient with a hemoglobin level of 10.5 g/dl

c. A female patient with a prealbumin level of 25 mg/dl

d. A male patient with a serum triglyceride level of 100 mg/dl

ANSWER: B

A

B

C

D

Feedback

This patient’s glucose level is within normal limits.

The expected hemoglobin level for a pregnant patient is 12 to 16 g/dl.

This patient’s prealbumin level is within normal limits.

This patient’s triglyceride level is within normal limits.


DIF: Cognitive Level: Analyze REF: 91| 93

TOP: Nursing Process: Assessment

MSC: NCLEX Patient Needs: Physiologic Integrity: Basic Care and Comfort: Nutrition and

Hydration

14. During a physical examination, the nurse notes that the patient’s skin is dry and flaking, with

patches of eczema, and suspects a nutritional deficiency. What additional data should the

nurse expect to find to confirm the suspicion?

a. Hair loss and hair that is easily removed from the scalp

b. Inflammation of the tongue and fissured tongue

c. Inflammation of peripheral nerves, and numbness and tingling in extremities

d. Fissures and inflammation of the mouth

ANSWER: A

A

B

C

D

Feedback

Hair loss (alopecia) and hair that is easily removed from the scalp (easy pluck

ability), such as dry, flaking skin, is caused by essential fatty acid deficiency.

Inflammation of the tongue (glossitis) and fissured tongue are manifestations of a

niacin deficiency.

Inflammation of peripheral nerves (neuropathy) and numbness and tingling in

extremities (paresthesia) are manifestations of a thiamine deficiency.

Fissures of the mouth (cheilosis) and inflammation of the mouth (stomatitis) are

manifestations of a pyridoxine deficiency.

DIF: Cognitive Level: Analyze REF: 88

TOP: Nursing Process: Assessment

MSC: NCLEX Patient Needs: Physiologic Integrity: Basic Care and Comfort: Nutrition and

Hydration

15. A male patient weighs 205 lb and his desired body weight (DBW) is 190 lb. How should the

nurse counsel this patient about his weight?

a. He has mild obesity and needs to increase exercise and assess his diet for nutrients

and calories.

b. He has moderate obesity and needs to consult a health care provider about weight

loss therapy.

c. He is within normal limits and need not be concerned at this time.

d. Further data are needed before an interpretation can be determined.

ANSWER: C

Feedback

A Using the DBW scale, mild obesity is 20% to 40% above DBW. This patient is

9% above his DBW.

B Using the DBW scale, moderate obesity is 40% to 100% greater than DBW. This

patient is 9% above his DBW.

C Ideally, the patient will fall between 90% and 110% of DBW, which for this

patient is between 171 lb and 209 lb. A calculation is required to answer the

question.

D According to the DBW scale, the patient will ideally be between 90% and 110%


of DBW; therefore, the nurse has enough information to make an interpretation.

DIF: Cognitive Level: Apply REF: 90

TOP: Nursing Process: Assessment

MSC: NCLEX Patient Needs: Health Promotion and Maintenance: Lifestyle Choices

16. A patient’s current body weight (304 lb) and his desirable body weight of 190 lb. How does

the nurse classify this patient’s weight?

a. Within expected range

b. Mildly obesity

c. Moderate obesity

d. Morbid obesity

ANSWER: C

Feedback

A A range of 90% to 110% of DBW is considered normal. This patient’s weight of

304 is 60% above his DBW.

B Mildly obesity is 20% to 40% above DBW. This patient’s weight of 304 is 60%

above his DBW.

C Moderate obesity is 40% to 100% above the DBW. This patient’s weight of 304 is

60% above his DBW.

D Morbid obesity is 100% greater than DBW. This patient’s weight of 304 is 60%

above his DBW.

DIF: Cognitive Level: Apply REF: 90

TOP: Nursing Process: Assessment

MSC: NCLEX Patient Needs: Health Promotion and Maintenance: Lifestyle Choices

17. Which patient has the least risk for unhealthy fat distribution?

a. The man whose triceps skinfold is at the 25th percentile

b. The woman whose triceps skinfold is at the 72nd percentile

c. The man whose waist circumference is 46 inches and hip circumference is 40

inches

d. The woman whose waist circumference is 30 inches and hip circumference is 38

inches

ANSWER: D

A

B

C

D

Feedback

Triceps skinfold is an estimate for total body fat, rather than risk. The expected

value is near the 50th percentile.

Triceps skinfold is an estimate for total body fat, rather than risk. The expected

value is near the 50th percentile.

This man’s waist-to-hip ratio is 1.15, which is higher than the 1.0 or less expected

value for a man.

This woman’s waist-to-hip ratio is 0.789, which is below 0.8, the expected value

for women.

DIF: Cognitive Level: Apply REF: 91

TOP: Nursing Process: Assessment


MSC: NCLEX Patient Needs: Health Promotion and Maintenance: Lifestyle Choices

18. A patient who has anorexia nervosa reports a healthy diet and no protein calorie malnutrition.

Which lab value best confirms this patient’s report?

a. Prealbumin

b. Serum albumin

c. Blood glucose

d. Serum cholesterol

ANSWER: A

Feedback

A Prealbumin is a reflection of protein and calorie intake for the previous 2 to 3

days.

B Serum albumin measures circulating protein. Albumin can be affected by a

number of factors, including fluid status, blood loss, liver function, and stress.

Fluctuation of albumin levels occurs over 3 to 4 weeks.

C Blood glucose measures can be affected by many factors such as stress, which

raises blood glucose levels, and is not a measure of protein calorie malnutrition.

D Serum cholesterol is part of the lipid profile, which provides data on fat

metabolism only, and is not a measure of protein calorie malnutrition.

DIF: Cognitive Level: Understand REF: 93

TOP: Nursing Process: Assessment

MSC: NCLEX Patient Needs: Physiologic Integrity: Basic Care and Comfort: Nutrition and

Hydration

MULTIPLE RESPONSE

1. A nurse is assessing an 80-year-old patient who is cared for at home by his 79-year-old wife.

Which data indicate this patient has malnutrition? Select all that apply.

a. Body mass index (BMI) of 17

b. Waist-to-hip ratio of 1.0

c. Weight loss of 6% since last month’s visit

d. Prealbumin level of 16 mg/dl

e. Hematocrit level of 50%

f. Hemoglobin level of 20 g/dl

ANSWER: A, C, F

Correct: A BMI of 18.5 to 24.9 is the normal range, and this patient is below normal. Severe

weight loss is a more than 2% weight change over 1 week. The expected hemoglobin for a

man is 14 to 18 g/dl. These values may also indicate dehydration.

Incorrect: Waist-to-hip ratio of 1.0 is an expected value for men. The expected level for

prealbumin is 15 to 36 mg/dl. This hematocrit level is within normal limits.

DIF: Cognitive Level: Analyze REF: 86-87| 90| 93

TOP: Nursing Process: Assessment

MSC: NCLEX Patient Needs: Physiologic Integrity: Basic Care and Comfort: Nutrition and

Hydration


COMPLETION

1. A man who is 6 feet 9 inches tall is told by his provider to lose weight so that he is closer to

his desired body weight. He asks the nurse, “How can I find out what my desired body weight

should be?” The nurse responds, “Let me show you how to calculate it. Your desired body

weight (DBW) should be lb.”

ANSWER:

232

6 feet 9 inches = 81 inches. DBW = 106 lb for the first 60 inches + 6 lb for every other inch,

which for this man equals 21 inches (81” – 60” = 21”). 21 6 = 126 +106 = 232 lb.

DIF: Cognitive Level: Apply REF: 90

TOP: Nursing Process: Implementation

MSC: NCLEX Patient Needs: Physiologic Integrity: Basic Care and Comfort: Nutrition and

Hydration

2. A woman who is 4 feet 11 inches tall is told by her provider to lose weight so that she is closer

to her desired body weight. She asks the nurse, “How can I find out what my desired body

weight should be?” The nurse responds, “Let me show you how to calculate it. Your desired

body weight (DBW) should be lb.”

ANSWER:

103.25

4 feet 11 inches = 59 inches. DBW = 105 lb for the 60 inches + 5 lb for every other inch.

However this woman is under 5 feet in height. Thus 105 lb/60 inches = 1.75 lb/inch. 1.75 59

inches = 103.25 lb.

DIF: Cognitive Level: Apply REF: 90

TOP: Nursing Process: Implementation

MSC: NCLEX Patient Needs: Physiologic Integrity: Basic Care and Comfort: Nutrition and

Hydration

3. A woman’s waist circumference is 32 inches and her hip circumference is 29 inches. Her

waist-to-hip ratio is .

ANSWER:

1.10

Calculate waist-to-hip by dividing waist measurement by the hip measurement, 32 inches/29 =

1.10.

DIF: Cognitive Level: Apply REF: 91

TOP: Nursing Process: Assessment

MSC: NCLEX Patient Needs: Physiologic Integrity: Basic Care and Comfort: Nutrition and

Hydration


Chapter 9: Skin, Hair, and Nails

Test Bank

MULTIPLE CHOICE

1. A patient asks the nurse if it is possible to grow new skin. What is the nurse’s most

appropriate response?

a. “Even if new skin growth is required, the melanocytes do not regenerate.”

b. “The avascular epidermis sheds slowly and is replaced completely every 4 weeks.”

c. “The outer layer of skin remains the same over the lifetime except for repairing

injuries.”

d. “Epidermal regeneration is impossible because it is avascular.”

ANSWER: B

A

B

C

D

Feedback

Melanocytes are not involved in regeneration. They secrete melanin, which

provides pigment for the skin and hair and serves as a shield against ultraviolet

radiation.

Within this deepest layer of epidermis, active cell generation takes place. As cells

are produced, they push up the older cells toward the skin surface. The entire

process takes about 30 days.

The dead cells are continuously sloughed off and replaced by new cells moving

up from the underlying epidermal layers.

Within this deepest layer of epidermis, active cell generation takes place.

DIF: Cognitive Level: Understand REF: 98

TOP: Nursing Process: Assessment

MSC: NCLEX Patient Needs: Physiologic Integrity: Reduction of Risk Potential: System Specific

Assessments

2. A nurse assessing a patient with liver disease expects to find which manifestation during the

examination?

a. Yellowish color in the axilla and groin

b. Yellow pigmentation in the sclera

c. Very pale skin on the palms

d. Ashen-gray color in the oral mucous membranes

ANSWER: B

A

B

C

D

Feedback

Instead of the axilla and groin, assess the sclera of the eyes, fingernails, palms of

hands, and oral mucosa.

Jaundice is manifested by a yellowish color in the sclera of the eyes and palms of

the hands in both light- and dark-skinned patients.

Pale skin may indicate anemia, but not jaundice. Yellow color of the palms

indicates jaundice.

Ashen-gray color may be seen in dark-skinned patients who are cyanotic.

DIF: Cognitive Level: Apply REF: 98| 101


TOP: Nursing Process: Assessment

MSC: NCLEX Patient Needs: Physiologic Integrity: Physiologic Adaptation: Alterations in Body

Systems

3. How does the nurse recognize jaundice in a dark-skinned patient?

a. Inspect the conjunctiva for ashen-gray color.

b. Inspect the nail beds for a deeper brown or purple skin tone.

c. Inspect the palms and soles for yellowish-green color.

d. Inspect the oral mucous membrane for yellow color.

ANSWER: C

A

B

C

D

Feedback

Ashen-gray color may be seen in dark-skinned patients who are cyanotic.

Brown or purple tone is seen in dark-skinned patients with erythema.

In dark-skinned patients, jaundice manifests as a yellowish-green color that can

be seen most obviously in the sclera, palms of hands, and soles of feet.

Mucous membranes do not change color from jaundice.

DIF: Cognitive Level: Understand REF: 103, Table 9-1

TOP: Nursing Process: Assessment

MSC: NCLEX Patient Needs: Physiologic Integrity: Physiologic Adaptation: Alterations in Body

Systems

4. What signs of cyanosis does a nurse inspect for in a dark-skinned patient?

a. Ashen-gray color of the oral mucous membranes

b. Blue color in the nail beds

c. Ashen-blue color in the palms and soles

d. Blue-gray color in the ear lobes and lips

ANSWER: A

A

B

C

D

Feedback

Cyanosis is manifested by ashen-gray color of the oral mucous membranes and

nail beds in a dark-skinned patient.

An ashen-gray color of the nail beds is expected in a dark-skinned patient, rather

than blue.

An ashen-gray color of the oral mucous membranes and nail beds are expected in

a dark-skinned patient.

An ashen-gray color of the oral mucous membranes and nail beds are expected in

a dark-skinned patient.

DIF: Cognitive Level: Apply REF: 103, Table 9-1

TOP: Nursing Process: Assessment

MSC: NCLEX Patient Needs: Physiologic Integrity: Physiologic Adaptation: Alterations in Body

Systems

5. When the patient’s chart includes a notation that petechiae are present, what finding does a

nurse expect during inspection?

a. Purplish-red pinpoint lesions

b. Deep purplish or red patches of skin


c. Small raised fluid-filled pinkish nodules

d. Generalized reddish discoloration of an area of skin

ANSWER: A

A

B

C

D

Feedback

Purplish-red pinpoint lesions describes the appearance of petechiae.

Petechiae are pinpoints, not as large as a patch.

Petechiae are pinpoints, not raised as a nodule.

Petechiae are pinpoints, not generalized.

DIF: Cognitive Level: Understand REF: 103, Table 9-1| 117

TOP: Nursing Process: Assessment

MSC: NCLEX Patient Needs: Physiologic Integrity: Physiologic Adaptation: Alterations in Body

Systems

6. When performing a skin assessment of an adult patient, the nurse expects what finding?

a. Reddened area does not blanch when gentle pressure is applied

b. Indentation of the finger remains in the skin after palpation

c. Flaking or scaling of the skin

d. Return of skin to its original position when pinched up slightly

ANSWER: D

A

B

C

D

Feedback

This is an indication of a stage I pressure ulcer.

This is a description of edema.

This may be an indication of dry skin, systemic disease, or nutritional deficiency.

This is an assessment of skin turgor; skin should return to its original position.

DIF: Cognitive Level: Apply REF: 106

TOP: Nursing Process: Assessment

MSC: NCLEX Patient Needs: Reduction of Risk Potential: System Specific Assessments

7. A nurse notices a patient’s nails are thin and depressed with the edges turned up. What

additional abnormal data should the nurse expect to find on this patient?

a. Pale conjunctiva

b. Jaundice

c. Ecchymosis

d. Rashes

ANSWER: A

A

B

C

D

Feedback

The abnormal nail finding was koilonychia, which occurs in patients with anemia

who frequently have pale conjunctiva.

Jaundice is due to increased serum bilirubin, indicating liver or gallbladder

disease, and does not create changes in nail structure.

Ecchymosis occurs after trauma to the blood vessel resulting in bleeding under

the tissue and does not cause changes in nail structure.

Rashes indicate an inflammation or allergic reaction that does cause changes in


the nails.

DIF: Cognitive Level: Analyze REF: 108

TOP: Nursing Process: Assessment

MSC: NCLEX Patient Needs: Physiologic Integrity: Physiologic Adaptation: Alterations in Body

Systems

8. A 45-year-old woman tells the nurse she is distressed by the presence of dark, coarse hair on

her face that has recently developed. What is the nurse’s most appropriate response to this

patient?

a. “This is simple vellus hair and it will decrease in amount over time.”

b. “Some women in your cultural group normally have dark hair on their faces.”

c. “This is unusual; female hair distribution should be limited to arms, legs, and

pubis.”

d. “Coarse dark hair could result from hormonal changes such as from menopause.”

ANSWER: D

A

B

C

D

Feedback

This response is not true. This example describes hirsutism, a condition

associated with an increase in the growth of facial, body, or pubic hair in women.

It does not decrease over time and the hair is not vellus.

Although it is true that women of some cultural groups normally have dark hair

on the face, women in these cultural groups have darker facial hair most of their

adult lives; the patient in this item has a new onset of hirsutism.

It is not true that female hair distribution should normally be limited to arms, legs,

and pubis. Women do have hair on their faces and other areas.

Coarse, dark hair on the face describes hirsutism, an increase in the growth of

facial, body, or pubic hair in women that can be associated with menopause or an

endocrine disorder.

DIF: Cognitive Level: Apply REF: 107

TOP: Nursing Process: Assessment

MSC: NCLEX Patient Needs: Physiologic Integrity: Physiologic Adaptation: Alterations in Body

Systems

9. What findings does a nurse expect when inspecting and palpating a patient’s nails?

a. A nail base angle of not more than 90 degrees

b. Whitish to clear nails in darker-skinned patients

c. Nail surface is smooth and rounded

d. Transverse depression running across the nails

ANSWER: C

A

B

C

D

Feedback

The expected angle of the nail base is 160 degrees.

Patients with darker-pigmented skin typically have nails that are yellow or brown,

and vertical banded lines may appear.

Nail surface that is smooth and rounded is an expected finding.

This is a description of Beau lines.


DIF: Cognitive Level: Apply REF: 107

TOP: Nursing Process: Assessment

MSC: NCLEX Patient Needs: Physiologic Integrity: Reduction of Risk Potential: System Specific

Assessments

10. A nurse notices that the angle of the patient’s proximal nail fold and the nail plate are almost a

flat line; about 160 degrees. How does the nurse interpret this finding?

a. This patient has chronic pulmonary disease.

b. This is an expected finding.

c. This is due to stress to the nails.

d. This is associated with anemia.

ANSWER: B

A

B

C

D

Feedback

This patient has chronic pulmonary disease, which causes clubbing (when the

angle of the nail base exceeds 180 degrees).

The expected angle of the nail base is 160 degrees.

This answer describes Beau lines, which appear as a groove or transverse

depression running across the nail. It results from a stressor that temporarily

impairs nail formation.

This is associated with anemia, which causes koilonychia, a thin, depressed nail

with the lateral edges turned upward.

DIF: Cognitive Level: Understand REF: 108-109

TOP: Nursing Process: Assessment

MSC: NCLEX Patient Needs: Physiologic Integrity: Reduction of Risk Potential: System Specific

Assessments

11. As a nurse is inspecting the nails of a patient with chronic hypoxemia and notices

enlargements of the ends of the fingers and angles of the nail base greater than a straight line

(exceeding 180 degrees). How does the nurse document these findings?

a. An expected finding

b. Koilonychia (spoon nail)

c. Clubbing

d. Leukonychia

ANSWER: C

A

B

C

D

Feedback

This is clubbing, which is not an expected finding.

Koilonychia is a thin, depressed nail with the lateral edges turned upward and is

associated with anemia.

Clubbing is present when the angle of the nail base exceeds 180 degrees. It is

caused by proliferation of the connective tissue resulting in an enlargement of the

distal fingers and is most commonly associated with chronic respiratory or

cardiovascular disease.

Leukonychia appears as white spots on the nail plate, usually caused by minor

trauma or manipulation of the cuticle.

DIF: Cognitive Level: Apply REF: 108-109


TOP: Nursing Process: Assessment

MSC: NCLEX Patient Needs: Physiologic Integrity: Physiologic Adaptation: Alterations in Body

Systems

12. While giving a history, a patient reports itching arms, legs, and chest after using a new soap.

What manifestations does the nurse expect to find on the arms, legs, and chest when

inspecting this patient’s skin?

a. Elevated irregularly shaped areas of edema of variable diameter

b. Elevated, firm, and rough lesions with flat surface greater than 1 cm in diameter

c. Elevated circumscribed superficial lesions less than 1 cm in diameter filled with

serous fluid

d. Elevated, firm circumscribed areas less than 1 cm in diameter

ANSWER: A

A

B

C

D

Feedback

This is a description of wheals, which occur as a result of allergic reactions.

This is a description of plaque.

This is a description of a vesicle.

This is a description of a papule.

DIF: Cognitive Level: Analyze REF: 112| 123-124

TOP: Nursing Process: Assessment

MSC: NCLEX Patient Needs: Physiologic Integrity: Physiologic Adaptation: Alterations in Body

Systems

13. While inspecting the skin, a nurse notices a lesion on the patient’s upper right arm. What is the

best way to document the size of this lesion?

a. Compare its size to the size of a coin.

b. Estimate its size to the nearest inch.

c. Use a centimeter ruler to measure the lesion.

d. Trace the lesion onto a piece of paper.

ANSWER: C

A

B

C

D

Feedback

Comparing its size to the size of a coin can be done if no measurement tool is

available, but the best way is to measure the lesion.

Estimating size to the nearest inch is not recommended due to inaccuracy.

A centimeter ruler to measure the size of lesions may be helpful. The lesion is

documented based on its characteristics, including location, distribution, color,

pattern, edges, flat or raised, and size.

Tracing the lesion onto a piece of paper can be done if no measurement tool is

available, but the best way is to measure the lesion.

DIF: Cognitive Level: Apply REF: 110-111, Box 9-2

TOP: Nursing Process: Assessment

MSC: NCLEX Patient Needs: Physiologic Integrity: Physiologic Adaptation: Alterations in Body

Systems


14. During shift report, a nurse learns that a patient has a macular rash. As the nurse inspects the

patient’s skin, what finding will confirm the rash?

a. Elevated, firm, well-defined lesions less than 1 cm in diameter

b. Depressed, firm, or scaly, rough lesions greater than 1 cm in diameter

c. Elevated, fluid-filled lesions less than 1 cm in diameter

d. Flat, well-defined, small lesions less than 1 cm in diameter

ANSWER: D

A

B

C

D

Feedback

Elevated, firm, well-defined lesions less than 1 cm in diameter is a description of

a papule.

Depressed, firm, or scaly, rough lesions greater than 1 cm in circumference is an

incorrect description.

Elevated, fluid-filled lesions less than 1 cm in diameter is a description of a

vesicle.

Flat, well-defined, small lesions less than 1 cm in diameter is a description of a

macule.

DIF: Cognitive Level: Apply REF: 111

TOP: Nursing Process: Assessment

MSC: NCLEX Patient Needs: Physiologic Integrity: Physiologic Adaptation: Alterations in Body

Systems

15. During inspection of a patient’s upper back, the nurse notices three small, elevated superficial

lesions filled with purulent fluid. How does the nurse document this finding?

a. As three cysts on the upper back

b. As several bullae on the back

c. As three pustules on the upper back

d. As three wheals on the upper back

ANSWER: C

A

B

C

D

Feedback

Cysts are elevated, circumscribed, encapsulated lesions.

Bullae are vesicles greater than 1 cm in diameter. This documentation is not

specific to the number or exact location.

Pustules are elevated, superficial lesions similar to vesicles but filled with

purulent fluid. This is a specific documentation of what the nurse saw (three

pustules) and their location (upper back).

Wheals are elevated irregular-shaped areas of cutaneous edema that are solid,

transient, and of variable diameter.

DIF: Cognitive Level: Understand REF: 113

TOP: Nursing Process: Assessment

MSC: NCLEX Patient Needs: Physiologic Integrity: Physiologic Adaptation: Alterations in Body

Systems

16. A nurse notices multiple lesions on a patient’s left hand that are 0.5 cm in width, elevated,

circumscribed, and filled with serous fluid. What kind of primary lesions are these?

a. Macules


b. Patches

c. Vesicles

d. Bullae

ANSWER: C

A

B

C

D

Feedback

Macules are flat, circumscribed areas that are a change in the color of the skin and

are less than 1 cm in diameter.

Patches are flat, nonpalpable, irregular-shaped macules greater than 1 cm in

diameter.

Vesicles are elevated, circumscribed, superficial (do not extend into dermis),

filled with serous fluid, and less than 1 cm in diameter. This documentation tells

the number and location of the lesions.

Bullae are large vesicles greater than 1 cm in diameter.

DIF: Cognitive Level: Understand REF: 113

TOP: Nursing Process: Assessment

MSC: NCLEX Patient Needs: Physiologic Integrity: Physiologic Adaptation: Alterations in Body

Systems

17. A nurse notices multiple lesions on the back of a patient’s left hand that are 0.5 cm in width,

elevated, circumscribed, and filled with serous fluid. How does the nurse document these

lesions?

a. As multiple macules on the dorsum of the left hand

b. As multiple vesicles on the dorsum of the left hand

c. As several patches on the left hand

d. As several bullae on the dorsum of the left hand

ANSWER: B

A

B

C

D

Feedback

Macules are flat, circumscribed areas that are a change in the color of the skin and

are less than 1 cm in diameter.

Vesicles are elevated, circumscribed, superficial (do not extend into dermis),

filled with serous fluid, and less than 1 cm in diameter. This documentation tells

the number and location of the lesions.

Patches are flat, nonpalpable, irregular-shaped macules greater than 1 cm in

diameter. This documentation does not include location of lesions.

Bullae are large vesicles greater than 1 cm in diameter.

DIF: Cognitive Level: Understand REF: 113

TOP: Nursing Process: Assessment

MSC: NCLEX Patient Needs: Physiologic Integrity: Physiologic Adaptation: Alterations in Body

Systems

18. A patient has come to the clinic complaining of a “bump” behind his right ear. Upon

inspection, the nurse notes a lesion that is elevated, solid, and 4 cm in diameter. What does the

nurse call this lesion when she reports her findings to the health care provider?

a. Tumor

b. Nodule


c. Keloid

d. Papule

ANSWER: A

A

B

C

D

Feedback

A tumor is an elevated and solid lesion, may or may not be clearly demarcated,

extends deeper in the dermis, and greater than 2 cm in diameter.

A nodule is an elevated, firm, circumscribed lesion that extends deeper into the

dermis than a papule and is 1 to 2 cm in diameter.

A keloid is an irregularly-shaped, elevated, progressively-enlarging scar that

grows beyond the boundaries of the wound.

A papule is an elevated, firm, circumscribed area less than 1 cm in diameter.

DIF: Cognitive Level: Understand REF: 112

TOP: Nursing Process: Assessment

MSC: NCLEX Patient Needs: Physiologic Integrity: Physiologic Adaptation: Alterations in Body

Systems

19. A nurse is inspecting the skin of a patient who has had skin problems after multiple piercings.

How will the nurse recognize the characteristics of keloids?

a. Roughened and thickened scales involving flexor surfaces

b. Hypertrophic scarring extending beyond the original wound edges

c. Thin, fibrous tissue replacing normal skin following injury

d. Loss of the epidermal layer, creating a hollowed-out or crusted area

ANSWER: B

A

B

C

D

Feedback

Roughened and thickened scales involving flexor surfaces is a description of

lichenification.

Hypertrophic scarring extending beyond the original wound edges is a description

of a keloid.

Thin, fibrous tissue replacing normal skin following injury is a description of a

scar.

Loss of the epidermal layer, creating a hollowed-out or crusted area is a

description of excoriation.

DIF: Cognitive Level: Understand REF: 114

TOP: Nursing Process: Assessment

MSC: NCLEX Patient Needs: Physiologic Integrity: Physiologic Adaptation: Alterations in Body

Systems

20. A patient reports the mole on the scalp has started itching and it bleeds when scratching it.

What other finding is a danger sign for pigmented skin lesions?

a. Symmetry of the lesion

b. Rounded border

c. Color variation

d. Size less than 6 mm wide

ANSWER: C


A

B

C

D

Feedback

Symmetry is an expected finding for moles. Asymmetric lesions are an early sign

of malignant melanoma.

A rounded border is an expected finding. A border that is poorly defined or

irregular is an early sign of malignant melanoma.

Uneven, variegated color is an early sign of malignant melanoma.

A size of less than 6 mm wide is an expected finding. A lesion greater than 6 mm

is an early sign of malignant melanoma.

DIF: Cognitive Level: Analyze REF: 111| 130

TOP: Nursing Process: Assessment

MSC: NCLEX Patient Needs: Physiologic Integrity: Physiologic Adaptation: Alterations in Body

Systems

21. A toddler patient has a small, slightly raised bright red area on the trunk. The child’s mother

reports that the lesion has been present since birth and has become a little larger. What type of

lesion does the nurse suspect?

a. Vascular nevi

b. Purpura

c. Ecchymosis

d. Cherry hemangioma is a benign tumor consisting of a mass of small blood vessels and can

vary in size. These are typically small, slightly raised lesions that are bright red in color

appearing on the face, neck and trunk of the body. These lesions increase in size with age.

ANSWER: D

A

B

C

D

Feedback

Vascular nevi is a type of angioma that involves the capillaries within the skin

producing an irregular macular patch that can vary from light red to dark red to

purple in color.

Purpura is a flat, reddish purple, nonblanchable discoloration in the skin greater

than 0.5 cm in diameter.

Ecchymosis is a reddish purple, nonblanchable spot of variable size.

Cherry hemangioma is a benign tumor consisting of a mass of small blood vessels

and can vary in size. These are typically small, slightly raised lesions that are

bright red in color appearing on the face, neck and trunk of the body. These

lesions increase in size with age.

DIF: Cognitive Level: Understand REF: 117-118

TOP: Nursing Process: Assessment

MSC: NCLEX Patient Needs: Physiologic Integrity: Physiologic Adaptation: Alterations in Body

Systems

22. A nurse notices several reddish purple, nonblanchable spots of different sizes on the arms and

legs of a patient with a low platelet count. How does the nurse distinguish ecchymosis from

purpura?

a. Ecchymosis is variable in size and a purpura is greater than 0.5 cm in diameter.

b. Ecchymosis does not blanch and purpura does blanch.

c. Ecchymosis has raised lesions and purpura has flat lesions.

d. Ecchymosis is irregularly shaped and purpura is round.


ANSWER: A

A

B

C

D

Feedback

Ecchymosis is variable in size and a purpura is greater than 0.5 cm in diameter.

This is an accurate statement.

Ecchymosis does not blanch and purpura does blanch. Both of these lesions are

nonblanchable.

Ecchymosis has raised lesions and purpura has flat lesions. Both of these lesions

are flat.

Ecchymosis is irregularly shaped and purpura is round. There is no specified

shape for either type of lesion.

DIF: Cognitive Level: Apply REF: 118, Table 9-4

TOP: Nursing Process: Assessment

MSC: NCLEX Patient Needs: Physiologic Integrity: Physiologic Adaptation: Alterations in Body

Systems

23. A patient is visiting an urgent care center after being hit in the back with a baseball. Upon

examination, the nurse notes a flat, nonblanchable spot 2.25 cm wide that is reddish-purple in

color. How does the nurse document this lesion?

a. As an angioma

b. As purpura

c. As petechiae

d. As ecchymosis

ANSWER: D

A

B

C

D

Feedback

An angioma is characterized by a small central red area with radiating spider-like

legs that blanches with pressure. The lesion of this patient does not blanch.

Purpura is flat, reddish purple, non-blanchable, and greater than 0.5 cm in size. It

is caused by infection or a bleeding disorder, not trauma.

Petechiae are tiny, flat, reddish-purple, non-blanchable spots in the skin less than

0.5 cm in diameter and appear as tiny red spots that are pinpoint to pinhead in

size.

Ecchymosis is reddish purple in color, nonblanchable and is caused by trauma

(being hit with a baseball) to the blood vessel which results in bleeding

underneath the skin. The size of the ecchymotic area varies depending on the

level of trauma.

DIF: Cognitive Level: Understand REF: 118

TOP: Nursing Process: Assessment

MSC: NCLEX Patient Needs: Physiologic Integrity: Physiologic Adaptation: Alterations in Body

Systems

24. A nurse is performing an admission physical examination on a patient who has been

bedridden for a month. The nurse notices a pressure ulcer on the patient’s left trochanter area

that involves partial-thickness skin loss with damage to the subcutaneous tissue. The nurse

reports this ulcer at what stage?

a. Stage I


b. Stage II

c. Stage III

d. Stage IV

ANSWER: B

A

B

C

D

Feedback

Stage I ulcers have persistent redness, but the epidermis is intact.

Stage II ulcers have partial-thickness skin loss of dermis. It appears as a shiny or

dry shallow open ulcer with pink wound bed without slough or bruising.

Stage III ulcers have full-thickness skin loss involving damage to or necrosis of

subcutaneous tissue that may extend to, but not through, underlying fascia.

Stage IV ulcers have full-thickness tissue loss with exposed bone, tendon, or

muscle. Slough or eschar may be within the wound bed.

DIF: Cognitive Level: Understand REF: 121-122

TOP: Nursing Process: Assessment

MSC: NCLEX Patient Needs: Physiologic Integrity: Physiologic Adaptation: Alterations in Body

Systems

25. A patient complains of itching on her feet. On inspection the nurse observes weeping vesicles

and skin that is softened and broken down between the toes? What explanation does the nurse

give the patient about the cause of this skin disorder?

a. “Your itching is caused by a bacterial infection.”

b. “Your itching is caused by an allergic reaction.”

c. “Your itching is caused by a viral infection.”

d. “Your itching is caused by a fungal infection.”

ANSWER: D

A

B

C

D

Feedback

Bacterial infections such as cellulitis cause redness, warmth, and tenderness,

rather than itching.

Allergic reactions such as contact dermatitis cause itching, but they appear as

localized erythema, and may also form edema, wheals, scales, or vesicles.

Viral infection such as herpes form grouped vesicles that are painful, rather than

itching.

This is a description of tinea pedia, which is caused by a number of dermophyte

fungal infections.

DIF: Cognitive Level: Analyze REF: 127

TOP: Nursing Process: Assessment

MSC: NCLEX Patient Needs: Physiologic Integrity: Physiologic Adaptation: Alterations in Body

Systems

26. A patient expresses concern that a new lesion may be melanoma. Which finding suggests a

malignant melanoma?

a. Nonblanching lesion

b. Irregular border

c. Diameter less than 5 mm

d. Black color of the lesion


ANSWER: B

A

B

C

D

Feedback

Blanching is not assessed in malignant skin lesions.

Irregular border or poorly defined border is an indication of a malignancy.

Diameter of a malignant skin lesion is usually greater than 6 mm.

Melanoma is a variety of colors.

DIF: Cognitive Level: Apply REF: 102| 129-130

TOP: Nursing Process: Assessment

MSC: NCLEX Patient Needs: Physiologic Integrity: Physiologic Adaptation: Alterations in Body

Systems

27. During a health fair, which recommendation is appropriate as a primary prevention measure to

reduce the risk for skin cancer?

a. Use a tanning booth instead of sunning outside if a tan is desired.

b. Wear protective clothing while in the sun.

c. Perform self-examination of skin monthly.

d. Use sunscreen with a sun protection except on overcast days.

ANSWER: B

A

B

C

D

Feedback

Avoiding tanning and sunning are part of primary prevention.

Wearing protective clothing while in the sun provides primary prevention for skin

cancer.

Performing self-examination of skin monthly is secondary prevention.

Sunscreen also needs to be used on overcast days.

DIF: Cognitive Level: Understand REF: 102

TOP: Nursing Process: Assessment

MSC: NCLEX Patient Needs: Health Promotion and Maintenance: Self-Care

28. A nurse notes that a 2-year-old child has multiple bruises over his body at different stages of

healing. What is the most appropriate action for the nurse at this time?

a. Obtain further data now to rule out abuse.

b. Remind parents that toddlers are clumsy and may fall, causing bruising.

c. Determine if this toddler has a coagulation disorder.

d. Recommend further observation at future visits.

ANSWER: A

A

B

C

D

Feedback

Further investigation is needed to rule out abuse. The important clue is bruises in

different stages in healing. Injuries to the skin are generally recognized in three

forms: bruises, bites, and burns.

Remind parents that toddlers are clumsy and may fall, causing bruising. The

important clue is bruises in different stages of healing.

A coagulation disorder can be ruled out by a laboratory test for platelets.

Recommend further observation at future visits. Action must be taken during this


visit if abuse is suspected.

DIF: Cognitive Level: Apply REF: 101| 118| 130-131

TOP: Nursing Process: Assessment

MSC: NCLEX Patient Needs: Physiologic Integrity: Physiologic Adaptation: Alterations in Body

Systems

MULTIPLE RESPONSE

1. What findings does the nurse expect when assessing skin, hair, and nails of a healthy male

adult? Select all that apply.

a. Transverse depression noticed across nails

b. Scalp is bald

c. Elevated, firm, circumscribed area less than 1 cm wide found on the fingers

d. Purpura and ecchymosis are noticed on arms and legs

e. Freckles are noted on face, back, arms, and legs

f. Skin turgor is elastic

ANSWER: B, E, F

Correct:Scalp is bald; freckles are noted on face, back, arms and legs; and skin turgor is

elastic . These are expected findings for a healthy adult male.

Incorrect: Transverse depression across the nails describes Beau lines. It results from a

stressor that temporarily impairs nail formation. An elevated, firm, circumscribed area, less

than 1 cm wide on the fingers describes a papule, such as a wart. Purpura and ecchymosis on

arms and legs are indications of bleeding.

DIF: Cognitive Level: Analyze REF: 106-107| 119

TOP: Nursing Process: Assessment

MSC: NCLEX Patient Needs: Physiologic Integrity: Reduction of Risk Potential: System Specific

Assessments

2. Which questions are appropriate to ask a patient when performing a symptom analysis for a

rash? Select all that apply.

a. “When did the rash first start?”

b. “Do you have a family history of rashes?”

c. “What makes the rash worse?”

d. “What do you do to make your rash better?”

e. “Describe the sensation from the rash, does it burn or itch?”

f. “Describe what the rash looked like initially.”

ANSWER: A, C, D, E, F

Correct: These are questions asked in a symptom analysis that includes the following

variables: onset of symptoms, location and duration of symptoms, characteristics, severity of

symptoms, related symptoms, alleviating factors, aggravating factors, and attempts at selftreatment.

Incorrect: This question relates to the patient’s history.

DIF: Cognitive Level: Analyze REF: 100

TOP: Nursing Process: Assessment

MSC: NCLEX Patient Needs: Physiologic Integrity: Physiologic Adaptation: Alterations in Body


Systems


Chapter 10: Head, Eyes, Ears, Nose, and Throat

Test Bank

MULTIPLE CHOICE

1. A patient is admitted with edema of the occipital lobe following a head injury. The nurse

correlates which finding with damage to this area?

a. Ipsilateral ptosis

b. Impaired vision

c. Pupillary constriction

d. Increased intraocular pressure

ANSWER: B

A

B

C

D

Feedback

Ipsilateral ptosis (drooping of the eye lid) is controlled by the oculomotor cranial

nerve (CN III) that is located in the midbrain. The nurse must correlate anatomy

with function and assessment.

The occipital lobe contains the visual context.

Pupillary constriction is controlled by the oculomotor cranial nerve (CN III) that

is located in the midbrain.

This abnormality is associated with glaucoma rather than injury to the occipital

lobe.

DIF: Cognitive Level: Analyze REF: 138

TOP: Nursing Process: Assessment

MSC: NCLEX Patient Needs: Physiologic Integrity: Physiologic Adaptation: Alteration in Body

Systems

2. The nurse is taking a health history on a patient who reports frequent stabbing headaches

occurring once a day lasting about an hour. Which statement by the patient is most indicative

of cluster headaches?

a. “I usually have nausea and vomiting with my headaches.”

b. “My whole head is constantly throbbing.”

c. “It feels like my head is in a vice.”

d. “The pain is on the left side over my eye, forehead, and cheek.”

ANSWER: D

A

B

C

D

Feedback

This is descriptive of migraines rather than cluster headaches.

This is descriptive of migraines rather than cluster headaches.

This is descriptive of tension rather than cluster headaches.

This description is consistent with cluster headaches.

DIF: Cognitive Level: Apply REF: 144

TOP: Nursing Process: Assessment

MSC: NCLEX Patient Needs: Physiologic Integrity: Physiologic Adaptation: Alteration in Body

Systems


3. A patient reports having migraine headaches on one side of the head that often start with an

aura and last 1 to 3 days. As a part of the symptom analysis, the patient reports which

associated symptoms of migraine headaches?

a. Nausea, vomiting, or visual disturbances

b. Nasal stuffiness or discharge

c. Ringing in the ears or dizziness

d. Red, watery eyes or drooping eyelids

ANSWER: A

A

B

C

D

Feedback

These are symptoms associated with migraine headaches.

This is a symptom associated with cluster headaches rather than migraine

headaches.

These symptoms are not associated with migraine headaches.

These are symptoms associated with cluster headaches rather than migraine

headaches.

DIF: Cognitive Level: Analyze REF: 144

TOP: Nursing Process: Assessment

MSC: NCLEX Patient Needs: Physiologic Integrity: Physiologic Adaptation: Alteration in Body

Systems

4. The nurse is taking a health history on a patient who reports frequent headaches with pain in

the front of the head, but sometimes felt in the back of the head. Which statement by the

patient is most indicative of tension headaches?

a. “I usually have nausea and vomiting with my headaches.”

b. “My whole head is constantly throbbing.”

c. “It feels like my head is in a vice.”

d. “The pain is on the left side over my eye, forehead, and cheek.”

ANSWER: C

A

B

C

D

Feedback

This is descriptive of migraines rather than tension headaches.

This is descriptive of migraines rather than tension headaches.

This is descriptive of tension headaches, which is consistent with the rest of the

data reported by the patient.

This is consistent with cluster headaches rather than tension headaches.

DIF: Cognitive Level: Apply REF: 144

TOP: Nursing Process: Assessment

MSC: NCLEX Patient Needs: Physiologic Integrity: Physiologic Adaptation: Alteration in Body

Systems

5. During symptom analysis, the nurse helps the patient distinguish between dizziness and

vertigo. Which description by the patient indicates vertigo?

a. “I felt faint, like I was going to pass out.”

b. “I just could not keep my balance when I sat up.”

c. “It seemed that the room was spinning around.”

d. “I was afraid that I was going to lose consciousness.”


ANSWER: C

A

B

C

D

Feedback

This is a description of lightheadedness, a form of dizziness.

This is a description of disequilibrium, a form of dizziness.

This is consistent with vertigo because it includes a sensation of motion.

This is a description of syncope, a form of dizziness.

DIF: Cognitive Level: Analyze REF: 144

TOP: Nursing Process: Assessment

MSC: NCLEX Patient Needs: Physiologic Integrity: Physiologic Adaptation: Alteration in Body

Systems

6. During symptom analysis, the nurse helps the patient distinguish between dizziness and

vertigo. Which description by the patient indicates dizziness?

a. “I felt faint, like I was going to pass out.”

b. “It felt like I was on a merry-go-round.”

c. “The room seemed to be spinning around.”

d. “My body felt like it was revolving and could not stop.”

ANSWER: A

A

B

C

D

Feedback

This is a description of lightheadedness, a form of dizziness.

This is consistent with objective vertigo because it includes a sensation of motion.

This is consistent with objective vertigo because it includes a sensation of motion.

This is consistent with subjective vertigo because it includes a sensation of one’s

body rotating in space.

DIF: Cognitive Level: Analyze REF: 144-145

TOP: Nursing Process: Assessment

MSC: NCLEX Patient Needs: Physiologic Integrity: Physiologic Adaptation: Alteration in Body

Systems

7. Which patient in the eye clinic should the nurse assess first?

a. The patient who reports a gradual clouding of vision

b. The patient who complains of sudden loss of vision

c. The patient who complains of double vision

d. The patient who complains of poor night vision

ANSWER: B

A

B

C

D

Feedback

A gradual clouding of vision is a symptom of cataracts that develop slowly and do

not require immediate assessment.

Sudden vision loss may indicate a detached retina and requires immediate

referral.

Double vision is a symptom of cataracts that develop slowly and do not require

immediate assessment.

Poor night vision is a symptom of cataracts that develop slowly and do not


require immediate assessment.

DIF: Cognitive Level: Apply REF: 144-145

TOP: Nursing Process: Assessment

MSC: NCLEX Patient Needs: Physiologic Integrity: Physiologic Adaptation: Alteration in Body

Systems

8. A patient complains of right ear pain. What findings does the nurse anticipate on inspecting

the patient’s ears?

a. Redness and edema of the pinna of the right ear

b. Report of pain when the nurse manipulates the right ear

c. Bulging and red tympanic membrane in the right ear

d. Increased cerumen in the right ear canal

ANSWER: C

A

B

C

D

Feedback

Redness and edema of the pinna of the right ear is consistent with external ear

pain that may be associated with otitis externa or swimmer’s ear.

Report of pain when the nurse manipulates the right ear is consistent with

external ear pain that may be associated with otitis externa or swimmer’s ear.

Bulging and red tympanic membrane in the right ear is consistent with internal

ear pain that may be associated with otitis media.

Increased cerumen in the right ear canal is not consistent with internal ear pain.

DIF: Cognitive Level: Apply REF: 145| 165

TOP: Nursing Process: Assessment

MSC: NCLEX Patient Needs: Physiologic Integrity: Physiologic Adaptation: Alteration in Body

Systems

9. During the history, a patient reports watery nasal drainage from allergies. Based on this

information, what does the nurse expect to find on inspection of the nares?

a. Enlarged and pale turbinates

b. Polyps within the nares

c. High vascularity of the turbinates

d. Dry and dull turbinates

ANSWER: A

A

B

C

D

Feedback

Enlarged and pale turbinates are expected findings for allergic rhinitis.

Polyps within the nares is not an expected finding.

High vascularity of the turbinates is not an expected finding.

Dry and dull turbinates is not an expected finding.

DIF: Cognitive Level: Apply REF: 145-146| 186

TOP: Nursing Process: Assessment

MSC: NCLEX Patient Needs: Physiologic Integrity: Physiologic Adaptation: Alteration in Body

Systems


10. A patient complains of nasal drainage and sinus headache. The nurse suspects a nasal

infection and anticipates observing which finding during examination?

a. Foul-smelling drainage

b. Purulent green-yellow drainage

c. Bloody drainage

d. Watery drainage

ANSWER: B

A

B

C

D

Feedback

Foul-smelling drainage is consistent with a foreign object in the nose.

Purulent green-yellow drainage is consistent with a nasal or sinus infection.

Bloody drainage is consistent with trauma to the nose.

Watery drainage is consistent with a nasal allergy.

DIF: Cognitive Level: Apply REF: 145

TOP: Nursing Process: Assessment

MSC: NCLEX Patient Needs: Physiologic Integrity: Physiologic Adaptation: Alteration in Body

Systems

11. A patient complains of itching, swelling, and drainage from the eyes with a postnasal drip and

sneezing. What type of nasal drainage does the nurse anticipate seeing during inspection of

this patient’s nares?

a. Clear

b. Malodorous

c. Yellow

d. Green

ANSWER: A

A

B

C

D

Feedback

The patient has allergic rhinitis, which produces clear drainage.

Malodorous drainage is associated with bacterial infection, which is not

consistent with the history given by this patient.

Yellow drainage is associated with bacterial infection, which is not consistent

with the history given by this patient.

Green drainage is associated with bacterial infection, which is not consistent with

the history given by this patient.

DIF: Cognitive Level: Apply REF: 145

TOP: Nursing Process: Assessment

MSC: NCLEX Patient Needs: Physiologic Integrity: Physiologic Adaptation: Alteration in Body

Systems

12. A patient reports a history of snorting cocaine and is concerned about his bloody nasal

drainage. What does the nurse expect to see on inspection of his nose?

a. Deviated septum

b. Pale turbinates

c. Perforated nasal septum

d. Localized erythema and edema


ANSWER: C

A

B

C

D

Feedback

Deviated septum may be from birth or trauma to the nose, but not from cocaine

use.

Pale turbinates are an indication of allergies.

Perforated nasal septum develops from cocaine use.

Localized erythema and edema are nonspecific and indicate inflammation

somewhere in the nose.

DIF: Cognitive Level: Apply REF: 170

TOP: Nursing Process: Assessment

MSC: NCLEX Patient Needs: Physiologic Integrity: Physiologic Adaptation: Alteration in Body

Systems

13. While taking a history, the nurse observes that the patient’s facial cranial nerves (CN VII) are

intact based on which behaviors of the patient?

a. The patient’s eyes move to the left, right, up, down, and obliquely during

conversation.

b. The patient moistens the lips with the tongue.

c. The sides of the mouth are symmetric when the patient smiles.

d. The patient’s eyelids blink periodically.

ANSWER: C

A

B

C

D

Feedback

This represents movement of the extraocular muscles, which are controlled by the

oculomotor, trochlear, and abducens cranial nerves (CN III, IV, and VI,

respectively).

This represents movement of the tongue, which is controlled by the hypoglossal

cranial nerve (CN XII).

This represents facial symmetry, which is controlled by the facial cranial nerve

(CN VII).

This represents function of the oculomotor cranial nerve (CN III).

DIF: Cognitive Level: Apply REF: 137| 171

TOP: Nursing Process: Assessment

MSC: NCLEX Patient Needs: Physiologic Integrity: Reduction of Risk Potential: System Specific

Assessments

14. To assess jaw movement of an adult patient, the nurse uses which technique?

a. Asking the patient to open the mouth and then passively moving the patient’s open

jaw from side to side

b. Placing two fingers in front of each ear and asking the patient to slowly open and

close the mouth

c. Asking the patient to open the mouth and to resist the nurse’s attempt to close the

mouth

d. Using the pads of all fingers to feel along the mandible for tenderness and nodules

ANSWER: B


A

B

C

D

Feedback

The patient’s jaw movement should be active, not passive.

This is the correct technique for palpating the jaw.

This technique assesses strength of the jaw, which is not typically evaluated.

Palpating under the middle of the mandible may reveal the submental lymph

node.

DIF: Cognitive Level: Apply REF: 149

TOP: Nursing Process: Assessment

MSC: NCLEX Patient Needs: Health Promotion and Maintenance: Techniques of Physical

Assessment

15. The nurse palpates the patient’s jaw movement, placing two fingers in front of each ear and

asking the patient to slowly open and close the mouth. What additional request does the nurse

ask the patient to do to assess the jaw?

a. Clinch the jaws together as tightly as possible.

b. Move the lower jaw from side to side.

c. Open the mouth as wide as possible, like a yawn.

d. Move the lower jaw forward and backward several times.

ANSWER: B

A

B

C

D

Feedback

This is not an assessment technique for the jaw.

This is the technique to complete assessment of the motion of the jaw.

This was completed when the nurse asked the patient to open and close the

mouth.

This is not an assessment technique for the jaw.

DIF: Cognitive Level: Apply REF: 149

TOP: Nursing Process: Assessment

MSC: NCLEX Patient Needs: Health Promotion and Maintenance: Techniques of Physical

Assessment

16. A patient is in a sitting position as the nurse palpates the temporal arteries and feels smooth,

bilateral pulsations. What is the appropriate action for the nurse at this time?

a. Auscultate the temporal arteries for bruits.

b. Palpate the arteries with the patient in supine position.

c. Document this as an expected finding.

d. Measure the patient’s blood pressure.

ANSWER: C

A

B

C

D

Feedback

This is not necessary for this patient at this time.

This is not necessary for this patient at this time.

These are consistent with expected assessment.

This will be done as a part of the assessment, but does not relate to the expected

palpation of this patient’s temporal arteries.

DIF: Cognitive Level: Apply REF: 149


TOP: Nursing Process: Assessment

MSC: NCLEX Patient Needs: Physiologic Integrity: Reduction of Risk Potential: System Specific

Assessments

17. What instructions does the nurse give the patient before using the Snellen visual acuity chart?

a. “Remove your eyeglasses before attempting to read the lowest line.”

b. “Stand 10 feet from the chart and read the first line aloud.”

c. “Hold a white card over one eye and read the smallest possible line.”

d. “Squint if necessary to improve the ability to read the largest letters.”

ANSWER: C

A

B

C

D

Feedback

Patients should wear their glasses when visual acuity is tested.

The patient should stand 20 feet from the Snellen chart.

This is the appropriate technique for using the Snellen chart.

The patient should not squint to see the chart.

DIF: Cognitive Level: Understand REF: 149

TOP: Nursing Process: Assessment

MSC: NCLEX Patient Needs: Health Promotion and Maintenance: Techniques of Physical

Assessment

18. Which cranial nerve is assessed by using the Snellen visual acuity chart?

a. Optic cranial nerve (CN II)

b. Oculomotor cranial nerve (CN III)

c. Abducens cranial nerve (CN IV)

d. Trochlear cranial nerve (CN VI)

ANSWER: A

A

B

C

D

Feedback

The optic cranial nerve (CN II) provides vision tested by the Snellen visual acuity

chart.

CN III controls pupillary constriction, eyelid movement, and eyeball movement.

CN IV controls eyeball movement.

CN VI controls eyeball movement.

DIF: Cognitive Level: Apply REF: 138| 150

TOP: Nursing Process: Assessment

MSC: NCLEX Patient Needs: Physiologic Integrity: Reduction of Risk Potential: System Specific

Assessments

19. Which finding on assessment of a patient’s eyes should the nurse document as abnormal?

a. An Asian American patient with an upward slant to the palpebral fissure

b. A Caucasian American patient whose sclerae are visible between the upper and

lower lids and the iris

c. An African American patient who has off-white sclerae with tiny black dots of

pigmentation near the limbus

d. An American Indian patient whose pupillary diameters are 5 mm bilaterally

ANSWER: B


A

B

C

D

Feedback

An Asian American patient with an upward slant to the palpebral fissure has an

expected racial variation.

A Caucasian American patient whose sclerae are visible between the upper and

lower lids and the iris has eyeball protrusion beyond the supraorbital ridge, which

indicates exophthalmos caused by hyperthyroidism.

An African American patient who has off-white sclerae with tiny black dots of

pigmentation near the limbus has an expected racial variation.

An American Indian whose pupils are 5 mm bilaterally is an expected finding.

DIF: Cognitive Level: Understand REF: 151

TOP: Nursing Process: Assessment

MSC: NCLEX Patient Needs: Physiologic Integrity: Physiologic Adaptation: Alteration in Body

Systems

20. A nurse shines a light toward the bridge of the patient’s nose and notices that the light

reflection in the right cornea is at the 9 o’clock position and in the left cornea at the 9 o’clock

position. What is the interpretation of this finding?

a. The extraocular muscles of both eyes are intact.

b. The cornea of each eye is transparent.

c. The sclera of each eye is clear.

d. The consensual reaction of both eyes is intact.

ANSWER: A

A

B

C

D

Feedback

The reflection of the light in both eyes in the same location indicates muscles

holding the eyes are symmetric.

The reflection of the light in both eyes in the same location indicates muscles

holding the eyes are symmetric.

The reflection of the light in both eyes in the same location indicates muscles

holding the eyes are symmetric.

Consensual reaction involves constriction of pupils.

DIF: Cognitive Level: Apply REF: 153

TOP: Nursing Process: Assessment

MSC: NCLEX Patient Needs: Physiologic Integrity: Physiologic Adaptation: Alteration in Body

Systems

21. How does a nurse assess movements of the eyes?

a. By assessing peripheral vision

b. By noting the symmetry of the corneal light reflex

c. By assessing the cardinal fields of gaze

d. By performing the cover-uncover test

ANSWER: C

A

B

Feedback

This tests the function of cranial nerve I (optic).

This indicates symmetry of eye muscles.


C

D

This tests the movement of the eye in all directions, which assesses the functions

of the cranial nerves III (oculomotor), IV (abducens), and VI (trochlear).

This is performed after the corneal light reflex is abnormal, indicating asymmetric

eye muscles.

DIF: Cognitive Level: Understand REF: 154-156

TOP: Nursing Process: Assessment

MSC: NCLEX Patient Needs: Health Promotion and Maintenance: Techniques of Physical

Assessment

22. On inspection of the external eye structures of an African American patient, the nurse notices

the sclerae are not white, but appear a darker shade with tiny black dots of pigmentation near

the limbus. How does the nurse document this finding?

a. As an indication of a type of anemia

b. As a hordeolum or sty

c. As jaundice

d. As an expected racial variation

ANSWER: D

A

B

C

D

Feedback

This may cause a pale conjunctiva.

This is an acute infection originating in the sebaceous gland of the eyelid.

Jaundice is a yellow color of the sclera associated with liver or gallbladder

disease.

This as an expected racial variation.

DIF: Cognitive Level: Remember REF: 153

TOP: Nursing Process: Assessment

MSC: NCLEX Patient Needs: Physiologic Integrity: Reduction of Risk Potential: System Specific

Assessments

23. A nurse shines a light in the right pupil to test constriction and notices that the left pupil

constricts as well. Based on these data, the nurse should take what action?

a. Document this finding as an abnormal finding.

b. Assess the patient for accommodation.

c. Document this finding as a consensual reaction.

d. Assess the patient’s corneal light reflex.

ANSWER: C

A

B

C

D

Feedback

This is a description of an expected finding—consensual reaction.

Accommodation is not assessed in response to consensual reaction; it tests the

function of the oculomotor cranial nerve (CN III).

This is a description of expected consensual reaction.

This item describes a consensual reaction rather than a corneal light reflex.

DIF: Cognitive Level: Apply REF: 154

TOP: Nursing Process: Assessment

MSC: NCLEX Patient Needs: Physiologic Integrity: Reduction of Risk Potential: System Specific


Assessments

24. When inspecting a patient’s eyes, the nurse assesses the presence of cranial nerve III

(oculomotor nerve) by observing the eyelids open and close bilaterally. What other technique

does a nurse use to test the function of this cranial nerve?

a. Pupillary constriction to light

b. Visual acuity

c. Peripheral vision

d. Presence of the red reflex

ANSWER: A

A

B

C

D

Feedback

Cranial nerve III (oculomotor) controls pupillary dilation and constriction, as well

as eyelid movement. Pupil dilation and ptosis may occur when CN III is

impaired.

Cranial nerve II (optic) provides vision.

Cranial nerve II (optic) provides peripheral vision.

The red reflex is not controlled by cranial nerve III, but is created by a light

illuminating the retina.

DIF: Cognitive Level: Apply REF: 154

TOP: Nursing Process: Assessment

MSC: NCLEX Patient Needs: Physiologic Integrity: Reduction of Risk Potential: System Specific

Assessments

25. How does a nurse recognize normal accommodation?

a. The patient has peripheral vision of 90 degrees left and right.

b. The patient’s eyes move up and down, side to side, and obliquely.

c. The right pupil constricts when a light is shown in the left pupil.

d. The patient’s pupils dilate when looking toward a distant object.

ANSWER: D

Feedback

A Normally a patient has 90 degrees peripheral vision temporally, but only 60

degrees nasally.

B This is an expected finding, but is not a test for accommodation. It is a test of

extraocular muscle function in the six cardinal fields of gaze.

C This is an expected finding for consensual reaction, rather than accommodation.

D This is an indication of accommodation.

DIF: Cognitive Level: Understand REF: 154

TOP: Nursing Process: Assessment

MSC: NCLEX Patient Needs: Physiologic Integrity: Reduction of Risk Potential: System Specific

Assessments

26. How does a nurse recognize a patient’s mydriasis?

a. The lens of each of the patient’s eyes is opaque.

b. There is involuntary rhythmical, horizontal movement of the patient’s eyes.

c. There is a white opaque ring encircling the patient’s limbus.


d. The patient’s pupils are 7 mm and do not constrict.

ANSWER: D

A

B

C

D

Feedback

An opaque lens is an abnormality that occurs when cataracts are present.

An involuntary rhythmical, horizontal movement of the patient’s eyes is a

description of nystagmus.

A white opaque ring encircling the patient’s limbus is a description of corneal

arcus seen in patients older than 60 years of age.

Mydriasis is pupil size greater than 6 mm and the pupil fails to constrict.

DIF: Cognitive Level: Understand REF: 154

TOP: Nursing Process: Assessment

MSC: NCLEX Patient Needs: Physiologic Integrity: Physiologic Adaptation: Alteration in Body

Systems

27. A nurse uses which technique to assess a patient’s peripheral vision?

a. The nurse asks the patient to keep the head still and by moving the eyes only,

follow the nurse’s finger as it moves side to side, up and down, and obliquely.

b. The nurse covers one of the patient’s eyes with a card and observes the uncovered

eye for movement, then removes the card and observes the just uncovered eye for

movement.

c. With the patient and nurse facing each other and a card covering their

corresponding eyes, the nurse moves an object into the visual field and the patient

reports when the object is seen.

d. The nurse shines a light on both corneas at the same time and notes the location of

the reflection in each eye.

ANSWER: C

A

B

C

D

Feedback

This technique tests extraocular muscle symmetry.

This cover-uncover technique is performed when the corneal light reflex is

asymmetric.

This is the confrontation test that tests peripheral vision.

This describes the corneal light reflex that tests the symmetry of the eye muscles.

DIF: Cognitive Level: Apply REF: 150

TOP: Nursing Process: Assessment

MSC: NCLEX Patient Needs: Health Promotion and Maintenance: Techniques of Physical

Assessment

28. During an eye assessment, a nurse asks the patient to cover one eye with a card as the nurse

covers his or her eye directly opposite the patient’s covered eye. The nurse moves an object

into the field of vision and asks the patient to tell when the object can be seen. This

assessment technique collects what data about the patient’s eyes?

a. Symmetry of extraocular muscles

b. Visual acuity in the uncovered eye

c. Peripheral vision of the uncovered eye

d. Consensual reaction of the uncovered eye


ANSWER: C

A

B

C

D

Feedback

Symmetry is tested by the corneal light reflex.

Visual acuity is tested using the Snellen chart.

This describes the confrontation test, which assesses peripheral vision.

Consensual reaction is tested by noticing the pupillary constriction of one eye

when a light is being shown into the other eye.

DIF: Cognitive Level: Apply REF: 156

TOP: Nursing Process: Assessment

MSC: NCLEX Patient Needs: Health Promotion and Maintenance: Techniques of Physical

Assessment

29. During an eye examination of an Asian patient, a nurse notices an involuntary rhythmical,

horizontal movement of the patient’s eyes. How does a nurse document this finding?

a. An expected racial variation

b. Nystagmus

c. Exophthalmus

d. Myopia

ANSWER: B

A

B

C

D

Feedback

This is not a racial variation.

An involuntary rhythmical, horizontal movement of the patient’s eyes is a

description of nystagmus.

Exophthalmus is the bulging of the eyeball forward, seen in patients with

hyperthyroidism.

Myopia is an elongated eyeball found in patients who are nearsighted.

DIF: Cognitive Level: Understand REF: 156

TOP: Nursing Process: Assessment

MSC: NCLEX Patient Needs: Physiologic Integrity: Physiologic Adaptation: Alteration in Body

Systems

30. A nurse shines a light toward the bridge of the patient’s nose and notices that the light

reflection in the right cornea is at the 2 o’clock position and in the left cornea at the 10 o’clock

position. Based on these data, the nurse should take what action?

a. Document these findings as normal.

b. Perform the cover-uncover test.

c. Perform the confrontation test.

d. Document these findings as abnormal.

ANSWER: B

A

B

Feedback

The findings are abnormal. The light should appear in the same location in each

cornea.

The nurse is performing the corneal light reflex test and the findings are

abnormal. Thus, when the corneal light reflex is asymmetric, the cover-uncover


C

D

test is performed to determine which eye has the weak extraocular muscle(s).

The confrontation test is used to assess peripheral visual fields and is not

appropriate to perform when the corneal light reflex is asymmetric.

The asymmetric corneal light reflex is abnormal, but the cover-uncover test

should follow the abnormal finding to determine which eye has the weak

extraocular muscle(s).

DIF: Cognitive Level: Analyze REF: 153

TOP: Nursing Process: Assessment

MSC: NCLEX Patient Needs: Physiologic Integrity: Physiologic Adaptation: Alteration in Body

Systems

31. During the history, a patient reports blurred vision, seeing double at times, and a glare from

headlights from oncoming cars at night. Based on this information, what finding does the

nurse expect to find on assessment of this patient’s eyes?

a. Anterior chamber depth is shallow.

b. Red reflex is absent.

c. Extraocular muscle movement is asymmetric.

d. Retinal arteries are wider than retinal veins.

ANSWER: B

A

B

C

D

Feedback

Shallow anterior chamber depth occurs in glaucoma.

The symptoms suggest cataracts. The red reflex cannot be seen because the light

cannot penetrate the opacity of the lens.

Extraocular muscle movement is asymmetric. Cataracts affect the lens rather than

the eye muscles.

Retinal arteries are wider than retinal veins. Cataracts affect the lens rather than

the retinal vessels.

DIF: Cognitive Level: Analyze REF: 158| 184

TOP: Nursing Process: Assessment

MSC: NCLEX Patient Needs: Physiologic Integrity: Physiologic Adaptation: Alteration in Body

Systems

32. What changes in using the ophthalmoscope should the nurse need to make when inspecting

the eye of a patient who is nearsighted?

a. Holding the ophthalmoscope in the right hand when inspecting the patient’s right

eye

b. Using the grid light of the lens aperture to visualize the internal structures of the

eye

c. Rotating the diopter to the red (minus) numbers

d. Asking the patient to look directly into the ophthalmoscope light

ANSWER: C

A

B

C

Feedback

This procedure is performed with all patients having an internal eye examination.

The grid is used to estimate the size of lesions.

This compensates for the longer eyeball of a myopic patient.


D

This is an instruction given to the patient to visualize the macula.

DIF: Cognitive Level: Apply REF: 158

TOP: Nursing Process: Assessment

MSC: NCLEX Patient Needs: Health Promotion and Maintenance: Techniques of Physical

Assessment

33. After seeing the red reflex and retinal vessels through the ophthalmoscope, how does the

nurse locate the optic disc?

a. By rotating the diopter to the block (positive) numbers until the optic disc comes

into focus

b. By following the retinal vessels inward toward the nose until optic disc is seen

c. By using the green beam light while looking outward toward the ear until the disc

is seen

d. By locating the macula and then looking temporally (toward the ear) until the disc

is seen

ANSWER: B

A

B

C

D

Feedback

This procedure is used for patients who are myopic.

This procedure locates the optic disc.

The green beam is used to identify retinal hemorrhages.

The macula lies temporal to the optic disc; thus the optic disc is in the opposite

direction.

DIF: Cognitive Level: Apply REF: 158-159

TOP: Nursing Process: Assessment

MSC: NCLEX Patient Needs: Health Promotion and Maintenance: Techniques of Physical

Assessment

34. When using an ophthalmoscope to examine the internal eye, how does the nurse distinguish

the retinal arteries from the retinal veins?

a. The arteries are narrower than veins.

b. The arteries are a darker red than veins.

c. The arteries have no light reflex and the veins have a narrow band of light in the

center.

d. The arteries have prominent pulsations and veins have no pulsations.

ANSWER: A

Feedback

A The artery-to-vein width should be 2:3 to 4:5.

B Arteries are lighter red than veins.

C Arteries have a narrow band of light in the center and veins have no light reflex.

D Arteries show little to no pulsations and venous pulsations may be visible.

DIF: Cognitive Level: Understand REF: 159

TOP: Nursing Process: Assessment

MSC: NCLEX Patient Needs: Health Promotion and Maintenance: Techniques of Physical

Assessment


35. Which finding warrants a referral for additional evaluation?

a. Earlobes hanging freely from the base of the pinna

b. Ears having painless nodules less than 1 cm in diameter at the helix

c. Ears measuring 8 cm in length

d. Pinna is 20 degrees lower than the outer canthus of the eye

ANSWER: D

A

B

C

D

Feedback

Earlobes hanging freely from the base of the pinna is an expected finding.

This is called a Darwin tubercle. It is a normal deviation and may be noted at the

helix of the ear.

A length of 8 cm is an expected finding.

The pinna of the ear should align directly with the outer canthus of the eye and be

angled no more than 10 degrees from a vertical position.

DIF: Cognitive Level: Apply REF: 160

TOP: Nursing Process: Assessment

MSC: NCLEX Patient Needs: Physiologic Integrity: Physiologic Adaptation: Alteration in Body

Systems

36. A nurse is assessing a patient who was hit at the base of the skull with a blunt instrument

causing a skull fracture. What assessment finding does this nurse anticipate during the

inspection?

a. Tinnitus, vertigo, and dizziness

b. Clear drainage from the ear and nose

c. Loss of hearing and smell

d. Purulent drainage from the ear and bloody drainage from the nose

ANSWER: B

A

B

C

D

Feedback

These are subjective and gathered during the history rather than inspection.

Although the patient may report having dizziness or vertigo, the finding of

tinnitus is inconsistent with a basilar skull fracture.

This may occur after a basilar skull fracture. The clear drainage may be

cerebrospinal fluid.

This is inconsistent with a basilar skull fracture.

Purulent drainage is inconsistent with a basilar skull fracture, and bloody drainage

usually does not come from the nose, but may be seen from the ear.

DIF: Cognitive Level: Analyze REF: 162

TOP: Nursing Process: Assessment

MSC: NCLEX Patient Needs: Physiologic Integrity: Physiologic Adaptation: Alteration in Body

Systems

37. An adult patient comes to the clinic complaining of right ear pain. What technique does the

nurse use to inspect this patient’s auditory canal?

a. Position the otoscope speculum 1.0 to 1.5 cm (about 0.5 inches) into the ear canal.

b. Remove cerumen from each canal before inserting otoscope.


c. Choose the smallest otoscope speculum that will fit the patient’s ear comfortably.

d. Pull the pinna slightly downward and backward before inserting the otoscope

speculum.

ANSWER: A

A

B

C

D

Feedback

This is the correct technique.

Removing cerumen is not necessary.

The largest speculum that comfortably fits in the ear canal is the one that should

be chosen.

For adults, the pinna is pulled up and backward to straighten the ear canal.

DIF: Cognitive Level: Apply REF: 162

TOP: Nursing Process: Assessment

MSC: NCLEX Patient Needs: Health Promotion and Maintenance: Techniques of Physical

Assessment

38. A nurse examines a patient’s auditory canal and tympanic membrane with an otoscope and

observes which finding as normal?

a. Clear fluid lining the auditory canal

b. A firm tympanic membrane without fluctuation with puffs of air

c. A small hole within the cone of light

d. A shiny, translucent tympanic membrane

ANSWER: D

A

B

C

D

Feedback

Clear fluid or bloody drainage following a head injury may indicate a basilar

skull fracture.

An expected response is that the tympanic membrane slightly fluctuates with

puffs of air.

A cone of light is expected, but a hole indicates perforation.

A shiny, translucent tympanic membrane is an expected finding.

DIF: Cognitive Level: Apply REF: 164

TOP: Nursing Process: Assessment

MSC: NCLEX Patient Needs: Physiologic Integrity: Physiologic Adaptation: Alteration in Body

Systems

39. A nurse observes a student using the whisper test to screen a patient with hearing loss. Which

behavior by the student requires a corrective comment from the nurse?

a. Instructing the patient to cover the ear not being tested

b. Standing beside the patient on the side of the ear being tested

c. Shielding the mouth to prevent the patient from reading lips

d. Whispering one or two syllable words and ask the patient to repeat what is heard

ANSWER: B

A

B

Feedback

This is the correct technique.

The student nurse should stand 1 to 2 feet in front or to the side of the patient.


C

D

This is the correct technique.

This is the correct technique.

DIF: Cognitive Level: Apply REF: 165

TOP: Nursing Process: Assessment

MSC: NCLEX Patient Needs: Health Promotion and Maintenance: Techniques of Physical

Assessment

40. A nurse reads in the history that a patient has a new onset of acute otitis media. Based on this

information, how does the nurse expect this patient’s tympanic membrane to appear?

a. Dull

b. Shiny

c. Red

d. Blue to deep red

ANSWER: C

A

B

C

D

Feedback

This indicates fibrosis or scarring.

This is normal for the tympanic membrane.

This indicates infection in the middle ear, such as otitis media.

This indicates blood behind the tympanic membrane, which may have occurred

secondary to injury.

DIF: Cognitive Level: Apply REF: 145| 163

TOP: Nursing Process: Assessment

MSC: NCLEX Patient Needs: Physiologic Integrity: Physiologic Adaptation: Alteration in Body

Systems

41. During the Rinne test, a nurse determines that the patient hears the tuning fork held on the

mastoid process for 15 seconds and hears the tuning fork held in front of the ear for 30

seconds. The same results are found in both ears. Based on this finding, what is the most

appropriate response of the nurse?

a. Repeat the test again using a 2000 Hz tuning fork.

b. Tell the patient that this represents an expected finding.

c. Refer the patient for additional testing to detect hearing abnormality.

d. Perform a Weber test to confirm the findings of the Rinne test.

ANSWER: B

A

B

C

D

Feedback

This is unnecessary because the finding of the Rinne test was normal.

This is a normal finding. Air conduction (30 seconds) is twice as long as bone

conduction (15 seconds).

This is unnecessary because the finding of the Rinne test was normal.

This is unnecessary because the finding of the Rinne test was normal.

DIF: Cognitive Level: Analyze REF: 166

TOP: Nursing Process: Assessment

MSC: NCLEX Patient Needs: Physiologic Integrity: Reduction of Risk Potential: System Specific

Assessments


42. During a Weber test, a patient with right ear hearing loss reports hearing sound longer in the

right ear than the left ear. What results should the nurse expect to find from this patient during

a Rinne test?

a. Air conduction will be twice as long as bone conduction (2:1 ratio).

b. Air conduction will be 1.5 times as long as bone conduction (1.5:1 ratio).

c. Bone conduction will be longer than air conduction.

d. Bone conduction will be equal to air conduction.

ANSWER: C

A

B

C

D

Feedback

This is an expected finding.

This finding is consistent with a sensorineural hearing loss, but this patient has a

conduction hearing loss based on the results of the Weber test.

This finding from the Rinne test indicates a conduction hearing loss, which is

consistent with the finding from the Weber test described in the question.

This finding is not consistent with the conductive hearing loss described.

DIF: Cognitive Level: Analyze REF: 166

TOP: Nursing Process: Assessment

MSC: NCLEX Patient Needs: Physiologic Integrity: Physiologic Adaptation: Alteration in Body

Systems

43. Which finding indicates that this patient has a sensorineural hearing loss?

a. The patient hears sound by air conduction longer than by bone conduction.

b. The patient hears sound from a vibrating tuning fork in the affected ear only.

c. The patient hears normal conversation at 40 dB and a whisper at 20 dB.

d. The patient hears the rubbing of fingers together from a distance of 4 inches from

each ear.

ANSWER: A

A

B

C

D

Feedback

In the Rinne test, hearing sound from a vibrating tuning fork longer by air

conduction than by bone conduction is consistent with a sensorineural hearing

loss.

This finding from the Weber test is consistent with a conduction hearing loss.

This is an expected finding using audiometry.

This is an expected finding using the finger rubbing screening hearing test.

DIF: Cognitive Level: Apply REF: 166

TOP: Nursing Process: Assessment

MSC: NCLEX Patient Needs: Physiologic Integrity: Physiologic Adaptation: Alteration in Body

Systems

44. How does the nurse perform a Weber test to assess hearing function?

a. Whispers three to four words into the patient’s ear and asks him to repeat the words

heard

b. Places a vibrating tuning fork in the middle of the head and asks the patient if the

sound is heard the same in both ears


c. Places a set of headphones over both ears, plays several tones, and asks the patient

to identify the sounds

d. Places a vibrating tuning fork on the mastoid process and asks the patient to signal

when he can no longer hear the sound

ANSWER: B

A

B

C

D

Feedback

This technique describes the whisper test.

This technique describes the Weber test.

This technique describes the use of an audiometer.

This technique describes part of the Rinne test.

DIF: Cognitive Level: Apply REF: 166

TOP: Nursing Process: Assessment

MSC: NCLEX Patient Needs: Health Promotion and Maintenance: Techniques of Physical

Assessment

45. How does the nurse perform a Rinne test of hearing function?

a. Whispers several words to the patient and requests that the patient repeat the words

heard

b. Places a vibrating tuning fork in the middle of the head and asks the patient if the

sound is heard the same in both ears or if it is louder in one ear than the other

c. Places a set of headphones over both ears, plays several tones, and asks the patient

to identify the sounds

d. Places a vibrating tuning fork on the mastoid process until the patient no longer

hears it, and then moves it in front of the ear until the patient no longer hears it

ANSWER: D

A

B

C

D

Feedback

This technique describes the whisper test.

This technique describes the Weber test.

This technique describes the use of an audiometer.

This technique describes the Rinne test.

DIF: Cognitive Level: Apply REF: 167

TOP: Nursing Process: Assessment

MSC: NCLEX Patient Needs: Health Promotion and Maintenance: Techniques of Physical

Assessment

46. A nurse assessing the hearing of a patient with presbycusis expects which finding on a test for

hearing?

a. Bone conduction will be longer than air conduction on the Rinne test (BC > AC).

b. Air conduction will be longer than bone conduction on the Rinne test (AC > BC).

c. Sound lateralizes to the affected ear on the Weber test.

d. Sound lateralizes to both ears equally on the Weber test.

ANSWER: B

A

Feedback

This finding indicates a conduction hearing loss, rather than a sensorineural


B

C

D

hearing loss.

This finding indicates a sensorineural health loss, the most common cause of

presbycusis.

This finding indicates a conduction hearing loss, rather than a sensorineural

hearing loss.

This is a normal finding on this test.

DIF: Cognitive Level: Analyze REF: 167-168

TOP: Nursing Process: Assessment

MSC: NCLEX Patient Needs: Physiologic Integrity: Physiologic Adaptation: Alteration in Body

Systems

47. While taking a history, the nurse notices that the patient’s family member repeats most of the

questions to the patient in a loud voice. Based on this information, what finding does the nurse

anticipate when assessing this patient’s hearing using an audioscope?

a. 5 dB hearing loss at all frequencies

b. 10 dB hearing loss at all frequencies

c. 20 dB hearing loss at all frequencies

d. 40 dB hearing loss at all frequencies

ANSWER: D

Feedback

A This decibel level is not tested by an audioscope .

B A 10 dB loss in high frequencies results in difficulty hearing quiet sounds, such as

a heartbeat.

C A 20 dB loss in high frequencies results in difficulty hearing high-pitched

consonants, such as a whisper.

D A 40 dB loss in all frequencies causes moderate difficulty in hearing normal

speech.

DIF: Cognitive Level: Apply REF: 168

TOP: Nursing Process: Assessment

MSC: NCLEX Patient Needs: Physiologic Integrity: Physiologic Adaptation: Alteration in Body

Systems

48. A patient is being seen in the clinic for suspected nasal obstruction from a foreign body. The

nurse recognizes which finding as most consistent with this diagnosis?

a. Unilateral foul-smelling drainage

b. Bilateral purulent green-yellow discharge

c. Bilateral bloody discharge

d. Unilateral watery discharge

ANSWER: A

A

B

C

D

Feedback

This is consistent with presence of a foreign object in one side of the nose.

This is consistent with a nasal or sinus infection.

This is consistent with localized trauma, such as a nasal fracture.

This is consistent with a history of head injury and may indicate skull fracture.


DIF: Cognitive Level: Apply REF: 162

TOP: Nursing Process: Assessment

MSC: NCLEX Patient Needs: Physiologic Integrity: Physiologic Adaptation: Alteration in Body

Systems

49. In assessing a patient with head injury, the nurse should be most concerned with which

finding?

a. Pain on palpation of the scalp

b. Unilateral clear, watery nasal discharge

c. A scalp laceration at the sight of injury

d. Complaints of dizziness

ANSWER: B

A

B

C

D

Feedback

This is expected after a head injury and is not a cause for concern.

This may be cerebrospinal fluid, indicating a skull fracture.

This is expected after a head injury and is not a cause for concern.

This is expected after a head injury and is not a cause for concern.

DIF: Cognitive Level: Understand REF: 162

TOP: Nursing Process: Assessment

MSC: NCLEX Patient Needs: Physiologic Integrity: Physiologic Adaptation: Alteration in Body

Systems

50. A patient complains of a lesion in his nose. Which technique does a nurse use to inspect the

nasal mucosa?

a. Inserts a nasal speculum horizontally into the patient’s affected nares

b. Inserts a nasal speculum obliquely into the patient’s affected nares

c. Uses a light source from the ophthalmoscope

d. Inserts a nasal speculum vertically into the patient’s affected nares

ANSWER: B

A

B

C

D

Feedback

Horizontal insertion puts pressure on the nasal septum, which is painful.

This is the appropriate technique for inspecting the nares.

The alternate light source is from an otoscope, rather than an ophthalmoscope.

The otoscope has an ear speculum that can be used when a nasal speculum is

unavailable.

Vertical insertion obstructs the nurse’s view of the internal nares.

DIF: Cognitive Level: Understand REF: 169-170

TOP: Nursing Process: Assessment

MSC: NCLEX Patient Needs: Health Promotion and Maintenance: Techniques of Physical

Assessment

51. When inspecting a patient’s nasal mucous membrane, which finding does the nurse expect to

see?

a. Deep pink turbinates

b. Red, edematous mucous membranes


c. Septum that angles to the left

d. Clear exudate

ANSWER: A

A

B

C

D

Feedback

These are expected for a nasal inspection.

These indicate a local infection within the nose.

This is abnormal.

This occurs with nasal allergies.

DIF: Cognitive Level: Understand REF: 170

TOP: Nursing Process: Assessment

MSC: NCLEX Patient Needs: Physiologic Integrity: Reduction of Risk Potential: System Specific

Assessments

52. A patient comes to the clinic for evaluation after a sinus infection. To evaluate the therapy, the

nurse uses transillumination to assess the sinuses and notes which finding indicating recovery

from a frontal sinus infection?

a. The soft palate illuminates brightly when the light source is placed against the

lateral nose.

b. No illumination is noted when the light source is placed firmly against the lateral

nose.

c. A bright glow illuminates the hard palate when the light source is placed against

each temporal bone.

d. A reddish light is noted above the eyebrows when the light is placed against each

supraorbital rim.

ANSWER: D

A

B

C

D

Feedback

This describes incorrect technique for transillumination.

An absence of a glow during transillumination of the sinuses may indicate that

the sinuses are congested.

This describes incorrect technique for transillumination.

Finding a reddish light above the eyebrows when the light is placed against each

supraorbital rim is consistent with frontal sinuses free of infection.

DIF: Cognitive Level: Apply REF: 171

TOP: Nursing Process: Assessment

MSC: NCLEX Patient Needs: Physiologic Integrity: Reduction of Risk Potential: System Specific

Assessments

53. A nurse suspects the patient has an infection of the maxillary sinuses. How can this suspicion

be confirmed?

a. Using a flashlight to illuminate the floor of the mouth

b. Pressing gently with both thumbs into the eyebrow ridges

c. Applying firm pressure with the thumbs below the cheekbones

d. Standing behind the patient and asking him or her to slowly rotate the head

ANSWER: C


A

B

C

D

Feedback

To transilluminate the maxillary sinuses, the nurse places the source of light

lateral to the nose, just beneath the medial aspect of the eye, and looks through

the patient’s open mouth for illumination of the hard palate.

This palpates the frontal sinuses rather than the maxillary sinuses.

This palpates the maxillary sinuses to detect tenderness, which may indicate sinus

congestion or infection.

This is not a correct technique to confirm infection of the maxillary sinuses.

DIF: Cognitive Level: Apply REF: 170

TOP: Nursing Process: Assessment

MSC: NCLEX Patient Needs: Health Promotion and Maintenance: Techniques of Physical

Assessment

54. After assessment of the nose and paranasal sinuses, which finding requires further

investigation by the nurse?

a. Nasal septum off the midline

b. Nose in the midline of the face

c. Middle turbinates deep pink in color

d. Noiseless exchange of air from each naris

ANSWER: A

A

B

C

D

Feedback

A deviated septum is an abnormal finding that needs further investigation.

This is an expected finding.

This is a normal finding.

This is a normal finding.

DIF: Cognitive Level: Understand REF: 170

TOP: Nursing Process: Assessment

MSC: NCLEX Patient Needs: Physiologic Integrity: Physiologic Adaptation: Alteration in Body

Systems

55. When inspecting a patient’s posterior wall of the pharynx and tonsils, a nurse documents

which finding as abnormal?

a. Both tonsils have a smooth surface.

b. Left and right tonsils meet at the midline.

c. Left and right tonsils extend beyond the posterior pillars.

d. Both tonsils have a glistening appearance.

ANSWER: B

Feedback

A A smooth surface is expected for the tonsils.

B This indicates an enlargement documented as 4+.

C This is an expected finding for the tonsils.

D This is an expected finding for the tonsils.

DIF: Cognitive Level: Apply REF: 174


TOP: Nursing Process: Assessment

MSC: NCLEX Patient Needs: Physiologic Integrity: Physiologic Adaptation: Alteration in Body

Systems

56. Wearing gloves, the nurse grasps the patient’s tongue with a gauze pad and palpates a small,

firm nodule on the left side of the tongue. Based upon this finding, what is the nurse’s

appropriate response?

a. Document that the patient’s tongue is normal on palpation.

b. Inspect the left submandibular salivary glands for redness.

c. Ask the patient to move the tongue in all directions.

d. Palpate cervical and submental lymph nodes for enlargement.

ANSWER: D

A

B

C

D

Feedback

The nodule is not an expected finding.

The salivary glands are not affected by a nodule of the tongue.

This assesses the hypoglossal cranial nerve or movement of the tongue, which is

not related to the nodule found.

The nodules may indicate a malignancy of the tongue, which may also cause

enlarged cervical or submental lymph nodes.

DIF: Cognitive Level: Analyze REF: 175

TOP: Nursing Process: Assessment

MSC: NCLEX Patient Needs: Physiologic Integrity: Physiologic Adaptation: Alteration in Body

Systems

57. A nurse assesses neck range of movement of several adults. Which patient has an expected

range of motion of the neck?

a. Patient A is unable to resist the nurse’s attempt to move the head upright.

b. Patient B bends the head to the right and left (ear to shoulder) 15 degrees.

c. Patient C flexes chin toward the chest 45 degrees.

d. Patient D hyperextends the head 30 degrees from midline.

ANSWER: C

Feedback

A This finding is abnormal.

B This finding is abnormal. The patient should be able to laterally bend the head 40

degrees from midline in each direction.

C This is an expected finding.

D The patient should be able to hyperextend the head 55 degrees from midline.

DIF: Cognitive Level: Understand REF: 176

TOP: Nursing Process: Assessment

MSC: NCLEX Patient Needs: Physiologic Integrity: Reduction of Risk Potential: System Specific

Assessments

58. What technique does a nurse use when palpating the right lobe of a patient’s thyroid gland

using the anterior approach?

a. Pushes the cricoid process to the left with the right thumb


b. Displaces the trachea to the right with the left thumb

c. Manipulates the thyroid between the thumb and index finger

d. Moves the sternocleidomastoid muscle to the right with the left thumb

ANSWER: B

A

B

C

D

Feedback

This is not a correct technique.

This is the correct technique for palpating the thyroid gland using the anterior

approach.

This is not a correct technique.

This is not a correct technique.

DIF: Cognitive Level: Apply REF: 177

TOP: Nursing Process: Assessment

MSC: NCLEX Patient Needs: Health Promotion and Maintenance: Techniques of Physical

Assessment

59. What technique does a nurse use when palpating the right lobe of a patient’s thyroid gland

using the posterior approach?

a. Pushes the cricoid process to the left with the right thumb and feels the right lobe

with the left hand

b. Uses the left hand to push the sternocleidomastoid muscle to the right and feels the

lobe with the right hand

c. Pushes the trachea to the right with the left hand and feels the right lobe with the

right hand

d. Places the fingers on either side of the trachea above the cricoid cartilage and feels

the right lobe

ANSWER: C

A

B

C

D

Feedback

This description is not a correct technique. In the posterior approach, the right

lobe is felt with the right hand.

This description is not a correct technique. The trachea, not the muscle, is moved

to the side.

This is the correct technique.

This description is not a correct technique. The fingers are placed below the

cricoid cartilage.

DIF: Cognitive Level: Apply REF: 177

TOP: Nursing Process: Assessment

MSC: NCLEX Patient Needs: Health Promotion and Maintenance: Techniques of Physical

Assessment

60. What instruction does a nurse give a patient to facilitate palpation of the right lobe of the

thyroid gland?

a. “Swallow for me one time.”

b. “Flex your head down and to the left.”

c. “Rotate your head to the right for me.”

d. “Hold your breath for a few seconds.”


ANSWER: A

A

B

C

D

Feedback

The patient is asked to swallow to make the thyroid lobe easier to palpate.

This is incorrect to palpate the right lobe. The patient flexes the neck toward the

side being palpated.

This is incorrect to palpate the right lobe. The patient flexes the neck toward the

side being palpated, but does not rotate the head.

This is not part of the thyroid palpation.

DIF: Cognitive Level: Apply REF: 177

TOP: Nursing Process: Assessment

MSC: NCLEX Patient Needs: Health Promotion and Maintenance: Techniques of Physical

Assessment

61. When palpating the right lobe of the patient’s thyroid gland using the anterior approach, the

nurse feels the tissue between which two structures?

a. Sternocleidomastoid and the trapezius muscles

b. Trapezius muscle and the trachea

c. Cricoid process and the trachea

d. Sternocleidomastoid muscle and the trachea

ANSWER: D

A

B

C

D

Feedback

This is not the correct location for palpating the thyroid gland using the anterior

approach.

This is not the correct location for palpating the thyroid gland using the anterior

approach.

This is not the correct location for palpating the thyroid gland using the anterior

approach.

This is the correct location for palpating the thyroid gland using the anterior

approach.

DIF: Cognitive Level: Apply REF: 177

TOP: Nursing Process: Assessment

MSC: NCLEX Patient Needs: Health Promotion and Maintenance: Techniques of Physical

Assessment

62. On palpation the nurse determines that the patient’s left thyroid lobe is larger than the right

thyroid lobe. What is the nurse’s most appropriate action at this time?

a. Refer the patient to the health care provider for further evaluation.

b. Document that the patient’s thyroid is normal on palpation.

c. Palpate the left thyroid lobe again using very firm pressure.

d. Ask the patient to flex the chin toward his chest and palpate again.

ANSWER: A

A

B

Feedback

The nurse found an abnormality that needs referral for follow-up.

This is not an appropriate action because the nurse found an abnormality.


C

D

Repeating the examination will yield the same abnormal finding.

Repeating the examination will yield the same abnormal finding.

DIF: Cognitive Level: Analyze REF: 178

TOP: Nursing Process: Assessment

MSC: NCLEX Patient Needs: Physiologic Integrity: Physiologic Adaptation: Alteration in Body

Systems

63. A teenager comes to the clinic complaining about the whiteheads and blackhead on his face

interfering with his social life. During the examination the nurse palpates an enlarged

submental lymph node. Where is this lymph node located?

a. In front of the ear

b. Under the mandible

c. At the base of the skull

d. Along the angle of the jaw

ANSWER: B

A

B

C

D

Feedback

This is the location of the preauricular lymph nodes.

This is the location of the submental lymph node.

This is the location of the occipital lymph nodes.

This is the location of the parotid lymph nodes.

DIF: Cognitive Level: Apply REF: 178

TOP: Nursing Process: Assessment

MSC: NCLEX Patient Needs: Health Promotion and Maintenance: Techniques of Physical

Assessment

64. How does the nurse test the function of the patient’s spinal accessory nerve (CN XI)?

a. Ask the patient to stick out the tongue and move it side to side.

b. Ask the patient to shrug the shoulders against the resistance of the nurse’s hands.

c. Ask the patient to open the mouth and observe the uvula rise when he says “ah.”

d. Ask the patient to move the chin to the chest and then up toward the ceiling.

ANSWER: B

A

B

C

D

Feedback

This is a test of the hypoglossal cranial nerve (XII).

This is the correct technique for assessing the spinal accessory cranial nerve (XI).

This is a test for cranial nerves IX (glossopharyngeal) and X (vagus).

This technique assesses the range motion of the neck.

DIF: Cognitive Level: Apply REF: 176

TOP: Nursing Process: Assessment

MSC: NCLEX Patient Needs: Health Promotion and Maintenance: Techniques of Physical

Assessment

65. A patient complains of sore throat, pain with swallowing, fever, and chills. The nurse suspects

tonsillitis and plans to palpate the anterior cervical lymph nodes. Where does the nurse place

his fingers to palpate these nodes?


a. In front of the ears

b. Under the mandibles

c. Along the angle of the mandibles

d. Adjacent to the sternocleidomastoid muscles

ANSWER: D

A

B

C

D

Feedback

This is the location of the preauricular lymph nodes.

This is the location of the submental and submandibular lymph nodes.

This is the location of the parotid lymph nodes.

This is the location of the anterior cervical lymph nodes.

DIF: Cognitive Level: Understand REF: 178

TOP: Nursing Process: Assessment

MSC: NCLEX Patient Needs: Physiologic Integrity: Physiologic Adaptation: System Specific

Assessments

66. What instructions does the nurse give the patient before palpating the right supraclavicular

lymph nodes?

a. “Lean your head backward and toward the right as far as comfortably possible.”

b. “Lie supine and turn your head away from the right side.”

c. “Draw up your shoulders forward, and flex your chin toward the right side.”

d. “Sit up, raise both arms over your head, and flex your chin away from the right

side.”

ANSWER: C

A

B

C

D

Feedback

This is incorrect. The patient should draw up (hunch) the shoulders forward rather

than leaning back.

The patient should be sitting, rather than lying down.

This is the technique for palpating the supraclavicular nodes.

The shoulders should be drawn up (hunched) forward, rather than raising the

arms.

DIF: Cognitive Level: Apply REF: 178

TOP: Nursing Process: Assessment

MSC: NCLEX Patient Needs: Health Promotion and Maintenance: Techniques of Physical

Assessment

67. A patient has had an infected facial wound for more than 3 months. How does the nurse

expect the patient’s enlarged lymph nodes to feel?

a. Soft, edematous, and tender

b. Round, tender, and movable

c. Hard, nontender, and nonmobile

d. Irregularly shaped, tender, and firm

ANSWER: B

A

Feedback

These are not characteristics of lymph nodes associated with inflammation.


B

C

D

These are characteristics of enlarged lymph nodes associated with inflammation.

These are characteristics of enlarged lymph nodes associated with a malignancy.

These are not characteristics of lymph nodes associated with inflammation.

DIF: Cognitive Level: Analyze REF: 178

TOP: Nursing Process: Assessment

MSC: NCLEX Patient Needs: Physiologic Integrity: Physiologic Adaptation: Alteration in Body

Systems

68. A nurse’s presentation to patients on risk factors for oral cancer includes which fact?

a. The peak incidence oral cancer is before 40 years of age.

b. Women have a higher risk than men.

c. Excessive alcohol consumption is a risk factor.

d. Eating a low fiber diet is a risk factor.

ANSWER: C

Feedback

A There is increased incidence after age 40 with peak incidence between ages 64

and 74.

B There is a 2:1 men-to-women incidence.

C Seventy-five to eighty percent of individuals who develop oral cancer consume

excessive amounts of alcohol.

D A low fiber diet increases the risk for colon cancer, but not oral cancer.

DIF: Cognitive Level: Understand REF: 181

TOP: Nursing Process: Assessment

MSC: NCLEX Patient Needs: Health Promotion and Maintenance: Health Promotion Programs

69. During the history, a 65-year-old male patient reports smoking two packs of cigarettes a day

for more than 40 years. With this knowledge, what does the nurse expect for during the

examination of this patient’s mouth?

a. Cracks and erythema in the corners of the mouth

b. Slightly rough papillae on the dorsal surface of the tongue

c. Smooth or beefy, red-colored, edematous tongue

d. Painless, nonhealing mouth ulcers

ANSWER: D

A

B

C

D

Feedback

This may be caused by vitamin B deficiencies.

These are an expected finding on the tongue.

This may be an indication of anemia.

This may indicate oral cancer.

DIF: Cognitive Level: Apply REF: 181

TOP: Nursing Process: Assessment

MSC: NCLEX Patient Needs: Physiologic Integrity: Physiologic Adaptation: Alteration in Body

Systems


70. A nurse’s presentation to patients on risk factors for macular degeneration includes which

fact?

a. The peak incidence is before 60 years of age.

b. Women have a higher risk than men.

c. Eating a low fat diet causes a vitamin A deficiency, which increases risk.

d. Cigarette smokers have twice the risk as nonsmokers.

ANSWER: D

A

B

C

D

Feedback

Macular degeneration exists in 25% of those between ages 65 and 74 years, and

33% of those older than 75 years.

There are no differences between genders for macular degeneration.

A diet high in monosaturated, polyunsaturated, and vegetable fats increases risk.

Smoking is a risk factor for macular degeneration.

DIF: Cognitive Level: Understand REF: 181

TOP: Nursing Process: Assessment

MSC: NCLEX Patient Needs: Health Promotion and Maintenance: Health Promotion Programs

MULTIPLE RESPONSE

1. During an examination of the head and neck of a healthy adult, the nurse expects which

findings? Select all that apply.

a. Small red lesions with white flakes scattered on the scalp

b. The head and facial bones are proportional for the size of the body

c. Depressions palpated on the right and left sides over the parietal bones

d. Head held flexed 15 degrees to the left

e. Face and jaw are symmetric and proportional

f. Temporomandibular joint moves smoothly

ANSWER: B, E, F

Correct: These are expected findings from an assessment of the head of a healthy adult.

Incorrect: Small red lesions with white flakes scattered on the scalp is an abnormal finding.

The scalp should be intact without lesion or flakes. Depressions palpated on the right and left

sides over the parietal bones is an abnormal finding. Perhaps this patient had skull tongs from

cervical traction at one time. Head held flexed 15 degrees to the left is an abnormal finding.

The head should be erect.

DIF: Cognitive Level: Analyze REF: 148| 179

TOP: Nursing Process: Assessment

MSC: NCLEX Patient Needs: Health Promotion and Maintenance

2. A nurse is assessing the eyes of a healthy 72-year-old adult. What findings does the nurse

expect? Select all that apply.

a. Bulbar conjunctiva pink and clear, with small red vessels noted

b. Sclera yellow and moist, cornea transparent

c. Extraocular movement symmetric with peripheral vision noted

d. Newspaper held at 18 inches to see clearly

e. Sclera visible between upper lid and iris


f. Gray to white circle noted where the sclera merges with the cornea

g. Light reflects on the cornea at 12 o’clock in each eye

ANSWER: A, C, G

Correct: These are expected findings from an assessment of the eyes of a healthy adult.

Incorrect: Sclera should be white and moist. Newspaper held at 18 inches to see clearly is

due to presbyopia due to the patient’s age. Patient must hold paper further away to see clearly.

The upper lid should cover the upper part of the iris. Sclera is visible in hyperthyroidism. A

gray to white circle is arcus senilis, which is an abnormal finding in older adults.

DIF: Cognitive Level: Apply REF: 137| 150| 152-153| 179

TOP: Nursing Process: Assessment

MSC: NCLEX Patient Needs: Health Promotion and Maintenance: Techniques of Physical

Assessment

3. During an eye assessment, the nurse asks the patient to keep the head stationary and by

moving the eyes only follow the nurse’s finger as it moves side to side, up and down, and

obliquely. This assessment technique collects what data about which cranial nerves? Select all

that apply.

a. Cranial nerve II (optic)

b. Cranial nerve III (oculomotor)

c. Cranial nerve IV (trochlear)

d. Cranial nerve VI (abducens)

e. Cranial nerve V (trigeminal)

ANSWER: B, C, D

Correct: Cranial nerve III (oculomotor), cranial nerve IV (trochlear), and cranial nerve VI

(abducens) provide muscle movement for the eyes.

Incorrect: Cranial nerve II (optic) provides vision. Cranial nerve V (trigeminal) provides

movement for the jaw and sensation for cornea, conjunctive, eyelids, teeth, tongue, and

mouth.

DIF: Cognitive Level: Apply REF: 154-155

TOP: Nursing Process: Assessment

MSC: NCLEX Patient Needs: Health Promotion and Maintenance: Techniques of Physical

Assessment

4. What findings does the nurse expect when assessing the ears of a healthy adult? Select all that

apply.

a. Cerumen noted in the outer ear canal

b. Pinna located below the external corner of the eye

c. Cone of light located in the 5 o’clock position in the left ear

d. Ratio of air conduction to bone conduction 2:1

e. Tympanic membrane pearly gray

f. Whispered words repeated accurately

ANSWER: A, D, E, F

Correct: These are all expected findings from an assessment of the ears of a healthy adult.

Incorrect: The pinna should align with the outer canthus of the eye. Cone of light should be

located in the 7 o’clock position in the left ear and the 5 o’clock position in the right ear.

DIF: Cognitive Level: Apply REF: 163-165| 168| 179


TOP: Nursing Process: Assessment

MSC: NCLEX Patient Needs: Health Promotion and Maintenance: Techniques of Physical

Assessment

5. Which findings does the nurse expect when assessing the mouth of a healthy adult? Select all

that apply.

a. Lips appear pink, smooth, moist, and symmetric

b. Teeth are white, yellow, or gray, with smooth edges

c. Exposed tooth neck and brown spots between teeth

d. Slight roughness on the dorsum of the tongue

e. Hard palate appears smooth, pale, and immovable

f. Mucous membranes are dry and intact

ANSWER: A, B, D, E

Correct: These are all expected findings from a mouth assessment of a healthy adult.

Incorrect: Receding gums expose tooth neck and may indicate gingival disease. Brown spots

may indicate caries. Dry and intact mucous membranes may indicate dehydration.

DIF: Cognitive Level: Analyze REF: 171| 173-174| 179

TOP: Nursing Process: Assessment

MSC: NCLEX Patient Needs: Physiologic Integrity: Reduction of Risk Potential: System Specific

Assessments

6. Nurses inquire about lifestyle behaviors in those patients with specific risk factors for

cataracts. Which characteristics are associated with risk factors for cataracts? Select all that

apply.

a. Smoking more than 20 cigarettes a day

b. Having parents with cataracts

c. Chronic consumption of alcohol

d. Having a chronic disease, such as diabetes mellitus

e. Being Asian

f. Being a man

ANSWER: A, C, D

Correct: These are all risk factors for cataracts.

Incorrect: Having parents with cataracts is not a genetic or familial disorder. Being Asian or a

man are not risk factors, but being an African-American or being a woman are risk factors.

DIF: Cognitive Level: Apply REF: 181

TOP: Nursing Process: Assessment

MSC: NCLEX Patient Needs: Health Promotion and Maintenance: Health Promotion Programs


Chapter 11: Lungs and Respiratory System

Test Bank

MULTIPLE CHOICE

1. A patient tells the nurse that she has smoked two packs of cigarettes a day for 20 years. The

nurse records this as how many pack-years?

a. 10

b. 20

c. 40

d. 60

ANSWER: C

Feedback

A This incorrect calculation was made by dividing 20 years by 2 packs.

B This is correct if the patient smoked 1 pack per day for 20 years.

C Two packs of cigarettes 20 years = 40 pack-years.

D This is correct if the patient smoked 3 packs per day for 20 years or 2 packs a day

for 30 years.

DIF: Cognitive Level: Apply REF: 197, Box 11-1

TOP: Nursing Process: Assessment

MSC: NCLEX Patient Needs: Physiologic Integrity: Reduction of Risk Potential: System Specific

Assessments

2. After taking a brief health history, a nurse needs to complete a focused assessment on which

patient?

a. A male who works as a painter

b. A male who plays basketball and hockey

c. A female who recently moved into a college dormitory

d. A female who has a history of gout

ANSWER: A

A

B

C

D

Feedback

The fumes and chemicals from the paint may expose the patient to respiratory

irritants. A baseline pulmonary assessment needs to be documented.

This patient is not at risk for pulmonary disease.

This patient is not at risk for pulmonary disease.

This patient is not at risk for pulmonary disease.

DIF: Cognitive Level: Apply REF: 198

TOP: Nursing Process: Assessment

MSC: NCLEX Patient Needs: Health Promotion and Maintenance: Health Screening

3. During a symptom analysis, a patient describes his productive cough and states his sputum is

thick and yellow. Based on these data, the nurse suspects which factor as the cause of these

symptoms?

a. Virus


b. Allergy

c. Fungus

d. Bacteria

ANSWER: D

A

B

C

D

Feedback

A virus usually produces a nonproductive cough.

An allergy usually produces clear sputum.

A fungus usually produces few symptoms. The sputum used to diagnose the

fungus is obtained from tracheal aspiration rather than the patient coughing up the

sputum.

Bacteria usually produce sputum that is yellow or green in color.

DIF: Cognitive Level: Apply REF: 198

TOP: Nursing Process: Assessment

MSC: NCLEX Patient Needs: Physiologic Integrity: Physiologic Adaptation: Alterations in Body

Systems

4. During the problem-based history, a patient reports coughing up sputum when lying on the

right side, but not when lying on the back or left side. The nurse suspects this patient may

have a lung abscess. What additional question does the nurse ask to gather more data?

a. “Does the sputum have an odor?”

b. “Do you have chest pain when you take a deep breath?”

c. “Have you also experienced tightness in your chest?”

d. “Have you coughed up any blood?”

ANSWER: A

A

B

C

D

Feedback

Sputum with odor and sputum production with change of position is associated

with lung abscess or bronchiectasis.

Chest pain on deep breathing is associated with pleural lining irritation.

Tightness in the chest is associated with asthma.

Coughing up rust-colored sputum is associated with pneumonia, but coughing up

blood may be associated with lung cancer.

DIF: Cognitive Level: Apply REF: 198

TOP: Nursing Process: Assessment

MSC: NCLEX Patient Needs: Physiologic Integrity: Physiologic Adaptation: Alterations in Body

Systems

5. Which question will give the nurse additional information about the nature of a patient’s

dyspnea?

a. “How often do you see the physician?”

b. “How has this condition affected your day-to-day activities?”

c. “Do you have a cough that occurs with the dyspnea?”

d. “Does your heart rate increase when you are short of breath?”

ANSWER: B

Feedback


A

B

C

D

This question does not relate specifically to the patient’s dyspnea.

This question provides data about the severity of the dyspnea and what actions

the patient has taken to cope with the dyspnea on a daily basis.

This question provides data, but does not give additional facts about the patient’s

dyspnea.

This is a closed-ended question that does not collect additional data about this

episode of dyspnea.

DIF: Cognitive Level: Apply REF: 199

TOP: Nursing Process: Assessment

MSC: NCLEX Patient Needs: Physiologic Integrity: Physiologic Adaptation: Alterations in Body

Systems

6. A patient complains of shortness of breath and having to sleep on three pillows to breathe

comfortably at night. During the nurse’s examination, what findings will suggest that the

cause of this patient’s dyspnea is due to heart disease rather than respiratory disease?

a. Increased anteroposterior diameter

b. Clubbing of the fingers

c. Bilateral peripheral edema

d. Increased tactile fremitus

ANSWER: C

A

B

C

D

Feedback

This is seen with lung hyperinflation and may be associated with emphysema.

This is associated with chronic hypoxia and may be associated with cystic fibrosis

or chronic obstructive pulmonary disease.

This indicates heart failure; dyspnea occurs because the heart cannot adequately

perfuse the lungs.

This occurs when vibrations are enhanced and is associated with consolidation

that may occur in pneumonia or tumor.

DIF: Cognitive Level: Analyze REF: 199

TOP: Nursing Process: Assessment

MSC: NCLEX Patient Needs: Physiologic Integrity: Physiologic Adaptation: Alterations in Body

Systems

7. During a history, a nurse notices a patient is short of breath, is using pursed-lip breathing, and

maintains a tripod position. Based on these data, what abnormal finding should the nurse

expect to find during the examination?

a. Increased tactile fremitus

b. Inspiratory and expiratory wheezing

c. Tracheal deviation

d. An increased anteroposterior diameter

ANSWER: D

A

B

Feedback

Increased tactile fremitus occurs when vibrations are enhanced and is associated

with consolidation that may occur in pneumonia or tumor.

Inspiratory and expiratory wheezing is associated with asthma.


C

D

Tracheal deviation is associated with tension pneumothorax.

An increased anteroposterior diameter is consistent with emphysema.

DIF: Cognitive Level: Analyze REF: 199| 207

TOP: Nursing Process: Assessment

MSC: NCLEX Patient Needs: Physiologic Integrity: Physiologic Adaptation: Alterations in Body

Systems

8. A nurse notices a patient’s chest wall moving in during inspiration and out during expiration.

What additional assessment must the nurse perform immediately?

a. Palpate for tracheal deviation.

b. Auscultate for bronchovesicular breath sounds in the lung periphery.

c. Palpate posterior thoracic muscles for tenderness.

d. Auscultate for absence of breath sounds in the lung periphery.

ANSWER: A

A

B

C

D

Feedback

Chest wall moving in during inspiration and out during expiration is paradoxical

chest wall movement. It can be caused by a tension pneumothorax, which

increases intrathoracic pressure in the thorax, causing tracheal deviation and

indicating mediastinal shift.

Tension pneumothorax does not create bronchovesicular breath sounds in the lung

periphery.

This is performed when the patient has air in the subcutaneous tissue or pleural

friction rub.

Absent breath sounds may be found in pneumothorax, but if the patient has a

tension pneumothorax, tracheal deviation is a more important sign.

DIF: Cognitive Level: Apply REF: 199| 213-214| 220

TOP: Nursing Process: Assessment

MSC: NCLEX Patient Needs: Physiologic Integrity: Physiologic Adaptation: Alterations in Body

Systems

9. A nurse inspects a patient’s hands and notices clubbing of the fingers. The nurse correlates this

finding with what condition?

a. Pulmonary infection

b. Trauma to the thorax

c. Chronic hypoxemia

d. Allergic reaction

ANSWER: C

A

B

C

D

Feedback

Pulmonary infection is acute and not associated with chronic hypoxia.

Trauma to the thorax is acute and not associated with chronic hypoxia.

Clubbing develops due to chronic hypoxemia, which occurs in chronic

obstructive pulmonary disease.

Allergic reaction is acute and not associated with chronic hypoxia.

DIF: Cognitive Level: Apply REF: 202


TOP: Nursing Process: Assessment

MSC: NCLEX Patient Needs: Physiologic Integrity: Physiologic Adaptation: Alterations in Body

Systems

10. A nurse is assessing a patient who was diagnosed with emphysema and chronic bronchitis 5

years ago. During the assessment of this patient’s integumentary system, what finding should

the nurse correlate to this respiratory disease?

a. Dry, flaky skin

b. Clubbing of the fingers

c. Hypertrophy of the nails

d. Hair loss from the scalp

ANSWER: B

A

B

C

D

Feedback

Dry, flaky skin occurs with dehydration.

Clubbing of the fingers develops due to chronic hypoxemia, which occurs in

chronic obstructive pulmonary disease.

Hypertrophy of the nails occurs with repeated trauma.

Hair loss from the scalp is alopecia, which occurs with many systemic diseases,

but not chronic pulmonary disease.

DIF: Cognitive Level: Apply REF: 202

TOP: Nursing Process: Assessment

MSC: NCLEX Patient Needs: Physiologic Integrity: Physiologic Adaptation: Alterations in Body

Systems

11. A nurse is auscultating the lungs of a healthy male patient and hears crackles on inspiration.

What action can the nurse take to ensure this is an accurate finding?

a. Make sure the bell of the stethoscope is used, rather than the diaphragm.

b. Hold stethoscope firmly to prevent movement when placed over chest hair.

c. Ask the patient not to talk while the nurse is listening to the lungs.

d. Change the patient’s position to ensure accurate sounds.

ANSWER: B

A

B

C

D

Feedback

Using the bell will provide inaccurate sounds, but not mimic crackles.

The stethoscope moving even slightly on chest hair can mimic the sound of

crackles.

When the patient talks during auscultation, it does interfere with data collection,

but the sound is a muffled voice.

Changing the position will not affect the outcome of the assessment if the initial

problem remains.

DIF: Cognitive Level: Apply REF: 203-204

TOP: Nursing Process: Assessment

MSC: NCLEX Patient Needs: Health Promotion and Maintenance: Techniques of Physical

Assessment


12. A patient is admitted to the emergency department with a tracheal obstruction. What sound

does the nurse expect to hear as this patient breathes?

a. Dull sounds on percussion

b. Soft, muffled rhonchi heard over the trachea

c. Bubbling or rasping sounds heard over the trachea

d. High-pitched sounds on inspiration and exhalation

ANSWER: D

A

B

C

D

Feedback

Dull sounds on percussion occur with pneumonia, pleural effusion, or atelectasis.

Soft, muffled rhonchi heard over the trachea is not a description of stridor.

Bubbling or rasping sounds heard over the trachea is not a description of stridor.

High-pitched sounds on inspiration and exhalation are consistent with stridor.

DIF: Cognitive Level: Understand REF: 206

TOP: Nursing Process: Assessment

MSC: NCLEX Patient Needs: Physiologic Integrity: Physiologic Adaptation: Alterations in Body

Systems

13. A nurse auscultates low-pitched, coarse snoring sounds in a patient’s lungs during inhalation.

What is the most appropriate action for the nurse to take at this time?

a. Palpate the posterior thorax for vocal fremitus.

b. Ask the patient to cough and repeat auscultation.

c. Auscultate the posterior thorax for vocal sounds.

d. Percuss the posterior thorax for tone.

ANSWER: B

A

B

C

D

Feedback

An abnormal vocal fremitus (decreased or increased vibrations) is not expected

for this patient.

The sounds indicate rhonchi, or secretions in the bronchi. The first action to take

is to determine if the rhonchi clear with coughing. If the rhonchi clear, there is no

need to further investigate this finding.

Abnormal vocal sounds (clear and loud sounds) are not expected for this patient.

An abnormal percussion tone (hyperresonance or dull) is not expected for this

patient.

DIF: Cognitive Level: Apply REF: 203-204

TOP: Nursing Process: Assessment

MSC: NCLEX Patient Needs: Physiologic Integrity: Physiologic Adaptation: Alterations in Body

Systems

14. A nurse had previously heard crackles over both lungs of a patient. As the patient improves,

what lung sounds does the nurse expect to hear in the patient’s lungs?

a. Vesicular breath sounds heard in peripheral lung fields

b. Bronchial breath sounds heard over the bronchi

c. Bronchovesicular breath sounds heard over the apices

d. Rhonchi heard over the main bronchi


ANSWER: A

A

B

C

D

Feedback

Vesicular breath sounds heard in peripheral lung fields are an expected finding for

healthy lungs.

Bronchial breath sounds are heard over the trachea.

Bronchovesicular breath sounds are heard anteriorly near the sternal border first

and second intercostals space.

Rhonchi are adventitious sounds indicating secretions in the bronchi.

DIF: Cognitive Level: Apply REF: 206

TOP: Nursing Process: Assessment

MSC: NCLEX Patient Needs: Physiologic Integrity: Reduction of Risk Potential: System Specific

Assessments

15. The nurse is comparing pitch and duration of the various types of a patient’s breath sounds

and recognizes which one of these as an expected finding?

a. Bronchial sounds are low-pitched and have a 2:1 inspiratory-versus-expiratory

ratio.

b. Bronchovesicular sounds have a moderate pitch and 1:1 expiratory-versusinspiratory

ratio.

c. Vesicular breath sounds are high-pitched and have a 1:2 inspiratory-versusexpiratory

ratio.

d. Wheezes are low-pitched and have a 2.5:1 inspiratory-versus-expiratory ratio.

ANSWER: B

A

B

C

D

Feedback

Bronchial sounds are high pitched with a duration of 1:2 inspiration-to-expiration

is the correct statement.

Bronchovesicular sounds having a moderate pitch and 1:1 expiratory-versusinspiratory

ratio is a normal finding.

Vesicular sounds are low pitched with a duration of 2.5:1 inspiration-to-expiration

is the correct statement.

Wheezes are high-pitched and have no specific duration because they are

adventitious sounds.

DIF: Cognitive Level: Understand REF: 205-206

TOP: Nursing Process: Assessment

MSC: NCLEX Patient Needs: Physiologic Integrity: Reduction of Risk Potential: System Specific

Assessments

16. On inspection, a nurse finds the patient’s anteroposterior diameter of the chest to be the same

as the lateral diameter. Based on this finding, what additional data does the nurse anticipate?

a. Increased vocal fremitus on palpation

b. Dull tones heard on percussion

c. Decreased breath sounds on auscultation

d. Complaint of sharp chest pain on inspiration

ANSWER: C


A

B

C

D

Feedback

Increased fremitus occurs when the vibrations feel enhanced. This is found when

lung tissues are congested or consolidated, which may occur in patients who have

pneumonia or a tumor.

Dull tones may be heard in patients with pneumonia, pleural effusion, or

atelectasis.

The equal anteroposterior and lateral diameters of the chest indicate air trapping

from enlarged or destroyed alveoli. This air trapping causes decreased to absent

breath sounds on auscultation.

Complaint of sharp chest pain on inspiration is pleuritic chest pain associated

with pleural lining irritation and may occur in a patient with pleurisy or

pneumonia.

DIF: Cognitive Level: Apply REF: 206-208| 219

TOP: Nursing Process: Assessment

MSC: NCLEX Patient Needs: Physiologic Integrity: Physiologic Adaptation: Alterations in Body

Systems

17. Where does a nurse expect to hear bronchovesicular lung sounds in a healthy adult?

a. In the lower lobes

b. Over the trachea

c. In the apices of the lungs

d. Near the sternal border

ANSWER: D

A

B

C

D

Feedback

Vesicular breath sounds are normally heard in the lower lobes.

Bronchial sounds are normally heard over the trachea.

Vesicular breath sounds are normally heard in the apices of the lungs.

Bronchovesicular breath sounds are normally heard over the central area of the

anterior thorax around the sternal border.

DIF: Cognitive Level: Remember REF: 205-206

TOP: Nursing Process: Assessment

MSC: NCLEX Patient Needs: Health Promotion and Maintenance: Health Screening

18. A nurse in the emergency department is assessing a patient with a moderate left

pneumothorax. What does this nurse expect to find during the respiratory examination?

a. Increased fremitus over the left chest

b. Tracheal deviation to the left side

c. Hyporesonant percussion tones over the left chest

d. Distant to absent breath sounds over the left chest

ANSWER: D

A

B

C

Feedback

Increased fremitus occurs over lung consolidation as in lobar pneumonia or

tumor.

If this patient had a tension pneumothorax, the trachea would deviate to the right.

Hyperresonant percussion tones are heard when the lung is overinflated as in


D

emphysema.

The air separating the lung from the chest where the nurse is auscultating creates

distant to absent breath sounds.

DIF: Cognitive Level: Apply REF: 206| 220

TOP: Nursing Process: Assessment

MSC: NCLEX Patient Needs: Physiologic Integrity: Physiologic Adaptation: Alterations in Body

Systems

19. A nurse suspects a patient has a chest wall injury and wants to collect more data about

thoracic expansion. Which is the appropriate technique to use?

a. Place the palmar side of each hand against the lateral thorax at the level of the

waist, ask the patient to take a deep breath, and observe lateral movement of the

hands.

b. Place both thumbs on either side of the patient’s T9 to T10 spinal processes, extend

fingers laterally, ask the patient to take a deep breath, and observe lateral movement

of the thumbs.

c. Place both thumbs on either side of the patient’s T7 to T8 spinal processes, extend

fingers laterally, ask the patient to exhale deeply, and observe lateral inward

movement of the thumbs.

d. Place the palmar side of each hand on the shoulders of the patient, ask the patient to

sit up straight and take a deep breath, and observe symmetric movement of the

shoulders.

ANSWER: B

A

B

C

D

Feedback

The palms of the hands are not used and hands are not placed on the lateral

thorax.

This is the correct technique to assess thoracic expansion.

The thoracic level is too high and the patient does not exhale.

The hands are not placed on the shoulders.

DIF: Cognitive Level: Understand REF: 209-210| 214

TOP: Nursing Process: Assessment

MSC: NCLEX Patient Needs: Health Promotion and Maintenance: Techniques of Physical

Assessment

20. A patient has right lower lobe pneumonia, creating a consolidation in that lung. In assessing

for vocal fremitus, the nurse found increased fremitus over the right lower lung. What finding

does the nurse anticipate when assessing vocal resonance to confirm the consolidation?

a. Bronchophony reveals the patient’s spoken “99” as clear and loud.

b. No sounds are expected since sounds cannot be transmitted through consolidation.

c. Egophony reveals indistinguishable sounds when the patient says “e-e-e.”

d. Whispered pectoriloquy reveals a muffled sound when the patient says “1-2-3.”

ANSWER: A

A

B

Feedback

This is an abnormal finding and occurs in consolidation.

The abnormal finding is hearing a clear sound.


C

D

This is a normal finding.

This is a normal finding.

DIF: Cognitive Level: Analyze REF: 213| 218

TOP: Nursing Process: Assessment

MSC: NCLEX Patient Needs: Physiologic Integrity: Physiologic Adaptation: Alterations in Body

Systems

21. A nurse is assessing for vocal (tactile) fremitus on a patient with pulmonary edema. Which is

the appropriate technique to use?

a. Systematically percuss the posterior chest wall following the same pattern that is

used for auscultation and listen for a change in tone from resonant to dull.

b. Place the pads of the fingers on the right and left thoraces and palpate the texture

and consistency of the skin feeling for a crackly sensation under the fingers.

c. Place the palms of the hands on the right and left thoraces, ask the patient to say

“99,” and feel for vibrations.

d. Place both thumbs on either side of the patient’s spinal processes, extend fingers

laterally, ask the patient to take a deep breath, and feel for vibrations.

ANSWER: C

A

B

C

D

Feedback

This is the technique for percussing the thorax for tones.

This is the technique for detecting crepitus.

This is the correct technique for vocal fremitus.

This is not the correct technique.

DIF: Cognitive Level: Understand REF: 210| 215

TOP: Nursing Process: Assessment

MSC: NCLEX Patient Needs: Health Promotion and Maintenance: Techniques of Physical

Assessment

22. A nurse examines a patient with a pleural effusion and finds decreased fremitus. What

additional abnormal finding should the nurse anticipate during further examination?

a. An increase in the anteroposterior to lateral ratio

b. Hyperresonance over the affected area

c. Absent breath sounds in the affected area

d. Increased vocal fremitus over the affected area

ANSWER: C

A

B

C

D

Feedback

An increase in the anteroposterior to lateral ratio occurs in overinflated lungs as

in emphysema.

Hyperresonance over the affected area occurs in overinflated lungs as in

emphysema.

Absent breath sound in the affected area is anticipated because the fluid in the

pleural space prevents breath sounds from being heard.

Increased vocal fremitus over the affected area is associated with consolidation

that occurs with pneumonia or tumor. Fremitus is decreased to absent in pleural

effusion.


DIF: Cognitive Level: Analyze REF: 206| 210| 215| 218

TOP: Nursing Process: Assessment

MSC: NCLEX Patient Needs: Physiologic Integrity: Physiologic Adaptation: Alterations in Body

Systems

23. A patient is suspected of having a lung consolidation. A nurse uses the three techniques for

assessing vocal resonance in this patient. What is the expected finding among the three

procedures that will help eliminate consolidation as a problem?

a. The nurse documents clearly hearing the patient say “99.”

b. The nurse documents hearing muffled sounds when the patient says “1-2-3.”

c. The nurse documents hearing no sounds when the patient says “e-e-e.”

d. The nurse documents clearly hearing the patient say “a-a-a.”

ANSWER: B

A

B

C

D

Feedback

Clear sounds are heard when a consolidation is present.

Muffled sounds of “1-2-3,” “e-e-e,” or “99” are heard when no consolidation is

found.

Clear sounds are heard when a consolidation is present.

Clear sounds are heard when a consolidation is present.

DIF: Cognitive Level: Analyze REF: 213

TOP: Nursing Process: Assessment

MSC: NCLEX Patient Needs: Physiologic Integrity: Physiologic Adaptation: Alterations in Body

Systems

24. In reviewing the patient’s record, the nurse notes that the patient has air in the subcutaneous

tissue. The nurse validates that this patient has crepitus with which finding?

a. Asymmetric expansion of the chest wall on inhalation

b. Increased transmission of vocal vibrations on auscultation

c. Crackling sensation under the skin of the chest on palpation

d. Coarse grating sounds heard over the mediastinum on inspiration

ANSWER: C

A

B

C

D

Feedback

Asymmetric chest expansion occurs with rib fracture or chest wall injury.

Increased vocal fremitus occurs with lung consolidation.

A crackling sensation is the finding when crepitus is present.

Coarse grating sounds heard over the mediastinum on inspiration does not

validate crepitus.

DIF: Cognitive Level: Apply REF: 209

TOP: Nursing Process: Assessment

MSC: NCLEX Patient Needs: Physiologic Integrity: Physiologic Adaptation: Alterations in Body

Systems

25. Which patient should the nurse assess first?

a. The patient whose respiratory rate is 26 breaths per minute and whose trachea


deviates to the right.

b. The patient who has pleuritic chest pain, bilateral crackles, a productive cough of

yellow sputum, and fever.

c. The patient who is short of breath, using pursed-lip breathing, and in a tripod

position.

d. The patient whose respiratory rate is 20 breaths/min, and has 8-word dyspnea and

expiratory wheezes.

ANSWER: A

A

B

C

D

Feedback

This is a description of a left tension pneumothorax. The key manifestation is

deviation of the trachea from midline, which indicates high intrathoracic pressure

from the left that is pushing the mediastinum out of alignment. The respiratory

rate indicates tachypnea.

This is a description of a patient with pneumonia who needs to be examined, but

this is not a life-threatening condition.

This is a description of a patient with emphysema, a chronic disease. This patient

may have these manifestations frequently and does not need to be examined

immediately.

This is a description of a patient who is having an asthma attack, but it is not a

life threatening attack; the respiratory rate is the upper limits of normal; the

dyspnea is abnormal, but not far from normal; and the wheezing is on expiration

only.

DIF: Cognitive Level: Analyze REF: 201| 213-214| 220

TOP: Nursing Process: Assessment

MSC: NCLEX Patient Needs: Physiologic Integrity: Physiologic Adaptation: Alterations in Body

Systems

26. A patient reports a productive cough with yellow sputum, fever, and a sharp pain when taking

a deep breath to cough. Based on these data, what abnormal finding will the nurse anticipate

on examination?

a. Decreased breath sounds on auscultation

b. Increased tactile fremitus and dull percussion tones

c. Inspiratory wheezing found on auscultation

d. Muffled sounds heard when the patient says “e-e-e”

ANSWER: B

A

B

C

D

Feedback

Decreased breath sounds on auscultation is consistent with emphysema or

atelectasis when alveoli are narrowed or destroyed.

The data describe purulent sputum and inflammation of the pleura that may occur

in pneumonia. Additional findings include increased tactile fremitus and dull

percussion tones, indicating congested or consolidated lung tissues.

Inspiratory wheezing found on auscultation is consistent with narrowing of

bronchi that may occur in asthma.

Muffled sounds heard when the patient says “e-e-e” is a normal finding on vocal

resonance (bronchophony or egophony).


DIF: Cognitive Level: Apply REF: 198-199| 210| 215| 218

TOP: Nursing Process: Assessment

MSC: NCLEX Patient Needs: Physiologic Integrity: Physiologic Adaptation: Alterations in Body

Systems

27. A nurse palpating the chest of a patient finds increased fremitus bilaterally. What is the

significance of this finding?

a. An expected finding

b. Chronic obstructive pulmonary disease

c. Bilateral pneumonia

d. Bilateral pneumothorax

ANSWER: C

A

B

C

D

Feedback

An increase in fremitus from normal is not an expected finding.

Air trapping in chronic obstructive pulmonary disease causes a decreased

fremitus.

Increased fremitus occurs when lung tissues are congested or consolidated, which

may occur in patients who have pneumonia or a tumor.

Air in the pleural space causes a decreased fremitus.

DIF: Cognitive Level: Apply REF: 210| 215| 218

TOP: Nursing Process: Assessment

MSC: NCLEX Patient Needs: Physiologic Integrity: Physiologic Adaptation: Alterations in Body

Systems

MULTIPLE RESPONSE

1. What are the functions of the upper airways? Select all that apply.

a. Conduct air to lower airway.

b. Provide area for gas exchange.

c. Prevent foreign matter from entering respiratory system.

d. Warm, humidify, and filter air entering lungs.

e. Provide transportation of oxygen and carbon dioxide between alveoli and cells.

ANSWER: A, C, D

Correct: These are functions of the upper airway.

Incorrect: Gas exchange occurs in the alveoli. The cardiovascular system provides

transportation of oxygen and carbon dioxide between alveoli and cells.

DIF: Cognitive Level: Remember REF: 193-194

TOP: Nursing Process: Assessment

MSC: NCLEX Patient Needs: Physiologic Integrity: Reduction of Risk Potential: System Specific

Assessments

2. On inspection, the nurse finds the patient’s anteroposterior diameter of the chest to be the

same as the lateral diameter. What other findings does this nurse expect during the

examination? Select all that apply.

a. Inspiratory wheezing found on auscultation


b. Hyperresonance heard on percussion

c. Decreased breath sounds heard on auscultation

d. Deceased diaphragmatic excursion on percussion

e. A sharp, abrupt pain reported when the patient breathes deeply

f. Decreased to absent vibration on vocal fremitus

ANSWER: B, C, D, F

Correct: These are all indications of enlargement or destruction of alveoli that occurs in

emphysema. Air is trapped, which increases the anteroposterior to lateral diameter creating a

barrel chest, and pushes the diaphragm down decreasing the excursion and causing

hyperresonance. The destroyed alveoli decrease the breath sounds and create absent vibration

on vocal fremitus.

Incorrect: Inspiratory wheezing found on auscultation indicates narrowed airways as found in

asthma. A sharp, abrupt pain reported when the patient breathes deeply is pleuritic chest pain

associated with pleural lining irritation that may occur in a patient with pleurisy or

pneumonia.

DIF: Cognitive Level: Analyze REF: 207| 210-213| 215| 219

TOP: Nursing Process: Assessment

MSC: NCLEX Patient Needs: Physiologic Integrity: Physiologic Adaptation: Alterations in Body

Systems

3. On examination, a nurse finds the patient has a productive cough with green sputum and

inspiratory crackles. What other findings does this nurse expect during the examination?

Select all that apply.

a. Dull tones to percussion

b. Increased vibration on vocal fremitus

c. Fever

d. Decreased diaphragmatic excursion

e. A sharp, abrupt pain reported when patient breathes deeply

f. Muffled sounds heard when the patient says “e-e-e”

ANSWER: A, B, C, E

Correct: These abnormal findings are consistent with consolidation that may occur with

pneumonia.

Incorrect: Decreased diaphragmatic excursion occurs when the lung is overinflated as in

emphysema. Muffled sounds when the patient says “e-e-e” is an expected finding. With a

consolidation, the sound of “e-e-e” would be clear.

DIF: Cognitive Level: Analyze REF: 198-199| 210-211| 215| 218

TOP: Nursing Process: Assessment

MSC: NCLEX Patient Needs: Physiologic Integrity: Physiologic Adaptation: Alterations in Body

Systems

4. A nurse is assessing the respiratory system of a healthy adult. Which findings does this nurse

expect to find? Select all that apply.

a. Thoracic expansion that is symmetric bilaterally

b. Respiratory rate of 24 breaths/min

c. Bronchophony revealing clear voice sounds

d. Breath sounds clear with vesicular breath sounds heard over most lung fields

e. Anteroposterior diameter of the chest about a 1:2 ratio of anteroposterior to lateral


diameter

f. Symmetric thorax with ribs sloping downward at about 45 degrees relative to the

spine

ANSWER: A, D, E, F

Correct: These are expected findings from a lung and respiratory assessment of a healthy

adult.

Incorrect: A respiratory rate of 24 breaths/min is considered tachypnea. Bronchophony

revealing clear voice sounds is not performed unless there is an indication of consolidation of

the lung, or if there was an abnormal finding of tactile fremitus. The expected finding is

muffled voiced sounds rather than clear.

DIF: Cognitive Level: Apply REF: 204| 207-208| 210| 214| 217

TOP: Nursing Process: Assessment

MSC: NCLEX Patient Needs: Physiologic Integrity: Reduction of Risk Potential: System Specific

Assessments

COMPLETION

1. A patient tells the nurse that he has smoked 1 packs of cigarettes a day for 14 years. The

nurse records this as pack-years?

ANSWER:

21

1 packs of cigarettes 14 years = 21 pack-years.

DIF: Cognitive Level: Apply REF: 197

TOP: Nursing Process: Assessment

MSC: NCLEX Patient Needs: Physiologic Integrity: Reduction of Risk Potential: System Specific

Assessments


Chapter 12: Heart and Peripheral Vascular System

Test Bank

MULTIPLE CHOICE

1. A nurse informs a patient that her blood pressure is 128/78. The patient asks what the number

128 means. What is the nurse’s appropriate response? The 128 represents the pressure in your

blood vessels when:

a. “The ventricles relax and the aortic and pulmonic valves open.”

b. “The ventricles contract and the mitral and tricuspid valves close.”

c. “The ventricles contract and the mitral and tricuspid valves open.”

d. “The ventricles relax and the aortic and pulmonic valves close.”

ANSWER: B

A

B

C

D

Feedback

The aortic and pulmonic valves open during systole, but ventricles fill during

diastole.

During systole the ventricles contract, creating a pressure that closes the

atrioventricular (AV) valves (mitral and tricuspid).

During systole the ventricles contract, creating a pressure that closes the AV

valves (mitral and tricuspid).

The ventricles are relaxed and the aortic and pulmonic valves close during

diastole, rather than systole.

DIF: Cognitive Level: Understand REF: 225

TOP: Nursing Process: Assessment

MSC: NCLEX Patient Needs: Physiologic Integrity: Reduction of Risk Potential: System Specific

Assessments

2. A nurse determines that a patient has a heart rate of 42 beats per minute. What might be a

cause of this heart rate?

a. Sinoatrial (SA) node failure

b. Atrial bradycardia

c. A well-conditioned heart muscle

d. Left ventricular hypertrophy

ANSWER: A

A

B

C

D

Feedback

If the SA node is ineffective, the atrioventricular node may initiate contraction,

but at a rate of 40 to 60 beats/min.

The heart rate reflects the ventricular rate rather than the atrial rate.

Although well-conditioned athletes may have slower heart rates, this rate is too

slow for even an athlete.

Left ventricular hypertrophy alters the strength of contraction rather than the heart

rate.

DIF: Cognitive Level: Apply REF: 226

TOP: Nursing Process: Assessment


MSC: NCLEX Patient Needs: Physiologic Integrity: Physiologic Adaptation: Alterations in Body

Systems

3. While taking a history, a nurse learns that a patient had rheumatic heart disease as a child.

Based on this information, what abnormal data might this nurse expect to find during an

examination?

a. An extra beat just before the S 2 heart sound heard during auscultation

b. A raspy machine-like or blowing sound heard during auscultation

c. A prominent thrust of the heart against the chest wall felt on palpation

d. A visible indentation of pericardial tissue noted during inspection

ANSWER: B

A

B

C

D

Feedback

An extra beat just before the S 1 heart sound heard during auscultation is a

description of the S 4 heart sound that occurs when there is hypertrophy of the

ventricle.

A raspy machine-like or blowing sound heard during auscultation is a description

of a murmur that can develop after rheumatic heart disease.

A prominent thrust of the heart against the chest wall felt on palpation is a

description of a heave, which may occur from left ventricular hypertrophy due to

increased workload.

A visible indentation of pericardial tissue noted during inspection is a description

of a retraction that begins in the intercostal spaces and occurs with increased

respiratory effort.

DIF: Cognitive Level: Analyze REF: 229| 259

TOP: Nursing Process: Assessment

MSC: NCLEX Patient Needs: Physiologic Integrity: Physiologic Adaptation: Alteration in Body

Systems

4. A nurse is completing a symptom analysis with a patient complaining of chest pain. When

asked what makes the chest pain worse, the patient reports that coughing and sneezing

increase the chest pain. Based on these data, what does the nurse suspect as the cause of this

patient’s chest pain?

a. Stable angina

b. Esophageal reflux disease

c. Mitral valve prolapse

d. Costochondritis

ANSWER: D

A

B

C

D

Feedback

Physical exertion, emotional stress, and cold worsen the chest pain associated

with stable angina.

A spicy or acidic meal, alcohol, or lying supine may worsen the chest pain

associated with esophageal reflux.

Only occasional position changes worsen the chest pain associated with mitral

valve prolapse.

Coughing, deep breathing, laughing, and sneezing worsen the chest pain

associated with costochondritis.


DIF: Cognitive Level: Apply REF: 230-231

TOP: Nursing Process: Assessment

MSC: NCLEX Patient Needs: Physiologic Integrity: Physiologic Adaptation: Alteration in Body

Systems

5. The patient describes her chest pain as “squeezing, crushing, and 12 on a scale of 10.” This

pain started more than an hour ago while she was resting, and she also feels nauseous. Based

on these findings, the nurse should assess for which associated symptoms?

a. Tachycardia, tachypnea, and hypertension

b. Dyspnea, diaphoresis, and palpitations

c. Hyperventilation, fatigue, anorexia, and emotional strain

d. Fever, dyspnea, orthopnea, and friction rub

ANSWER: B

A

B

C

D

Feedback

Tachycardia, tachypnea, and hypertension are symptoms associated with cocaineinduced

chest pain.

Dyspnea, diaphoresis, and palpitations are symptoms associated with unstable

angina.

Hyperventilation, fatigue, anorexia, and emotional strain are symptoms associated

with panic disorder.

Fever, dyspnea, orthopnea, and friction rub are symptoms associated with

pericarditis.

DIF: Cognitive Level: Analyze REF: 230-231

TOP: Nursing Process: Assessment

MSC: NCLEX Patient Needs: Physiologic Integrity: Physiologic Adaptation: Alteration in Body

Systems

6. When auscultating the heart of a patient with pericarditis, the nurse expects to hear which

sound?

a. A systolic murmur

b. An S 3 heart sound

c. A friction rub

d. An S 4 heart sound

ANSWER: C

A

B

C

D

Feedback

Most systolic murmurs are caused by obstruction of the outflow of the semilunar

valves or by incompetent AV valves.

An S 3 heart sound occurs when there is heart failure.

Two classic findings of pericarditis are pericardial friction rub and chest pain.

An S 4 heart sound occurs when there is hypertrophy of the ventricle.

DIF: Cognitive Level: Apply REF: 230-231

TOP: Nursing Process: Assessment

MSC: NCLEX Patient Needs: Physiologic Integrity: Physiologic Adaptation: Alteration in Body

Systems


7. Which patient’s statement helps a nurse distinguish between chest pain originating from

pericarditis rather than from angina?

a. “No, I have not done anything to strain chest muscles.”

b. “If I take a deep breath, the pain gets much worse.”

c. “This pain feels like there’s an elephant sitting on my chest.”

d. “Whenever this pain happens, it goes right away if I lie down.”

ANSWER: B

A

B

C

D

Feedback

Chest pain from muscle strain may be aggravated by movement of arms.

The chest pain from pericarditis is aggravated by deep breathing, coughing, or

lying supine.

“This pain feels like there’s an elephant sitting on my chest” is associated with a

myocardial infarction.

Chest pain relieved by rest occurs with angina.

DIF: Cognitive Level: Analyze REF: 230-231

TOP: Nursing Process: Assessment

MSC: NCLEX Patient Needs: Physiologic Integrity: Physiologic Adaptation: Alteration in Body

Systems

8. While taking a history, a nurse learns that this patient experiences shortness of breath

(dyspnea). If the cause of the dyspnea is a cardiovascular problem, the nurse expects which

abnormal finding on examination?

a. Flat jugular neck veins

b. Red, shiny skin on the legs

c. Weak, thready peripheral pulses

d. Edema of the feet and ankles

ANSWER: D

A

B

C

D

Feedback

Flat jugular veins indicate a fluid deficit, which is not associated with dyspnea.

Red, shiny skin on the legs is associated with peripheral arterial disease and is not

associated with dyspnea.

Weak, thready peripheral pulses indicate fluid deficit, which is not associated

with dyspnea.

This patient may have heart failure. Edema of the feet occurs with right

ventricular heart failure. Dyspnea occurs with left ventricular heart failure.

DIF: Cognitive Level: Analyze REF: 230| 232-233

TOP: Nursing Process: Assessment

MSC: NCLEX Patient Needs: Physiologic Integrity: Physiologic Adaptation: Alteration in Body

Systems

9. A nurse is assessing a patient’s peripheral circulation. Which finding indicates venous

insufficiency of this patient’s legs?

a. Paresthesias and weak, thin peripheral pulses

b. Leg pain that can be relieved by walking


c. Edema that is worse at the end of the day

d. Leg pain that increases when the legs are lowered

ANSWER: C

A

B

C

D

Feedback

Paresthesias and weak, thin peripheral pulses are characteristics of arterial

insufficiencies rather than venous.

Pain caused by arterial insufficiency gets worse by walking, because walking

requires additional arterial blood.

Dependent edema is an indication of venous insufficiency.

Arterial pain is relieved by lowering the leg and aggravated by elevating the legs.

DIF: Cognitive Level: Apply REF: 233| 255

TOP: Nursing Process: Assessment

MSC: NCLEX Patient Needs: Physiologic Integrity: Physiologic Adaptation: Alteration in Body

Systems

10. A patient reports having leg pain while walking that is relieved with rest. Based on these data,

the nurse expects which finding on inspection and palpation of this patient?

a. 1+ edema of the feet and ankles bilaterally

b. The circumference of the right leg is larger than the left leg

c. Patchy petechiae and purpura of the lower extremities

d. Cool feet with capillary refill of toes greater than 3 seconds

ANSWER: D

A

B

C

D

Feedback

Edema of 1+ of the feet and ankles bilaterally is an indication of a venous

problem rather than an arterial problem.

When one leg is larger in circumference than the other, it could be due to

lymphedema or a deep vein thrombosis.

Petechiae and purpura of the lower extremities indicate a bleeding problem, such

as low platelets, rather than an arterial problem.

The pain while walking that is relieved by rest is called intermittent claudication

and is an indication of arterial insufficiency. Cool feet and prolonged capillary

refill also occur due to arterial insufficiency.

DIF: Cognitive Level: Analyze REF: 233| 241

TOP: Nursing Process: Assessment

MSC: NCLEX Patient Needs: Physiologic Integrity: Physiologic Adaptation: Alteration in Body

Systems

11. How does a nurse accurately palpate carotid pulses?

a. Two fingers of each hand are placed firmly over the right and left temples at the

same time.

b. One finger is placed gently in the space between the biceps and triceps muscles.

c. Two fingers are placed at the thumb side of the forearm at the wrist.

d. One finger is placed along the right and then the left medial sternocleidomastoid

muscle.


ANSWER: D

A

B

C

D

Feedback

Two fingers of each hand placed firmly over the right and left temples at the same

time is the correct procedure for palpating the temporal pulse.

One finger placed gently in the space between the biceps and triceps muscles is

the correct procedure for palpating the brachial pulse.

Two fingers placed at the thumb side of the forearm at the wrist is the correct

procedure for palpating the radial pulse.

One finger placed along the right and then the left medial sternocleidomastoid

muscle is the correct procedure for palpating the carotid pulses, checking each

side separately.

DIF: Cognitive Level: Understand REF: 234-235

TOP: Nursing Process: Assessment

MSC: NCLEX Patient Needs: Health Promotion and Maintenance: Techniques of Physical

Assessment

12. To document the palpation of a pulse, the nurse is correct in making which notation about the

rhythm?

a. “Rhythm 100 beats/min”

b. “Irregular rhythm”

c. “Rhythm noted at +2”

d. “Bounding rhythm”

ANSWER: B

A

B

C

D

Feedback

This notation refers to the rate rather than the rhythm.

The rhythm should be an equal pattern or spacing between beats. Irregular

rhythms without any pattern should be noted.

This notation refers to the amplitude rather than the rhythm.

This notation refers to the contour rather than the rhythm.

DIF: Cognitive Level: Apply REF: 236

TOP: Nursing Process: Assessment

MSC: NCLEX Patient Needs: Physiologic Integrity: Reduction of Risk Potential: System Specific

Assessments

13. A nurse expects which finding during a cardiovascular assessment of a healthy adult?

a. Visible, consistent pulsations of the jugular vein

b. Pink nail beds with a 90-degree angle at the base

c. Capillary refill of the toes greater than 5 seconds

d. Bruits heard on auscultation of the carotid arteries

ANSWER: A

A

B

Feedback

Visible, consistent pulsations of the jugular vein is an expected finding.

Pink nail beds with a 90-degree angle at the base is not a normal finding; the

angle at the base should be 160 degrees.


C

D

Capillary refill of the toes greater than 5 seconds is not a normal finding.

Capillary refills should be 2 seconds or less.

Bruits heard on auscultation of the carotid arteries is not a normal finding. Bruits

indicate occlusion of a blood vessel.

DIF: Cognitive Level: Apply REF: 235| 237

TOP: Nursing Process: Assessment

MSC: NCLEX Patient Needs: Physiologic Integrity: Reduction of Risk Potential: System Specific

Assessments

14. Which pulse may be a challenge for a nurse to palpate?

a. Temporal

b. Femoral

c. Popliteal

d. Dorsalis pedis

ANSWER: C

A

B

C

D

Feedback

The temporal pulse is palpated over the temporal bone on each side of the head.

For the femoral pulse, palpate below the inguinal ligament, midway between the

symphysis pubis and anterior superior iliac.

For the popliteal pulse, palpate the popliteal artery behind the knee in the

popliteal fossa to assess perfusion. This pulse may be difficult to find.

For the dorsalis pedis pulse, palpate on the inner aspect of the ankle below and

slightly behind the medial malleolus (ankle bone).

DIF: Cognitive Level: Understand REF: 241-242

TOP: Nursing Process: Assessment

MSC: NCLEX Patient Needs: Health Promotion and Maintenance: Techniques of Physical

Assessment

15. When assessing a patient with aortic valve stenosis, the nurse listens for which sound to detect

a thrill?

a. Sustained thrust of the heart against the chest wall during systole

b. Visible sinking of the tissues between and around the ribs

c. Fine, palpable vibration felt over the precordium

d. Bounding pulse noted bilaterally

ANSWER: C

A

B

C

D

Feedback

A sustained thrust of the heart against the chest wall during systole is a

description of a lift.

A visible sinking of the tissues between and around the ribs is a description of a

retraction.

A thrill is a palpable vibration over the precordium or artery.

A thrill feels like a palpable vibration rather than a bounding pulse.

DIF: Cognitive Level: Understand REF: 243| 248

TOP: Nursing Process: Assessment


MSC: NCLEX Patient Needs: Physiologic Integrity: Physiologic Adaptation: Alteration in Body

Systems

16. A nurse is having difficulty auscultating a patient’s heart sounds because the lung sounds are

too loud. What does the nurse ask the patient to do to improve hearing the heart sounds?

a. Lie in a supine position.

b. Cough.

c. Hold his or her breath for a few seconds.

d. Sit up and lean forward.

ANSWER: C

A

B

C

D

Feedback

Lying in a supine position will not reduce the noise of breathing.

Coughing may clear some secretions, but when the lung sounds are so noisy that

the heart sounds are difficult to hear, coughing is not sufficient to eliminate the

noise from respirations.

Holding the breath for a few seconds eliminates the noise of breathing long

enough to hear several cardiac cycles of heart sounds. The holding of the breath

can be repeated if needed to hear the heart sounds again.

Sitting up and leaning forward brings the heart closer to the thoracic wall, but will

not eliminate noise produced by the lungs.

DIF: Cognitive Level: Analyze REF: 244

TOP: Nursing Process: Assessment

MSC: NCLEX Patient Needs: Physiologic Integrity: Physiologic Adaptation: Alteration in Body

Systems

17. While assessing edema on a male patient’s lower leg, the nurse notices that there is a slight

imprint of his fingers where he palpated the patient’s leg. How does the nurse document this

finding?

a. No edema

b. 1+ edema

c. 2+ edema

d. 3+ edema

ANSWER: B

A

B

C

D

Feedback

No pit left after palpation indicates no edema.

A barely perceptible pit is detected after palpation.

A deeper pit that rebounds in a few seconds after palpation is 2+ edema.

A deep pit that rebounds in 10 to 20 seconds after palpation is 3+ edema.

DIF: Cognitive Level: Apply REF: 238

TOP: Nursing Process: Assessment

MSC: NCLEX Patient Needs: Physiologic Integrity: Physiologic Adaptation: Alteration in Body

Systems

18. Where does a nurse place a stethoscope to auscultate the mitral valve area? Choose the letter

that corresponds to the correct stethoscope placement.


a. A

b. B

c. D

d. E

ANSWER: D

A

B

C

D

Feedback

A is the location of the aortic valve area—second intercostal space, right sternal

border.

B is the location of the pulmonic valve area—fifty-second intercostal space, left

sternal border.

D is the location of the tricuspid valve area—fourth intercostal space, left sternal

border.

E is the location of the mitral valve area—the fifth intercostal space,

midclavicular line.

DIF: Cognitive Level: Understand REF: 245-246

TOP: Nursing Process: Assessment

MSC: NCLEX Patient Needs: Physiologic Integrity: Reduction of Risk Potential: System Specific

Assessments

19. Which valve does a nurse auscultate when the stethoscope is placed on the fourth intercostal

space at the left of the sternal border?

a. Pulmonic

b. Tricuspid

c. Mitral

d. Aortic

ANSWER: B

A

B

C

Feedback

Pulmonic valve sounds are best heard in the second intercostal space at the left of

the sternal border.

Tricuspid valve sounds are best heard in the fourth intercostal space at the left of

the sternal border.

Mitral valve sounds are best heard in the fifth intercostal space at the

midclavicular line.


D

Aortic valve sounds are best heard in the second intercostal space at the right of

the sternal border.

DIF: Cognitive Level: Remember REF: 245-246

TOP: Nursing Process: Assessment

MSC: NCLEX Patient Needs: Health Promotion and Maintenance: Techniques of Physical

Assessment

20. A patient reports that he has coronary artery disease with ventricular hypertrophy. Based on

these data, what finding should the nurse expect during assessment?

a. S 4 heart sound

b. Clubbing of fingers

c. Splitting of the S 1 heart sound

d. Pericardial friction rub

ANSWER: A

A

B

C

D

Feedback

An S 4 heart sound signifies a noncompliant or “stiff’’ ventricle. Coronary artery

disease is a major cause of a stiff ventricle.

Clubbing of fingers occurs due to chronic hypoxia rather than a stiff ventricle.

Splitting of the S 1 heart sound indicates a valve problem rather than ventricular

hypertrophy. When the mitral and tricuspid valves do not close at the same time,

S 1 sounds as if it were split into two sounds instead of one.

Pericardial friction rubs are caused by inflammation of the layers of the

pericardial sac.

DIF: Cognitive Level: Apply REF: 225-226| 248

TOP: Nursing Process: Assessment

MSC: NCLEX Patient Needs: Physiologic Integrity: Physiologic Adaptation: Alteration in Body

Systems

21. What does the S 2 heart sound represent?

a. The beginning of systole.

b. The closure of the aortic and pulmonic valves.

c. The closure of the tricuspid and mitral values

d. A split heard sound on exhalation

ANSWER: B

A

B

C

D

Feedback

The beginning of systole is the S 1 heart sound.

The second heart sound is made by the closing of these valves, which indicates

the beginning of diastole.

The tricupid and mitral valves create the S 1 heart sound.

A split sound on exhalation is not a correct statement.

DIF: Cognitive Level: Remember REF: 226| 244

TOP: Nursing Process: Assessment

MSC: NCLEX Patient Needs: Physiologic Integrity: Reduction of Risk Potential: System Specific

Assessments


22. How is the first heart sound (S 1 ) created?

a. Pulmonic and tricuspid valves close.

b. Mitral and aortic valves close.

c. Aortic and pulmonic valves close.

d. Mitral and tricuspid valves close.

ANSWER: D

A

B

C

D

Feedback

The pulmonic and tricuspid valves are the valves of the right side of the heart, and

they do not close simultaneously in the cardiac cycle.

The mitral and aortic valves are the valves of the left side of the heart, and they

do not close simultaneously in the cardiac cycle.

The aortic and pulmonic valves are the semilunar valves that create the second

heart sound.

The first heart sound (S 1 ) is made by the closing of the mitral (M1) and tricuspid

(T1) valves.

DIF: Cognitive Level: Remember REF: 226| 244

TOP: Nursing Process: Assessment

MSC: NCLEX Patient Needs: Physiologic Integrity: Reduction of Risk Potential: System Specific

Assessments

23. A nurse learns from a report that a patient has aortic stenosis. Where does the nurse place the

stethoscope to hear this stenotic valve?

a. Second intercostal space, right sternal border

b. Second intercostal space, left sternal border

c. Fourth intercostal space, left sternal border

d. Fifth intercostal space, left midclavicular line

ANSWER: A

A

B

C

D

Feedback

Second intercostal space, right sternal border is the location for listening to the

aortic valve.

Second intercostal space, left sternal border is the location for listening to the

pulmonic valve.

Fourth intercostal space, left sternal border is the location for listening to the

tricuspid valve.

Fifth intercostal space, left midclavicular line is the location for listening to the

mitral valve.

DIF: Cognitive Level: Apply REF: 244-246

TOP: Nursing Process: Assessment

MSC: NCLEX Patient Needs: Health Promotion and Maintenance: Techniques of Physical

Assessment

24. A nurse who is auscultating a patient’s heart hears a harsh sound, a raspy machine-like

blowing sound, after S 1 and before S 2 . How does this nurse document this finding?

a. An opening snap


b. A diastolic murmur

c. A systolic murmur

d. A pericardial friction rub

ANSWER: C

A

B

C

D

Feedback

An opening snap is caused by the opening of the mitral or tricuspid valve and is

an abnormal sound heard in diastole when either valve is thickened, stenotic, or

deformed. The sounds are high pitched and occur early in diastole.

A diastolic murmur is heard after the S 2 heart sound at the beginning of diastole.

The blowing sound is a murmur. The nurse determines whether it is a systolic or a

diastolic murmur based on where it is heard during the cardiac cycle. S 1 indicates

the beginning of systole; the sound is made by the closing of the mitral and

tricuspid valves, which is followed by ventricular contraction or systole.

Pericardial friction rubs have a rubbing sound that is usually present in both

diastole and systole, and is best heard over the apical area.

DIF: Cognitive Level: Apply REF: 248-249

TOP: Nursing Process: Assessment

MSC: NCLEX Patient Needs: Physiologic Integrity: Physiologic Adaptation: Alteration in Body

Systems

25. A nurse determines that a patient’s jugular venous pressure is 3.5 inches. What additional data

does the nurse expect to find?

a. Weight loss

b. Tented skin turgor

c. Peripheral edema

d. Capillary refill greater than 5 seconds

ANSWER: C

A

B

C

D

Feedback

Weight loss occurs with loss of fluid rather than fluid overload.

Tented skin turgor occurs with fluid loss rather than fluid overload.

The pressure should not rise more than 1 inch (2.5 cm) above the sternal angle. A

pressure of 3.5 inches indicates fluid volume excess, which causes peripheral

edema due to excessive fluid in blood vessels.

Capillary refill greater than 5 seconds occurs with arterial insufficiency rather

than fluid overload.

DIF: Cognitive Level: Analyze REF: 252-253| 260-261

TOP: Nursing Process: Assessment

MSC: NCLEX Patient Needs: Physiologic Integrity: Physiologic Adaptation: Alteration in Body

Systems

26. How does a nurse assess the competence of venous valves in patients who have varicose

veins?

a. Notes how quickly veins fill after lifting one leg above the level of the heart

b. Assesses for Homan sign in both lower extremities while the patient is supine

c. Assesses capillary refill on the toes of both feet while the patient is sitting in the


chair

d. Measures the circumference of both calves and compares the results

ANSWER: A

A

B

C

D

Feedback

Noting how quickly veins fill after lifting one leg above the level of the heart is

the procedure to test for incompetent veins.

Homan sign is an unreliable test for deep vein thrombosis.

Assessing capillary refill assesses perfusion (blood flow from arteries) rather than

competence of venous valves.

Measuring the circumference of both calves and comparing the results is used to

assess deep vein thrombosis.

DIF: Cognitive Level: Apply REF: 255

TOP: Nursing Process: Assessment

MSC: NCLEX Patient Needs: Physiologic Integrity: Physiologic Adaptation: Alteration in Body

Systems

27. Which patient does the nurse identify as the one at greatest risk for hypertension?

a. Woman with coronary artery disease

b. Hispanic male

c. Obese male with diabetes mellitus

d. Postmenopausal woman

ANSWER: C

A

B

C

D

Feedback

Although hypertension is a risk factor for coronary artery disease, coronary artery

disease is not a risk factor for hypertension.

Although male gender is a risk factor, African-American men have a greater risk

than Hispanic men.

Obese men with diabetes mellitus have three risk factors: obesity, gender, and

comorbidity of diabetes mellitus.

Postmenopausal women do not have an increased risk for developing

hypertension.

DIF: Cognitive Level: Apply REF: 260

TOP: Nursing Process: Assessment

MSC: NCLEX Patient Needs: Health Promotion and Maintenance: Health Promotion Programs

28. After two separate office visits, the nurse suspects that a patient is developing Stage 1

hypertension based on which consecutive blood pressure readings?

a. Visit 1, 118/78; Visit 2, 116/76

b. Visit 1, 130/88; Visit 2, 134/88

c. Visit 1, 144/92; Visit 2, 150/90

d. Visit 1, 162/100; Visit 2, 166/104

ANSWER: C

Feedback


A These readings are within normal limits.

B These readings are prehypertension because the systolic pressures are 120 to 139

and diastolic pressures are greater than 80 mm Hg.

C These readings are stage 1 because the systolic pressures are 140 to 159 and

diastolic pressures are 90 mm Hg or greater.

D These readings are stage 2 because the systolic pressures are greater than 160 and

diastolic pressures are 100 mm Hg or greater.

DIF: Cognitive Level: Analyze REF: 261

TOP: Nursing Process: Assessment

MSC: NCLEX Patient Needs: Physiologic Integrity: Physiologic Adaptation: Alteration in Body

Systems

MULTIPLE RESPONSE

1. During a health fair, the nurse is alert for which risk factors for hypertension? Select all that

apply.

a. Excessive protein intake

b. Having parents with hypertension

c. Excessive alcohol intake

d. Being Asian

e. Experiencing persistent stress

f. Elevated serum lipids

ANSWER: B, C, E, F

Correct: These are all risk factors for hypertension.

Incorrect: Excessive protein is not a risk factor for hypertension, but excessive sodium intake

is a risk factor. Being Asian is not a risk factor, but being African-American is a risk factor.

DIF: Cognitive Level: Analyze REF: 261

TOP: Nursing Process: Assessment

MSC: NCLEX Patient Needs: Physiologic Integrity: Reduction of Risk Potential: Potential for

Alteration in Body Systems

2. A patient with heart failure reports having a cough with frothy sputum and awakening during

the night to urinate. Based on this information, what abnormal data might this nurse expect to

find during an examination? Select all that apply.

a. S 4 heart sound

b. Dyspnea

c. Jugular vein distention

d. Pericardial friction rub

e. Edema of ankle and feet at the end of the day

f. S 3 heart sound

ANSWER: B, C, E, F

Correct: All of these manifestations are consistent with fluid overload that occurs in heart

failure because the cardiac output is decreased.


Incorrect: S 4 heart sounds signifies a noncompliant or “stiff’’ ventricle. Hypertrophy of the

ventricle precedes a noncompliant ventricle. Also, coronary artery disease is a major cause of

a stiff ventricle. Pericardial friction rubs are caused by inflammation of the layers of the

pericardial sac.

DIF: Cognitive Level: Analyze REF: 230| 233| 238| 247| 252-253| 260

TOP: Nursing Process: Assessment

MSC: NCLEX Patient Needs: Physiologic Integrity: Physiologic Adaptation: Alterations in Body

Systems

3. What findings does the nurse expect when assessing the cardiovascular system of a healthy

adult? Select all that apply.

a. Heart rate of 102 beats/min

b. S 1 and S 2 present with regular rhythm

c. Capillary refill greater than 3 seconds

d. Blood pressure of 124/86

e. Warm, elastic turgor

f. Pulse of smooth contour with 2+ amplitude

ANSWER: B, E, F

Correct: These are all expected findings.

Incorrect: A heart rate of 102 beats/min is tachycardia. Capillary refill should be 2 seconds or

less. Blood pressure of 124/86 is prehypertension. Normal is less than 120 and less than 80

mm Hg.

DIF: Cognitive Level: Analyze REF: 238-239| 241| 244| 259

TOP: Nursing Process: Assessment

MSC: NCLEX Patient Needs: Physiologic Integrity: Reduction of Risk Potential: System Specific

Assessments

COMPLETION

1. A patient’s blood pressure using the posterior tibial pulse is 104/72 while blood pressure using

the brachial pulse is 112/84. This patient’s ankle-brachial index (ABI) is .

ANSWER:

0.92

Posterior tibial systolic pressure (104) divided by the brachial systolic pressure (112) = 0.92.

The systolic pressures are the numbers used to calculate the ABI.

DIF: Cognitive Level: Apply REF: 256

TOP: Nursing Process: Assessment

MSC: NCLEX Patient Needs: Physiologic Integrity: Reduction of Risk Potential: System Specific

Assessments


Chapter 13: Abdomen and Gastrointestinal System

Test Bank

MULTIPLE CHOICE

1. A patient tells the nurse, “I’ve been having pain in my belly for several days that gets worse

after eating.” Which datum from the symptom analysis is consistent with the nurse’s suspicion

of peptic ulcer disease?

a. Gnawing epigastric pain radiates to the back or shoulder that worsens after eating.

b. Sharp midepigastric pain radiates to the jaw.

c. Intermittent cramping pain in the left lower quadrant is relieved by defecation.

d. Colicky pain is felt near the umbilicus with vomiting and constipation.

ANSWER: A

A

B

C

D

Feedback

Gnawing epigastric pain that radiates to the back or shoulder and worsens after

eating is a symptom that is consistent with peptic ulcer disease.

Sharp midepigastric pain that radiates to the jaw is not a symptom of peptic ulcer

disease.

Intermittent cramping pain in the left lower quadrant relieved by defecation is a

symptom of diverticular disease rather than peptic ulcer disease.

Colicky pain felt near the umbilicus with vomiting and constipation is a symptom

of an intestinal obstruction rather than peptic ulcer disease.

DIF: Cognitive Level: Apply REF: 268-269

TOP: Nursing Process: Assessment

MSC: NCLEX Patient Needs: Physiologic Integrity: Physiologic Adaptation: Alteration in Body

Systems

2. During an assessment for abdominal pain, a patient reports a colicky abdominal pain and pain

in the right shoulder that gets worse after eating fried foods. What question does the nurse ask

to confirm the suspicion of cholelithiasis?

a. “Have you noticed any swelling in your ankles or feet at the end of the day?”

b. “Have you noticed a change in the color of your urine or stools?”

c. “Have you vomited up any blood in the last 24 hours?”

d. “Have you experienced fever, chills, or sweating?”

ANSWER: B

A

B

C

D

Feedback

This question is related to fluid retention, which may be asked if the patient has

renal or heart failure.

Gallstones can obstruct the flow of bile to the gastrointestinal tract making urine

darker and stools lighter in color.

This question applies if the patient has peptic ulcer disease or esophageal varices.

This question applies if the patient has gastroenteritis or a urinary tract infection.

DIF: Cognitive Level: Analyze REF: 268-269

TOP: Nursing Process: Assessment


MSC: NCLEX Patient Needs: Physiologic Integrity: Reduction of Risk Potential: System Specific

Assessments

3. A patient reports having frequent heartburn. Which question does the nurse ask in response to

this information?

a. “Has your abdomen been distended when you feel the heartburn?”

b. “What have you eaten in the last 24 hours?”

c. “Is there a history of heart disease in your family?”

d. “How long after eating do you have heartburn?”

ANSWER: D

A

B

C

D

Feedback

The question “Has your abdomen been distended when you feel the heartburn?”

is not related to the heartburn. Distention usually is related to intestinal

obstruction or liver disease.

The question “What have you eaten in the last 24 hours?” relates more to

gastroenteritis. Indigestion is usually caused by food eaten in the last meal rather

than in the last 24 hours.

The question “Is there a history of heart disease in your family?” points to

myocardial ischemia. Although heartburn may be a symptom of myocardial

ischemia, asking the patient about the family history is not relevant in this case.

Asking “How long after eating do you have heartburn?” can aid in determining if

the patient has gastroesophageal reflux disease or a hiatal hernia. Both are

common disorders that cause indigestion a few hours after meals.

DIF: Cognitive Level: Apply REF: 268-270

TOP: Nursing Process: Assessment

MSC: NCLEX Patient Needs: Health Promotion and Maintenance: Techniques of Physical

Assessment

4. A patient reports having abdominal distention. The nurse notices that the patient’s sclerae are

yellow. What question is appropriate for the nurse to ask in response to this information?

a. “Has there been a change in your usual pattern of urination?”

b. “Have you had any nausea or vomiting?”

c. “Has there been a change in your bowel habits?”

d. “Have you had indigestion or heartburn?”

ANSWER: B

A

B

C

D

Feedback

“Has there been a change in your usual pattern of urination?” is not a question

related to the abdominal distention and jaundice.

“Have you had any nausea or vomiting?” is an appropriate question because the

nurse suspects the patient may have a liver disease based on the abdominal

distention and jaundice. The nurse interprets the relationship with data gathered

from the history and the observation.

“Has there been a change in your bowel habits?” is a question that may be related

to the abdominal distention, but not the jaundice.

“Have you had indigestion or heartburn?” is not a question related to the

abdominal distention and jaundice. It applies more to gastric disorders, such as


gastroesophageal reflux disease or hiatal hernia.

DIF: Cognitive Level: Analyze REF: 269-270

TOP: Nursing Process: Assessment

MSC: NCLEX Patient Needs: Physiologic Integrity: Physiologic Adaptation: Alteration in Body

Systems

5. A patient reports having abdominal distention and having vomited several times yesterday and

today. What question is appropriate for the nurse to ask in response to this information?

a. “Has there been a change in your usual pattern of urination?”

b. “Did you have heartburn before the vomiting?”

c. “What did the vomitus look like?”

d. “Have you noticed a change in the color of your urine or stools?”

ANSWER: C

A

B

C

D

Feedback

“Has there been a change in your usual pattern of urination?” is not a question

related to abdominal distention and vomiting.

Have you noticed a change in the color of your urine or stools?” is not a question

related to abdominal distention and vomiting. It is related to elevated bilirubin

from liver or gallbladder disease and is accompanied by jaundice.

“What did the vomitus look like?” is an appropriate question because the

characteristics of the vomitus may help determine its cause. Acute gastritis leads

to vomiting of stomach contents, obstruction of the bile duct results in greenishyellow

vomitus, and an intestinal obstruction may cause a fecal odor to the

vomitus.

This is not a question related to the abdominal distention and vomiting. Heartburn

applies more to gastric disorders, such as gastroesophageal reflux disease or hiatal

hernia.

DIF: Cognitive Level: Apply REF: 268| 270

TOP: Nursing Process: Assessment

MSC: NCLEX Patient Needs: Physiologic Integrity: Physiologic Adaptation: Alteration in Body

Systems

6. A patient reports a change in the usual pattern of urination. What question does the nurse ask

to determine if incontinence is the reason for these symptoms?

a. “Do you have the feeling that you cannot wait to urinate?”

b. “Are you urinating a large amount each time you go to the bathroom?”

c. “Has the color of your urine changed lately?”

d. “Have you noticed any swelling in your ankles at the end of the day?”

ANSWER: A

A

B

Feedback

“Do you have the feeling that you cannot wait to urinate?” is a question that asks

about urgency, a symptom of incontinence.

Are you urinating a large amount each time you go to the bathroom?” is not a

question related to incontinence. Usually patients with incontinence void

frequently in small amounts.


C

D

Has the color of your urine changed lately?” is a question that is asked when the

nurse suspects the patient has gallbladder or liver disease.

Have you noticed any swelling in your ankles at the end of the day?” is a question

that relates to patients who have renal or heart disease.

DIF: Cognitive Level: Apply REF: 270-271

TOP: Nursing Process: Assessment

MSC: NCLEX Patient Needs: Physiologic Integrity: Physiologic Adaptation: Alteration in Body

Systems

7. In assessing a patient with renal disease, the nurse palpates edema in both ankles and feet.

Based on this finding, what question does the nurse ask the patient?

a. “Have you had any pain in your abdomen?”

b. “Have you had an unexpected weight gain?”

c. “Have you noticed a change in the color of your skin?”

d. “Have you had any nausea or vomiting?”

ANSWER: B

A

B

C

D

Feedback

“This question does not relate to renal disease. The pain experienced with renal

disease is usually flank pain over the costovertebral angle.

This question relating to weight gain and edema suggests fluid retention that

occurs with renal or heart disease, particularly renal failure.

This question does not relate to renal disease. It might relate to liver or

gallbladder disease if the change in skin color was yellow, indicating jaundice.

This question usually relates to disorders within the gastrointestinal tract itself

and not renal disease.

DIF: Cognitive Level: Apply REF: 270-271

TOP: Nursing Process: Assessment

MSC: NCLEX Patient Needs: Physiologic Integrity: Physiologic Adaptation: Alteration in Body

Systems

8. A patient reports having abdominal distention. The nurse observes that the patient’s sclerae are

yellow. Which abnormal finding does the nurse anticipate on examination of this patient’s

abdomen?

a. Decreased bowel sounds in all quadrants

b. Glistening or taut skin of the abdomen

c. Bulge in the abdomen when coughing

d. Bruit around the umbilicus

ANSWER: B

A

B

Feedback

Decreased bowel sounds in all quadrants may be present if the abdominal

distention was from an intestinal obstruction, but the observation of jaundice

suggests liver disease, which does not decrease bowel sounds.

Glistening or taut skin of the abdomen is consistent with ascites that appear as

abdominal distention. Jaundice and ascites suggest liver disease. There would

also be an increase in abdominal girth.


C

D

A bulge in the abdomen when coughing is a finding associated with abdominal or

incisional hernias.

Bruit around the umbilicus is a finding associated with an abdominal aortic

aneurysm.

DIF: Cognitive Level: Analyze REF: 270

TOP: Nursing Process: Assessment

MSC: NCLEX Patient Needs: Physiologic Integrity: Physiologic Adaptation: Alteration in Body

Systems

9. When inspecting a patient’s abdomen, which finding does the nurse note as normal?

a. Engorgement of veins around the umbilicus

b. Sudden bulge at the umbilicus when coughing

c. Visible peristalsis in all quadrants

d. Silver-white striae extending from the umbilicus

ANSWER: D

A

B

C

D

Feedback

Engorgement of veins around the umbilicus is an abnormal finding.

Sudden bulge at the umbilicus when coughing is an abnormal finding and may

indicate a hernia.

Visible peristalsis in all quadrants is an abnormal finding.

Silver-white striae extending from the umbilicus is a normal finding, particularly

in women who have been pregnant or in any adult who has lost weight after

having an obese abdomen.

DIF: Cognitive Level: Understand REF: 272

TOP: Nursing Process: Assessment

MSC: NCLEX Patient Needs: Health Promotion and Maintenance: Techniques of Physical

Assessment

10. When inspecting a patient’s abdomen, the nurse notes which finding as abnormal?

a. Protruding abdomen with skin that is lighter in color than the arms and legs

b. Marked rhythmic pulsation to the left of the midline

c. Faint, fine vascular network

d. Small shadows created by changes in contour

ANSWER: B

A

B

C

D

Feedback

Obesity may cause a protruding abdomen and although obesity is not an indicator

of health, it does not necessarily indicate a disease is present.

Marked rhythmic pulsation to the left of the midline is an abnormal finding that

may indicate an abdominal aortic aneurysm.

A faint, fine vascular network is a normal finding. If the vessels were engorged, it

would be an abnormal finding.

Small shadows created by changes in contour are a normal finding and they are

seen by using a light source to inspect the contour.

DIF: Cognitive Level: Understand REF: 272


TOP: Nursing Process: Assessment

MSC: NCLEX Patient Needs: Health Promotion and Maintenance: Techniques of Physical

Assessment

11. On inspection of a female patient’s abdomen, the nurse asks the patient to raise her head

without using her arms and notes a midline bulge. What is the appropriate response of the

nurse at this time?

a. Ask the patient to cough to see if the bulge reappears.

b. Auscultate the patient’s abdomen for hypoactive bowel sounds.

c. Document this as a normal finding and continue the examination.

d. Perform light and deep palpation of the abdomen.

ANSWER: C

A

B

C

D

Feedback

Ask the patient to cough to see if the bulge reappears. A bulge that appears with

coughing is an abnormal finding revealed by the increase in intrathoracic pressure

during the cough.

Auscultating the patient’s abdomen for hypoactive bowel sounds is not indicated

because the bulge is a normal finding.

Document this as a normal finding and continue the examination. This is a normal

finding on a patient

Performing light and deep palpation of the abdomen are not indicated because the

bulge is a normal finding.

DIF: Cognitive Level: Understand REF: 273

TOP: Nursing Process: Assessment

MSC: NCLEX Patient Needs: Physiologic Integrity: Physiologic Adaptation: Alteration in Body

Systems

12. A nurse notices abdominal distention when inspecting a patient’s abdomen. What action does

the nurse take next to gain further objective data?

a. Place a measuring tape around the superior iliac crests.

b. Assist the patient to turn on to the left side and then the right side.

c. Ask the patient to cough while lying supine.

d. Use the fingertips to sharply strike one side of the abdomen.

ANSWER: A

A

B

C

D

Feedback

This is the procedure for measuring abdominal girth.

This procedure is unnecessary. The distention will remain in a side-lying position.

Having the patient cough is used to assess for bulges rather than distention.

This is part of the procedure to test for a fluid wave, which is not indicated in this

patient.

DIF: Cognitive Level: Understand REF: 273

TOP: Nursing Process: Assessment

MSC: NCLEX Patient Needs: Physiologic Integrity: Physiologic Adaptation: Alteration in Body

Systems


13. A nurse inspects the abdomen for skin color, surface characteristics, and surface movement.

What part of the abdominal assessment does the nurse perform next?

a. Palpate lightly for tenderness and muscle tone.

b. The tip of the middle finger of the dominant hand strikes the nail of the middle

finger touching the skin of the abdomen.

c. Palpate deeply for masses or aortic pulsation.

d. Percuss for tones.

ANSWER: B

A

B

C

D

Feedback

Palpating lightly for tenderness and muscle tone is performed after auscultation.

Auscultation for bowel sounds occurs before palpating and percussing the

abdomen.

Palpating deeply for masses or aortic pulsation is performed after light palpation.

Percussion for tones is performed after palpation.

DIF: Cognitive Level: Apply REF: 273

TOP: Nursing Process: Assessment

MSC: NCLEX Patient Needs: Health Promotion and Maintenance: Techniques of Physical

Assessment

14. How does the nurse accurately assess bowel sounds?

a. Press the diaphragm of the stethoscope firmly against the abdomen in each

quadrant.

b. Hold the diaphragm of the stethoscope lightly against the abdomen in each

quadrant.

c. Press the bell of the stethoscope firmly against the abdomen in each quadrant.

d. Hold the bell of the stethoscope lightly against the abdomen in each quadrant.

ANSWER: B

A

B

C

D

Feedback

Pressing the diaphragm of the stethoscope firmly against the abdomen in each

quadrant is not the correct technique for listening to bowel sounds.

Holding the diaphragm lightly against the abdomen in each quadrant is the correct

technique for listening to bowel sounds.

The bell is used to listen to vascular sounds of the abdomen, which are normally

not heard.

The bell is used to listen to vascular sounds of the abdomen, which are normally

not heard.

DIF: Cognitive Level: Understand REF: 273

TOP: Nursing Process: Assessment

MSC: NCLEX Patient Needs: Health Promotion and Maintenance: Techniques of Physical

Assessment

15. When auscultating a patient’s abdomen using the bell of the stethoscope, the nurse hears soft,

low-pitched murmurs over the right and left upper midline. What do these sounds indicate?

a. Expected peristalsis

b. Femoral artery stenosis


c. Renal artery stenosis

d. Hyperactive bowel sounds

ANSWER: C

A

B

C

D

Feedback

Expected peristalsis would be heard using the diaphragm of the stethoscope and

would be a gurgling sound.

Femoral artery stenosis is a vascular sound heard with the bell, but located in the

lower abdomen.

Renal artery stenosis is a vascular sound heard with the bell and located in the

upper abdomen.

Hyperactive bowel sounds would be heard using the diaphragm and would be

present in all quadrants.

DIF: Cognitive Level: Understand REF: 274

TOP: Nursing Process: Assessment

MSC: NCLEX Patient Needs: Physiologic Integrity: Physiologic Adaptation: Alteration in Body

Systems

16. What sound does a nurse expect to hear when using the bell of the stethoscope over the

epigastric area of the abdomen of a healthy patient?

a. Bowel sounds

b. Venous hum

c. Soft, low-pitched murmur

d. No sounds

ANSWER: D

A

B

C

D

Feedback

Bowel sounds are heard with the diaphragm of the stethoscope.

Venous hum is not a normal finding.

Soft, low-pitched murmur is not a normal finding.

The bell is used to listen for vascular sounds and normally no vascular sounds are

heard in the abdomen.

DIF: Cognitive Level: Understand REF: 274

TOP: Nursing Process: Assessment

MSC: NCLEX Patient Needs: Physiologic Integrity: Physiologic Adaptation: Alteration in Body

Systems

17. What instructions does the nurse give a patient before palpating the abdomen?

a. Bend the knees.

b. Take a deep breath and hold it.

c. Take a deep breath and cough.

d. Place the hands over the head.

ANSWER: A

A

B

Feedback

Bend the knees to relax the abdominal muscles.

This action is not needed to assess the abdomen.


C

D

This action is used to detect bulges in the abdomen, but not used before palpation.

This action is not needed to assess the abdomen.

DIF: Cognitive Level: Remember REF: 275

TOP: Nursing Process: Assessment

MSC: NCLEX Patient Needs: Physiologic Integrity: Physiologic Adaptation: Alteration in Body

Systems

18. A patient reports intermittent cramping abdominal pain that is relieved by having a bowel

movement. The patient complains of having the pain at this time, which is why she is seeking

care. Which abnormal finding does the nurse anticipate finding on examination of this

patient’s abdomen?

a. Decreased bowel sounds

b. Bulge in the abdomen when coughing

c. Palpable mass in the left lower quadrant

d. Bruit around the umbilicus

ANSWER: C

A

B

C

D

Feedback

Decreased bowel sounds are not expected if the patient is having bowel

movements.

Bulge in the abdomen when coughing is a finding associated with abdominal or

incisional hernias.

Palpable mass in the left lower quadrant is expected when interpreted with other

data—age of the patient, intermittent cramping abdominal pain relieved by a

bowel movement—as consistent with diverticular disease.

Bruit around the umbilicus is a finding associated with an abdominal aortic

aneurysm.

DIF: Cognitive Level: Analyze REF: 269| 276

TOP: Nursing Process: Assessment

MSC: NCLEX Patient Needs: Physiologic Integrity: Physiologic Adaptation: Alteration in Body

Systems

19. Using deep palpation of a patient’s epigastrium, a nurse feels a rhythmic pulsation of the

aorta. Based on this finding, what is the nurse’s most appropriate response?

a. Auscultate this area using the bell of the stethoscope.

b. Percuss the area for tones.

c. Ask the patient if there is pain in this area.

d. Document this as a normal finding.

ANSWER: D

A

B

C

D

Feedback

Auscultating this area using the bell of the stethoscope is not necessary because

this is a normal finding. Vascular sounds are usually not heard.

Percussing the area for tones is not necessary because this is a normal finding.

Asking the patient if there is pain in this area is not necessary because this is a

normal finding.

Document this as a normal finding. The aorta is often palpable at the epigastrium.


DIF: Cognitive Level: Apply REF: 276

TOP: Nursing Process: Assessment

MSC: NCLEX Patient Needs: Physiologic Integrity: Reduction of Risk Potential: System Specific

Assessments

20. What technique does a nurse use when performing deep palpation of a patient’s abdomen?

a. Places the left hand under the ribs to lift them up

b. Asks the patient to breathe slowly through the mouth

c. Positions the patient on the right side with knees flexed

d. Uses the heel of the hand to depress the abdomen

ANSWER: B

A

B

C

D

Feedback

Placing the left hand under the ribs to lift them up is the technique for palpating

the liver.

Asking the patient to breathe slowly through the mouth while the nurse uses the

pads of the fingers to depress the abdomen is the correct procedure.

Positioning the patient on the right side with knees flexed is an alternate strategy

for palpating the spleen.

Using the heel of the hand to depress the abdomen is not a correct technique; the

pads of the fingers are used.

DIF: Cognitive Level: Understand REF: 275

TOP: Nursing Process: Assessment

MSC: NCLEX Patient Needs: Health Promotion and Maintenance: Techniques of Physical

Assessment

21. To correctly percuss the abdomen, a nurse places the distal aspect of the middle finger of the

nondominant hand against the skin of the abdomen, and the other fingers are spread apart and

slightly lifted off the skin. How does the nurse use the fingers of the dominant hand?

a. The pad of the middle finger strikes the distal interphalangeal joint of the middle

finger touching the skin of the abdomen.

b. The tip of the middle finger strikes the nail of the middle finger touching the skin

of the abdomen.

c. The tip of the middle finger strikes the distal interphalangeal joint of the middle

finger touching the skin of the abdomen.

d. The pads of the index and middle fingers strike the nail of the middle finger

touching the skin of the abdomen.

ANSWER: C

A

B

C

D

Feedback

This description is incorrect because the tip of the finger is used rather than the

pad.

This description is incorrect because the distal joint is struck rather than the nail.

This is the correct technique.

This description is incorrect because the tip of the middle finger strikes the distal

joint.


DIF: Cognitive Level: Understand REF: 276

TOP: Nursing Process: Assessment

MSC: NCLEX Patient Needs: Health Promotion and Maintenance: Techniques of Physical

Assessment

22. Which sound does a nurse expect to hear when percussing a patient’s abdomen?

a. Tympany over all quadrants

b. Resonance over the upper quadrants and tympany in the lower quadrants

c. Dull sounds over the upper quadrants and hollow sounds over the lower quadrants

d. Dull sounds over the stomach and resonant sounds over the bladder

ANSWER: A

A

B

C

D

Feedback

Tympany over all quadrants is a normal finding, which represents the presence of

gas.

Resonance in the upper quadrants and tympany over the lower quadrants are not

normal findings. There would be tympany in the lower quadrants, but also in the

upper quadrants.

Dull sounds over the lower quadrants and hollow sounds over the upper quadrants

are not normal findings. There would be tympany over the lower quadrants, but

also in the upper quadrants.

Dull sounds over the stomach and resonant sounds over the bladder are not

normal findings.

DIF: Cognitive Level: Understand REF: 276

TOP: Nursing Process: Assessment

MSC: NCLEX Patient Needs: Physiologic Integrity: Reduction of Risk Potential: System Specific

Assessments

23. A nurse expects which finding when assessing the abdomen of a patient who has been unable

to void for 12 hours?

a. Absent bowel sounds

b. Hyperactive bowel sounds

c. Tympanic tones over the lower abdomen

d. Dull tones over the suprapubic area

ANSWER: D

A

B

C

D

Feedback

Absent bowel sounds is incorrect because the bowel sounds would not be affected

by a full bladder.

Hyperactive bowel sounds is incorrect because the bowel sounds would not be

affected by a full bladder.

Tympanic tones over the lower abdomen is incorrect because tympany sound is

created by gas in the abdomen.

Dull tones over the suprapubic area would be found. The urine in the bladder

would create a dull sound when the bladder is percussed similar to the sounds

when an abdominal mass is present.

DIF: Cognitive Level: Apply REF: 276


TOP: Nursing Process: Assessment

MSC: NCLEX Patient Needs: Physiologic Integrity: Reduction of Risk Potential: System Specific

Assessments

24. When assessing an adult’s liver, the nurse percusses the lower border and finds it to be 5 cm

below the costal margin. What is the nurse’s appropriate action at this time?

a. Document this as an expected finding for this adult.

b. Palpate the gallbladder for tenderness.

c. Palpate the upper liver border on deep inspiration.

d. Use the hooking technique to palpate the lower border of the liver.

ANSWER: C

A

B

C

D

Feedback

Documenting this as a normal finding for an adult patient is incorrect because this

finding indicates an enlarged liver.

Palpating the gallbladder for tenderness is not indicated for an enlarged liver.

Palpating the upper border of the liver on deep inspiration is the correct technique

to use when an enlarged liver is found (as indicated by the liver being percussed 5

cm below the costal margin).

Using the hooking technique to palpate the lower border of the liver is not needed

because the liver is enlarged.

DIF: Cognitive Level: Apply REF: 277

TOP: Nursing Process: Assessment

MSC: NCLEX Patient Needs: Physiologic Integrity: Reduction of Risk Potential: System Specific

Assessments

25. Which location does a nurse select when palpating a patient’s liver?

a. A (right lower quadrant)

b. B (right upper quadrant)

c. C (left upper quadrant)

d. D (left lower quadrant)

ANSWER: B

A

B

C

Feedback

The majority of the liver is located in the right upper quadrant of the abdomen.

The majority of the liver is located in the right upper quadrant of the abdomen.

C is the left upper quadrant.


D

D is the left lower quadrant.

DIF: Cognitive Level: Understand REF: 273| 277

TOP: Nursing Process: Assessment

MSC: NCLEX Patient Needs: Health Promotion and Maintenance: Techniques of Physical

Assessment

26. On palpation of the left upper quadrant of the abdomen of a female patient, the nurse notes

tenderness. This finding may indicate a disorder in which organ?

a. Spleen

b. Gallbladder

c. Sigmoid colon

d. Left ovary

ANSWER: A

A

B

C

D

Feedback

The spleen is located in the left upper quadrant of the abdomen.

The gallbladder is located in the right upper quadrant of the abdomen.

The sigmoid colon is located in the left lower quadrant of the abdomen.

The left ovary is located in the left lower quadrant of the abdomen.

DIF: Cognitive Level: Apply REF: 265| 278

TOP: Nursing Process: Assessment

MSC: NCLEX Patient Needs: Physiologic Integrity: Physiologic Adaptation: Alteration in Body

Systems

27. The nurse recognizes which clinical finding as expected on palpation of the abdomen?

a. Inability to palpate the spleen

b. Left kidney rounded at 2 cm below the costal margin

c. Slight tenderness of the gallbladder on light palpation

d. Bounding pulsation of the aorta over the umbilicus

ANSWER: A

A

B

C

D

Feedback

Inability to palpate the spleen is the expected finding on palpating the abdomen.

A rounded left kidney at 2 cm below the costal margin is not an expected finding.

Kidneys are usually not palpated.

Slight tenderness of the gallbladder on light palpation is not an expected finding;

the gallbladder is usually not palpable.

Bounding pulsation of the aorta over the umbilicus would be an abnormal

finding, perhaps indicating an aneurysm.

DIF: Cognitive Level: Understand REF: 280

TOP: Nursing Process: Assessment

MSC: NCLEX Patient Needs: Physiologic Integrity: Reduction of Risk Potential: System Specific

Assessments


28. The nurse observes a patient rocking back and forth on the examination table in pain. Based

on the patient’s history, the nurse suspects kidney stones. What additional examination

technique does the nurse perform to confirm this suspicion?

a. Palpating the flank area for rebound tenderness

b. Percussing the bladder for fullness

c. Percussing the costal vertebral margins for tenderness

d. Palpating McBurney point for tenderness

ANSWER: C

A

B

C

D

Feedback

Palpating the flank area for rebound tenderness is the correct location (flank

area), but rebound tenderness is performed on the abdomen to detect peritoneal

inflammation.

Percussing the bladder for fullness would provide data about bladder distention,

but is not a technique to detect for kidney stones.

Percussing the costal vertebral margins for tenderness is the appropriate technique

to detect kidney stones. The nurse recognizes the relationship between the history

and the observation with further assessment techniques needed to confirm kidney

stones.

Palpating McBurney point for tenderness is a technique to detect appendicitis.

DIF: Cognitive Level: Analyze REF: 281-282

TOP: Nursing Process: Assessment

MSC: NCLEX Patient Needs: Physiologic Integrity: Physiologic Adaptation: Alteration in Body

Systems

29. Which techniques does a nurse use to palpate a patient’s right kidney?

a. Asks the patient to take a deep breath, elevates the patient’s eleventh and twelfth

ribs with the left hand, and deeply palpates for the right kidney with the right hand

b. Asks the patient to exhale, elevates the patient’s eleventh and twelfth ribs with the

left hand, and deeply palpates for the right kidney with the right hand

c. Asks the patient to take a deep breath, elevates the patient’s right flank with the left

hand, and deeply palpates for the right kidney with the right hand

d. Asks the patient to exhale, elevates the patient’s right flank with the left hand, and

deeply palpates for the right kidney with the right hand

ANSWER: C

A

B

C

Feedback

Asking the patient to take a deep breath, elevating the patient’s eleventh and

twelfth ribs with the left hand, and deeply palpating for the right kidney with the

right hand is incorrect because the flank is elevated rather than the ribs.

Asking the patient to exhale, elevating the patient’s eleventh and twelfth ribs with

the left hand, and deeply palpating for the right kidney with the right hand is

incorrect because the flank is elevated rather than the ribs and the patient is asked

to inhale rather than exhale.

Asking the patient to take a deep breath, elevating the patient’s right flank with

the left hand, and deeply palpating for the right kidney with the right hand is the

correct technique.


D

Asking the patient to exhale, elevating the patient’s right flank with the left hand,

and deeply palpating for the right kidney with the right hand is incorrect because

the patient is asked to inhale rather than exhale.

DIF: Cognitive Level: Apply REF: 280

TOP: Nursing Process: Assessment

MSC: NCLEX Patient Needs: Health Promotion and Maintenance: Techniques of Physical

Assessment

30. When assessing the abdomen of a patient who has fluid in the peritoneal cavity, the nurse

expects what change to occur when the patient turns from supine to the left side?

a. Movement of the tympanic tones from lateral in the supine position to closer to

midline when lying on the left side

b. Movement of the dull tones from lateral in the supine position to closer to midline

when lying on the left side

c. Change in bowel sounds from hypoactive in the supine position to hyperactive

when lying on the left side

d. Change in bowel sounds from hyperactive in the supine position to hypoactive

when lying on the left side

ANSWER: B

A

B

C

D

Feedback

Movement of the tympanic tones from lateral in the supine position to closer to

midline when lying on the left side is incorrect because the tone will be dull,

rather than tympanic, due to the fluid.

Movement of the dull tones from lateral in the supine position to closer to midline

when lying on the left side is the expected change when assessing for shifting

dullness.

A change in bowel sounds from hypoactive in the supine position to hyperactive

when lying on the left side is incorrect because bowel sounds would not be

affected by the fluid.

A change in bowel sounds from hyperactive in the supine position to hypoactive

when lying on the left side is incorrect because bowel sounds would not be

affected by the fluid.

DIF: Cognitive Level: Understand REF: 282-283

TOP: Nursing Process: Assessment

MSC: NCLEX Patient Needs: Health Promotion and Maintenance: Techniques of Physical

Assessment

31. The patient reports right lower quadrant (RLQ) pain that is worse with coughing. Based on the

patient’s history, the nurse suspects appendicitis. What additional examination technique does

the nurse perform to confirm this suspicion?

a. Placing the hand over the lower right thigh and asking the patient to flex the knee

while pushing down on the knee to resist it and noting if the patient complains of

pain

b. Palpating deeply a point of the abdomen, located halfway between the umbilicus

and the left anterior iliac crest

c. Asking the patient to flex the right hip and knee to 90 degrees, then abducting the


leg and noting if the patient complains of pain

d. Pressing down in an area away from the RLQ at a 90-degree angle to the abdomen,

then releasing the fingers quickly and noting any complaint of pain

ANSWER: D

A

B

C

D

Feedback

This is an incorrect description of the iliopsoas muscle test.

This is an incorrect description of the testing for McBurney point. McBurney

point is located to the right of the umbilicus.

This is an incorrect description of the obturator muscle test.

This describes rebound tenderness, which is performed to detect peritoneal

inflammation.

DIF: Cognitive Level: Analyze REF: 284-285

TOP: Nursing Process: Assessment

MSC: NCLEX Patient Needs: Physiologic Integrity: Reduction of Risk Potential: System Specific

Assessments

32. When palpating the abdomen to determine a floating mass, a nurse presses on the abdomen at

a 90-degree angle with the fingertips. Which finding indicates a mass?

a. An increase in abdominal girth

b. A complaint from the patient of a dull pain in the flank area

c. A freely movable mass will float upward and touch the fingertips

d. Fluid in the abdomen will shift upward and touch the fingertips

ANSWER: C

A

B

C

D

Feedback

An increase in abdominal girth does not occur as a result of ballottement.

A complaint from the patient of a dull pain in the flank area is not an expected

finding.

A freely movable mass floating upward and touching the fingertips is the

expected finding (ballottement).

Fluid in the abdomen shifting upward and touching the fingertips does not occur;

it is the mass on the abdomen that shifts upward.

DIF: Cognitive Level: Apply REF: 285

TOP: Nursing Process: Assessment

MSC: NCLEX Patient Needs: Health Promotion and Maintenance: Techniques of Physical

Assessment

33. A 75-year-old male patient asks how to reduce his risk of esophageal cancer. What is the

nurse’s most appropriate response?

a. “Don’t worry about it, esophageal cancers have a low incidence in men.”

b. “You should not be concerned about esophageal cancer at your age.”

c. “You should consider limiting your alcohol intake to two drinks per day.”

d. “Increasing the fiber and protein in your diet can help you lower your risk.”

ANSWER: C

Feedback


A

B

C

D

Men have a rate three times that of women.

The risk increases with age, with the peak between 70 and 80 years.

“You should consider limiting your alcohol intake to two drinks per day. Longterm

alcohol intake increases your risk for esophageal cancer.”

Although fiber and protein are important for the diet, their intake does not affect

the risk of esophageal cancer.

DIF: Cognitive Level: Understand REF: 287

TOP: Nursing Process: Intervention

MSC: NCLEX Patient Needs: Health Promotion and Maintenance: Disease Prevention

34. Which patient has the lowest risk for colon cancer?

a. Patient A is 50 years old, is obese, and has type 2 diabetes mellitus.

b. Patient B is 60 years old, has alcoholism, and smokes a pack of cigarettes daily.

c. Patient C is 55 years old, has ulcerative colitis, and inflammatory bowel disease.

d. Patient D is 45 years old and has diverticulosis.

ANSWER: D

A

B

C

D

Feedback

Patient A has three risk factors for colon cancer.

Patient B has three risk factors for colon cancer.

Patient C has two risk factors for colon cancer.

Patient D has the lowest risk of colon cancer. Ninety percent of colon cancers

occur in adults older than 50 years of age. Although this patient does have a

disorder of the colon, it is not linked to an increased risk of colon cancer.

DIF: Cognitive Level: Analyze REF: 287

TOP: Nursing Process: Assessment

MSC: NCLEX Patient Needs: Health Promotion and Maintenance: Disease Prevention

35. Which assessment technique is the nurse performing in the figure below?

a. Direct percussion

b. Indirect percussion

c. Light palpation

d. Deep palpation

ANSWER: B

A

B

Feedback

Direct percussion is performed with one hand.

Indirect percussion is the technique shown.


C Light palpation is performed using the pads of the fingers depressing the tissue 1

to 2 cm, usually on the abdomen.

D Deep palpation is performed using the pads of the fingers depressing the tissue 4

to 6 cm, usually on the abdomen.

DIF: Cognitive Level: Apply REF: 282

TOP: Nursing Process: Assessment

MSC: NCLEX Patient Needs: Health Promotion and Maintenance: Techniques of Physical

Assessment

MULTIPLE RESPONSE

1. A nurse suspects appendicitis in a patient with abdominal pain. Which findings are suggestive

of appendicitis? Select all that apply.

a. Pain radiating to the right shoulder

b. Pain around the umbilicus

c. Pain relieved by lying still

d. Right lower quadrant pain

e. Increased peristalsis

ANSWER: B, C, D

Correct: These are all descriptions of pain related to appendicitis.

Incorrect: Pain radiating to the right shoulder is associated with gallbladder disease.

Increased peristalsis can be associated with gastroenteritis or diarrhea.

DIF: Cognitive Level: Understand REF: 278-279

TOP: Nursing Process: Assessment

MSC: NCLEX Patient Needs: Physiologic Integrity: Physiologic Adaptation: Alteration in Body

Systems

2. Alcoholism increases the risk of cancers of the gastrointestinal tract. Which cancer risk is

increased in patients with alcoholism? Select all that apply.

a. Esophageal cancer

b. Stomach cancer

c. Pancreatic cancer

d. Liver cancer

e. Colon cancer

f. Bladder cancer

ANSWER: A, B, D, E

Correct: The risk of esophageal, stomach, liver, and colon cancers are increased by heavy

intake of alcohol.

Incorrect: The risk of pancreatic and bladder cancers are increased with tobacco. However,

the risk for esophageal, stomach, liver, and colon cancers are also increased with tobacco use.

DIF: Cognitive Level: Understand REF: 285-285

TOP: Nursing Process: Assessment

MSC: NCLEX Patient Needs: Health Promotion and Maintenance: Disease Prevention

OTHER


1. Put in correct order the steps used to palpate the liver.

A. Place your right hand parallel to the right costal margin.

B. Ask the patient to take a deep breath.

C. Place your right hand parallel to the right costal margin.

D. Lift up the eleventh and twelfth ribs with the left hand.

E. Press your right hand down and under the coastal margin.

F. Ask the patient to take some deep breaths

ANSWER:

D, C, A, E, B, F

DIF: Cognitive Level: Apply REF: 287

TOP: Nursing Process: Assessment

MSC: NCLEX Patient Needs: Health Promotion and Maintenance: Techniques of Physical

Assessment


Chapter 14: Musculoskeletal System

Test Bank

MULTIPLE CHOICE

1. Which description of pain from the patient makes a nurse suspect the patient’s pain is

originating from a muscle?

a. “Crampy”

b. “Dull and deep”

c. “Boring and intense”

d. “Sharp upon movement”

ANSWER: A

A

B

C

D

Feedback

Muscle pain is often described as “crampy.”

Bone pain typically is described as “deep” and “dull.”

Bone pain typically is described as “boring” and “intense.”

Muscle pain usually remains crampy on movement.

DIF: Cognitive Level: Understand REF: 301

TOP: Nursing Process: Assessment

MSC: NCLEX Patient Needs: Physiologic Integrity: Physiologic Adaptation: Alteration in Body

Systems

2. A nurse asks a patient to describe his new onset of leg pain. He slept well through the night,

but this morning he suddenly developed pain in his left lower leg that is red and too painful to

touch. Nothing relieves the pain. Based on these data, the nurse suspects which disorder is

causing this pain?

a. Rheumatoid arthritis

b. Osteoarthritis

c. Gout

d. Tendonitis

ANSWER: C

Feedback

A Patients with rheumatoid arthritis often have morning stiffness lasting 1 to 2

hours.

B Patients with osteoarthritis experience pain when bearing weight that is relieved

by rest.

C Sudden onset of pain and erythema in the great toe, ankle, and lower leg suggests

gout (also called gouty arthritis).

D Tendonitis may awaken the patient, especially when the patient is lying on the

affected limb.

DIF: Cognitive Level: Apply REF: 301| 332

TOP: Nursing Process: Assessment

MSC: NCLEX Patient Needs: Physiologic Integrity: Physiologic Adaptation: Alteration in Body

Systems


3. During a history, the patient reports having gout. Based on this information, what findings

does the nurse anticipate during a focused assessment?

a. Warm, tender, and deformed wrists and peripheral interphalangeal (PIP) joints

bilaterally

b. Edema, warmth, and redness of one great toe and pea-like nodules in the ear lobes

c. Enlarged and tender PIP or distal interphalangeal (DIP) joints on one or several

fingers

d. Tenderness with pronation and supination of the elbow and point tenderness on the

lateral epicondyle

ANSWER: B

A

B

C

D

Feedback

This is a description of findings of a patient who has rheumatoid arthritis.

Bilateral joint involvement is common.

This is a description of gout. The pealike nodules are tophi, collections of uric

acid in subcutaneous tissue.

This is a description of findings of a patient who has osteoarthritis. Enlarged and

tender PIP joints refer to Heberden nodes and DIP joints refer to Bouchard nodes.

This is a description of epicondylitis (tennis elbow).

DIF: Cognitive Level: Apply REF: 301| 332

TOP: Nursing Process: Assessment

MSC: NCLEX Patient Needs: Physiologic Integrity: Physiologic Adaptation: Alteration in Body

Systems

4. A patient reports joint pain interfering with sleep and morning joint stiffness for the first hour

after getting out of bed. Considering this report, what abnormal findings does the nurse

anticipate during the examination?

a. Abrupt onset of local tenderness, edema, and decreased range of motion of the

shoulder and hip bilaterally

b. Decreased range of motion of one hip and knee with pain on flexion and crepitus

during movement of these joints

c. Erythema in one great toe, ankle, and lower leg that is painful to the touch

d. Hot, painful, deformed, and edematous wrists and peripheral interphalangeal joints

bilaterally

ANSWER: D

A

B

C

D

Feedback

This examination finding is more consistent with bursitis.

This examination finding is more consistent with osteoarthritis.

This examination finding is more consistent with gout.

The history and these examination findings are consistent with rheumatoid

arthritis. Joints are involved bilaterally in rheumatoid arthritis because it is a

systemic autoimmune disorder.

DIF: Cognitive Level: Analyze REF: 301| 318| 332

TOP: Nursing Process: Assessment

MSC: NCLEX Patient Needs: Physiologic Integrity: Physiologic Adaptation: Alteration in Body


Systems

5. In assessing a patient with a history of poliomyelitis, the nurse suspects the right leg muscles

are smaller than the left leg. What is the best approach for the nurse to confirm or reject this

suspicion?

a. Palpating both legs using the pads of the thumb and index fingers and comparing

one side with another

b. Using a tape to measure each leg’s circumference at the same location, above or

below the nearest joint

c. Using a goniometer to measure the upper and lower legs with the patient in supine

and standing positions

d. Palpating the legs using the tips of the thumb and index fingers, and comparing the

findings with the Lovett scale

ANSWER: B

A

B

C

D

Feedback

This describes an appropriate procedure to determine tenderness of muscles, but

not muscle size.

This technique is correct, provides a baseline for future comparisons, and

provides measurements for side-to-side comparisons.

The goniometer is used to measure the degree of joint flexion and extension

rather than muscle size.

The Lovett scale is used to grade and record muscle strength, rather than muscle

size.

DIF: Cognitive Level: Apply REF: 304

TOP: Nursing Process: Assessment

MSC: NCLEX Patient Needs: Physiologic Integrity: Physiologic Adaptation: System Specific

Assessments

6. In assessing the joint range of motion of a patient’s knees, the nurse notices the flexion is less

than expected in both knees. What is the next appropriate action for the nurse?

a. Documenting this finding as expected for this patient because it occurs in both

knees

b. Palpating the suprapatellar pouch on each side of the quadriceps for contour,

tenderness, and edema

c. Using a goniometer to measure the flexion in both knees and comparing the results

with expected flexion

d. Applying opposing force to the lower leg while the patient tries to maintain flexion

and extension

ANSWER: C

A

B

C

Feedback

This is not an expected finding. Even when the finding is bilateral, when it is

abnormal, additional examination is needed.

This is the procedure for palpating the knees, in which data are collected, but it

may not have a direct bearing on the lack of flexion of the knees.

When a joint seems to have increased or decreased range of motion, use a

goniometer to measure the angle.


D

This is the procedure for testing leg muscles for strength, in which data are

collected, but it may not have a direct bearing on the lack of flexion of the knees.

DIF: Cognitive Level: Apply REF: 307

TOP: Nursing Process: Assessment

MSC: NCLEX Patient Needs: Physiologic Integrity: Physiologic Adaptation: System Specific

Assessments

7. The nurse asks the patient to hold the arms straight out, perpendicular to the floor, and the

nurse tries to push the patient’s arms down. This procedure tests the strength of which

muscles?

a. Triceps

b. Biceps

c. Trapezius

d. Deltoid

ANSWER: D

A

B

C

D

Feedback

The nurse would test the triceps muscle strength by having the patient extend the

arm and resist while the nurse tries to push the arm to a flexed position.

The nurse would test the muscle strength of the biceps by having the patient flex

the forearm while the nurse applies resistance.

The nurse would test the trapezius by having the patient shrug the shoulders

against resistance.

The patient uses the deltoid muscles to resist the action of the nurse.

DIF: Cognitive Level: Apply REF: 308| 315

TOP: Nursing Process: Assessment

MSC: NCLEX Patient Needs: Health Promotion and Maintenance: Techniques of Physical

Assessment

8. To assess the triceps and biceps muscle strength, the nurse applies resistance to the patient’s

arm. What should be done to ensure the appropriate muscle is being assessed?

a. The patient pushes up against the nurse’s hand to abduct the triceps muscle and

pushes down against the nurse’s hand to adduct the biceps muscle.

b. The patient pushes forward against the nurse’s hand to extend the triceps muscle

and pulls backward against the nurse’s hand to flex the biceps muscle.

c. The patient pulls backward against the nurse’s hand to flex the triceps muscle and

pushes forward against the nurse’s hand to extend the biceps muscle.

d. The patient pushes up against the nurse’s hand to abduct the biceps muscle and

pushes down against the nurse’s hand to adduct the triceps muscle.

ANSWER: B

A

B

C

D

Feedback

Abducting and adducting test range of motion rather than muscle strength.

This is the correct technique for assessing these muscles.

This is a reversal of the correct technique.

Abducting and adducting test range of motion rather than muscle strength.


DIF: Cognitive Level: Apply REF: 308| 315

TOP: Nursing Process: Assessment

MSC: NCLEX Patient Needs: Health Promotion and Maintenance: Techniques of Physical

Assessment

9. The nurse notes that there is an audible clicking sound when the patient opens and closes the

mouth. What is the appropriate response of the nurse at this time?

a. Recording this as an abnormal finding, requiring additional assessment

b. Measuring the distance between each side of the mandible and the eyes

c. Applying resistance to the maxilla and asking the patient to repeat the motion

d. Documenting this finding as expected if no other signs or symptoms are found

ANSWER: D

A

B

C

D

Feedback

This would be an incorrect action because clicking of the mandible with no other

associated signs or symptoms is a normal finding.

This is not a correct assessment technique to use.

This is not a correct assessment technique to use.

The mandible should move smoothly and painlessly. An audible or palpable

snapping or clicking in the absence of other symptoms is not unusual.

DIF: Cognitive Level: Apply REF: 309-310

TOP: Nursing Process: Assessment

MSC: NCLEX Patient Needs: Physiologic Integrity: Physiologic Adaptation: System Specific

Assessments

10. A nurse palpates the patient’s jaw movement by placing two fingers in front of each ear and

asking the patient to slowly open and close the mouth. What movement does the nurse ask the

patient to do next?

a. Move the jaw side to side.

b. Swallow.

c. Smile.

d. Clench the teeth together.

ANSWER: A

A

B

C

D

Feedback

Moving the jaw side to side assesses the range of motion of the jaw; asking the

patient to protrude and retract the jaw also assesses range of motion.

Swallowing assesses cranial nerve IX (glossopharyngeal) and X (vagus).

Smiling assesses cranial nerve VII (facial).

Clenching the teeth together assesses cranial nerve VII (facial).

DIF: Cognitive Level: Apply REF: 309-310

TOP: Nursing Process: Assessment

MSC: NCLEX Patient Needs: Health Promotion and Maintenance: Techniques of Physical

Assessment

11. When assessing the neck of a healthy adult, a nurse expects which findings?

a. A convex contour of the posterior cervical spine


b. Bending of the head to the right and left (ear to shoulder) 15 degrees

c. Turning the chin to the right shoulder and then the left shoulder

d. Hyperextension of the head 30 degrees from midline

ANSWER: C

A

B

C

D

Feedback

The posterior cervical spine should be concave.

The patient should be able to laterally bend the head 40 degrees from midline in

each direction.

This is an expected finding.

The patient should be able to hyperextend the head 55 degrees from midline.

DIF: Cognitive Level: Understand REF: 310-311

TOP: Nursing Process: Assessment

MSC: NCLEX Patient Needs: Physiologic Integrity: Physiologic Adaptation: System Specific

Assessments

12. A patient asks, “Why is touching my toes necessary? This is a sports physical examination,

not exercise class.” What is the most appropriate response by the nurse?

a. “This is the best way to check for symmetry of your arms.”

b. “I am looking at the stretch of your ham strings.”

c. “This allows me to see how straight your spinal column is.”

d. “I am assessing the flexion of your spine.”

ANSWER: C

A

B

C

D

Feedback

This is an incorrect technique for assessing arm symmetry.

The hamstrings are not normally assessed.

This is the correct technique for inspecting the spine and for detecting scoliosis.

This is not a correct statement.

DIF: Cognitive Level: Apply REF: 312-313

TOP: Nursing Process: Assessment

MSC: NCLEX Patient Needs: Physiologic Integrity: Physiologic Adaptation

13. How does a nurse document the finding from the patient shown below?

a. Kyphosis


b. Lordosis

c. Osteoporosis

d. Scoliosis

ANSWER: D

A

B

C

D

Feedback

Kyphosis is a posterior curvature (convexity) of the thoracic spine.

Lordosis is an anterior curvature (concavity) of the spine.

Osteoporosis occurs when the bones become porous and fracture easily.

Deviation of the spine or asymmetry of shoulder or iliac height is an abnormal

finding. This figure shows a patient who has scoliosis, a lateral curvature of the

spine.

DIF: Cognitive Level: Understand REF: 312-313

TOP: Nursing Process: Assessment

MSC: NCLEX Patient Needs: Physiologic Integrity: Physiologic Adaptation: Alteration in Body

Systems

14. A patient reports a history of compression of the left cranial nerve XI (spinal accessory nerve)

from an old sports injury. Based on this information, what technique does the nurse include in

the focused assessment?

a. Asking the patient to rotate the head against resistance of the nurse’s hand on the

patient’s chin

b. Asking the patient to flex the chin to the chest against resistance of the nurse’s hand

on the patient’s forehead

c. Asking the patient to extend the head back against resistance of the nurse’s hand on

the back of the patient head

d. Asking the patient to shrug the shoulders while the nurse attempts to push them

down

ANSWER: D

A

B

C

D

Feedback

This technique tests the sternocleidomastoid muscle.

This technique tests the sternocleidomastoid muscle.

This technique tests the sternocleidomastoid muscle.

This is the technique to test strength of the trapezius muscle that is innervated by

the cranial nerve XI.

DIF: Cognitive Level: Analyze REF: 314

TOP: Nursing Process: Assessment

MSC: NCLEX Patient Needs: Physiologic Integrity: Physiologic Adaptation: System Specific

Assessments

15. The nurse in the figure below is assessing function and strength of which muscle?


a. Sternocleidomastoid

b. Trapezius

c. Deltoid

d. Pectoralis major

ANSWER: B

A

B

C

D

Feedback

The sternocleidomastoid is tested by the patient flexing the chin to the chest while

the nurse tries to manually force the head upright.

The trapezius muscle is tested by the patient shrugging the shoulders while the

nurse attempts to push them down.

The deltoid muscle is tested by the patient holding the arms up while the nurse

tries to push them down.

The pectoralis major muscle is palpated for tenderness, but its strength is not

usually assessed.

DIF: Cognitive Level: Understand REF: 314

TOP: Nursing Process: Assessment

MSC: NCLEX Patient Needs: Physiologic Integrity: Physiologic Adaptation: Alteration in Body

Systems

16. When a nurse asks a patient to place the right arm behind the back, so that the back of the

hand is touching the lower spine, the nurse is testing for which range of motion?

a. Pronation of the elbow

b. Hyperextension of the elbow

c. Internal rotation and adduction of the shoulder

d. External rotation and abduction of the shoulder

ANSWER: C

A

B

C

D

Feedback

Pronation of the elbow is tested by pronating the palm on a flat surface.

Hyperextension of the elbow. This is not a motion of the elbow.

Internal rotation and adduction of the shoulder is tested by this maneuver.

External rotation and abduction of the shoulder is tested by asking the patient to

place the hand behind the head.

DIF: Cognitive Level: Apply REF: 315


TOP: Nursing Process: Assessment

MSC: NCLEX Patient Needs: Health Promotion and Maintenance: Techniques of Physical

Assessment

17. When a nurse asks a patient to place the right arm behind the head, the nurse is testing for

which range of motion?

a. Flexion of the elbow

b. Hyperextension of the shoulder

c. Internal rotation and adduction of the shoulder

d. External rotation and abduction of the shoulder

ANSWER: D

A

B

C

D

Feedback

Flexion of the elbow requires the patient to flex the elbow, but the elbow flexion

is usually tested by asking the patient to bend the elbow so that the fingers are

touching the shoulder.

Hyperextension of the shoulder is tested by moving the arm straight backward.

Internal rotation and adduction of the shoulder is tested by asking the patient to

place the hand behind the back.

External rotation and abduction of the shoulder is tested by this maneuver.

DIF: Cognitive Level: Apply REF: 315

TOP: Nursing Process: Assessment

MSC: NCLEX Patient Needs: Health Promotion and Maintenance: Techniques of Physical

Assessment

18. The nurse asks the patient to rest the left arm on a table and to move the lower arm so that the

palm of the hand is up and then down. What motion is the nurse testing?

a. Adduction and abduction of the wrist

b. Supination and pronation of the wrist

c. Adduction and abduction of the elbow

d. Supination and pronation of the elbow

ANSWER: D

A

B

C

D

Feedback

The movements of the wrist are flexion, extension, and ulnar and radial deviation.

The movements of the wrist are flexion, extension, and ulnar and radial deviation.

This is not a movement of the elbow. The elbow moves in flexion, extension,

pronation, and supination.

Supination and pronation of the elbow is tested by this maneuver.

DIF: Cognitive Level: Apply REF: 316-317

TOP: Nursing Process: Assessment

MSC: NCLEX Patient Needs: Health Promotion and Maintenance: Techniques of Physical

Assessment

19. On inspection of a patient’s hands, the nurse notices ulnar deviation and swan-neck

deformities bilaterally and correlates this finding with which disorder?

a. Osteoarthritis


b. Osteoporosis

c. Rheumatoid arthritis

d. Gout

ANSWER: C

A

B

C

D

Feedback

The findings are consistent with rheumatoid arthritis.

The findings are consistent with rheumatoid arthritis.

Ulnar deviation, swan-neck, and boutonnière deformities of interphalangeal joints

are manifestations of rheumatoid arthritis.

The findings are consistent with rheumatoid arthritis.

DIF: Cognitive Level: Apply REF: 317| 332

TOP: Nursing Process: Assessment

MSC: NCLEX Patient Needs: Physiologic Integrity: Physiologic Adaptation: Alteration in Body

Systems

20. With the patient lying supine, a nurse raises the patient’s leg to flex the hip. The patient

complains of pain when the leg is raised to 40 degrees. The nurse correlates this finding with

which disorder?

a. Lumbar nerve compression

b. Cervical disk herniation

c. Osteoarthritis

d. Bursitis

ANSWER: A

A

B

C

D

Feedback

To evaluate for nerve root irritation or lumbar disk herniation, perform straight

leg raises. Pain in the back of the leg with 30 to 60 degrees of elevation indicates

pressure on a lumbar peripheral nerve by an intervertebral disk.

Straight leg raises evaluate for herniated disks, but not in the cervical disks in the

neck.

Osteoarthritis is a degenerative disease of articular cartilage that affects weightbearing

joints such as vertebrae, hips, knees, and ankles. Straight leg raises is not

a technique to assess for osteoarthritis.

Bursa become inflamed by constant friction around joints and may be precipitated

by arthritis or injury. The hip is a common site, but not vertebrae. Manifestations

include painful range of motion, point tenderness, and erythema of the affected

joint.

DIF: Cognitive Level: Apply REF: 329| 333

TOP: Nursing Process: Assessment

MSC: NCLEX Patient Needs: Physiologic Integrity: Physiologic Adaptation: Alteration in Body

Systems

21. With the patient in a supine position, how does a nurse test the external rotation of the

patient’s right hip?

a. Asking the patient to move the right leg laterally with the right knee straight

b. Asking the patient to flex the right knee and turn medially toward the left side


(inward)

c. Asking the patient to place the right heel on the left patella

d. Asking the patient to raise the right leg straight up and perpendicular to the body

ANSWER: C

A

B

C

D

Feedback

Moving the right leg laterally with the right knee straight assesses abduction of

the right hip.

Flexing the right knee and turning medially toward the left side (inward)

internally rotates the right hip.

Placing the right heel on the left patella externally rotates the right hip.

Raising the right leg straight up and perpendicular to the body flexes the right hip.

DIF: Cognitive Level: Apply REF: 320-321

TOP: Nursing Process: Assessment

MSC: NCLEX Patient Needs: Health Promotion and Maintenance: Techniques of Physical

Assessment

22. What movement from the patient does a nurse request to assess for hyperextension of the hip?

a. Raise one leg at a time while lying prone.

b. Raise one leg at a time while lying supine.

c. Move one leg at a time laterally, away from midline, while lying prone.

d. Move one leg at a time medially, toward midline, while lying supine.

ANSWER: A

A

B

C

D

Feedback

This procedure tests hyperextension of the hip.

This procedure tests hip flexion.

This procedure tests hip abduction.

This procedure tests hip adduction.

DIF: Cognitive Level: Apply REF: 321-322

TOP: Nursing Process: Assessment

MSC: NCLEX Patient Needs: Health Promotion and Maintenance: Techniques of Physical

Assessment

23. How does a nurse assess the eversion and inversion of a patient’s ankle?

a. For eversion, ask the patient to turn the sole of the foot away from the body and for

inversion turn the sole of the foot toward the midline.

b. For eversion, ask the patient to turn the sole of the inward toward the midline and

for inversion turn the sole of the foot away from the body.

c. For eversion, ask the patient to walk on his toes and, for inversion, to walk on his

heels.

d. For eversion, ask the patient to point the toes forward and, for inversion, to point

the toes backward.

ANSWER: A

A

Feedback

This is the correct maneuver for eversion and inversion of the ankle.


B

C

D

This movement is the opposite of the correct movement.

This is the test for muscle strength of the ankles and feet.

This is the maneuver for dorsiflexion and plantar flexion of the ankle.

DIF: Cognitive Level: Apply REF: 324-325

TOP: Nursing Process: Assessment

MSC: NCLEX Patient Needs: Health Promotion and Maintenance: Techniques of Physical

Assessment

24. While giving a history, the patient reports having carpal tunnel syndrome. Based on this

information, what technique does the nurse include in a focused assessment?

a. Ask the patient to press the pads of the right and left fingers against each other and

hold for 1 minute.

b. Ask the patient to push the hand against the nurse’s forearm while attempting to

flex the wrist.

c. Ask the patient to flex both wrists and press the dorsal aspects of the hands together

for 1 minute.

d. Hold pressure to the radial and ulnar pulses and watch for blanching.

ANSWER: C

A

B

C

D

Feedback

This is not the correct technique for Phalen sign.

This is not the correct technique for Phalen sign.

This is the correct technique for Phalen sign.

This is the technique for Allen’s test, which is used to detect arterial circulation of

the hand, rather than Phalen sign, which is used to test for carpel tunnel

syndrome.

DIF: Cognitive Level: Apply REF: 325

TOP: Nursing Process: Assessment

MSC: NCLEX Patient Needs: Health Promotion and Maintenance: Techniques of Physical

Assessment

25. In teaching the group of patients about osteoporosis, the nurse identifies which one of these

participants as having the highest risk for this disease?

a. A small-boned, thin white American woman

b. An American Indian man who smokes

c. A Hispanic woman who has completed menopause

d. An African American man with a family history of osteoporosis

ANSWER: A

A

B

C

D

Feedback

A small-boned, thin white American woman has three risk factors for

osteoporosis: female gender, white race, and small body size.

This patient has one risk factor: smoking.

This patient has two risk factors: female gender and menopause.

This patient has one risk factor: family history.

DIF: Cognitive Level: Understand REF: 331


TOP: Nursing Process: Assessment

MSC: NCLEX Patient Needs: Health Promotion and Maintenance: Health Promotion Programs

MULTIPLE RESPONSE

1. Which findings are expected from a musculoskeletal assessment of a left-handed healthy

adult? Select all that apply.

a. Cervical concave, thoracic convex, and lumbar concave contours of the spine

b. Muscle strength of 3/5 bilaterally

c. Circumference of left upper arm larger than right upper arm

d. Lumbar and thoracic spine flexion of 75 degrees

e. External rotation and abduction of left arm of 90 degrees

f. Flexion of right and left knees of 90 degrees

ANSWER: A, C, D, E

Correct: Cervical concave, thoracic convex, and lumbar concave contours of the spine are

expected findings of the spine. The circumference of the left upper arm larger than the right

upper arm is considered an expected finding because this patient is left-handed, which may

account for the increase in circumference. Lumbar and thoracic spine flexion of 75 degrees is

an expected finding of the spine. Ninety-degree external rotation and abduction of the left arm

is an expected finding of the spine.

Incorrect: The expected muscle strength is 5/5. The expected flexion is 120 degrees.

DIF: Cognitive Level: Understand REF: 303-304| 313| 315| 323

TOP: Nursing Process: Assessment

MSC: NCLEX Patient Needs: Physiologic Integrity: Reduction of Risk Potential: System Specific

Assessments

2. Nurses inquire about lifestyle behaviors of patients with risk factors for osteoarthritis. Which

risk factors for osteoarthritis does the nurse ask about? Select all that apply.

a. Estrogen deficiency

b. Physical inactivity

c. Overuse of joints

d. Smoking

e. Obesity

f. Age

ANSWER: B, C, E

Correct: Lack of exercise weakens muscles that support joints. Overuse of joints damages

cartilage in joints. Being overweight puts stress on joints.

Incorrect: Estrogen deficiency, smoking, and age are risk factors for osteoporosis rather than

osteoarthritis.

DIF: Cognitive Level: Understand REF: 331

TOP: Nursing Process: Assessment

MSC: NCLEX Patient Needs: Health Promotion and Maintenance: Disease Prevention

3. Which movements does a nurse expect to find when assessing the hip range of motion of a

healthy person? Select all that apply.

a. Pronation and supination

b. Flexion and extension


c. Internal and external rotation

d. Adduction and abduction

e. Hyperextension

ANSWER: B, C, D, E

Correct: These are all expected motions for the hip joint.

Incorrect: Pronation and supination are not expected motions for the hip joint.

DIF: Cognitive Level: Understand REF: 320-322

TOP: Nursing Process: Assessment

MSC: NCLEX Patient Needs: Physiologic Integrity: Reduction of Risk Potential: System Specific

Assessments

4. Which movements does a nurse expect to find when assessing the ankle range of motion of a

healthy person? Select all that apply.

a. Inversion and eversion

b. Plantar flexion and dorsiflexion

c. Pronation and supination

d. Adduction and abduction

e. Rotation

ANSWER: A, B, D, E

Correct: These are all expected motions for the ankle joint.

Incorrect: Pronation and supination are not expected motions for the ankle joint.

DIF: Cognitive Level: Understand REF: 324-325

TOP: Nursing Process: Assessment

MSC: NCLEX Patient Needs: Physiologic Integrity: Reduction of Risk Potential: System Specific

Assessments


Chapter 15: Neurologic System

Test Bank

MULTIPLE CHOICE

1. A nurse assesses a patient with a head injury who has slowing intellectual functioning,

personality changes, and emotional lability. The nurse correlates these findings with which

area of the brain?

a. Frontal lobe

b. Parietal lobe

c. Thalamus

d. Temporal lobe

ANSWER: A

A

B

C

D

Feedback

The frontal lobe controls intellectual function, awareness of self, personality, and

autonomic responses related to emotion.

The parietal lobe receives sensory input such as position sense, touch, shape, and

texture of objects.

The thalamus is a relay and integration station from the spinal cord to the cerebral

cortex and other parts of the brain.

The temporal lobe contains the primary auditory cortex. It also interprets auditory,

visual, and somatic sensory inputs that are stored in thought and memory.

DIF: Cognitive Level: Understand REF: 337

TOP: Nursing Process: Assessment

MSC: NCLEX Patient Needs: Physiologic Integrity: Physiologic Adaptation: Alteration in Body

Systems

2. In assessing a patient with damage to the occipital lobe, the nurse correlates which clinical

manifestation to this injury?

a. Intentional tremors

b. Visual changes

c. Decreased hearing

d. Inability to formulate words

ANSWER: B

A

B

C

D

Feedback

Intentional tremors are caused by cerebellar problems.

The occipital lobe contains the visual cortex.

The temporal lobe contains the auditory cortex.

The ability to formulate words comes from the Broca area in the frontal lobe.

DIF: Cognitive Level: Understand REF: 337

TOP: Nursing Process: Assessment

MSC: NCLEX Patient Needs: Physiologic Integrity: Physiologic Adaptation: Alteration in Body

Systems


3. While obtaining a symptom analysis from a patient who has an inner ear infection, the nurse

helps the patient distinguish between dizziness and vertigo. Which description by the patient

indicates vertigo?

a. “I felt lightheaded when I stood up.”

b. “I just could not keep my balance when I sat up.”

c. “It seemed that the room was spinning around.”

d. “I was afraid that I was going to lose consciousness.”

ANSWER: C

A

B

C

D

Feedback

This is a description of dizziness that is often associated with transient ischemia

attacks.

This is a description of disequilibrium, a form of dizziness.

This report is consistent with vertigo because it includes a sensation of motion.

This is a description of presyncope, a form of dizziness.

DIF: Cognitive Level: Apply REF: 344-345

TOP: Nursing Process: Assessment

MSC: NCLEX Patient Needs: Physiologic Integrity: Physiologic Adaptation: Alteration in Body

Systems

4. While obtaining a symptom analysis from a patient who had a transient ischemic attack, the

nurse helps the patient distinguish between dizziness and vertigo. Which description by the

patient indicates dizziness?

a. “I felt lightheaded when I stood up.”

b. “It felt like I was on a merry-go-round.”

c. “The room seemed to be spinning around.”

d. “My body felt like it was revolving and could not stop.”

ANSWER: A

A

B

C

D

Feedback

This is a description of dizziness that is often associated with transient ischemia

attacks.

This report is consistent with objective vertigo because it includes a sensation of

motion.

This report is consistent with objective vertigo because it includes a sensation of

motion.

This report is consistent with subjective vertigo because it includes a sensation of

one’s body rotating in space.

DIF: Cognitive Level: Apply REF: 344

TOP: Nursing Process: Assessment

MSC: NCLEX Patient Needs: Physiologic Integrity: Physiologic Adaptation: Alteration in Body

Systems

5. Which patient behavior indicates to the nurse that the patient’s facial cranial nerve (CN VII) is

intact?

a. The patient’s eyes move to the left, right, up, down, and obliquely.

b. The patient moistens the lips with the tongue.


c. The sides of the mouth are symmetric when the patient smiles.

d. The patient’s eyelids blink periodically.

ANSWER: C

A

B

C

D

Feedback

This finding represents movement of the extraocular muscles, which are

controlled by the oculomotor, trochlear, and abducens cranial nerves (CN III, IV,

and VI, respectively).

This finding represents movement of the tongue, which is controlled by the

hypoglossal cranial nerve (CN XII).

This finding represents facial symmetry, which is controlled by the facial cranial

nerve (CN VII).

This finding represents function of the oculomotor cranial nerve (CN III).

DIF: Cognitive Level: Apply REF: 352-353, Table 15-1

TOP: Nursing Process: Assessment

MSC: NCLEX Patient Needs: Physiologic Integrity: Reduction of Risk Potential: System Specific

Assessments

6. A nurse assessing a patient who had a cerebrovascular accident involving the Broca area

suspects expressive or nonfluent aphasia. What communication abilities does the nurse

anticipate from this patient?

a. The patient understands speech but is unable to translate ideas into meaningful

speech.

b. The patient is unable to comprehend speech and thus does not respond verbally.

c. The patient is able to understand speech but has difficulty forming words, creating

muffled speech.

d. The patient is unable to comprehend speech and responds inappropriately to

conversation.

ANSWER: A

A

B

C

D

Feedback

The inability to translate ideas into meaningful speech or writing is termed

expressive aphasia or nonfluent aphasia and is associated with lesions in the

Broca area in the frontal lobe.

The inability to comprehend the speech of others is called receptive aphasia or

fluent aphasia and is associated with lesions in the Wernicke area in the temporal

lobe.

This speech pattern is more consistent with patients who have involvement of

muscles of speech rather than neurologic deficits.

This speech pattern is not relevant to this patient.

DIF: Cognitive Level: Apply REF: 337| 339| 346

TOP: Nursing Process: Assessment

MSC: NCLEX Patient Needs: Physiologic Integrity: Physiologic Adaptation: Alteration in Body

Systems

7. The nurse hears in a report that a patient has receptive or fluent aphasia. What communication

abilities does the nurse anticipate from this patient?


a. The patient understands speech but is unable to translate ideas into meaningful

speech.

b. The patient is able to understand speech but has difficulty forming words creating

muffled speech.

c. The patient is unable to comprehend speech and thus does not respond verbally.

d. The patient is emotionally liable and cries easily, which interferes with the ability

to communicate.

ANSWER: C

A

B

C

D

Feedback

The inability to translate ideas into meaningful speech or writing is termed

expressive aphasia or nonfluent aphasia and is associated with lesions in the

Broca area in the frontal lobe.

This speech pattern is more consistent with patients who have involvement of

muscles of speech rather than neurologic deficits.

This deficit is called receptive aphasia or fluent aphasia and is associated with

lesions in the Wernicke area in the temporal lobe.

This speech pattern is not relevant to this patient.

DIF: Cognitive Level: Apply REF: 337| 339| 346

TOP: Nursing Process: Assessment

MSC: NCLEX Patient Needs: Physiologic Integrity: Physiologic Adaptation: Alteration in Body

Systems

8. What is the earliest and most sensitive indication of altered cerebral function?

a. Unequal pupils

b. Loss of deep tendon reflexes

c. Paralysis on one side of the body

d. Change in level of consciousness

ANSWER: D

A

B

C

D

Feedback

Pupillary function represents function of the oculomotor cranial nerve and the

midbrain.

Deep tendon reflexes represent function of the spinal cord and reflex arcs.

Movement represents function of the spinal cord and posterior frontal lobe.

Maintaining consciousness represents the functions of and communication

between the frontal lobe and reticular activating system.

DIF: Cognitive Level: Remember REF: 347

TOP: Nursing Process: Assessment

MSC: NCLEX Patient Needs: Physiologic Integrity: Physiologic Adaptation: Alteration in Body

Systems

9. A patient reports having difficulty swallowing. Based on this information, how does the nurse

assess the cranial nerve related to swallowing?

a. Ask the patient about feeling the blunt end of a paper clip along the jaw line.

b. Observe the rising of the soft palate when the patient says “Ahh.”

c. Observe the symmetry of the face when the patient talks.


d. Assess taste on the anterior part of the tongue.

ANSWER: B

A

B

C

D

Feedback

This tests the sensory function of the trigeminal cranial nerve (CN V).

This tests the glossopharyngeal cranial nerve (CN IX), which is involved in

swallowing. The nurse must correlate difficulty swallowing with the cranial

nerves involved with that function and how to test it. The cranial nerves involved

are IX, X, and XII.

This tests the motor function of the facial cranial nerve (CN VII).

This tests the sensory portion of the facial cranial nerve (CN VII).

DIF: Cognitive Level: Analyze REF: 341| 346| 354

TOP: Nursing Process: Assessment

MSC: NCLEX Patient Needs: Health Promotion and Maintenance: Techniques of Physical

Assessment

10. A patient reports having difficulty swallowing. Based on this information, how does the nurse

assess the appropriate cranial nerve?

a. Ask the patient to stick out the tongue and move it in all directions.

b. Ask the patient to move the head to the right and left.

c. Observe the symmetry of the face when the patient talks.

d. Assess for taste on the anterior part of the tongue.

ANSWER: A

A

B

C

D

Feedback

This tests the hypoglossal cranial nerve (CN XII) that is involved in swallowing.

The nurse must correlate difficulty swallowing with the cranial nerves involved

with that function and how to test them. The cranial nerves involved are IX, X,

and XII.

This tests the function of the spinal accessory cranial nerve (CN XI).

This tests the motor function of the facial cranial nerve (CN VII).

This tests the sensory portion of the facial cranial nerve (CN VII).

DIF: Cognitive Level: Analyze REF: 341| 346| 354

TOP: Nursing Process: Assessment

MSC: NCLEX Patient Needs: Health Promotion and Maintenance: Techniques of Physical

Assessment

11. In assessing a patient’s deep tendon reflexes, a nurse finds a patient has a 4+ triceps response.

How does the nurse interpret this finding?

a. A hyperactive response

b. A diminished response

c. An absent response

d. An expected response

ANSWER: A

A

Feedback

Deep tendon reflexes are graded from 0 to 4+ and 4+ is a hyperactive response.


B A diminished response is 1+.

C An absent response is 0.

D An expected response is 2+.

DIF: Cognitive Level: Understand REF: 342-343| 349-350

TOP: Nursing Process: Assessment

MSC: NCLEX Patient Needs: Physiologic Integrity: Physiologic Adaptation: Alteration in Body

Systems

12. The nurse holds the patient’s relaxed arm with elbow flexed at a 90-degree angle, places a

thumb over a tendon in the antecubital fossa, and strikes the thumb with the pointed end of the

reflex hammer. Which deep tendon reflex is the nurse assessing?

a. Brachioradialis

b. Biceps

c. Triceps

d. Deltoid

ANSWER: B

A

B

C

D

Feedback

The technique described is not the correct one for assessing the brachioradial

deep tendon reflex.

This is the correct technique for assessing the biceps deep tendon reflex.

The technique described is not the correct one for assessing the triceps deep

tendon reflex.

There is no reflex to test the deltoid muscle.

DIF: Cognitive Level: Understand REF: 342-343| 349-350

TOP: Nursing Process: Assessment

MSC: NCLEX Patient Needs: Health Promotion and Maintenance: Techniques of Physical

Assessment

13. A patient has a compression fracture of the cervical spine at C7 to C8 that is impairing deep

tendon reflexes. Which response will the nurse expect from the affected deep tendon reflex?

a. Diminished to absent pronation of the arm

b. Diminished to absent flexion of the elbow

c. Diminished to absent extension of the elbow

d. Diminished to absent adduction of the upper arm

ANSWER: C

A

B

C

D

Feedback

Diminished to absent pronation of the arm is an abnormal response from the

brachioradial deep tendon reflex that is innervated from C5 to C6.

Diminished to absent flexion of the elbow is an abnormal response from the

biceps deep tendon reflex that is innervated from C5 to C6.

Diminished to absent extension of the elbow is an abnormal response from the

triceps deep tendon reflex that is innervated from C6, C7, and C8.

Diminished to absent adduction of the upper arm is not a response of any deep

tendon reflex.


DIF: Cognitive Level: Analyze REF: 342-343| 349-350

TOP: Nursing Process: Assessment

MSC: NCLEX Patient Needs Physiologic Integrity: Physiologic Adaptation: Alteration in Body

Systems

14. A nurse holds the patient’s relaxed left arm, with elbow flexed at a 90-degree angle, in one

hand. The nurse palpates and then strikes the appropriate tendon just above the elbow with

either end of the reflex hammer. What is the expected response for this deep tendon reflex?

a. Flexion of the left elbow

b. Pronation of the left forearm

c. Supination of the left arm

d. Extension of the left elbow

ANSWER: D

A

B

C

D

Feedback

Flexion of the left elbow would be a normal response for the biceps deep tendon

reflex.

Pronation of the left forearm would be a normal response for the brachioradialis

deep tendon reflex.

Supination of the left arm is not a response of any deep tendon reflex.

Extension of the left elbow is the normal response of the triceps deep tendon

reflex.

DIF: Cognitive Level: Analyze REF: 349-350

TOP: Nursing Process: Assessment

MSC: NCLEX Patient Needs: Health Promotion and Maintenance: Techniques of Physical

Assessment

15. A nurse holds the patient’s relaxed arm with the elbow flexed at a 90-degree angle, places a

thumb over the appropriate tendon in the antecubital fossa, and strikes the thumb with the

pointed end of the reflex hammer. What is the expected response for this deep tendon reflex?

a. Flexion of the left elbow

b. Pronation of the left forearm

c. Supination of the left arm

d. Extension of the left elbow

ANSWER: D

A

B

C

D

Feedback

Pronation of the left forearm is a normal response for the brachioradialis deep

tendon reflex.

Supination of the left arm is not a response of any deep tendon reflex.

Extension of the left elbow is the normal response of the triceps deep tendon

reflex.

Flexion of the left elbow is a normal response for the biceps deep tendon reflex.

DIF: Cognitive Level: Analyze REF: 349-350

TOP: Nursing Process: Assessment

MSC: NCLEX Patient Needs: Health Promotion and Maintenance: Techniques of Physical

Assessment


16. How does a nurse test the brachioradial deep tendon reflex?

a. Uses the end of the handle on the reflex hammer to stroke the lateral aspect of the

sole of the patient’s foot from heel to ball

b. Asks the patient to slightly pronate the relaxed forearm into the nurse’s hand and

strikes the appropriate tendon with the reflex hammer

c. Holds the patient’s relaxed arm with the elbow flexed at a 90-degree angle in one

hand, and palpates and strikes the appropriate tendon just above the elbow with the

flat end of the reflex hammer

d. Holds the patient’s relaxed arm with the elbow flexed at a 90-degree angle, places a

thumb over the appropriate tendon in the antecubital fossa, and strikes the thumb

with the pointed end of the reflex hammer

ANSWER: B

A

B

C

D

Feedback

This is the technique to test plantar flexion, the Babinski reflex.

This is the technique to assess the brachioradial deep tendon reflex.

This is the technique to test the triceps deep tendon reflex.

This is the technique to test the biceps deep tendon reflex.

DIF: Cognitive Level: Understand REF: 349-350

TOP: Nursing Process: Assessment

MSC: NCLEX Patient Needs: Health Promotion and Maintenance: Techniques of Physical

Assessment

17. A nurse dorsiflexes a patient’s right ankle 90 degrees and then uses a reflex hammer to strike

the appropriate tendon. What is the expected response for this deep tendon reflex?

a. Extension of the right lower leg

b. Plantar flexion of the right toes

c. Dorsiflexion of the right foot

d. Plantar flexion of the right foot

ANSWER: D

A

B

C

D

Feedback

This is the expected response for the patellar deep tendon reflex.

This is the expected response for the plantar reflex (Babinski).

This is an incorrect response because the nurse is holding the patient’s foot in

dorsiflexion, therefore dorsiflexion would not be an expected response.

Plantar flexion is the expected response of the Achilles deep tendon reflex.

DIF: Cognitive Level: Analyze REF: 349-350

TOP: Nursing Process: Assessment

MSC: NCLEX Patient Needs: Health Promotion and Maintenance: Techniques of Physical

Assessment

18. The nurse moves a wisp of cotton lightly across the anterior scalp, paranasal sinuses, and

lower jaw to test the function of which cranial nerve?

a. CN IV (trochlear nerve)

b. CN V (trigeminal nerve)


c. CN VI (abducens nerve)

d. CN VII (facial nerve)

ANSWER: B

A

B

C

D

Feedback

The CN IV (trochlear nerve) supplies downward and inward eye movement.

The CN V (trigeminal cranial nerve) supplies sensation to the cornea, iris,

lacrimal glands, conjunctiva, eyelids, forehead, nose, nasal and mouth mucosa,

teeth, tongue, ear, and facial skin.

The CN VI (abducens nerve) supplies lateral eye movement.

The CN VII (facial nerve) supplies movement of facial expression muscles except

the jaw, closes the eyes, and allows labial speech sounds (b, m, w, and rounded

vowels).

DIF: Cognitive Level: Understand REF: 341| 352

TOP: Nursing Process: Assessment

MSC: NCLEX Patient Needs: Health Promotion and Maintenance: Techniques of Physical

Assessment

19. A nurse who is assessing a patient’s eyes finds that the pupils are equal, round, and react to

light and accommodation (PERRLA). These findings verify the expected functioning of which

cranial nerve?

a. Optic cranial nerve (CN II)

b. Oculomotor cranial nerve (CN III)

c. Trochlear cranial nerve (CN IV)

d. Abducens cranial nerve (CN VI)

ANSWER: B

A

B

C

D

Feedback

The optic cranial nerve (CN II) provides vision.

The oculomotor cranial nerve (CN III) provides these eye functions.

The trochlear cranial nerve (CN IV) provides eye movement downward and

inward.

The abducens cranial nerve (CN VI) provides lateral eye movement.

DIF: Cognitive Level: Understand REF: 341| 352

TOP: Nursing Process: Assessment

MSC: NCLEX Patient Needs: Physiologic Integrity: Reduction of Risk Potential: System Specific

Assessments

20. In assessing a patient with a tumor in the pons, the nurse expects to find which abnormalities

due to pressure on cranial nerves?

a. Dilated pupils and ptosis

b. Facial asymmetry and impaired hearing

c. Difficulty swallowing

d. Impaired gag reflex

ANSWER: B

Feedback


A

B

C

D

These abnormalities represent pressure on the oculomotor (CN III) that exits from

the midbrain.

These abnormalities represent pressure on the facial and acoustic cranial nerves.

The nurse correlates the cranial nerves that exit from the pons which are

trigeminal (CN V), abducens (CN VI), facial (CN VII), and acoustic (CN VIII).

This abnormality represents pressure on the three cranial nerves that affect

swallowing: glossopharyngeal (CN IX), vague (CN X), and hypoglossal (CN

XII). These cranial nerves exit the brainstem in the medulla oblongata.

This reflex is controlled by the vagus cranial nerve (CN IX), which exits the

brainstem in the medulla oblongata.

DIF: Cognitive Level: Analyze REF: 338| 341| 352, Figure 15-5

TOP: Nursing Process: Assessment

MSC: NCLEX Patient Needs: Physiologic Integrity: Physiologic Adaptation: Alteration in Body

Systems

21. The nurse assesses the glossopharyngeal nerve (CN IX) by testing which reflex?

a. Corneal reflex

b. Gag reflex

c. Blink reflex

d. Cough reflex

ANSWER: B

A

B

C

D

Feedback

The corneal reflex is controlled by the trigeminal cranial nerve (CN V).

Movement of the posterior pharynx and gag reflex test is controlled by the

glossopharyngeal cranial nerve (CN IX).

The blink reflex is another name for the corneal reflex.

The cough reflex is controlled from the medulla oblongata.

DIF: Cognitive Level: Understand REF: 341| 354

TOP: Nursing Process: Assessment

MSC: NCLEX Patient Needs: Health Promotion and Maintenance: Techniques of Physical

Assessment

22. Which cranial nerve is assessed when a nurse asks a patient to stick out the tongue and move

it side to side?

a. Vagus nerve (CN X)

b. Facial nerve (CN VII)

c. Abducens nerve (CN VI)

d. Hypoglossal nerve (CN XII)

ANSWER: D

A

B

Feedback

The vagus cranial nerve provides movement for voluntary muscles of phonation

(guttural speech sounds) and swallowing.

The facial cranial nerve provides movement for facial expression muscles except

the jaw, closes the eyes, and allows labial speech sounds (b, m, w, and rounded

vowels).


C

D

The abducens cranial nerve provides for lateral eye movement.

The hypoglossal cranial nerve provides tongue movement for speech sound

articulation (l, t, n) and swallowing.

DIF: Cognitive Level: Understand REF: 341| 354

TOP: Nursing Process: Assessment

MSC: NCLEX Patient Needs: Health Promotion and Maintenance: Techniques of Physical

Assessment

23. As a patient is walking down the hall, the nurse notices the patient’s staggering, unsteady gait.

What findings does the nurse anticipate on the neurologic examination?

a. When the patient stands with feet together, eyes open and then closed, an upright

posture is maintained.

b. When the patient touches the end of each finger to the thumb of the same hand, a

tremor is observed in the fingers.

c. When the patient is giving a history to the nurse, a tremor is noticed as the patient’s

hands rest in the lap.

d. When lying supine, the patient is able to move the heel of one foot down the shin of

the other leg.

ANSWER: B

A

B

C

D

Feedback

This is a result of a negative Romberg test. This patient has a cerebellar problem,

which would result in a positive Romberg test.

This patient has a cerebellar problem as evidenced by the staggering gait (noted at

the beginning of the encounter) and the intention tremor on movement (noted

during the examination).

This describes a tremor at rest that occurs in patients with parkinsonism rather

than with cerebellar problems.

This describes a normal response on an examination of cerebellar function.

DIF: Cognitive Level: Apply REF: 340| 357-358

TOP: Nursing Process: Assessment

MSC: NCLEX Patient Needs: Physiologic Integrity: Physiologic Adaptation: Alteration in Body

Systems

24. A nurse asks the patient to stand with feet together, arms resting at the sides, with eyes open

and then with the eyes closed. Which response by the patient indicates an expected cerebellar

function?

a. Sways slightly and maintains upright posture with feet together

b. Is unable to stand upright after turning around in a circle once

c. Steps sideways when standing with feet together and eyes closed

d. Has to move arms horizontally to maintain balance

ANSWER: A

A

B

Feedback

Maintaining balance indicates function of the cerebellum in the Romberg test.

Losing balance is an abnormal response, but turning in a circle is not a part of the

Romberg test.


C

D

This is an abnormal response for the Romberg test (a positive Romberg test).

This is an abnormal response for the Romberg test (a positive Romberg test).

DIF: Cognitive Level: Apply REF: 355

TOP: Nursing Process: Assessment

MSC: NCLEX Patient Needs: Physiologic Integrity: Physiologic Adaptation: Alteration in Body

Systems

25. The nurse asks the patient to stand with feet together, arms resting at the sides, with eyes open

and then with the eyes closed. Which response by the patient indicates a problem in the

cerebellum?

a. Maintains balance when eyes are open, but loses balance with eyes closed

b. Is unable to stand upright after turning around in a circle once

c. Steps sideways when standing with feet together and eyes closed

d. Sways slightly and maintains upright posture with feet together

ANSWER: C

A

B

C

D

Feedback

This is an abnormal response, but is indicative of a proprioceptive problem rather

than a cerebellar problem.

Losing balance is an abnormal response, but turning in a circle is not a part of the

Romberg test.

This is an abnormal response documented as a “positive Romberg” and indicates

cerebellar dysfunction.

This is an expected response documented as a “negative Romberg,” indicating

appropriate cerebellar function for balance.

DIF: Cognitive Level: Apply REF: 355

TOP: Nursing Process: Assessment

MSC: NCLEX Patient Needs: Physiologic Integrity: Physiologic Adaptation: Alteration in Body

Systems

26. What is the patient’s expected response when the nurse is assessing graphesthesia?

a. Lies supine and runs one heel along the opposite shin

b. Identifies a familiar object placed in the hands

c. Describes where a sensation of a vibrating tuning fork is felt

d. Identifies a letter or number drawn in the hand

ANSWER: D

A

B

C

D

Feedback

This activity tests cerebellar function of the lower extremities.

This is a test of stereognosis that tests the function of the parietal lobe and

sensory tracts.

This is a test of vibratory sense that tests sensory tracts.

This is a test of graphesthesia that assesses the parietal lobe and sensory tracts.

DIF: Cognitive Level: Understand REF: 359-360

TOP: Nursing Process: Assessment

MSC: NCLEX Patient Needs: Health Promotion and Maintenance: Techniques of Physical


Assessment

27. What is the patient’s expected response when the nurse is assessing stereognosis?

a. Identifies an object placed in the hand

b. Distinguishes numbers or letters traced in the palm of the hand

c. Touches the index finger of the nondominant hand to the nose

d. Walks heel to toe in a straight line

ANSWER: A

A

B

C

D

Feedback

A nurse tests stereognosis by asking the patient to close his or her eyes and

placing a small, familiar object in the patient’s hand, asking him or her to identify

it. Stereognosis tests sensory nerve tracts and parietal lobe function.

This activity tests graphesthesia, a test of sensory nerve tracts and parietal lobe

function.

This activity tests cerebellar function of the upper extremities.

This activity tests cerebellar function of the lower extremities.

DIF: Cognitive Level: Understand REF: 337| 359

TOP: Nursing Process: Assessment

MSC: NCLEX Patient Needs: Health Promotion and Maintenance: Techniques of Physical

Assessment

28. A nurse correlates a patient’s altered stereognosis with a neurologic dysfunction in which part

of the nervous system?

a. Midbrain or pons

b. Temporal lobe or ascending nerve tracts

c. Frontal lobe or motor nerve tracts

d. Parietal lobe or sensory nerve tracts

ANSWER: D

A

B

C

D

Feedback

Sensory and motor tracts travel though the midbrain and pons, but they are not

tested with stereognosis.

Ascending tracts carry sensory data, but the temporal lobe provides functions of

hearing rather than perception of touch.

Motor tracts carry impulses for movement and they exit from the frontal lobe,

which also helps to maintain consciousness.

A parietal lobe or sensory nerve tract dysfunction prevents a patient from

identifying a familiar object by touch, which is a definition of stereognosis.

DIF: Cognitive Level: Apply REF: 337| 359

TOP: Nursing Process: Assessment

MSC: NCLEX Patient Needs: Physiologic Integrity: Physiologic Adaptation: Alteration in Body

Systems

29. Which part of the nervous system is a nurse assessing when he places a vibrating tuning fork

on a patient’s wrist or ankle?

a. Frontal lobe and motor tracts


b. Parietal lobe and sensory tracts

c. Hypothalamus and sensory tracts

d. Cerebellum and motor tracts

ANSWER: B

A

B

C

D

Feedback

The frontal lobe sends impulses governing movement through motor (efferent)

tracts.

The parietal lobe receives sensory input, such as vibratory sense and pain,

through sensory (afferent) tracts.

The hypothalamus functions include regulation of body temperature, hunger, and

thirst and formation of autonomic nervous system responses.

The cerebellum provides equilibrium and coordination of movement.

DIF: Cognitive Level: Understand REF: 337| 340| 358-359

TOP: Nursing Process: Assessment

MSC: NCLEX Patient Needs: Physiologic Integrity: Reduction of Risk Potential: System Specific

Assessments

30. A patient has a herniated disk compressing the lumbar spine at L2, L3, and L4 that is

impairing deep tendon reflexes. Which response does a nurse expect from this patient?

a. Diminished contraction of the gastrocnemius muscle with plantar flexion of the

foot

b. Diminished contraction of the quadriceps muscle with extension of the lower leg

c. Diminished plantar flexion of the toes

d. Diminished dorsiflexion of the foot and flexion of the toes

ANSWER: B

A

B

C

D

Feedback

This is an abnormal response from the Achilles tendon reflex that is innervated

from S1 and S2.

This is an abnormal response from the patellar deep tendon reflex that is

innervated from L2, L3, and L4.

This is an abnormal response from the plantar reflex or a positive Babinski sign.

This is an abnormal response of ankle clonus.

DIF: Cognitive Level: Analyze REF: 343| 350

TOP: Nursing Process: Assessment

MSC: NCLEX Patient Needs: Physiologic Integrity: Physiologic Adaptation: Alteration in Body

Systems

31. What technique does the nurse use to test the patellar deep tendon reflex?

a. Using the end of the handle on the reflex hammer, the nurse strokes the lateral

aspect of the sole of the patient’s foot from heel to ball.

b. Ask the patient to flex one knee to 90 degrees, while the nurse dorsiflexes the ankle

and strikes the appropriate tendon on the foot with the flat end of the reflex

hammer.

c. Ask the patient to flex one knee to 45 degrees, while the nurse plantar flexes the

ankle and strikes the appropriate tendon of the ankle with the pointed end of the


reflex hammer.

d. Ask the patient to flex one knee to 90 degrees, while the nurse strikes the

appropriate tendon in the knee with the blunt end of the reflex hammer.

ANSWER: D

A

B

C

D

Feedback

This is the technique for testing plantar flex or the Babinski reflex.

This is the technique for testing the Achilles deep tendon reflex.

This is not a correct technique for testing any reflex.

This is the technique for testing the patella deep tendon reflex.

DIF: Cognitive Level: Understand REF: 350

TOP: Nursing Process: Assessment

MSC: NCLEX Patient Needs: Health Promotion and Maintenance: Techniques of Physical

Assessment

32. What technique does the nurse use to test ankle clonus?

a. Strokes the lateral aspect of the sole of the patient’s foot from heel to ball with a

reflex hammer

b. Supports the patient’s knee in flexed position and sharply dorsiflexes the foot and

maintains the flexion

c. Plantar flexes the ankle and strikes the appropriate tendon of the ankle with the

hammer

d. Everts the ankle and slowly moves the ankle into plantar flexion and quickly

release the foot

ANSWER: B

A

B

C

D

Feedback

This is the technique for testing plantar flex or the Babinski reflex.

This is the correct technique for assessing ankle clonus.

This is not a correct technique for testing any reflex.

This is not a correct technique for testing any reflex.

DIF: Cognitive Level: Understand REF: 350

TOP: Nursing Process: Assessment

MSC: NCLEX Patient Needs: Health Promotion and Maintenance: Techniques of Physical

Assessment

33. Which response does a nurse expect when testing ankle clonus of a healthy woman?

a. No movement of the foot

b. Plantar flexion of the foot

c. Extension of the lower leg

d. Dorsiflexion of the foot

ANSWER: A

A

B

Feedback

No movement of the foot is the expected response from a healthy woman.

Plantar flexion of the foot is not a response to ankle clonus.


C

D

Extension of the lower leg is not a response to ankle clonus.

Dorsiflexion of the foot is an abnormal response of ankle clonus.

DIF: Cognitive Level: Understand REF: 350

TOP: Nursing Process: Assessment

MSC: NCLEX Patient Needs: Physiologic Integrity: Physiologic Adaptation: Alteration in Body

Systems

MULTIPLE RESPONSE

1. To complete a symptom analysis, which questions does a nurse ask patient who recently had a

seizure for the first time? Select all that apply.

a. “Did you have any warning signs before the seizure started?”

b. “Did you lose consciousness during the seizure?”

c. “Did the room seem to be spinning around before the seizure?”

d. “Did you urinate during the seizure?”

e. “What did you hear while you were seizing?”

f. “How did you feel after the seizure?”

ANSWER: A, B, D, F

Correct: These are all appropriate questions to ask to gather more data about this patient’s

first seizure.

Incorrect: “Did the room seem to be spinning around before the seizure?” This question is

about vertigo, which does not relate to this patient. “What did you hear while you were

seizing?” The answer to this question is not needed in the data for this patient.

DIF: Cognitive Level: Analyze REF: 345

TOP: Nursing Process: Assessment

MSC: NCLEX Patient Needs: Physiologic Integrity: Physiologic Adaptation: Alteration in Body

Systems

2. Which characteristics are risk factors for cerebrovascular accident? Select all that apply.

a. Excessive alcohol intake

b. Smoking

c. Eating large amounts of smoked foods

d. Obesity

e. Atherosclerosis

f. High blood pressure

ANSWER: A, B, D, E, F

Correct: All of these are risk factors for cerebrovascular accident.

Incorrect: Eating large amounts of smoked foods is a risk factor for stomach cancer.

DIF: Cognitive Level: Remember REF: 362

TOP: Nursing Process: Assessment

MSC: NCLEX Patient Needs: Physiologic Integrity: Reduction of Risk Potential: Potential for

Alteration in Body Systems

3. Which manifestations does a nurse correlate with a patient with suspected meningitis? Select

all that apply.

a. Ptosis


b. Loss of balance when standing with feet together and the eyes closed

c. Confusion, agitation, and irritability

d. Severe headache

e. Stiff neck

f. Lethargy

ANSWER: C, D, E, F

Correct: Confusion, agitation, and irritability; severe headache—this is a symptom of

meningeal irritation due to inflammation of the meninges; stiff neck; lethargy. Patients may

have changes in level of consciousness.

Incorrect: Ptosis is drooping of eyelids controlled by the oculomotor cranial nerve. Loss of

balance when standing with feet together and the eyes closed. This describes a positive

Romberg test indicating a cerebellar problem.

DIF: Cognitive Level: Understand REF: 362

TOP: Nursing Process: Assessment

MSC: NCLEX Patient Needs: Physiologic Integrity: Physiologic Adaptation: Alteration in Body

Systems


Chapter 16: Breasts and Axillae

Test Bank

MULTIPLE CHOICE

1. In teaching a patient about breast self-examination, why does the nurse emphasize palpation

of the axillary areas?

a. Because deep muscles in that area can mask changes

b. Because some patients avoid this area because of tenderness

c. Because most lymph draining from the breast flows through this area

d. Because supporting ligaments in this area may present as tissue changes

ANSWER: C

A

B

C

D

Feedback

Because deep muscles in that area can mask changes. This answer is incorrect.

Palpating the axilla for enlarged lymph nodes is very important.

Because some patients avoid this area because of tenderness. This answer is

incorrect. Palpating the axilla for enlarged lymph nodes is very important.

Because most lymph draining from the breast flows through this area. More than

75% of lymph drainage from the breast flows outward toward the axillary lymph

node.

Because supporting ligaments in this area may present as tissue changes. This

answer is incorrect. Palpating the axilla for enlarged lymph nodes is very

important.

DIF: Cognitive Level: Understand REF: 367| 370| 375

TOP: Nursing Process: Assessment

MSC: NCLEX Patient Need Category: Physiologic Integrity: Reduction of Risk Potential: System

Specific Assessments

2. In reviewing the charts of several patients in the clinic, a nurse recognizes which patient as

being at highest risk of breast cancer?

a. A woman who had her first child at age 26

b. A woman who reached menopause at age 58

c. A woman who breastfed all four of her children

d. A woman who states that she reached menarche at age 14

ANSWER: B

A

B

C

D

Feedback

A woman who had her first child at age 26 has a low risk.

A woman who reached menopause at age 58. A long menstrual history

(menopause after age 50) increases risk.

A woman who breastfed all four of her children has a low risk.

A woman who states that she reached menarche at age 14 has a low risk because

the menarche was after 12 years of age.

DIF: Cognitive Level: Understand REF: 381

TOP: Nursing Process: Assessment


MSC: NCLEX Patient Need Category: Physiologic Integrity: Reduction of Risk Potential: Potential

for Alteration in Body Systems

3. While giving a presentation about breast health, a nurse informs patients about which

recommendation?

a. Women in their 30s should have annual clinical breast examinations.

b. Women at high risk of breast cancer should have semiannual mammograms.

c. Women who are postmenopausal require clinical breast examination every 5 years.

d. A screening mammogram is recommended for all women beginning at age 50

years.

ANSWER: D

A

B

C

D

Feedback

Clinical breast examinations are recommended as part of a periodic health

examination at least every 3 years for average-risk, asymptomatic women in their

20s and 30s, and annually for asymptomatic women age 40 and older.

Having semiannual mammograms is more often than necessary.

Clinical breast examinations are recommended as part of a periodic health

examination at least every 3 years for average-risk, asymptomatic women in their

20s and 30s, and annually for asymptomatic women age 40 and older.

This is the recommendation of the U.S. Preventive Services Task Force.

DIF: Cognitive Level: Remember REF: 370

TOP: Nursing Process: Intervention

MSC: NCLEX Patient Need Category: Health Promotion and Maintenance: Health Promotion

Programs

4. Based on the history, a nurse determines that the patient with which finding requires further

assessment?

a. Occasional discharge from nipples

b. Supernumerary nipples along the milk line

c. Rash in the axillae associated with change in deodorant

d. Mild breast swelling that fluctuates with the menstrual cycle

ANSWER: A

A

B

C

D

Feedback

Nipple discharge is usually an abnormal finding. A specimen of the discharge

should be collected.

Supernumerary nipples along the milk line are a normal finding in some women.

A rash in the axillae associated with change in deodorant can be solved by

changing deodorant and treating the rash.

Mild breast swelling that fluctuates with the menstrual cycle is consistent with

fibrocystic breast disease.

DIF: Cognitive Level: Analyze REF: 369| 374| 377

TOP: Nursing Process: Assessment

MSC: NCLEX Patient Need Category: Physiologic Integrity: Physiologic Adaptation: Alteration in

Body Systems


5. During a breast examination of a healthy female, the nurse recognizes which finding as

normal?

a. Asymmetrical venous pattern

b. Unequal nipple size

c. Supernumerary nipples along the milk line

d. Pink discharge from one nipple when manipulated

ANSWER: C

A

B

C

D

Feedback

The venous patterns should be bilaterally similar.

Nipple size should be symmetric.

Supernumerary nipples are considered a normal variation, although they are

uncommon.

Nipple discharge is usually considered an abnormal finding.

DIF: Cognitive Level: Remember REF: 372-373| 379

TOP: Nursing Process: Assessment

MSC: NCLEX Patient Need Category: Physiologic Integrity: Reduction of Risk Potential: System

Specific Assessments

6. A patient comes to the clinic complaining of a new onset of nipple discharge. After inspection

of the breast and discharge, what action of the nurse has the highest priority?

a. Palpating both breasts comparing amount of discharge

b. Asking the patient about breast pain

c. Asking the patient to raise her arms and comparing the movement of the breasts

d. Obtaining a specimen of the discharge for cytology

ANSWER: D

A

B

C

D

Feedback

Getting a specimen is more important than palpating breasts at this time.

Asking the patient about breast pain is not a priority action at this time.

Asking the patient to raise her arms and comparing the movement of the breasts is

not a priority action at this time.

If a patient has nipple discharge, a specimen should be collected for cytologic

examination to detect malignant cells.

DIF: Cognitive Level: Analyze REF: 374| 377

TOP: Nursing Process: Assessment

MSC: NCLEX Patient Need Category: Physiologic Integrity: Physiologic Adaptation: Alteration in

Body Systems

7. What is the purpose of asking a female to lean forward during the breast examination?

a. To accentuate the Montgomery glands

b. To observe for symmetry of the suspensory ligaments

c. To compare nipple symmetry

d. To identify any breast masses in the subcutaneous tissues

ANSWER: B

Feedback


A

B

C

D

Montgomery glands are located within the areolar surface and would be difficult

to inspect when the patient leans forward.

Observing these ligaments can be accomplished with the patient leaning forward.

Nipple symmetry is better evaluated with the patient sitting up or lying supine.

Palpation is a better way to identify a breast mass in subcutaneous tissue.

DIF: Cognitive Level: Understand REF: 374-375

TOP: Nursing Process: Assessment

MSC: NCLEX Patient Need Category: Health Promotion and Maintenance: Techniques of Physical

Assessment

8. Which technique does a nurse use to palpate the patient’s axillary lymph nodes?

a. With the patient sitting, the nurse places fingers of both hands deep into the axilla,

one hand on either side, and firmly pushes the axillary tissue toward the center to

feel for enlarged nodes.

b. With the patient lying supine with arms at the sides, the nurse uses the tips of the

fingers of one hand to palpate the axilla moving from the posterior to the anterior

aspect of the axilla to feel for enlarged nodes.

c. With the patient lying supine with the hand behind the head of the side being

assessed, the nurse uses the pads of fingers of one hand to systematically palpate

the axilla using small circular motions to feel for enlarged nodes.

d. With the patient sitting, the nurse places fingers of one hand deep into the axilla

and firmly slides the fingers along the patient’s middle, anterior, and posterior of

the axilla to feel for enlarged nodes.

ANSWER: D

A

B

C

D

Feedback

This is incorrect technique.

This is incorrect technique.

This is incorrect technique.

This is the correct technique.

DIF: Cognitive Level: Understand REF: 375-376

TOP: Nursing Process: Assessment

MSC: NCLEX Patient Need Category: Health Promotion and Maintenance: Techniques of Physical

Assessment

9. When examining the lymph nodes of an adult female patient, the nurse recognizes which

finding as normal?

a. Visible superficial nodes

b. Palpable supraclavicular nodes

c. Nonpalpable lymph nodes in the axilla

d. Enlarged, fixed nodes in the neck

ANSWER: C

A

B

Feedback

Lymph nodes are not normally palpable or visible.

Lymph nodes are not normally palpable. Enlarged supraclavicular nodes often

indicate a malignancy.


C

D

Lymph nodes are not normally palpable.

Lymph nodes are not normally palpable.

DIF: Cognitive Level: Apply REF: 375

TOP: Nursing Process: Assessment

MSC: NCLEX Patient Need Category: Physiologic Integrity: Reduction of Risk Potential: System

Specific Assessments

10. A nurse performing a breast examination on a female patient places the patient in a supine

position, places a pillow under the right shoulder, and asks the patient to place her right lower

arm above her head. What is the reason for this position?

a. Flatten the breast tissue evenly over the chest wall.

b. Help the patient to relax and feel more comfortable.

c. Reveal lumps deep in the breast more easily.

d. Expose any drainage from the nipples.

ANSWER: A

A

B

C

D

Feedback

This is the reason for the position.

Flattening the breast tissue, not relaxation, is the reason.

Lumps are detected by palpation rather than inspection.

Drainage is revealed by palpation of the nipples.

DIF: Cognitive Level: Apply REF: 376

TOP: Nursing Process: Assessment

MSC: NCLEX Patient Need Category: Health Promotion and Maintenance: Techniques of Physical

Assessment

11. What instructions does the nurse give a female patient when she is learning to perform breast

self-examination?

a. Press the pads of the fingers firmly to compress breast tissue against the rib cage.

b. Lie in front of a mirror and observe for dimpling of the skin.

c. Lift the fingers from the chest wall during palpation to better define the breast

tissue.

d. Apply gentle pressure while moving the fingers in a pattern across the breast.

ANSWER: D

A

B

C

D

Feedback

Gentle rather than firm pressure is used.

A patient stands in front of a mirror to inspect symmetry; she does not lie in front

of a mirror.

The fingers should not be lifted from the breast to prevent breaking the continuity

of palpation.

This is appropriate technique.

DIF: Cognitive Level: Apply REF: 376

TOP: Nursing Process: Intervention

MSC: NCLEX Patient Need Category: Health Promotion and Maintenance: Techniques of Physical

Assessment


12. In assessing the breast of a male patient, the nurse places him in which position?

a. Standing with hands over the head

b. Supine with the hand on the side being examined placed behind the head

c. Sitting with arms at the side

d. Bending forward 45 degrees at the waist

ANSWER: C

A

B

C

D

Feedback

This is part of the examination for female patients.

This is part of the examination for female patients.

This is the appropriate position for the male breast examination.

This is part of the examination for female patients.

DIF: Cognitive Level: Understand REF: 378

TOP: Nursing Process: Assessment

MSC: NCLEX Patient Need Category: Health Promotion and Maintenance: Techniques of Physical

Assessment

13. During a breast examination of a male patient, the nurse recognizes which finding as normal?

a. Bilateral nontender flat breasts with symmetric nipple and areolar areas

b. A fibrous layer of subcutaneous breast tissue that is thicker than in women

c. Breast tenderness on the dominant side but not on the other side

d. Bilateral symmetry of breasts with absence of hair in the areolar areas

ANSWER: A

A

B

C

D

Feedback

This is a description of a normal finding.

This is not a normal finding.

This is not a normal finding.

This is not a normal finding.

DIF: Cognitive Level: Understand REF: 378-379

TOP: Nursing Process: Assessment

MSC: NCLEX Patient Need Category: Physiologic Integrity: Reduction of Risk Potential: System

Specific Assessments

14. Which statement by a 40-year old man would be most indicative of possible breast cancer?

a. “I had embarrassing breast enlargement when I was a teenager.”

b. “I think I felt a hard spot in my left breast, but it does not hurt.”

c. “My right breast has always been a little smaller than the left.”

d. “My father’s breasts got larger after he was older.”

ANSWER: B

A

B

C

Feedback

This describes gynecomastia.

A breast malignancy usually manifests in one breast as a hard, painless, irregular

nodule, often fixed.

One larger breast is not indicative of breast cancer and may be a normal variation.


D

This report is not indicative of breast cancer.

DIF: Cognitive Level: Understand REF: 379

TOP: Nursing Process: Assessment

MSC: NCLEX Patient Need Category: Physiologic Integrity: Physiologic Adaptation: Alteration in

Body Systems

15. What technique does a nurse use when performing a breast examination on a patient who has

had a mastectomy?

a. Excludes palpation of the axillary area where there was lymph node dissection

b. Inspects and palpates both the operative and the nonoperative sides

c. Avoids palpating the scar to prevent causing the patient any discomfort

d. Palpates only the muscle tissue on the affected side

ANSWER: B

A

B

C

D

Feedback

Axilla is still palpated after node dissection.

Women who have had a mastectomy require the same breast assessment as all

other women.

The scar tissue should not be tender and the technique of palpation on the

operative side should be the same as the nonoperative side.

Women who have had a mastectomy require the same breast assessment as all

other women.

DIF: Cognitive Level: Apply REF: 380

TOP: Nursing Process: Assessment

MSC: NCLEX Patient Need Category: Health Promotion and Maintenance: Techniques of Physical

Assessment

16. In a presentation on breast cancer risk factors, a nurse would be accurate in making which

statement?

a. “Women who breastfeed their children are at increased risk of breast cancer.”

b. “Women who are more than 30% overweight are at increased risk of breast cancer.”

c. “African American women have the highest risk of breast cancer.”

d. “Women who have children before age 30 are at increased risk of breast cancer.”

ANSWER: B

A

B

C

D

Feedback

Breastfeeding is not a risk factor for breast cancer.

Obesity especially after age 50 or increased weight gain as an adult increases

breast cancer risk.

White women have the highest incidence of breast cancer.

First full-term pregnancy after age 30 increases risk.

DIF: Cognitive Level: Understand REF: 381

TOP: Nursing Process: Intervention

MSC: NCLEX Patient Need Category: Physiologic Integrity: Reduction of Risk Potential: Potential

for Alteration in Body Systems


17. A patient comes to the clinic because she found a mass in her left breast that is present during

and after her menstrual periods. On palpation the nurse finds a mass in the left breast that is

round, rubbery, mobile, and nontender. This finding is consistent with which breast disorder?

a. Fibrocystic breast disease

b. Invasive breast cancer

c. Mastitis

d. Fibroadenoma

ANSWER: D

A

B

C

D

Feedback

Findings of this disorder affect both breasts and produce tenderness that subsides

between menstrual periods.

Masses of breast cancer are irregular, hard, and fixed.

Mastitis is an inflammation of the entire breast that produces tissue that is red,

edematous, tender, and warm to the touch.

These findings are consistent with fibroadenoma.

DIF: Cognitive Level: Apply REF: 381-382

TOP: Nursing Process: Assessment

MSC: NCLEX Patient Need Category: Physiologic Integrity: Physiologic Adaptation: Alteration in

Body Systems

18. A patient had a left radical mastectomy last year. The nurse assesses for painless and

nonpitting swelling of the arm on that side. Which complication of a mastectomy is the nurse

assessing for?

a. Infection

b. Lymphedema

c. Inflammation

d. Lymphoma

ANSWER: B

A

B

C

D

Feedback

Localized signs of infection include redness, heat, and pain; there can also be

edema, which, when associated with infection, is pitting.

Lymphedema is a localized accumulation of lymph fluid in the interstitial spaces

caused by removal of the lymph nodes.

Localized signs of inflammation include redness, heat, and pain; there can also be

edema, which, when associated with inflammation, is pitting.

The sign of lymphoma is an enlarged lymph node rather than generalized

swelling.

DIF: Cognitive Level: Apply REF: 380

TOP: Nursing Process: Assessment

MSC: NCLEX Patient Need Category: Physiologic Integrity: Physiologic Adaptation: Alteration in

Body Systems

19. In assessing a patient with lymphedema after a mastectomy, the nurse expects which finding?

a. Fragile, thin, pale skin covering the area of lymphedema

b. Several brownish-red discolorations in the center of the affected arm


c. Unilateral nonpitting edema of the affected arm

d. Pitting edema of affected arm

ANSWER: C

A

B

C

D

Feedback

This is an incorrect description of lymphedema.

This is more consistent with a description of chronic venous insufficiency than

lymphedema.

Lymphedema is a localized accumulation of lymph fluid in the interstitial spaces

caused by removal of the lymph nodes.

The fluid accumulation in lymphedema usually is not enough to cause pitting.

DIF: Cognitive Level: Apply REF: 380

TOP: Nursing Process: Assessment

MSC: NCLEX Patient Need Category: Physiologic Integrity: Physiologic Adaptation: Alteration in

Body Systems

20. A nurse is performing a breast examination of a patient who complains of pain in both breasts

that occurs around the time of her menstrual period. The nurse expects which findings during

the breast examination?

a. Masses in the breasts that are round, soft, mobile, and well-delineated

b. Masses in the breasts that are round, firm, mobile, and well-delineated

c. Masses in the breasts that are irregular, hard, and fixed

d. Breast tissue that is red, edematous, tender, and warm to the touch

ANSWER: A

A

B

C

D

Feedback

This finding is consistent with fibrocystic breast disease. The findings are almost

the same as fibroadenoma (except for consistency of the masses), but the nurse

must correlate the history (occurs during the menses) with the findings for

fibrocystic disease.

These findings are consistent with fibroadenoma.

These findings are consistent with breast cancer.

These findings are consistent with mastitis.

DIF: Cognitive Level: Apply REF: 381

TOP: Nursing Process: Assessment

MSC: NCLEX Patient Need Category: Physiologic Integrity: Physiologic Adaptation: Alteration in

Body Systems

21. The nurse notices dimpling of the skin surrounding a palpable mass in the right breast of a

female patient. What is the most appropriate action for the nurse to take next?

a. Record this as an expected finding.

b. Palpate the area of dimpling for pain.

c. Palpate the borders of the area of dimpling for irregularity.

d. Tell the patient that dimpling indicates the mass is benign.

ANSWER: C

Feedback


A

B

C

D

This finding is not normal.

Malignant breast masses are not tender.

Malignant breast masses are irregular and poorly delineated. As the mass grows,

there may be breast asymmetry, discoloration (erythema or ecchymosis),

unilateral vein prominence, peau d’orange, ulceration, dimpling, puckering, or

retraction of the skin.

Dimpling may indicate a malignant mass rather than a benign mass.

DIF: Cognitive Level: Apply REF: 371-372| 383

TOP: Nursing Process: Assessment

MSC: NCLEX Patient Need Category: Physiologic Integrity: Physiologic Adaptation: Alteration in

Body Systems

22. A nurse becomes suspicious that a patient may have breast cancer based on which abnormal

finding?

a. An irregularly shaped hard mass in one breast

b. Bilateral, small, nontender nodes close to the surface

c. Multiple rubbery-feeling lumps with well-defined borders

d. A mobile, firm lump located in the upper outer quadrant of the left breast

ANSWER: A

A

B

C

D

Feedback

Malignant masses are solitary, irregularly-shaped, unilateral, nontender, and

immobile.

Malignant masses are unilateral.

Malignant masses have irregular, poorly defined borders.

Breast cancer often occurs in the upper outer quadrant, but is immobile.

DIF: Cognitive Level: Understand REF: 376| 383

TOP: Nursing Process: Assessment

MSC: NCLEX Patient Need Category: Physiologic Integrity: Physiologic Adaptation: Alteration in

Body Systems

23. The nurse would give immediate attention to the patient who presents with which complaint?

a. Bilateral breast swelling

b. Unilateral nipple discharge

c. A breast lump that changes during the menstrual cycle

d. Unequal breast size

ANSWER: B

A

B

C

D

Feedback

Bilateral breast swelling is frequently related to fibrocystic breast disease.

A breast malignancy usually manifests in one breast, and a serosanguineous or

clear nipple discharge may be present.

A breast lump that changes during the menstrual cycle is frequently related to

fibrocystic breast disease.

Unequal breast size that does not include a mass in one breast is not an immediate

concern.


DIF: Cognitive Level: Understand REF: 374| 377

TOP: Nursing Process: Assessment

MSC: NCLEX Patient Need Category: Physiologic Integrity: Physiologic Adaptation: Alteration in

Body Systems

MULTIPLE RESPONSE

1. Which life style behaviors do nurses ask about to identify patients with risk factors for breast

cancer? Select all that apply.

a. Obesity after age 50

b. Smoking more than one pack of cigarettes a day

c. Never having given birth to a viable infant

d. Drinking two to five alcoholic beverages a day

e. Estrogen replacement therapy for more than 5 years

f. High blood pressure for more than 3 years

ANSWER: A, C, D, E

Correct: Obesity after age 50, never having given birth to a viable infant (nulliparity),

drinking two to five alcoholic beverages a day, and estrogen replacement therapy for more

than 5 years are all risk factors for breast cancer.

Incorrect: Although smoking and high blood pressure are risk factors for many disorders,

they are not risk factors for breast cancer.

DIF: Cognitive Level: Understand REF: 381

TOP: Nursing Process: Assessment

MSC: NCLEX Patient Need Category: Physiologic Integrity: Reduction of Risk Potential: Potential

for Alteration in Body Systems


Chapter 17: Reproductive System and the Perineum

Test Bank

MULTIPLE CHOICE

1. During the initial inspection of the female genitalia, the nurse recognizes which finding as

normal?

a. The labia minora are hair-covered and lying within the labia majora.

b. The cervical os in the multiparous woman has the shape of a small circle.

c. The vaginal vestibule lies between the labia minora and contains the urinary

meatus.

d. The openings of Skene and Bartholin glands are visible posteriorly.

ANSWER: C

A

B

C

D

Feedback

The labia majora, rather than the labia minora, are covered with hair.

The os of parous women is the shape of a slit.

This description is of normal female anatomy.

The opening of these glands is on either side of the vaginal vestibule.

DIF: Cognitive Level: Understand REF: 386| 401

TOP: Nursing Process: Assessment

MSC: NCLEX Patient Needs: Physiologic Integrity: Reduction of Risk Potential: System Specific

Assessments

2. The pregnant patient tells the nurse that she has had three pregnancies and two live births to

date. How does the nurse record this in the patient’s history?

a. Gravida 3, para 3

b. Gravida 3, para 2

c. Gravida 2, para 3

d. Gravida 2, para 2

ANSWER: B

A

B

C

D

Feedback

Gravida 3, para 3 represents three pregnancies and three that reached 20 weeks or

longer.

Gravida 3, para 2 represents three pregnancies and two that reached 20 weeks or

longer, which is consistent with what the patient reported.

Gravida 2, para 3 represents two pregnancies and three that reached 20 weeks or

longer, which is not possible.

Gravida 2, para 2 represents two pregnancies and two that reached 20 weeks or

longer.

DIF: Cognitive Level: Apply REF: 394

TOP: Nursing Process: Assessment

MSC: NCLEX Patient Needs: Physiologic Integrity: Reduction of Risk Potential: System Specific

Assessments


3. A mother asks a nurse when her daughter should get immunized again for human papilloma

virus (HPV). What is the nurse’s most appropriate response to this question?

a. “Your daughter does not need this immunization until she becomes sexually

active.”

b. “The recommended age for this immunization is between ages 25 and 30 years of

age.”

c. “Between the ages of 11 and 26 years is the recommended time for this

immunization.”

d. “When she begins having menstrual periods is the best time for this immunization.”

ANSWER: C

A

B

C

D

Feedback

This is not the recommendation of the CDC.

This is not the recommendation of the CDC.

This is the recommendation from the Centers for Disease Control and Prevention

(CDC).

This is not the recommendation of the CDC.

DIF: Cognitive Level: Apply REF: 398

TOP: Nursing Process: Assessment

MSC: NCLEX Patient Needs: Health Promotion and Maintenance: Health Promotion Programs

4. A patient asks when she should make an appointment for her first Pap (Papanicolaou) test to

screen for cervical cancer. What is the nurse’s most appropriate response?

a. “There is no need for Pap tests until after you have become pregnant.”

b. “All women should have the first Pap test after reaching menarche.”

c. “All women should have the first Pap test after they are 19 years of age.”

d. “All women should have the first Pap test when they become sexually active or at

age 21.”

ANSWER: D

A

B

C

D

Feedback

The recommendation is when females become sexually active or 21 years old.

The recommendation is when females become sexually active or 21 years old.

All females should be screened when they become sexually active or 21 years

old, whichever happens first.

This is the recommendation from the U.S. Preventive Services Task Force. All

females should be screened when they become sexually active or age 21,

whichever happens first.

DIF: Cognitive Level: Apply REF: 399

TOP: Nursing Process: Assessment

MSC: NCLEX Patient Needs: Health Promotion and Maintenance: Health Promotion Programs

5. A patient asks when she can stop having Pap (Papanicolaou) tests. What is the nurse’s most

appropriate response?

a. “Until you are no longer sexually active.”

b. “Through age 65.”

c. “Until you begin menopause.”


d. “Through the end of menopause.”

ANSWER: B

A

B

C

D

Feedback

The recommendation from the U.S. Preventive Services Task Force is through

age 65.

This is the recommendation from the U.S. Preventive Services Task Force. All

females should be screened after the onset of sexual activity through age 65.

The recommendation from the U.S. Preventive Services Task Force is through

age 65.

The recommendation from the U.S. Preventive Services Task Force is through

age 65.

DIF: Cognitive Level: Apply REF: 399

TOP: Nursing Process: Assessment

MSC: NCLEX Patient Needs: Health Promotion and Maintenance: Health Promotion Programs

6. When performing a well woman examination, the nurse expects what findings?

a. The inner surface of the vestibule is deep pink and moist with a smooth texture.

b. The inguinal skin appears wrinkled and moist with sparse hair distribution.

c. The labia minora is deeply pigmented, and the tissue is ragged and asymmetrical.

d. Pubic hair is distributed evenly over the mons and shaped as a triangle with the

apex over the mons.

ANSWER: A

A

B

C

D

Feedback

This is a normal finding of female external genitalia.

The inguinal area would normally not be wrinkled.

The labia minora is not normally ragged and asymmetric.

Normally the base of the triangle is over the mons.

DIF: Cognitive Level: Understand REF: 401

TOP: Nursing Process: Assessment

MSC: NCLEX Patient Needs: Physiologic Integrity: Reduction of Risk Potential: System Specific

Assessments

7. The nurse documents which finding as expected on inspection of the anus?

a. Skin tone darker and coarser than that of the surrounding skin

b. Sphincter lightly closed when the patient is relaxed

c. Large amount of stiff, curling hair surrounding the anus

d. Slight protrusion under the skin when the patient strains or bears down

ANSWER: A

A

B

C

D

Feedback

This is the normal finding.

The anal sphincter should be tight.

The anus is typically hairless.

A protrusion may be a hemorrhoid, which is not an expected finding.


DIF: Cognitive Level: Understand REF: 392| 402

TOP: Nursing Process: Assessment

MSC: NCLEX Patient Needs: Physiologic Integrity: Reduction of Risk Potential: System Specific

Assessments

8. On inspection of the internal structure of the vagina, the nurse notes a rounded protrusion on

the posterior wall of the vagina. How does the nurse document this finding?

a. Rectocele

b. Cystocele

c. Bartholin cyst

d. Nabothian cyst

ANSWER: A

A

B

C

D

Feedback

Rectocele is a hernia type of protrusion of the rectum against the posterior wall of

the vagina.

Cystocele is a hernia type of protrusion of the bladder against the posterior wall

of the vagina.

The Bartholin glands are external structures.

Nabothian cysts appear on the cervix.

DIF: Cognitive Level: Understand REF: 403| 437

TOP: Nursing Process: Assessment

MSC: NCLEX Patient Needs: Physiologic Integrity: Physiologic Adaptation: Alteration in Body

Systems

9. During the examination of the internal genitalia with the speculum, the nurse records which

finding as normal?

a. A healed laceration of the cervix in a nulliparous patient

b. A large amount of thick white drainage from the cervical os

c. Deviation of the cervix toward the posterior vaginal wall

d. Pink cervix with a small ring of reddened tissue near the os

ANSWER: D

A

B

C

D

Feedback

This is an abnormal finding; a laceration is not expected in nulliparous women.

This is an abnormal finding; this drainage may indicate a sexually transmitted

disease.

This is an abnormal finding.

This is a normal finding.

DIF: Cognitive Level: Understand REF: 406

TOP: Nursing Process: Assessment

MSC: NCLEX Patient Needs: Physiologic Integrity: Reduction of Risk Potential: System Specific

Assessments

10. The nurse recognizes that a Papanicolaou (Pap) test is indicated for which patient?

a. A 12-year-old who has not yet reached menarche.


b. A 30-year-old who had a normal Pap test 12 months ago.

c. A 45-year-old who had a total hysterectomy for cervical cancer.

d. A 55-year-old who had a total hysterectomy to treat endometriosis.

ANSWER: C

A

B

C

D

Feedback

A Pap test is not indicated for this patient.

A Pap test is not indicated for this patient.

In women who have undergone a hysterectomy in which the cervix was removed,

Pap testing is not required unless the hysterectomy was performed because of

cervical cancer or its precursors.

A Pap test is not indicated for this patient.

DIF: Cognitive Level: Understand REF: 406

TOP: Nursing Process: Assessment

MSC: NCLEX Patient Needs: Physiologic Integrity: Reduction of Risk Potential: System Specific

Assessments

11. What technique does the nurse use to obtain a cervical tissue sample for a Papanicolaou (Pap)

test?

a. A Cervex-Brush is inserted into the cervix and rotated to obtain a sample of

ectocervical and endocervical cells.

b. A wooden spatula scrapes the cervix to obtain a sample of endocervical cells.

c. A pipette is placed inside the cervical os and rotated to obtain a thick layer of

endocervical and ectocervical cells.

d. A cotton-tipped applicator is used on the outside of the cervix to obtain ectocervical

cells.

ANSWER: A

A

B

C

D

Feedback

This is the correct technique.

A wooden spatula is not used and ectocervical cells are needed as well as

endocervical cells.

A pipette is not used and the cells are not contained in a thick layer.

Using a brush as opposed to a cotton-tipped applicator has improved the quality

of the sample of endocervical cells and ectocervical cells.

DIF: Cognitive Level: Understand REF: 407-408

TOP: Nursing Process: Assessment

MSC: NCLEX Patient Needs: Health Promotion and Maintenance: Techniques of Physical

Assessment

12. A nurse expects which normal findings when performing a bimanual palpation of the cervix

and uterus?

a. The uterus feels firm and slightly nodular.

b. The cervix feels soft, smooth, and slightly rounded.

c. The uterus of a nonpregnant patient cannot be felt with the internal fingers.

d. The cervix is tender when moved laterally.


ANSWER: B

A

B

C

D

Feedback

The uterus normally does not feel nodular.

These are the normal findings from a bimanual examination of the cervix.

The uterus of a nonpregnant woman can be palpated.

The cervix normally is not tender; if it is, it may indicate a sexually transmitted

disease.

DIF: Cognitive Level: Remember REF: 409

TOP: Nursing Process: Assessment

MSC: NCLEX Patient Needs: Physiologic Integrity: Reduction of Risk Potential: System Specific

Assessments

13. A nurse expects which normal findings when palpating a patient’s ovaries?

a. Nodular and nonmovable

b. Smooth, fluid-filled, and nonmovable

c. Smooth, firm, and about the size of a walnut

d. Spongy, mobile, and about the size of a peanut

ANSWER: C

A

B

C

D

Feedback

Normally the ovaries do not feel nodular and nonmovable.

Normally the ovaries do not feel fluid-filled and nonmovable.

This is a correct description of a normal finding when palpating an ovary.

Normally the ovaries do not feel spongy or mobile.

DIF: Cognitive Level: Remember REF: 411

TOP: Nursing Process: Assessment

MSC: NCLEX Patient Needs: Physiologic Integrity: Reduction of Risk Potential: System Specific

Assessments

14. After a rectal examination of a patient with obstructive jaundice, the nurse expects the stool to

be what color?

a. Tan

b. Pale yellow

c. Black

d. Bright red

ANSWER: A

A

B

C

D

Feedback

Tan stool indicates a lack of bile caused by obstructive jaundice.

Pale yellow stools indicate a malabsorption syndrome.

Black stools indicate upper intestinal tract bleeding or excessive iron or bismuth

ingestion.

Bright red indicates bleeding from the lower rectum or hemorrhoids.

DIF: Cognitive Level: Understand REF: 414

TOP: Nursing Process: Assessment


MSC: NCLEX Patient Needs: Physiologic Integrity: Physiologic Adaptation: Alteration in Body

Systems

15. On inspection of the external male genitalia, the nurse notes which finding as abnormal?

a. The scrotum is covered with dark rugous skin.

b. The skin covering the penis is hairless and loose.

c. The urinary meatus is located on the upper surface of the penis.

d. The left side of the scrotum hangs slightly lower than the right.

ANSWER: C

A

B

C

D

Feedback

This is a normal finding.

This is a normal finding.

This is called epispadias.

This is a normal finding.

DIF: Cognitive Level: Understand REF: 417

TOP: Nursing Process: Assessment

MSC: NCLEX Patient Needs: Physiologic Integrity: Physiologic Adaptation: Alteration in Body

Systems

16. The nurse observes that the urinary meatus is located on the under surface of the penis. How

does the nurse document this finding?

a. Balanitis

b. Phimosis

c. Epispadias

d. Hypospadias

ANSWER: D

A

B

C

D

Feedback

Balanitis is inflammation of the glans that occurs in patients with phimosis.

Phimosis is a very tight foreskin that cannot be retracted over the glans.

Epispadias occur when the urinary meatus is on the upper (dorsum) surface of the

penis.

Hypospadias occur when the urinary meatus is on the undersurface of the penis.

DIF: Cognitive Level: Understand REF: 417

TOP: Nursing Process: Assessment

MSC: NCLEX Patient Needs: Physiologic Integrity: Physiologic Adaptation: Alteration in Body

Systems

17. In inspecting the scrotum, the nurse documents which finding as normal?

a. The epididymides are round, solid nodular masses.

b. The scrotum is deeply pigmented with a rugous surface.

c. The scrotal skin is a lighter color than the body skin.

d. The vas deferens is palpable bilaterally.

ANSWER: B

Feedback


A

B

C

D

Normally the epididymis is a tubular, comma-shaped structure.

This is an expected finding.

Normally the scrotal skin is more deeply pigmented than the body skin.

Normally the vas deferens is not palpable.

DIF: Cognitive Level: Understand REF: 419

TOP: Nursing Process: Assessment

MSC: NCLEX Patient Needs: Physiologic Integrity: Reduction of Risk Potential: System Specific

Assessments

18. Which assessment technique does a nurse use to assess the inguinal region and femoral area of

a male patient as he is standing and straining?

a. Palpates the femoral artery

b. Palpates the inguinal lymph nodes

c. Observes for a bulge through the inguinal region

d. Observes for discoloration of the inguinal ring

ANSWER: C

A

B

C

D

Feedback

The nurse palpates the femoral artery when the patient is lying supine.

The nurse palpates the inguinal lymph nodes when the patient is lying supine.

The nurse observes for a bulge that may indicate a hernia; the normal finding is

no bulge.

The nurse cannot see the inguinal ring; it must be palpated.

DIF: Cognitive Level: Apply REF: 419

TOP: Nursing Process: Assessment

MSC: NCLEX Patient Needs: Health Promotion and Maintenance: Techniques of Physical

Assessment

19. When palpating the epididymis, the nurse considers which finding to be abnormal?

a. The epididymis is located on the posterolateral surface of each testis.

b. The epididymis feels like a tubular, comma-shaped structure.

c. The epididymis collapses on palpation.

d. The epididymis has an irregular, nodular surface.

ANSWER: D

A

B

C

D

Feedback

This is an expected finding.

This is an expected finding.

This is an expected finding.

The surface should be smooth and nontender.

DIF: Cognitive Level: Understand REF: 420

TOP: Nursing Process: Assessment

MSC: NCLEX Patient Needs: Physiologic Integrity: Physiologic Adaptation: Alteration in Body

Systems

20. When does a nurse use transillumination of the scrotum?


a. When the patient has tortuosity of the veins along the spermatic cord

b. When the patient has an indirect hernia

c. When there is a mass or fluid in the epididymis

d. When there is twisting of the testicle and spermatic cord

ANSWER: C

A

B

C

D

Feedback

This is a description of a varicocele, which does not transilluminate.

Hernias do not transilluminate.

This is a description of a spermatocele, which does transilluminate, as does a

hydrocele.

This is a description of testicular torsion.

DIF: Cognitive Level: Understand REF: 420

TOP: Nursing Process: Assessment

MSC: NCLEX Patient Needs: Physiologic Integrity: Physiologic Adaptation: Alteration in Body

Systems

21. What procedure does a nurse use to assess the inguinal ring of a male patient for a hernia?

a. Asks the patient to lie supine, lifts the scrotum, asks the patient to take a deep

breath, and observes for a bulge

b. Asks the patient to lean over the examination table, inserts a gloved finger into the

lower part of the scrotum into the inguinal canal, asks the patient to cough, and

palpates for a bulge

c. Asks the patient to lie on the side not being assessed, inserts a gloved finger into

the lower part of the scrotum into the inguinal canal, asks the patient to exhale

completely, and palpates for a bulge

d. Asks the patient to stand, inserts a gloved finger into the lower part of the scrotum

into the inguinal canal, asks the patient to cough, and palpates for a bulge

ANSWER: D

A

B

C

D

Feedback

This has the patient in the wrong position with the wrong technique and wrong

instruction given.

This has the patient in the wrong position.

This has the patient in the wrong position with the wrong instruction given.

This describes the correct procedure.

DIF: Cognitive Level: Understand REF: 421

TOP: Nursing Process: Assessment

MSC: NCLEX Patient Needs: Health Promotion and Maintenance: Techniques of Physical

Assessment

22. The nurse places a male patient in which position for rectal examination?

a. Lithotomy position

b. Prone with the knees fully extended

c. Bending over the table, with feet everted

d. Left lateral position with knees and hips flexed


ANSWER: D

A

B

C

D

Feedback

This is the appropriate position for a rectal examination for a female patient.

This position is not used; a knee chest position may be used instead.

This is the appropriate position of a rectal examination for a male patient, except

that the feet are inverted, rather than everted.

This is the appropriate position for a rectal examination for a male patient.

DIF: Cognitive Level: Understand REF: 421-422

TOP: Nursing Process: Assessment

MSC: NCLEX Patient Needs: Health Promotion and Maintenance: Techniques of Physical

Assessment

23. During an internal examination of a patient’s anus, the nurse notes that the patient has a

hypertonic sphincter. What is the most relevant action for the nurse to take at this time?

a. Ask the patient about anxiety or pain related to the examination.

b. Inquire if the patient has had any neurologic injury that causes a hypertonic

sphincter.

c. Refer the patient to the physician for evaluation.

d. Question the patient about a history of anal trauma.

ANSWER: A

A

B

C

D

Feedback

This is the relevant action for the nurse to collect more data from the patient

about the reaction to the examination.

This datum probably would have been gathered during the history, so that the

nurse would have anticipated an abnormal finding.

This is not the action of most importance at this time. The nurse needs to collect

more data from the patient before considering a referral.

This is not the action of most importance at this time. The nurse needs to collect

more data from the patient before asking about trauma.

DIF: Cognitive Level: Understand REF: 421

TOP: Nursing Process: Assessment

MSC: NCLEX Patient Needs: Health Promotion and Maintenance: Techniques of Physical

Assessment

24. What normal finding does a nurse expect to find when palpating a male patient’s prostate

gland?

a. Is approximately 4.5 cm in diameter and is highly mobile

b. Feels smooth, firm, and slightly mobile

c. Is deeply divided into three lobes, each approximately 2 cm in length

d. Feels hard, asymmetrical, and has a palpable ridge that divides the gland into two

lobes

ANSWER: B

A

Feedback

A normal prostate is 1.5 inches in diameter and slightly mobile.


B

C

D

This is a normal finding.

The prostate consists of two lobes.

The prostate feels firm, smooth, and symmetric.

DIF: Cognitive Level: Understand REF: 423

TOP: Nursing Process: Assessment

MSC: NCLEX Patient Needs: Physiologic Integrity: Reduction of Risk Potential: System Specific

Assessments

25. A 50-year-old patient asks the nurse about her risk of developing a cancer of the reproductive

system. What is the appropriate response by the nurse?

a. “Human papilloma virus infection and cigarette smoking are major risk factors for

cervical cancer.”

b. “Some of the risk factors for endometrial cancer include being age 40 or older and

having a history of infertility.”

c. “Ovarian cancer is not often seen in women under age 50 or those who have a

family history of breast cancer.”

d. “Women who have had menstrual irregularities for many years are at lower risk of

developing any of the reproductive system cancers.”

ANSWER: A

A

B

C

D

Feedback

Human papilloma virus infection and cigarette smoking are risk factors for

cervical cancer.

These are not risk factors for endometrial cancer.

These are not risk factors for ovarian cancer.

These are not risk factors for gynecologic cancers.

DIF: Cognitive Level: Understand REF: 424

TOP: Nursing Process: Intervention

MSC: NCLEX Patient Needs: Health Promotion and Maintenance: Health Promotion Programs

26. The nurse recognizes which patient has the highest risk of endometrial cancer?

a. A 24-year old woman with menarche at age 9

b. A 30-year old woman who started menstruating at age 19

c. A 42-year old woman who reached menopause at age 40

d. A 64-year old woman who had irregular, heavy menstrual cycles

ANSWER: A

A

B

C

D

Feedback

Early menarche is a risk factor.

This age of menarche is not a risk factor for endometrial cancer.

This age of menopause is not a risk factor for endometrial cancer. Patients who

have late onset menopause are at risk.

An irregular, heavy menstrual cycle is not a risk factor for endometrial cancer.

DIF: Cognitive Level: Understand REF: 424

TOP: Nursing Process: Assessment

MSC: NCLEX Patient Needs: Health Promotion and Maintenance: Health Promotion Programs


27. Which patient does the nurse recognize as having the highest risk for ovarian cancer?

a. A 24-year-old nulliparous woman who has a history of multiple sexual partners

b. A 32-year-old woman who has had six live births and a history of human papilloma

virus (HPV) infection

c. A 55-year-old woman who reached menarche at age 12 and menopause at age 54

d. A 64-year-old nulliparous woman who has taken hormone replacement therapy for

eight years

ANSWER: D

A

B

C

D

Feedback

This patient has no risk factor for ovarian cancer.

This patient has a risk factor for cervical cancer (HPV), but not ovarian cancer.

This patient has no risk factors for ovarian cancer.

This patient has a risk factor for ovarian cancer.

DIF: Cognitive Level: Understand REF: 424

TOP: Nursing Process: Assessment

MSC: NCLEX Patient Needs: Health Promotion and Maintenance: Health Promotion Programs

28. The nurse correlates which factor to an increased risk of endometrial cancer in women with

early menarche or late menopause?

a. Total number of ovulatory cycles

b. Less hormone stimulation

c. Need for estrogen replacement in these patients

d. Extended duration of the menstrual cycle in these patients

ANSWER: A

A

B

C

D

Feedback

More ovulatory cycles increases risk. These risk factors represent an increased

cumulative exposure to estrogen.

Hormone stimulation does not increase risk in endometrial cancer.

Estrogen replacement does not increase risk in endometrial cancer.

Extended duration of the menstrual cycle is not a risk factor.

DIF: Cognitive Level: Understand REF: 424

TOP: Nursing Process: Assessment

MSC: NCLEX Patient Needs: Health Promotion and Maintenance: Health Promotion Programs

29. A patient complains of dysuria, yellow-green vaginal discharge, and vulvar itching. The nurse

suspects which sexually transmitted disease?

a. Syphilis

b. Gonorrhea

c. Genital warts

d. Chlamydia

ANSWER: B

Feedback


A

B

C

D

Primary syphilis produces a single, firm, painless open sore or chancre with

indurated borders at the site of entry on the genitals.

Gonorrhea causes a yellow or green vaginal discharge, dysuria, pelvic or

abdominal pain, and vaginal itching and burning.

Genital warts appear as soft, papillary, pink to brown, elongated lesions that may

occur singularly or in clusters on the internal genitalia, the external genitalia, and

the anal-rectal region.

Chlamydia infection is asymptomatic in up to 75% of women because it often

does not cause enough inflammation to produce symptoms.

DIF: Cognitive Level: Remember REF: 425

TOP: Nursing Process: Assessment

MSC: NCLEX Patient Needs: Physiologic Integrity: Physiologic Adaptation: Alteration in Body

Systems

30. In assessing a patient with suspected Chlamydia, the nurse’s actions are guided by which

characteristic of this disease?

a. Chlamydia is frequently asymptomatic and requires screening.

b. Chlamydia is associated with a yellow-green vaginal discharge.

c. Chlamydia is accompanied by heavy bleeding and headache.

d. Chlamydia is only seen in immunocompromised patients.

ANSWER: A

A

B

C

D

Feedback

This answer is consistent with clinical findings of Chlamydia.

This answer is consistent with clinical findings of gonorrhea.

Chlamydia is not accompanied by heavy bleeding and headache.

Chlamydia is seen in patients with healthy immune systems who are not

immunocompromised.

DIF: Cognitive Level: Understand REF: 425

TOP: Nursing Process: Assessment

MSC: NCLEX Patient Needs: Physiologic Integrity: Physiologic Adaptation: Alteration in Body

Systems

31. A nurse examines a patient and finds a single, firm, painless open sore with indurated borders

on the vulva. The nurse correlates this finding with which disorder?

a. Human papillomavirus (HPV) infection

b. Herpes infection

c. Gonorrhea

d. Syphilis

ANSWER: D

A

B

C

D

Feedback

HPV infection causes wartlike growths.

Herpes infection forms vesicles rather than chancres.

Gonorrhea produces a yellow or green vaginal discharge.

The clinical finding is consistent with a chancre found in syphilis.


DIF: Cognitive Level: Apply REF: 425

TOP: Nursing Process: Assessment

MSC: NCLEX Patient Needs: Physiologic Integrity: Physiologic Adaptation: Alteration in Body

Systems

32. A nurse expects to find which manifestations in the male patient who has both Chlamydia and

gonorrhea?

a. Painful urination and purulent urethral discharge

b. A single, firm painless open sore on the shaft of the penis

c. Red superficial vesicles on the shaft of the penis

d. A single or a cluster of wartlike growth in the anal-rectal area

ANSWER: A

A

B

C

D

Feedback

These are manifestations of Chlamydia and gonorrhea. Dysuria means painful

urination.

This is a description of a lesion consistent with syphilis.

This is a description of lesions consistent with herpes genitalis.

This is a description of lesions consistent with genital warts caused by human

papillomavirus.

DIF: Cognitive Level: Apply REF: 427

TOP: Nursing Process: Assessment

MSC: NCLEX Patient Needs: Physiologic Integrity: Physiologic Adaptation: Alteration in Body

Systems

33. While giving a history, the patient reports having herpes genitalis. Based on this information,

which finding does the nurse anticipate during the assessment?

a. Small vesicles on the genitalia

b. Single, firm, painless, open sore

c. Pain when palpating the cervix

d. Malodorous greenish-yellow vaginal discharge

ANSWER: A

A

B

C

D

Feedback

Small vesicles on the genitalia are consistent with genital herpes.

Single, firm, painless, open sore is consistent with primary syphilis.

Pain when palpating the cervix is consistent with Chlamydia.

Malodorous greenish-yellow vaginal discharge is consistent with trichomonas.

DIF: Cognitive Level: Apply REF: 427

TOP: Nursing Process: Assessment

MSC: NCLEX Patient Needs: Physiologic Integrity: Physiologic Adaptation: Alteration in Body

Systems

34. In teaching a class of adolescents about sexually transmitted diseases, a nurse includes which

information about the human papillomavirus (HPV)?

a. HPV is fragile and not easily transmitted.

b. Wartlike growths in the genital area are a sign of HPV infection.


c. There is a specific blood test needed to screen for HPV.

d. Heavy, purulent vaginal discharge is the primary sign of HPV.

ANSWER: B

A

B

C

D

Feedback

HPV is highly contagious.

This is the correct description of the clinical findings of HPV.

There is no specific blood test to detect HPV.

HPV is a virus that develops warts.

DIF: Cognitive Level: Understand REF: 427

TOP: Nursing Process: Intervention

MSC: NCLEX Patient Needs: Physiologic Integrity: Physiologic Adaptation: Alteration in Body

Systems

35. The patient is unable to tolerate a bimanual pelvic examination due to pain in ovaries and

fallopian tubes. Which disorder does the nurse suspect?

a. Tertiary syphilis

b. Genital herpes

c. Human papillomavirus (HPV) infection

d. Pelvic inflammatory disease

ANSWER: D

A

B

C

D

Feedback

Tertiary syphilis does not cause much pain.

Genital herpes does not cause much pain.

HPV infection does not cause much pain.

Typically, the pain is so severe that the patient with pelvic inflammatory disease

is unable to tolerate bimanual pelvic examination.

DIF: Cognitive Level: Understand REF: 428

TOP: Nursing Process: Assessment

MSC: NCLEX Patient Needs: Physiologic Integrity: Physiologic Adaptation: Alteration in Body

Systems

36. A patient with testicular torsion is experiencing which abnormality?

a. Abnormal dilation and tortuosity of the veins along the spermatic cord

b. Twisting of the testicle and spermatic cord

c. A cystic mass filled with sperm and seminal fluid in the epididymis

d. An accumulation of fluid in the scrotum

ANSWER: B

A

B

C

Feedback

Abnormal dilation and tortuosity of the veins along the spermatic cord describes a

varicocele.

Twisting of the testicle and spermatic cord describes testicular torsion.

A cystic mass filled with sperm and seminal fluid in the epididymis describes a

spermatocele.


D

An accumulation of fluid in the scrotum describes a hydrocele.

DIF: Cognitive Level: Understand REF: 430

TOP: Nursing Process: Assessment

MSC: NCLEX Patient Needs: Physiologic Integrity: Physiologic Adaptation: Alteration in Body

Systems

37. How does a nurse recognize when a patient has a testicular torsion?

a. The nurse sees a light red glow on transillumination of the scrotum.

b. The nurse palpates testicular edema that is painless.

c. The patient reports a pulling sensation and dull ache of the scrotum.

d. The patient complains of sudden onset of severe pain with edema of the scrotum.

ANSWER: D

A

B

C

D

Feedback

This is a clinical finding of hydrocele.

This is a clinical finding of spermatocele.

This is a clinical finding of varicocele.

These are clinical findings of testicular torsion.

DIF: Cognitive Level: Understand REF: 430

TOP: Nursing Process: Assessment

MSC: NCLEX Patient Needs: Physiologic Integrity: Physiologic Adaptation: Alteration in Body

Systems

38. In educating a male patient about testicular cancer, the nurse includes which statement?

a. The highest incidence of this cancer is in men between 20 and 34 years of age.

b. The incidence of this cancer is correlated with human papillomavirus (HPV)

infection.

c. The risk of this cancer increases with multiple sexual partners.

d. This type of cancer more commonly affects uncircumcised males.

ANSWER: A

A

B

C

D

Feedback

This is a true statement of risk.

HPV infection is correlated with cervical cancer, rather than testicular cancer.

The risk of sexually transmitted diseases increases with multiple sexual partners

rather than testicular cancer.

This is not a risk factor.

DIF: Cognitive Level: Remember REF: 431

TOP: Nursing Process: Intervention

MSC: NCLEX Patient Needs: Health Promotion and Maintenance: Health Promotion Programs

39. While taking a history of a patient with an enlarged prostate, the nurse expects the patient to

report which symptom?

a. Painful urination with each voiding

b. Blood in the urine upon arising

c. Waking from sleep to urinate


d. Incontinence throughout the day

ANSWER: C

A

B

C

D

Feedback

There is no pain associated with an enlarged prostate.

This is not a manifestation of a prostate disorder.

Compression of the urethra by the enlarged prostate may cause men to be

awakened from sleep to urinate (nocturia).

The urinary problem associated with a prostate disorder is difficulty in starting

the urinary stream because the enlarged prostate gland compresses the urethra,

thus incontinence is not associated with a prostate disorder.

DIF: Cognitive Level: Understand REF: 397| 431

TOP: Nursing Process: Assessment

MSC: NCLEX Patient Needs: Physiologic Integrity: Physiologic Adaptation: Alteration in Body

Systems

40. The nurse correlates which patient complaint with suspected enlargement of the prostate

gland?

a. Constipation

b. Change in bowel patterns

c. Weak urine stream

d. Increased mucus in urine

ANSWER: C

A

B

C

D

Feedback

Enlargement of the prostate gland does not cause constipation.

This is a warning sign for colon cancer rather than a manifestation of an enlarged

prostate.

Enlargement of the prostate gland compresses the urethra causing a weak urinary

stream.

This may be an indication of a urinary tract infection rather than an enlarged

prostate.

DIF: Cognitive Level: Understand REF: 396| 423| 431

TOP: Nursing Process: Assessment

MSC: NCLEX Patient Needs: Physiologic Integrity: Physiologic Adaptation: Alteration in Body

Systems

41. While giving a history, a patient reports having a weak urinary stream and feeling that his

bladder is not empty after urination. Based on these data, what finding does the nurse

anticipate upon examination?

a. An enlarged prostate gland palpated on the anterior wall of the rectum

b. An indirect hernia palpated through the inguinal ring when the patient coughs

c. The foreskin of the penis cannot be returned to position after retraction behind the

glans

d. A nodular prostate gland palpated on the posterior wall of the rectum

ANSWER: A


A

B

C

D

Feedback

The enlarged prostate compresses the urethra, causing difficulties with voiding.

A hernia would not interfere with voiding.

This is a description of paraphimosis.

The posterior prostate is palpated on the anterior surface of the rectum.

DIF: Cognitive Level: Apply REF: 396| 423| 431

TOP: Nursing Process: Assessment

MSC: NCLEX Patient Needs: Physiologic Integrity: Physiologic Adaptation: Alteration in Body

Systems

42. A patient tells the nurse that he has been informed he has internal hemorrhoids and asks

whether there are different types of hemorrhoids. What is the nurse’s most appropriate

response?

a. “Internal hemorrhoids are usually seen outside the anus and appear blue.”

b. “Sometimes patients have other diseases, such as anal warts, that may be mistaken

for internal hemorrhoids.”

c. “Internal hemorrhoids are found higher in the rectum and usually can’t be felt

unless they are infected or prolapsed.”

d. “Both internal and external hemorrhoids arise from the same general area and

produce the same kinds of symptoms.”

ANSWER: C

A

B

C

D

Feedback

Internal hemorrhoids are found high in the rectum and may be felt if infected, but

are not seen during an anal examination.

Anal warts have a distinct appearance that is different from hemorrhoids.

This is a correct statement.

Internal hemorrhoids are found higher in the rectum, while external hemorrhoids

are found outside the external rectal sphincter.

DIF: Cognitive Level: Understand REF: 433

TOP: Nursing Process: Assessment

MSC: NCLEX Patient Needs: Physiologic Integrity: Physiologic Adaptation: Alteration in Body

Systems

43. During a history, a patient reports rectal bleeding, a warning sign of colorectal cancer. The

nurse correlates which clinical finding with colorectal cancer?

a. Thick, blood-tinged mucus within the rectum

b. A pus-filled cavity in the anorectal area

c. An irregular mass with raised edges on the rectal wall

d. A small, smooth nodule protruding from the rectum

ANSWER: C

A

B

C

Feedback

This is not an indication of colorectal cancer.

This is an indication of an infection in the anal area.

This finding is indicative of colorectal cancer.


D

This may be a polyp protruding from the anus.

DIF: Cognitive Level: Understand REF: 397| 422| 434-435

TOP: Nursing Process: Assessment

MSC: NCLEX Patient Needs: Physiologic Integrity: Physiologic Adaptation: Alteration in Body

Systems

MULTIPLE RESPONSE

1. Which comments by a male patient during a health history suggest erectile dysfunction?

Select all that apply.

a. “I have had type 1 diabetes mellitus since I was 8 years old.”

b. “I frequently have urinary tract infections.”

c. “I am taking medications to control my blood pressure.”

d. “I have an enlarged prostate gland.”

e. “I take a diuretic every morning.”

ANSWER: A, C, E

Correct: A chronic complication of diabetes can cause impotence. An adverse reaction of

some types of antihypertensive and diuretic medications can cause impotence.

Incorrect: Urinary tract infections are unrelated to impotence. Having an enlarged prostate

causes problems with urinating, but not with erections.

DIF: Cognitive Level: Analyze REF: 396

TOP: Nursing Process: Assessment

MSC: NCLEX Patient Needs: Physiologic Integrity: Physiologic Adaptation: Alteration in Body

Systems

2. Which patients meet the criteria for Chlamydia screening? Select all that apply.

a. A 40-year-old woman who is sexually active and uses barrier protection

consistently

b. A 15-year-old female woman who is sexually active with one partner

c. A 22-year-old woman who is sexually active and uses barrier protection

inconsistently

d. A 23-year-old woman who has had four sexual partners in the last 3 months

e. A 34 year-old woman who uses barrier protection inconsistently with multiple

sexual partners

f. A 36-year-old pregnant woman making the first prenatal visit

ANSWER: B, C, D, E, F

Correct: These patients meet the criteria for screening for Chlamydia.

Incorrect: This patient does not meet criteria based on age and use of protection.

DIF: Cognitive Level: Analyze REF: 398

TOP: Nursing Process: Assessment

MSC: NCLEX Patient Needs: Health Promotion and Maintenance: Health Promotion Programs

3. Which questions are appropriate for a symptom analysis of a patient with benign prostatic

hyperplasia? Select all that apply.

a. “How often have you found that you stopped and started again several times when

you urinated?”


b. “How often have you had to urinate again less than 2 hours after you finished

urinating?”

c. “How often have you been incontinent of urine?”

d. “How often have you had constipation due to the enlarged prostate?”

e. “How often have you had to push or strain to begin urination?”

f. “How often have you had to get up during the night to urinate?”

ANSWER: A, B, E, F

Correct: These questions from the American Urological Association Symptom Index for

Benign Prostatic Hyperplasia are used to screen men for an enlarged prostate.

Incorrect: The urinary problem is difficulty in starting the urinary stream due to the prostate

gland compressing the urethra, thus incontinence is not a manifestation of an enlarged

prostate. Although the enlarged prostate is palpated through the rectum, it does not become

large enough to cause constipation.

DIF: Cognitive Level: Apply REF: 396-397

TOP: Nursing Process: Assessment

MSC: NCLEX Patient Needs: Physiologic Integrity: Physiologic Adaptation: Alteration in Body

Systems

4. Which patients have risk factors for endometrial cancer? Select all that apply.

a. The patient who never had children.

b. The patient who has given birth to nine children.

c. The patient whose body mass index is 39.

d. The patient whose menopause began at age 60.

e. The patient whose father had colon cancer.

f. The patient who has had human papillomavirus (HPV) infections.

ANSWER: A, C, D, E

Correct: A family history of endometrial, breast, colon, or ovarian cancer is considered a risk

factor.

Incorrect: Nulliparity or infertility is a risk factor. HPV is a risk factor for cervical cancer

rather than endometrial cancer.

DIF: Cognitive Level: Analyze REF: 424

TOP: Nursing Process: Assessment

MSC: NCLEX Patient Needs: Health Promotion and Maintenance: Health Promotion Programs


Chapter 18: Developmental Assessment Throughout the Life Span

Test Bank

MULTIPLE CHOICE

1. Which statement best illustrates Erikson’s theory of development?

a. The main goal is to establish equilibrium between self and environment.

b. One progresses through stages that involve specific psychosocial tasks.

c. There are four distinct, sequential levels of cognitive development.

d. Cognitive development occurs from birth to around age 15.

ANSWER: B

A

B

C

D

Feedback

This statement is not consistent with Erikson’s theory of development.

This statement is consistent with Erikson’s theory of development.

This is more consistent with Piaget’s theory.

This is more consistent with Piaget’s theory.

DIF: Cognitive Level: Understand REF: 439-440

TOP: Nursing Process: Assessment

MSC: NCLEX Patient Needs: Health Promotion and Maintenance: Growth and Development

2. A nurse asks a 15-year-old boy to think of an explanation for a simple puzzle. When he is

unable to come up with any answer at all, the nurse recognizes that he may not yet have

successfully mastered which of Piaget’s levels of cognitive development?

a. Sensorimotor

b. Preoperational

c. Concrete operations

d. Formal operations

ANSWER: D

Feedback

A The sensorimotor stage develops in children between the ages of birth to 2 years,

when thoughts are dominated by physical manipulation of objects and events.

B The preoperational stage develops in children between the ages of 2 to 7 years,

when children function symbolically using language as a major tool.

C The concrete operations stage develops in children between the ages of 7 to 11

years, when mental reasoning processes assume logical approaches to solving

concrete problems.

D The formal operations level develops in children between the ages of 11 and 15

years, during which true logical thoughts and manipulation of abstract concepts

emerge.

DIF: Cognitive Level: Apply REF: 440

TOP: Nursing Process: Assessment

MSC: NCLEX Patient Needs: Health Promotion and Maintenance: Growth and Development

This study source was downloaded by 100000838401522 from CourseHero.com on 03-03-2022 08:55:23 GMT -06:00

https://www.coursehero.com/file/20908977/c18/


3. When performing a physical assessment on a 7-month-old infant, the nurse notes that the child

is able to smile responsively and unable to roll from the prone to the supine position. What is

the most appropriate action for this nurse?

a. Reassure the parents that the infant is “performing like an 8-month-old.”

b. Document the infant’s growth and development as “within normal limits.”

c. Continue to assess the infant for other signs of developmental delay.

d. Give the caretaker specific directions for specialized exercises.

ANSWER: C

A

B

C

D

Feedback

This would be an incorrect statement because this infant has delayed

development.

This would be an incorrect statement because this infant has delayed

development.

An infant should be able to roll from prone to supine at 4 months.

This infant needs to be referred for additional evaluation.

DIF: Cognitive Level: Analyze REF: 441-442

TOP: Nursing Process: Assessment

MSC: NCLEX Patient Needs: Health Promotion and Maintenance: Growth and Development

4. A 7-month-old infant weighs 11.6 lb compared with a birth weight of 7.1 lb and has a head

circumference of 19 inches. What does the nurse document about this infant?

a. Underweight and normal head circumference

b. Underweight and larger-than-normal head circumference

c. Overweight and smaller-than-normal head circumference

d. Normal weight and larger-than-normal head circumference

ANSWER: B

A

B

C

D

Feedback

This 7-month-old infant should weigh more than 14 lb and have a head

circumference of 16.5 to 17.5 inches.

This 7-month-old infant should weigh more than 14 lb and have a head

circumference of 16.5 to 17.5 inches. In general, infants double their birth weight

by 4 to 5 months of age. Expected head circumference for term newborns

averages from 13 to 14 inches (33 to 36 cm) and increases 0.5 inch (1.3 cm)

monthly for the first six months.

This 7-month-old infant should weigh more than 14 lb and have a head

circumference of 16.5 to 17.5 inches.

This 7-month-old infant should weigh more than 14 lb and have a head

circumference of 16.5 to 17.5 inches.

DIF: Cognitive Level: Apply REF: 441

TOP: Nursing Process: Assessment

MSC: NCLEX Patient Needs: Health Promotion and Maintenance: Growth and Development

5. A nurse notices that an infant tries to reach for a toy that the mother has hidden in her hand.

This illustrates that the child has developed an understanding of which concept?

a. Object permanence

This study source was downloaded by 100000838401522 from CourseHero.com on 03-03-2022 08:55:23 GMT -06:00

https://www.coursehero.com/file/20908977/c18/


b. Trust versus mistrust

c. Autonomy

d. Parallel play

ANSWER: A

A

B

C

D

Feedback

Object permanence, meaning that objects and people still exist when they are out

of sight.

This is Erikson’s developmental stage for infants when they develop trust

relationships with the mother or primary caregiver. Important criteria are the

quality and consistency of the mother-child relationship.

This is Erikson’s developmental stage for toddlers who yearn for independence;

however, they lack judgment to maintain their safety.

Parallel play is a social skill of a 2-year-old toddler.

DIF: Cognitive Level: Understand REF: 443

TOP: Nursing Process: Assessment

MSC: NCLEX Patient Needs: Health Promotion and Maintenance: Growth and Development

6. The parents of a toddler express concern that the child is not progressing the same way as

their other children did at that age. What is the most appropriate suggestion the nurse can give

the parents about monitoring the progress of the toddler?

a. Advising the parents to take the toddler to the clinic every 2 months for

reevaluation

b. Teaching the parents how to use the Denver II test to assess for gross motor

movement, language, fine motor movement, and personal-social skills

c. Suggesting that the child needs more time to reach the milestones and that

additional monitoring is not necessary

d. Informing the parents about the ages and stages questionnaire (ASQ), which

identifies developmental delays in children from 4 to 60 months

ANSWER: D

A

B

C

D

Feedback

This suggestion is time-consuming, expensive, and not practical.

The Denver II test requires special equipment and training, and is designed for

health care professionals.

The suggestion for the parents to take no action is not appropriate in this case.

This is the best suggestion for these parents at this time. The ASQ is designed for

parents to report data on their children.

DIF: Cognitive Level: Apply REF: 444

TOP: Nursing Process: Assessment

MSC: NCLEX Patient Needs: Health Promotion and Maintenance: Growth and Development

7. Which characteristics are expected during an assessment of a normal toddler?

a. Half of adult height achieved by age two, potbelly, and sway back.

b. Thirty-two erupted teeth by age 2, tripled birth weight by age 30 months.

c. Desire for autonomy coupled with sufficient judgment to ensure safety.

d. Head circumference greater than chest circumference, high frustration tolerance.

This study source was downloaded by 100000838401522 from CourseHero.com on 03-03-2022 08:55:23 GMT -06:00

https://www.coursehero.com/file/20908977/c18/


ANSWER: A

A

B

C

D

Feedback

These are characteristics of toddlers.

Toddlers have 20 teeth erupted and infants triple their birth weight.

The Erikson developmental stage for toddlers is autonomy versus shame and

doubt.

The first part is true for toddlers, but they do not have a high frustration tolerance.

DIF: Cognitive Level: Understand REF: 444

TOP: Nursing Process: Assessment

MSC: NCLEX Patient Needs: Health Promotion and Maintenance: Growth and Development

8. A nurse makes observations about a toddler’s motor development. Which behavior is an

example of fine motor behavior?

a. Sitting up in a chair

b. Walking while holding on to the edge of a table

c. Creeping up the stairs

d. Stacking blocks to make a tower

ANSWER: D

A

B

C

D

Feedback

Sitting up in a chair is an example of gross motor skills.

Walking while holding on to the edge of a table is an example of gross motor

skills.

Creeping up the stairs is an example of gross motor skills.

Stacking blocks to make a tower is an example of fine motor skills with hands

and fingers.

DIF: Cognitive Level: Understand REF: 445

TOP: Nursing Process: Assessment

MSC: NCLEX Patient Needs: Health Promotion and Maintenance: Growth and Development

9. A nurse is assessing a preschooler who is able to draw a three-part human figure, hop on one

foot, and recognize three colors. The nurse recognizes these characteristics as consistent for

what age?

a. 3 years old

b. 4 years old

c. 5 years old

d. 6 years old

ANSWER: C

A

B

C

D

Feedback

These characteristics are too advanced for 3-year-old children.

These characteristics are too advanced for 4-year-old children.

These are characteristics consistent with developmental expectations of 5-yearold

children.

These characteristics are below those expected for 6-year-old children who are in

This study source was downloaded by 100000838401522 from CourseHero.com on 03-03-2022 08:55:23 GMT -06:00

https://www.coursehero.com/file/20908977/c18/


first grade.

DIF: Cognitive Level: Understand REF: 447, Table 18-5

TOP: Nursing Process: Assessment

MSC: NCLEX Patient Needs: Health Promotion and Maintenance: Growth and Development

10. A parent tells the nurse about having difficulty disciplining a 5-year-old child. What

characteristic does the nurse teach this parent to improve the discipline of this child?

a. Children at this age are incapable of delaying gratification.

b. At age 5 years, children are not interested in attaining rights and privileges of

individuality.

c. Five-year-olds should demonstrate basic social skills and respond to others’

expectations.

d. At age 5 years, children use highly inappropriate methods of expressing frustration.

ANSWER: C

A

B

C

D

Feedback

This behavior describes a toddler.

This behavior is not consistent with a preschooler.

This describes expected developmental tasks of preschoolers.

This behavior is more consistent with a toddler.

DIF: Cognitive Level: Apply REF: 445-447

TOP: Nursing Process: Assessment

MSC: NCLEX Patient Needs: Health Promotion and Maintenance: Growth and Development

11. The mother of a 7-year-old boy takes him for a checkup at the local clinic. The nurse notes

that the child has gained 4.9 lb and has grown 2.5 inches in 1 year. Based on these findings,

what is the most appropriate action of this nurse?

a. Recommend that the child be placed on a low-fat, high-protein diet.

b. Counsel the mother to increase the amount of calcium in the child’s diet.

c. Ask the mother to return with the child next week for a more comprehensive

growth and development study.

d. Inform the mother that the child’s developmental rate is within the expected ranges

for his age.

ANSWER: D

A

B

C

D

Feedback

This child’s weight is within expected ranges.

This child’s height is within expected ranges.

This child’s weight and height are within expected ranges.

For school-age children, growth continues at a slow pace, with an average 5 lb

weight gain and 2 inch height increase per year.

DIF: Cognitive Level: Apply REF: 446

TOP: Nursing Process: Assessment

MSC: NCLEX Patient Needs: Health Promotion and Maintenance: Growth and Development

This study source was downloaded by 100000838401522 from CourseHero.com on 03-03-2022 08:55:23 GMT -06:00

https://www.coursehero.com/file/20908977/c18/


12. What suggestions does the nurse make to parents to support the development of their 8-yearold

child?

a. They buy the child a computer to foster a sense of self-worth.

b. The emphasis is placed on the importance of being a success at all costs.

c. The child is rewarded for cooperation and healthy competition with peers.

d. Social relationships outside the home are limited to one or two close friends.

ANSWER: C

A

B

C

D

Feedback

This action is not needed for the development of a child at this age.

This action is not needed for the development of a child at this age.

Cooperation and healthy competition are developmental tasks for school-age

children.

This action would interfere with extending abilities to relate to others, a

developmental task of this age group.

DIF: Cognitive Level: Apply REF: 446| 448

TOP: Nursing Process: Assessment

MSC: NCLEX Patient Needs: Health Promotion and Maintenance: Growth and Development

13. The parents of a 14-year-old boy express concern that their son’s behavior ranges from cleancut

and personable to “grungy” and sullen. They have tried talking with him about this and

have tried disciplining him, but he continues to show different sides, and they are confused.

What is the nurse’s assessment for the behavior of this teenager?

a. The teenager is dangerously labile.

b. This behavior is normal experimentation.

c. This boy is being rebelliously hostile.

d. This behavior may require hospitalization.

ANSWER: B

A

B

C

D

Feedback

The behaviors described fall within expected behavior for adolescents.

Adolescents may behave in new and different ways, much to the chagrin of their

parents, as they “try on” differing roles and values. Adolescents begin testing and

evaluating previously accepted notions about life, living, spirituality, relating, and

being.

The behaviors described fall within expected behavior for adolescents.

The behaviors described fall within expected behavior for adolescents.

DIF: Cognitive Level: Apply REF: 448, Box 18-5

TOP: Nursing Process: Assessment

MSC: NCLEX Patient Needs: Health Promotion and Maintenance: Growth and Development

14. During a counseling session, which statement by an adolescent indicates he is adjusting to

expected developmental tasks?

a. “I wish my parents would just leave me alone.”

b. “I’m hoping to go to college.”

c. “I don’t have any friends.”

d. “It’s terrible being taller than all my friends.”

This study source was downloaded by 100000838401522 from CourseHero.com on 03-03-2022 08:55:23 GMT -06:00

https://www.coursehero.com/file/20908977/c18/


ANSWER: B

A

B

C

D

Feedback

This comment does not demonstrate adjustment, although it is a problem some

adolescents face.

This indicates a developmental task of adolescence.

This comment does not demonstrate adjustment.

This comment does not demonstrate adjustment, although it is a problem some

adolescents face.

DIF: Cognitive Level: Apply REF: 448-449

TOP: Nursing Process: Assessment

MSC: NCLEX Patient Needs: Health Promotion and Maintenance: Growth and Development

15. Which behavior indicates successful achievement of the major developmental tasks of a

young adulthood?

a. Accepting physical changes

b. Achieving emotional independence

c. Mastering money management

d. Getting an appropriate education

ANSWER: D

A

B

C

D

Feedback

Accepting physical changes is a developmental task of adolescence.

Achieving emotional independence is a developmental task of adolescence.

Mastering money management is a developmental task of school-age children.

Getting an appropriate education is a developmental task of young adults.

DIF: Cognitive Level: Apply REF: 448-449

TOP: Nursing Process: Assessment

MSC: NCLEX Patient Needs: Health Promotion and Maintenance: Growth and Development

16. Which statement reflects a 21-year-old woman’s achievement of an expected developmental

task?

a. “I am planning to get married next year.”

b. “I don’t plan anything without asking my boyfriend first.”

c. “I don’t know which direction I’ll take after college.”

d. “I am living with my parents and may stay for a while.”

ANSWER: A

A

B

C

D

Feedback

This statement reflects an expected developmental task of a young adult.

This statement does not reflect an expected developmental task of a young adult.

This statement does not reflect an expected developmental task of a young adult.

This statement does not reflect an expected developmental task of a young adult.

DIF: Cognitive Level: Understand REF: 448-449

TOP: Nursing Process: Assessment

MSC: NCLEX Patient Needs: Health Promotion and Maintenance: Growth and Development

This study source was downloaded by 100000838401522 from CourseHero.com on 03-03-2022 08:55:23 GMT -06:00

https://www.coursehero.com/file/20908977/c18/


17. Interviewing patients in middle adulthood, the nurse recognizes which behavior as an

expected developmental task for this age group?

a. Finding meaning in life

b. Establishing autonomy as an individual

c. Increased self-understanding

d. Dissatisfaction with one’s interpersonal relationships

ANSWER: C

A

B

C

D

Feedback

This is a developmental task of older adults.

This is a developmental task of young adults.

This is a developmental task of middle adulthood.

This behavior would not be consistent with normal adaptation.

DIF: Cognitive Level: Apply REF: 449

TOP: Nursing Process: Assessment

MSC: NCLEX Patient Needs: Health Promotion and Maintenance: Growth and Development

18. During middle adulthood, which immunization may be recommended?

a. PPV (pneumococcal pneumonia vaccine)

b. Hepatitis B virus vaccine, third dose

c. Human papillomavirus (HPV)

d. Td (tetanus and diphtheria toxoids)

ANSWER: D

Feedback

A This immunization is given to adults older than 65.

B This immunization is given between birth and age 10.

C This immunization is given between ages 11 and 24.

D This is the only immunization recommended for this age group. Tetanus and

diphtheria toxoid is recommended every 10 years.

DIF: Cognitive Level: Understand REF: 451

TOP: Nursing Process: Assessment

MSC: NCLEX Patient Needs: Health Promotion and Maintenance: Growth and Development

19. A nurse recognizes that which patient has not yet successfully completed Erikson’s final

developmental stage?

a. A 78-year-old widower who has gone to the mental health clinic for counseling

after the recent death of his wife

b. A 60-year-old man who tells the nurse that he is feeling fine and really does not

need any help from anyone

c. An 81-year-old woman who states that she enjoys having her grandchildren visit

but is usually glad when it is time for them to go home

d. A 75-year-old woman who tells the nurse that she wishes her friends were alive and

she wishes she could change the choices she made over the years

ANSWER: D

This study source was downloaded by 100000838401522 from CourseHero.com on 03-03-2022 08:55:23 GMT -06:00

https://www.coursehero.com/file/20908977/c18/


A

B

C

D

Feedback

Dealing with the death of a spouse is a developmental task of this age group, and

this patient is seeking assistance to help him cope.

This man is meeting the developmental tasks for his age.

Enjoying grandchildren is a developmental task of this age group.

Erikson’s final stage is ego integrity versus despair, and this woman is voicing

despair and regret about previous decisions made in her life.

DIF: Cognitive Level: Analyze REF: 451-452

TOP: Nursing Process: Assessment

MSC: NCLEX Patient Needs: Health Promotion and Maintenance: Growth and Development

20. Which behavior illustrates a developmental task for a “young-old” older adult?

a. Adapting to living alone

b. Adjusting to loss of physical strength, illness, and emotional stress

c. Managing leisure time

d. Accepting possible institutional living arrangements

ANSWER: C

A

B

C

D

Feedback

This is a developmental task of those 85 years and older.

This is a developmental task of those older than 85 years of age.

This is a developmental task of the “young-old” adult (65 to 85 years of age).

This is a developmental task of those older than 85 years of age.

DIF: Cognitive Level: Understand REF: 452

TOP: Nursing Process: Assessment

MSC: NCLEX Patient Needs: Health Promotion and Maintenance: Growth and Development

This study source was downloaded by 100000838401522 from CourseHero.com on 03-03-2022 08:55:23 GMT -06:00

https://www.coursehero.com/file/20908977/c18/


Chapter 19: Assessment of the Infant, Child, and Adolescent

Test Bank

MULTIPLE CHOICE

1. An adolescent patient appears reluctant to discuss sensitive issues with her parents present.

What is the nurse’s most appropriate intervention?

a. Tell the patient that it is very important to be honest and specific.

b. Provide time when the adolescent is alone with the nurse.

c. Reassure the patient that anything said in the interview is considered confidential.

d. Ask the parents to answer the questions if the patient is not willing to answer.

ANSWER: B

A

B

C

D

Feedback

Although this statement is true, the adolescent should have time alone with the

nurse, if needed, to answer or ask personal questions.

As children reach adolescence, they should be given the option to provide

sensitive parts of the history without their parents present.

Although this statement is true, the adolescent should have time alone with the

nurse, if needed, to answer or ask personal questions.

This intervention is not appropriate when the patient is present and able to answer

questions. In addition, the parents may not know the information needed by the

nurse about the adolescent.

DIF: Cognitive Level: Apply REF: 456

TOP: Nursing Process: Assessment

MSC: NCLEX Patient Needs: Psychosocial Integrity: Therapeutic Communication

2. What does the nurse teach to parents to prevent sudden infant death syndrome (SIDS)?

a. Place the baby on back to sleep.

b. Place the baby on side to sleep.

c. Not to feed the baby for 3 hours before sleep.

d. Place the baby on her stomach to sleep.

ANSWER: A

A

B

C

D

Feedback

The American Academy of Pediatrics recommends positioning infants on their

backs; the slogan to help people remember is “Back to Sleep.”

The side-lying position is not recommended for sleep because of the risk of

aspiration.

Not feeding the baby for 3 hours before sleep is not a prevention for SIDS.

The prone position is not recommended for sleep due to the risk of aspiration.

DIF: Cognitive Level: Apply REF: 458, Box 19-3

TOP: Nursing Process: Assessment

MSC: NCLEX Patient Needs: Health Promotion: Lifestyle Choices


3. In taking a history from an adolescent girl about diet and nutrition, a nurse specifically asks

which question?

a. “How frequently do you eat fast food or junk food?”

b. “Which carbonated drinks do you drink most often?”

c. “Do you have any food restrictions or diet routines?”

d. “What are your favorite fruits and vegetables?”

ANSWER: C

A

B

C

D

Feedback

Asking the frequency of fast food or junk food consumption does not give data

about what food is eaten.

Knowing the amount of carbonated drinks provides more useful data.

Adolescents should be asked specifically about their perception of their current

weight and behaviors associated with eating disorders, including food restrictions,

extreme diet/exercise routines, binging or purging, and the use of laxatives to

screen for eating disorders.

Knowing how frequently these foods are eaten provides more useful data.

DIF: Cognitive Level: Understand REF: 458

TOP: Nursing Process: Assessment

MSC: NCLEX Patient Needs: Health Promotion and Maintenance: Health and Wellness

4. A nurse is assessing a child who is able to dress herself, jump rope, identify colors, and follow

rules when playing games. These are expected developmental achievements of a child of what

age?

a. 3 years old

b. 4 years old

c. 5 years old

d. 6 years old

ANSWER: C

A

B

C

D

Feedback

These are developmental behaviors too advanced for a 3-year-old child.

These are developmental behaviors too advanced for a 4-year-old child.

These are developmental behaviors consistent with a 5-year-old child.

These developmental behaviors are typically achieved and surpassed by a 6-yearold

child.

DIF: Cognitive Level: Understand REF: 459

TOP: Nursing Process: Assessment

MSC: NCLEX Patient Needs: Health Promotion and Maintenance: Growth and Development

5. A 4-year-old child has had a tonsillectomy and the nurse is preparing to ask him about his

pain. Which technique is the most appropriate method for pain assessment for this patient?

a. Asking him if the pain hurts “a little or a lot”

b. Asking him to rate the pain on a scale of 0 to 10

c. Using the visual analog scale to rate the pain

d. Using the Wong/Baker FACES rating scale


ANSWER: D

A

B

C

D

Feedback

Using adjectives such as these is not reliable to assess pain in patients of any age.

This scale is appropriate for adolescents and adults, but a child cannot understand

the concept of using numbers to rate pain.

This type of scale is appropriate for adults, but a child cannot understand the

concept of using a straight line to rate pain.

This tool is appropriate for children who can point to the child’s face that best

represents how they are feeling.

DIF: Cognitive Level: Apply REF: 459| 461-462

TOP: Nursing Process: Assessment

MSC: NCLEX Patient Needs: Physiologic Integrity: Physiologic Adaptation: Alteration in Body

Systems

6. Which assessment technique is appropriate to measure the 8-month-old’s vital signs during a

well-baby check?

a. Assess temperature using a rectal thermometer.

b. Observe the infant’s abdomen when counting respirations.

c. Take the infant from the parent’s arms to assess pulse.

d. Measure blood pressure in the leg.

ANSWER: B

A

B

C

D

Feedback

Rectal temperatures should be taken as a last resort because children tend to fear

intrusive procedures and because of the risk for rectal perforation. The

recommended sites for temperature measurement in newborns, infants, and

children to age 5 are the axillary or tympanic sites.

Infants usually breathe diaphragmatically, which requires observation of

abdominal movement.

For the older infant ( 6 months) and toddler, the nurse may find that having the

caregiver hold the baby or toddler decreases fear and distress, thus making it

easier for the nurse to conduct the examination.

This infant is too young for blood pressure measurement. The National High

Blood Pressure Education Program recommends that blood pressure be measured

in children from age 3 through adolescence as part of routine health care visits.

DIF: Cognitive Level: Apply REF: 463

TOP: Nursing Process: Assessment

MSC: NCLEX Patient Needs: Health Promotion and Maintenance: Techniques of Physical

Assessment

7. An American Indian mother expresses concern about an irregularly shaped, dark area over her

neonate’s sacrum and buttocks. What is the nurse’s most appropriate response to this mother?

a. “This area will continue to grow until the infant is 10 to 15 months old.”

b. “This is a birth mark, which usually disappears by age 5 years.”

c. “This skin abnormality will require follow-up care.”

d. “This is a birth mark and they usually disappear by age 1 or 2 years.”


ANSWER: D

A

B

C

D

Feedback

This description refers to cavernous hemangioma that requires frequent

reassessment.

This description refers to a “stork bite” (telangiectasis).

This is an inappropriate response.

This description refers to a Mongolian spot.

DIF: Cognitive Level: Apply REF: 466

TOP: Nursing Process: Assessment

MSC: NCLEX Patient Needs: Physiologic Integrity: Reduction of Risk Potential: System Specific

Assessments

8. How does a nurse document a large, flat bluish capillary area on a neonate’s cheek?

a. Mongolian spot

b. Stork bite (telangiectasis)

c. Port-wine stain (nevus flammeus)

d. Strawberry hemangioma

ANSWER: C

A

B

C

D

Feedback

A Mongolian spot appears as an irregularly shaped, darkened, flat area over the

sacrum and buttocks.

A stork bite (telangiectasis) is a common vascular birthmark that appears as a

small red or pink spot often seen on the back of the neck.

Port-wine stains appear as large, flat, bluish purple capillary areas. They are

frequently found on the face along the distribution of the fifth cranial nerve

(trigeminal).

A strawberry hemangioma appears as a slightly raised, reddened area with a sharp

demarcation line that may be 2 to 3 cm in diameter.

DIF: Cognitive Level: Apply REF: 466

TOP: Nursing Process: Assessment

MSC: NCLEX Patient Needs: Physiologic Integrity: Reduction of Risk Potential: System Specific

Assessments

9. How does a nurse collect baseline measurements of a 6-month-old infant?

a. Measure the chest circumference around the lower ribs.

b. Ask the parent how much the infant’s weight has changed since birth.

c. Measure the head just above the ears and eyebrows.

d. Ask the parent to hold the infant while the nurse measures the length.

ANSWER: C

A

B

Feedback

The nurse measures chest circumference, but the tape measure is placed around

the nipples rather than the ribs.

An infant platform scale covered with a paper drape is used for weighing

newborns, infants, and small children.


C

D

The nurse measures head circumference using this procedure until about 2 years

old.

An infant’s height is measured while the infant is lying supine.

DIF: Cognitive Level: Apply REF: 467-468

TOP: Nursing Process: Assessment

MSC: NCLEX Patient Needs: Health Promotion and Maintenance: Techniques of Physical

Assessment

10. How does a nurse assess the head circumference of an infant?

a. Places a ruler behind the infant’s head, noting the head width.

b. Uses a plastic headband placed around the infant’s head from crown to chin.

c. Places a measuring tape around the head above the eyebrows and occipital

prominence.

d. Uses a measuring tape to find the distance between the ears and eyes and between

the eyes and occiput.

ANSWER: C

A

B

C

D

Feedback

This is an incorrect technique.

This is an incorrect technique.

This is the correct technique for measuring head circumference.

This is an incorrect technique.

DIF: Cognitive Level: Understand REF: 467

TOP: Nursing Process: Assessment

MSC: NCLEX Patient Needs: Health Promotion and Maintenance: Techniques of Physical

Assessment

11. During a well-baby check for several 4-month-old infants, a nurse recognizes that which

infant needs further assessment of an abnormal finding?

a. The infant who is unable to sit independently

b. The infant whose head circumference and chest circumference are equal

c. The infant whose weight has doubled since birth

d. The infant whose length falls in the 90th percentile on growth charts

ANSWER: B

Feedback

A This is not an expected motor skill for a 4-month-old; it is expected at 6 months

of age.

B At four months of age, the head circumference should be larger than the chest

circumference.

C This is a normal finding; infants generally double their birth weight by age 4 to 5

months.

D This is not an abnormal finding, especially if weight is normal; the height of the

parents should also be considered.

DIF: Cognitive Level: Analyze REF: 467-468

TOP: Nursing Process: Assessment


MSC: NCLEX Patient Needs: Health Promotion and Maintenance: Growth and Development

12. Which finding indicates to a nurse that a neonate has a cephalhematoma?

a. Well-defined edematous area over one cranial bone

b. Molding of the cranium that causes generalized cerebral edema

c. Diffuse edema over two or more cranial bones

d. Anterior fontanelle that is deeply depressed

ANSWER: A

A

B

C

D

Feedback

This is a subperiosteal hematoma under the scalp that occurs secondary to birth

trauma. The area, which appears as a soft, well-defined swelling over the cranial

bone, generally is reabsorbed within the first month of life.

Molding is secondary to the head passing through the birth canal and generally

lasts less than a week.

Cephalhematoma occurs over one cranial bone rather than several, and is welldefined

rather than diffuse.

Anterior fontanelles are soft but not depressed.

DIF: Cognitive Level: Understand REF: 468

TOP: Nursing Process: Assessment

MSC: NCLEX Patient Needs: Physiologic Integrity: Physiologic Adaptation: Alteration in Body

Systems

13. During assessment of an infant, the nurse notes that when the infant cries, the fontanelles

bulge slightly. What is the most appropriate action for the nurse at this time?

a. Note in the record that the child is microcephalic.

b. Assess the fontanelles again when the child is not crying.

c. Check the child for signs of malnutrition and dehydration.

d. Use transillumination for further assessment of the skull.

ANSWER: B

A

B

C

D

Feedback

Microcephaly occurs when the head circumference is less than expected, which is

not the finding here.

The fontanelles frequently bulge when an infant is crying. The nurse will palpate

the anterior and posterior fontanelles for fullness while the infant is in an upright

position and calm.

An elevated fontanelle when the infant is calm may indicate overhydration or

fluid volume excess.

Transillumination is not needed in this case.

DIF: Cognitive Level: Apply REF: 468

TOP: Nursing Process: Assessment

MSC: NCLEX Patient Needs: Health Promotion and Maintenance: Growth and Development

14. A mother who sees her newborn just after vaginal delivery is distraught because the child’s

head is elongated. Which response is most appropriate by the nurse?

a. “This is due to a small area of bleeding that will go away in 1 to 2 months.”


b. “This may indicate a congenital deformity; the pediatrician will evaluate this.”

c. “This will require surgery to prevent hydrocephalus from developing.”

d. “This is not unusual after a vaginal delivery and will go away in about a week.”

ANSWER: D

A

B

C

D

Feedback

This response refers to cephalohematoma, which is not described here.

The newborn has molding that will resolve in a week.

The newborn has molding that will resolve in a week.

This is molding, which occurs when cranial bones override each other. Molding is

secondary to the head passing through the birth canal and generally lasts less than

a week.

DIF: Cognitive Level: Apply REF: 468

TOP: Nursing Process: Assessment

MSC: NCLEX Patient Needs: Physiologic Integrity: Reduction of Risk Potential: System Specific

Assessments

15. A nurse shines the light from the ophthalmoscope into the eyes of a newborn and observes a

bright, round, red-orange glow seen through both pupils. How does the nurse document this

finding?

a. An expected red reflex

b. Eyelid capillary hemangiomas

c. Bilateral conjunctivitis

d. Ophthalmia neonatorum

ANSWER: A

A

B

C

D

Feedback

The red reflex appears as a bright, round, red-orange glow seen through both

pupils.

Eyelid capillary hemangiomas are abnormalities that appear on the eyelid, but

disappear spontaneously.

Redness, lesions, nodules, discharge, or crusting of conjunctiva indicate

conjunctivitis.

Ophthalmia neonatorum is an eye disorder that produces purulent conjunctivitis

and keratitis.

DIF: Cognitive Level: Apply REF: 468

TOP: Nursing Process: Assessment

MSC: NCLEX Patient Needs: Physiologic Integrity: Reduction of Risk Potential: System Specific

Assessments

16. What finding does a nurse expect when assessing a one-month old’s eyes and vision?

a. The newborn distinguishes most colors

b. Tears when the newborn cries

c. The newborn following a bright toy or light

d. The newborn’s blink reflex is present

ANSWER: C


A

B

C

D

Feedback

Testing for color vision occurs between ages 4 and 8 years.

There are no tears until about 2 to 3 months of age.

This is an accurate statement.

The blink reflex is present in normal newborns and infants before one month.

DIF: Cognitive Level: Understand REF: 469

TOP: Nursing Process: Assessment

MSC: NCLEX Patient Needs: Health Promotion and Maintenance: Growth and Development

17. Which finding rules out defects in the cornea, lens, and vitreous chamber of an infant?

a. Bilateral red reflex

b. Symmetric corneal light reflex

c. Bilateral blink reflex

d. Symmetric eye movements

ANSWER: A

A

B

C

D

Feedback

Presence of the red reflex eliminates the presence of most serious defects of the

cornea, aqueous chamber, lens, and vitreous chamber.

Symmetric corneal light reflex tests for symmetric eye muscle function and

eliminates the presence of strabismus.

Bilateral blink reflex tests the function of cranial nerve V (trigeminal).

Symmetry of extraocular muscles tests cranial nerves III (oculomotor), IV

(trochlear), and VI (abducens).

DIF: Cognitive Level: Understand REF: 468-469

TOP: Nursing Process: Assessment

MSC: NCLEX Patient Needs: Physiologic Integrity: Reduction of Risk Potential: System Specific

Assessments

18. In assessing the eyes of a 4-month-old infant, a nurse shines a penlight in the infant’s eyes and

notices that the light reflection is not in the same location in each eye. What is the nurse’s

most appropriate response to this finding?

a. Perform the cover-uncover test.

b. Document it as an expected finding at this age.

c. Document abnormal function of cranial nerves IV (trochlear) and VI (abducens).

d. Refer the infant to an ophthalmologist.

ANSWER: B

A

B

C

D

Feedback

Performing the cover-uncover test is not indicated unless there is asymmetric

light reflex after 6 months of age.

Transient strabismus is common during the first few months of life due to a lack

of binocular vision. If it continues after 6 months of age; however, a referral to an

ophthalmologist is needed.

The data do not support this interpretation.

Referral is not needed unless the finding is present after the infant is 6 months


old.

DIF: Cognitive Level: Apply REF: 469

TOP: Nursing Process: Assessment

MSC: NCLEX Patient Needs: Physiologic Integrity: Reduction of Risk Potential: System Specific

Assessments

19. What technique does a nurse use to inspect the ear canal of a 1-year-old child?

a. Uses a light source without a speculum to minimize any trauma to the ear canal

b. Places the child in an upright position with the head flexed slightly downward

c. Applies gentle traction to the lower portion of the ear and pulls upward and

laterally

d. Uses an assistant to hold the child’s arms down and keep the child’s head turned to

one side

ANSWER: D

A

B

C

D

Feedback

A nurse uses the largest speculum that fits comfortably into the child’s ear canal.

Because the nurse must have both hands free to hold the ear and maneuver the

otoscope, another individual must act as a “holder.” The child’s head is turned to

one side.

The correct technique is to grasp the lower portion of the pinna and apply gentle

traction down and slightly backward (as opposed to pulling the pinna up and back

for the adult). This maneuver straightens the canal of the ear.

Instruct the holder to secure the infant’s arms at the sides with one hand, and turn

and hold the infant’s head to one side with the other hand.

DIF: Cognitive Level: Apply REF: 469

TOP: Nursing Process: Assessment

MSC: NCLEX Patient Needs: Health Promotion and Maintenance: Techniques of Physical

Assessment

20. In inspecting the eyes and ears of an infant, the nurse documents which finding as normal?

a. The external ear is in direct line with the outer margin of the eyelid.

b. The ear lobe is within 10° of alignment with the outer margin of the eyelid.

c. A lateral upward slant of the eyes aligns them with the helix of the ear.

d. The inner margin of the eye is directly aligned with the helix of the ear.

ANSWER: A

A

B

C

D

Feedback

This is the expected alignment of the ears and eyes of an infant.

This is not an expected finding.

This finding occurs in infants with Down syndrome.

This is not an expected finding.

DIF: Cognitive Level: Understand REF: 470

TOP: Nursing Process: Assessment

MSC: NCLEX Patient Needs: Physiologic Integrity: Reduction of Risk Potential: System Specific

Assessments


21. Which behavior would be most indicative of hearing impairment in a 1-year-old child?

a. Failure to respond to mother’s voice

b. Crying when a loud noise occurs unexpectedly

c. Saying only single-syllable words

d. Disinterest in playing with musical toys

ANSWER: A

A

B

C

D

Feedback

By age 4 to 6 months, an infant should turn the head toward the source of the

sound and should respond to the parent’s voice.

This behavior indicates an ability to hear loud noises.

This is an expected finding for a child this age.

Although an infant by age 4 to 6 months should respond to music toys, a

disinterest does not indicate a hearing problem.

DIF: Cognitive Level: Understand REF: 470

TOP: Nursing Process: Assessment

MSC: NCLEX Patient Needs: Physiologic Integrity: Physiologic Adaptation: Alteration in Body

Systems

22. Which finding during inspection of the mouth of a 1-month-old infant requires further

investigation?

a. A small loose tooth in the lower jaw

b. Tongue overlapping the floor of the mouth

c. Whitish epithelial cells on the roof of the mouth

d. White patches on the tongue that scrape off easily

ANSWER: B

A

B

C

D

Feedback

This is not an abnormal finding; occasionally, a natal loose tooth may be found.

The infant’s tongue should be appropriate to the size of the mouth and fit well

into the floor of the mouth.

These small, white epithelial cells on the palate or gums are called Bohn nodules

or Epstein pearls and are an expected finding.

Whitish patches seen along the mucosa that scrape off easily are milk deposits.

White patches that scrape off but leave a red area that may bleed indicate

candidiasis lesions.

DIF: Cognitive Level: Apply REF: 471

TOP: Nursing Process: Assessment

MSC: NCLEX Patient Needs: Physiologic Integrity: Physiologic Adaptation: Alteration in Body

Systems

23. In performing a respiratory assessment of a 1-month-old infant, the nurse recognizes which

finding as abnormal?

a. Sneezing

b. Coughing

c. Abdominal breathing


d. Predominantly nose breathing

ANSWER: B

A

B

C

D

Feedback

Sneezing is a common finding for an infant and is therapeutic because it helps to

clear the nose.

Coughing at this age is considered abnormal and indicates a problem.

Infants use abdominal breathing rather than diaphragmatic breathing.

Infants are obligate nose breathers until about 3 months old.

DIF: Cognitive Level: Understand REF: 472

TOP: Nursing Process: Assessment

MSC: NCLEX Patient Needs: Physiologic Integrity: Physiologic Adaptation: Alteration in Body

Systems

24. The nurse suspects respiratory distress in a newborn infant who exhibits which manifestation?

a. Respiratory rate of 36 breaths/min

b. Sternal retractions

c. Nasal breathing

d. Irregular breathing pattern

ANSWER: B

A

B

C

D

Feedback

Respiratory rate of 36 per minute in the newborn and infant is within the expected

range of 30 to 60 breaths/min.

Several respiratory findings indicate that an infant is in respiratory distress,

including stridor, grunting, sternal or supraclavicular retractions, and nasal

flaring.

Infants are obligate nose breathers until about 3 months old.

The respiratory pattern in the newborn may be irregular, having a Cheyne-Stokes

type of pattern.

DIF: Cognitive Level: Understand REF: 471-472

TOP: Nursing Process: Assessment

MSC: NCLEX Patient Needs: Physiologic Integrity: Physiologic Adaptation: Alteration in Body

Systems

25. When assessing an infant, the nurse recognizes which finding requires immediate attention?

a. Cheyne-Stokes type of respiratory pattern

b. 1:1 anteroposterior to lateral chest diameter

c. Stridor and nasal flaring

d. Bronchovesicular lung sounds in the periphery

ANSWER: C

A

B

Feedback

The respiratory pattern in the newborn may be irregular, having a Cheyne-Stokes

type of pattern.

Unlike the adult, the infant has a round thorax with an equal anteroposterior and

lateral diameter.


C

D

Stridor and nasal flaring warrants immediate medical attention. Stridor is a highpitched,

piercing sound that is primarily heard in a distressed infant during

inspiration.

The predominant breath sound heard in the peripheral lung fields of infants is

bronchovesicular.

DIF: Cognitive Level: Analyze REF: 472

TOP: Nursing Process: Assessment

MSC: NCLEX Patient Needs: Physiologic Integrity: Physiologic Adaptation: Alteration in Body

Systems

26. Which statement by a mother makes the nurse assess the infant girl for cardiovascular

problems?

a. “She has gained 2 lb since our last visit.”

b. “She naps twice a day for almost 2 hours each time.”

c. “She gets so tired and out of breath when she takes her bottle.”

d. “She gets fussy after I feed her and seems to have lots of gas.”

ANSWER: C

A

B

C

D

Feedback

This is an expected weight gain for an infant.

This is an expected sleep pattern for an infant.

Heart problems are indicated when caregivers report the child stops eating to

catch her breath.

This finding may be related to the food eaten, rather than a cardiovascular

problem.

DIF: Cognitive Level: Analyze REF: 472

TOP: Nursing Process: Assessment

MSC: NCLEX Patient Needs: Physiologic Integrity: Physiologic Adaptation: Alteration in Body

Systems

27. During the assessment of a newborn within hours after birth, a nurse determines which finding

as abnormal?

a. Capillary refill time of less than 1 second

b. Apical pulse felt at the second intercostal space

c. Splitting of heart sounds

d. Cyanosis of the hands and feet

ANSWER: B

A

B

C

D

Feedback

This is a normal finding; capillary refill in infants is rapid—less than 1 second.

This finding is abnormal because the apical pulse of the newborn normally is felt

in the fourth or fifth intercostal space.

Splitting of heart sounds is common in infants until about 48 hours after birth

because of the transition from fetal circulation to systemic and pulmonic

circulation.

Acrocyanosis (cyanosis of hands and feet) without central cyanosis is of little

concern and usually disappears within hours to days after birth.


DIF: Cognitive Level: Analyze REF: 472

TOP: Nursing Process: Assessment

MSC: NCLEX Patient Needs: Physiologic Integrity: Physiologic Adaptation: Alteration in Body

Systems

28. On assessment of an infant’s abdomen, the nurse notes which finding as normal?

a. Easily palpable spleen

b. Flat to slightly concave abdominal contour

c. Lower liver border 2 inches below the costal margin

d. Small protrusion between the rectus muscles when crying

ANSWER: D

A

B

C

D

Feedback

The spleen is generally not palpable, although the tip may be felt in the left upper

quadrant (far left costal margin).

Inspecting the abdomen of a healthy infant finds a symmetric, soft, and round

abdomen with a slight protrusion.

The edge of the infant’s liver should be 1 to 2 cm below the right rib cage (costal

margin).

Diastasis swelling and a gap between the rectus muscles may be noted as an

expect finding during crying.

DIF: Cognitive Level: Understand REF: 472

TOP: Nursing Process: Assessment

MSC: NCLEX Patient Needs: Physiologic Integrity: Reduction of Risk Potential: System Specific

Assessments

29. The nurse documents which finding as normal after performing the Barlow-Ortolani

maneuver on an infant?

a. The clavicles are immobile and without crepitus.

b. Each shoulder remains in a “hunched up” position.

c. No clicking is noted when the hips are abducted and adducted.

d. Both feet are held in the varus position when stroked on the soles.

ANSWER: C

A

B

C

D

Feedback

Normal findings include stable and smooth clavicles, without crepitus.

The Barlow-Ortolani maneuver assesses hip location.

The Barlow-Ortolani maneuver assesses hip location, and the movement should

feel smooth and produce no clicking.

The feet should be flexible and not fixed. Normally, the hindfoot aligns with the

lower leg and the forefoot turns inward slightly.

DIF: Cognitive Level: Understand REF: 473

TOP: Nursing Process: Assessment

MSC: NCLEX Patient Needs: Physiologic Integrity: Reduction of Risk Potential: System Specific

Assessments


30. Which technique does a nurse use to assess hip location of a newborn?

a. With newborn’s knees flexed, the nurse adducts the legs, then abducts them,

moving the knees apart and down to touch the table.

b. With the newborn supine, the nurse flexes and extends the hips, and then passively

moves each leg through internal and external rotation.

c. The nurse holds the newborn in a vertical position with the feet flat on the table and

palpates each hip for location.

d. With the newborn supine, the nurse measures the length of each leg from the

trochanter to the lateral malleolus (ankle).

ANSWER: A

A

B

C

D

Feedback

This describes the Barlow-Ortolani maneuver to assess hip location and

determine dislocation.

This describes an incorrect technique.

This describes an incorrect technique.

This describes an incorrect technique.

DIF: Cognitive Level: Apply REF: 473

TOP: Nursing Process: Assessment

MSC: NCLEX Patient Needs: Health Promotion and Maintenance: Techniques of Physical

Assessment

31. A nurse assessing a 3-month-old infant suspects hydrocephalus based on which finding?

a. Soft anterior fontanelle

b. Lack of head control while sitting

c. Increasing head circumference

d. Marked asymmetry of the head

ANSWER: C

A

B

C

D

Feedback

This is an expected finding. Bulging fontanelle is a manifestation of

hydrocephalus.

By 4 months old, most infants demonstrate head control by holding the head erect

and midline when in an upright position.

The abnormal accumulation of cerebrospinal fluid increases the head

circumference because the sutures of the skull are not ossified and the anterior

fontanelle is not closed.

Marked asymmetry of the head may indicate craniosynostosis, a premature

ossification of one or more of the cranial sutures.

DIF: Cognitive Level: Understand REF: 475

TOP: Nursing Process: Assessment

MSC: NCLEX Patient Needs: Physiologic Integrity: Physiologic Adaptation: Alteration in Body

Systems

32. To assess the reflexes of a 5-month-old infant lying supine, the nurse turns the infant’s head to

the left side so that the chin is over the shoulder. What is the expected response for this reflex?

a. Left arm and leg abduct and the right arm and leg adduct.


b. Left arm and leg extend and the right arm and leg flex.

c. Infant turns the chin from the left to the right side.

d. Infant begins a sucking motion with the lips and tongue.

ANSWER: B

Feedback

A The arms and legs flex and extend, rather than abduct and adduct.

B This is the expected response for the tonic neck reflex that appears from birth to 6

weeks old and disappears at 4 to 6 months old.

C This is not an expected finding for any infant reflexes.

D This is a description of the sucking reflex.

DIF: Cognitive Level: Apply REF: 474

TOP: Nursing Process: Assessment

MSC: NCLEX Patient Needs: Health Promotion and Maintenance: Growth and Development

33. A nurse tests a reflex on a 9-month-old infant’s right foot by stroking the surface of the

infant’s foot, moving from the sole laterally up and across to the great toe. What is the

expected response?

a. Flexion of the right toes

b. Extension of the right ankle

c. Dorsiflexion of the right foot

d. Fanning of the toes of the right foot

ANSWER: D

A

B

C

D

Feedback

This is not an expected response from any reflex.

This is not an expected response from any reflex.

This is part of the clonus reflex test.

This is the expected response for the Babinski reflex.

DIF: Cognitive Level: Apply REF: 474-475

TOP: Nursing Process: Assessment

MSC: NCLEX Patient Needs: Health Promotion and Maintenance: Growth and Development

34. In assessing a neonate, the nurse notices that one testicle has not descended into the scrotal

sac. What is the most appropriate response for the nurse as a result of this finding?

a. Document the findings and refer this neonate for further examination for an

undescended testicle.

b. Place a finger over the upper inguinal ring and gently push downward to try to push

the testicle into the scrotum.

c. Use a light source to transilluminate the affected scrotal sack to determine if fluid is

preventing the descent of the testicle.

d. Insert the fifth finger into the inguinal ring to palpate for a hernia that may have

prevented the testicle from descending.

ANSWER: B

Feedback


A

B

C

D

This response is premature without determining if the testicle can be manually

descended into the scrotum.

If the testicle can be pushed into the scrotum, it is considered descended even

though it retracts into the inguinal canal.

Transillumination is used to detect a hydrocele, which is a common finding in

infants but not related to an undescended testicle.

This response is not appropriate.

DIF: Cognitive Level: Analyze REF: 476

TOP: Nursing Process: Assessment

MSC: NCLEX Patient Needs: Physiologic Integrity: Reduction of Risk Potential: System Specific

Assessments

35. After obtaining a history from the parents and inspecting the skin, the nurse determines which

child needs further evaluation?

a. The child who has a 1-cm red spot on the back of the neck, a fever of 100° F, and

clear nasal drainage.

b. The child who has a 2-cm slightly-raised, reddened area with a sharp demarcation

line on the back of the neck.

c. The child has a 2-cm abrasion on the right knee, a 3-cm abrasion on the left knee,

and scrapes on both palms.

d. The child who has several flat, bluish discolorations of the skin on the abdomen

and back from 2 to 6 cm.

ANSWER: D

A

B

C

D

Feedback

This child has a stork bite birthmark on the back of the neck and an upper

respiratory allergy or viral infection.

This is a strawberry hemangioma, a birthmark that disappears by 5 years of age.

This child probably fell down while running, skinned both knees, and tried to

break the fall with the hands.

Bruising in unusual areas (such as upper arms, back, buttocks, and abdomen) or

multiple bruises found at different stages of healing should be further investigated

to determine if there is abuse.

DIF: Cognitive Level: Analyze REF: 477

TOP: Nursing Process: Assessment

MSC: NCLEX Patient Needs: Physiologic Integrity: Physiologic Adaptation: Alteration in Body

Systems

36. Which tool is most appropriate for testing the vision of a 5-year-old child?

a. Denver II test

b. Snellen E chart

c. Allen picture cards

d. Snellen standard chart

ANSWER: B

A

Feedback

Vision can be assessed when performing developmental tests such as the Denver


B

C

D

II, but the Snellen E provides more objective data.

Use the Snellen E chart for children 3 to 6 years of age (see Chapter 11). Have

children point their fingers in the direction of the “arms” of the E.

Use the Allen picture cards to screen for visual acuity in 2.5- to 3-year-old

children.

The Snellen standard chart is used for adolescents and adults; it is too difficult to

use for children.

DIF: Cognitive Level: Understand REF: 478

TOP: Nursing Process: Assessment

MSC: NCLEX Patient Needs: Health Promotion and Maintenance: Techniques of Physical

Assessment

37. When examining lymph nodes in a 7-year-old child, the nurse records which finding as

abnormal?

a. “Shotty” nodes in the cervical areas

b. Palpable submandibular nodes

c. Nodes that are tender 1 week after a tetanus vaccination

d. Tender, fixed nodes greater than 1 cm

ANSWER: D

A

B

C

D

Feedback

The term “shotty” may be used to describe small, firm, and mobile nodes, usually

occurring as a normal variation in children.

Cervical and submandibular nodal enlargements are frequent in older children.

Enlarged, tender nodes may occur after immunizations or upper respiratory

infection.

Abnormal findings are tender, fixed nodes greater than 1 cm.

DIF: Cognitive Level: Analyze REF: 479-480

TOP: Nursing Process: Assessment

MSC: NCLEX Patient Needs: Physiologic Integrity: Physiologic Adaptation: Alteration in Body

Systems

38. After assessment of each child, the nurse determines which child needs to be referred for

further evaluation?

a. A 4-year-old child with a predominantly nasal breathing pattern

b. A 6-year-old child with a 1:2 anteroposterior-to-transverse-chest ratio

c. A 7-year-old child with a predominantly thoracic breathing pattern

d. A 9-year-old child with bronchovesicular breath sounds in peripheral lungs

ANSWER: D

A

B

C

Feedback

At this age, children have a nasal and abdominal breathing pattern and do not

develop a thoracic breathing pattern until 6 or 7 years of age.

By age 5 or 6, the rounded thorax of the child approximates the 1:2 ratio of

anteroposterior to lateral diameter of the adult.

By age 6 or 7, the child’s breathing pattern should change from primarily nasal

and abdominal to thoracic in girls and abdominal in boys.


D

Although bronchovesicular breath sounds in peripheral lung areas are expected in

a young child, vesicular sounds are expected at this age.

DIF: Cognitive Level: Analyze REF: 480

TOP: Nursing Process: Assessment

MSC: NCLEX Patient Needs: Physiologic Integrity: Physiologic Adaptation: Alteration in Body

Systems

39. A nurse refers which child for further assessment?

a. A 2-year-old who has a jugular venous hum after playing

b. A 4-year-old who has a resting heart rate of 100

c. A 5-year-old who positions herself in a squat after running a few feet

d. A 7-year-old who has a strong femoral pulse readily detected on palpation

ANSWER: C

A

B

C

D

Feedback

An expected finding in children is a venous hum in the jugular vein.

This is an expected resting heart rate for a child; the expected range for a toddler

is from 80 to 110 and for a school-age child from 60 to 110 beats/min.

Squatting may be a compensatory position for a child with a heart defect.

This is an expected finding.

DIF: Cognitive Level: Analyze REF: 481

TOP: Nursing Process: Assessment

MSC: NCLEX Patient Needs: Physiologic Integrity: Physiologic Adaptation: Alteration in Body

Systems

40. How does a nurse respond to parents of a 5-year-old who are worried that their child has a

protruding abdomen?

a. Assesses the child to differentiate a normal “potbelly” from a hernia

b. Suggests that the parents administer an appropriate dose of a laxative at bedtime

c. Refers the parents to a nutritionist to develop an appropriate weight-loss diet for the

child

d. Informs the parents that a protruding abdomen is always an abnormal finding in

this age group

ANSWER: A

A

B

C

D

Feedback

Toddlers normally exhibit a rounded (potbelly) abdomen while both standing and

lying down.

The nurse needs to collect more data before making this recommendation.

The nurse needs to collect more data before making this recommendation.

This is an inappropriate statement based on the description of this child.

DIF: Cognitive Level: Apply REF: 481

TOP: Nursing Process: Assessment

MSC: NCLEX Patient Needs: Health Promotion and Maintenance: Growth and Development


41. During a musculoskeletal assessment of a school-age child, a nurse documents which finding

as expected?

a. A positive Trendelenburg sign on one side

b. Lumbar lordosis, especially in African American children

c. Varus rotation when the knees are greater than 1 inch apart

d. Valgus rotation of less than 1 inch with the knees touching

ANSWER: D

A

B

C

D

Feedback

Trendelenburg sign (or gait) tests for hip dysplasia and the function of the gluteus

medius muscle.

Lordosis is seen more frequently in African American children but should not be

seen in children older than 6 years of age.

Varus rotation (medial malleolus touching with knees greater than 1 inch [2.5 cm]

apart) requires further evaluation for tibial torsion; it may be normal through 18

to 24 months of age.

Valgus rotation (medial malleolus greater than 1 inch [2.5 cm] apart with knees

touching) is normal in 2- to 3.5-year-old children and may be present up to 12

years old.

DIF: Cognitive Level: Understand REF: 482

TOP: Nursing Process: Assessment

MSC: NCLEX Patient Needs: Physiologic Integrity: Reduction of Risk Potential: System Specific

Assessments

42. While examining the genitalia of a 6-year-old girl, a nurse notices which finding as expected?

a. Clear mucoid vaginal discharge

b. Prepuce and clitoris are prominent

c. Flat labia majora with thin labia minora

d. Sparse pubic hair over the inner thighs

ANSWER: C

A

B

C

D

Feedback

Vaginal discharge is an abnormal finding at this age.

Normally at this age the clitoris is relatively small.

Until approximately age 7, the labia majora are flat, the labia minora are thin.

Evidence of pubic hair may be seen by the time the child reaches pubescence,

usually between ages 8 and 11.

DIF: Cognitive Level: Understand REF: 485

TOP: Nursing Process: Assessment

MSC: NCLEX Patient Needs: Physiologic Integrity: Reduction of Risk Potential: System Specific

Assessments

43. The nurse places an 8-year-old boy in which position for examination of his genitalia?

a. Supine with legs extended to either side

b. Lying on his left side with knees bent

c. Reclining with knees flexed

d. Standing with legs spread apart


ANSWER: C

A

B

C

D

Feedback

The examination is easiest to perform if the child is sitting.

The examination is easiest to perform if the child is sitting.

The examination is easiest to perform if the child is sitting in either a slightly

reclining position with his knees flexed or heels near the buttock or sitting with

his knees spread and ankles crossed.

The examination is easiest to perform if the child is sitting.

DIF: Cognitive Level: Understand REF: 485

TOP: Nursing Process: Assessment

MSC: NCLEX Patient Needs: Health Promotion and Maintenance: Techniques of Physical

Assessment

44. Which disorder, if any, does a nurse screen for when examining a healthy adolescent?

a. Muscle weakness

b. Limited joint range of motion

c. Curvature of the spine

d. No screening is needed when the adolescent is healthy

ANSWER: C

A

B

C

D

Feedback

Screening for muscle weakness in a well adolescent is not indicated.

Screening for limited range of motion in a well adolescent is not indicated.

Adolescents are screened for scoliosis, kyphosis, and lordosis. Postural kyphosis

is almost always accompanied by a compensatory lordosis, an abnormally

concave lumbar curvature.

Adolescents are screened for scoliosis.

DIF: Cognitive Level: Understand REF: 486-487

TOP: Nursing Process: Assessment

MSC: NCLEX Patient Needs: Health Promotion and Maintenance: Growth and Development

45. During a physical examination, a 12-year-old girl expresses concern to the nurse that her

breasts are different sizes. Which response is most appropriate for the nurse?

a. “This happens normally to many girls your age. Full breast development takes an

average of 3 years.”

b. “I can talk with your mother about a referral to a physician who can perform further

examination and tests.”

c. “Have you started your menstrual period yet, because breast development is

irregular until menstruation begins?”

d. “This is called ‘precocious breast development’ and your breasts will become more

equal just before your growth spurt starts.”

ANSWER: A

A

Feedback

The right and left breasts may develop at different rates. It is important to reassure

the patient that this is common and, in time, the development may equalize. Full


B

C

D

development of the breast takes an average of 3 years (range 1.5 to 6 years).

Breasts may develop at different rates.

Menarche begins when the breasts reach Tanner stages 3 or 4, which is

approximately age 12.

Precocious breast development is the development of breasts before 8 years old.

DIF: Cognitive Level: Apply REF: 487

TOP: Nursing Process: Assessment

MSC: NCLEX Patient Needs: Health Promotion and Maintenance: Growth and Development


Chapter 20: Assessment of the Pregnant Patient

Test Bank

MULTIPLE CHOICE

1. A patient who is 30 weeks pregnant tells the nurse, “I have had low blood pressure all my life,

and now it is 136/74. What’s wrong with me?” What is the most appropriate response by this

nurse?

a. “A woman’s blood pressure usually drops several points during pregnancy, but

yours hasn’t.”

b. “The blood pressure increases because your blood volume increases to supply you

and the baby with enough blood.”

c. “Yes, this is a significant change from your baseline, and I advise you to see your

obstetrician at your earliest convenience.”

d. “If you spend more time lying down, I think your blood pressure should return to

normal in a few days.”

ANSWER: B

A

B

C

D

Feedback

The blood pressure may decrease slightly in the second trimester and then return

to the usual level during the third trimester. With an increase in blood volume, the

patient may experience an increase in blood pressure above baseline.

Blood volume increases by 1500 mL to meet the needs of an enlarged uterus and

fetal tissue, causing increased cardiac workload (increased heart rate and blood

pressure).

This referral is unnecessary.

With an increase in blood volume, the patient may experience an increase in

blood pressure above baseline.

DIF: Cognitive Level: Apply REF: 496

TOP: Nursing Process: Assessment

MSC: NCLEX Patient Needs: Physiologic Integrity: Reduction of Risk Potential: System Specific

Assessments

2. A nurse instructs the patient about which expected skin changes during pregnancy?

a. Nipples becoming thicker

b. Hands and feet becoming pale and cool

c. Blotchy, brown pigmentation of the abdomen

d. Stretch marks on the expanding abdomen

ANSWER: D

A

B

C

D

Feedback

Nipples and areolae become more prominent and deeply pigmented.

Blood volume increases and edema often develops in the feet.

The blotchy, brown pigmentation occurs on the face and is called chloasma, or

the mask of pregnancy.

The increasing size of the abdomen contributes to striae gravidarum (stretch

marks) over the abdomen.


DIF: Cognitive Level: Understand REF: 496| 505

TOP: Nursing Process: Assessment

MSC: NCLEX Patient Needs: Physiologic Integrity: Physiologic Adaptation: Alterations in Body

Systems

3. How does a nurse determine the Goodell sign?

a. Assesses the softening of the lower uterine segment

b. Palpates for softening of the cervix

c. Assesses the breasts for fullness and tenderness

d. Inspects the cervix for a bluish coloration

ANSWER: B

A

B

C

D

Feedback

Assesses the softening of the lower uterine segment is a description of Hegar

sign.

Palpates for softening of the cervix is a description of Goodell sign.

Assesses the breasts for fullness and tenderness is a presumptive sign of

pregnancy rather than a probable sign.

Inspects the cervix for a bluish coloration is a description of Chadwick sign.

DIF: Cognitive Level: Understand REF: 496, Table 20-1

TOP: Nursing Process: Assessment

MSC: NCLEX Patient Needs: Health Promotion and Maintenance: Ante/Intra/Postpartum and

Newborn Care

4. The nurse recognizes which clinical manifestation as a positive sign of pregnancy?

a. Cessation of menstruation

b. Visualization of the fetus by ultrasound

c. Nausea and increased abdominal girth

d. Positive pregnancy test (hCG)

ANSWER: B

A

B

C

D

Feedback

Cessation of menstruation is a presumptive sign of pregnancy.

This is a positive sign of pregnancy.

Nausea and increased abdominal girth are presumptive signs of pregnancy.

Positive pregnancy test (hCG) is a probable sign of pregnancy.

DIF: Cognitive Level: Understand REF: 496, Table 20-1

TOP: Nursing Process: Assessment

MSC: NCLEX Patient Needs: Health Promotion and Maintenance: Ante/Intra/Postpartum and

Newborn Care

5. Using Nägele’s rule, what is the estimated delivery date of a patient whose last menstrual

period started on December 1?

a. August 1

b. August 10

c. September 4


d. September 8

ANSWER: D

Feedback

A Subtract 3 months rather than 4, and add 7 days.

B Subtract 3 months rather than 4, and add 7 days rather than 9.

C After subtracting 3 months, add 7 days instead of 3.

D September 8 is determined by using the first day of the last menstrual period,

subtracting 3 months, and adding 7 days.

DIF: Cognitive Level: Apply REF: 497

TOP: Nursing Process: Assessment

MSC: NCLEX Patient Needs: Health Promotion and Maintenance: Ante/Intra/Postpartum and

Newborn Care

6. What is the meaning of “G5, T1, P0, A3, L1” found in a patient’s history?

a. One birth at term

b. Three living children

c. Five grown children

d. One delivery not at term

ANSWER: A

A

B

C

D

Feedback

These data report that this patient has had five pregnancies, one term birth, no

preterm births, three abortions, and one living child.

This patient has had three abortions.

This patient has had five pregnancies.

This patient has had no preterm births.

DIF: Cognitive Level: Apply REF: 497

TOP: Nursing Process: Assessment

MSC: NCLEX Patient Needs: Health Promotion and Maintenance: Ante/Intra/Postpartum and

Newborn Care

7. A pregnant woman who drinks alcoholic beverages while pregnant increases the risk for

which disorder?

a. Low infant birth weight

b. Birth defects

c. Abruptio placentae

d. Gestational diabetes mellitus

ANSWER: B

A

B

C

D

Feedback

Low infant birth weight is not directly caused by alcohol intake.

Alcohol is a teratogen; no safe level of alcohol ingestion has been identified for

pregnant women.

Maternal hypertension is the risk factor for abruptio placentae.

Gestational diabetes mellitus is not caused by alcohol intake.


DIF: Cognitive Level: Understand REF: 498

TOP: Nursing Process: Assessment

MSC: NCLEX Patient Needs: Physiologic Integrity: Reduction of Risk Potential

8. A nurse refers which pregnant patient for additional assessment?

a. A woman at 36 weeks of gestation who has 30% effacement of the cervix

b. A woman at 19 weeks of gestation who has noticed fetal movement every day this

week

c. A woman at 20 weeks of gestation who has gained 4 lb in the last 2 weeks

d. A woman at 28 weeks of gestation who has a systolic blood pressure of 40 mg Hg

over baseline

ANSWER: D

A

B

C

D

Feedback

This is an expected finding at this point in the third trimester.

This is an expected finding.

This is an expected weight gain.

This finding may indicate pregnancy-induced hypertension, characterized by

systolic blood pressure of at least 30 mm Hg above baseline.

DIF: Cognitive Level: Analyze REF: 500

TOP: Nursing Process: Assessment

MSC: NCLEX Patient Needs: Physiologic Integrity: Physiologic Adaptation: Alterations in Body

Systems

9. A patient’s prepregnant weight was 131 lb, within the desirable range for her height. What is

the expected weight for this pregnant patient?

a. 131 lb at 1 week postpartum

b. 140 lb at the end of the first trimester

c. 145 lb at the end of the second trimester

d. 176 lb at the beginning of the third trimester

ANSWER: C

A

B

C

D

Feedback

There are no specific weight guidelines for after birth, but it is unlikely that she

would be at her prepregnant weight 1 week after delivery.

The expected range for the first trimester is weight gain of 3 to 5 lb; the patient

gained 9 lb.

This 14-lb weight gain is within expected ranges (12 to 15 lb [5.5 to 6.8 kg]) for

the second trimester.

This 45-lb weight gain is 10 lb more than desired; the expected range for the third

trimester is 12 to 15 lb greater than the second trimester, which would be 163 to

166 lb.

DIF: Cognitive Level: Apply REF: 501

TOP: Nursing Process: Assessment

MSC: NCLEX Patient Needs: Health Promotion and Maintenance: Ante/Intra/Postpartum and

Newborn Care


10. A pregnant patient presents to the clinic with a 3 lb/wk weight gain for 2 successive weeks.

The nurse is most concerned that this patient is demonstrating signs of which condition?

a. Gestational diabetes mellitus

b. Preeclampsia

c. Placenta enlargement

d. Multiple gestations

ANSWER: B

A

B

C

D

Feedback

A rapid increase in weight could indicate gestational diabetes, but a gain of this

much in 2 successive weeks warrants considering preeclampsia.

If a woman gains more than 2 lb (0.9 kg) in any 1 week, preeclampsia should be

suspected.

Rapid weight gain is not associated with placental enlargement.

A rapid increase in weight could indicate multiple gestation, but a gain of this

much in 2 successive weeks warrants considering preeclampsia.

DIF: Cognitive Level: Analyze REF: 501

TOP: Nursing Process: Assessment

MSC: NCLEX Patient Needs: Health Promotion and Maintenance: Ante/Intra/Postpartum and

Newborn Care

11. A pregnant woman’s weight before becoming pregnant was 137 lb. What weight is expected

for this woman during her second trimester?

a. 149 lb

b. 151 lb

c. 155 lb

d. 164 lb

ANSWER: C

A

B

C

D

Feedback

This weight is obtained when you forget to include the 3 to 5 lb for the first

trimester.

The prepregnant weight was 137 lb. Add 3 to 5 lb for the first trimester, making

the ranges from 140 to 142 lb. The second trimester is 12 to 15 lb in addition to

the first trimester weight. This 140 to 142 lb + 12 to 15 lb = a range from 152 to

157 lb.

The prepregnant weight was 137 lb. Add 3 to 5 lb for the first trimester, making

the ranges from 140 to 142 lb. The second trimester is 12 to 15 lb in addition to

the first trimester weight. This 140 to 142 lb + 12 to 15 lb = a range from 152 to

157 lb.

This is the weight for the third trimester when an additional 12 to 15 lb can be

added.

DIF: Cognitive Level: Apply REF: 501

TOP: Nursing Process: Assessment

MSC: NCLEX Patient Needs: Health Promotion and Maintenance: Ante/Intra/Postpartum and

Newborn Care


12. A nurse documents as abnormal which finding of a pregnant patient?

a. Facial swelling in a woman who is 20 weeks pregnant

b. 1+ pitting ankle edema in a woman who is 26 weeks pregnant

c. Pinkish-red blotches of the hands in a woman at 32 weeks gestation

d. Blotchy, brownish pigmentation of the face in a woman at 36 weeks gestation

ANSWER: A

A

B

C

D

Feedback

Excessive edema (particularly if noted in the hands and face in addition to the

lower extremities) is considered pathologic and may be an indication of

pregnancy induced hypertension.

Some edema is expected during pregnancy and 1+ is a modest amount of edema.

Pinkish-red blotches or diffuse mottling of the hands due to an increase in

estrogen is termed palmar erythema and is considered an expected finding.

Blotchy, brownish pigmentation of the face—chloasma, or the mask of pregnancy

—is an expected finding.

DIF: Cognitive Level: Apply REF: 502

TOP: Nursing Process: Assessment

MSC: NCLEX Patient Needs: Physiologic Integrity: Physiologic Adaptation: Alterations in Body

Systems

13. In prioritizing patient care, the nurse would give immediate attention to the pregnant patient

with which clinical manifestation?

a. Darkened eyelids

b. Seeing spots

c. Excessive eye dryness

d. Pale conjunctiva

ANSWER: B

A

B

C

D

Feedback

Darkened eyelids may occur normally and is not a reason for concern.

Chromatopsia may be noted, characterized by unusual color perception, seeing

spots, or blindness in the lateral visual field, and requires immediate follow-up.

Excessive eye dryness occurs normally in pregnancy and can be treated with

artificial tears.

Pale conjunctiva may indicate anemia, which needs to be treated, but it is not an

urgent problem.

DIF: Cognitive Level: Analyze REF: 502

TOP: Nursing Process: Assessment

MSC: NCLEX Patient Needs: Safe and Effective Care Environment: Management of Care:

Establishing Priorities

14. The nurse documents which clinical finding as an expected change associated with advancing

pregnancy?

a. Slight persistent ankle edema

b. Hypoplasia of the thyroid gland


c. Increased diaphragmatic excursion

d. Heart murmur after 20 weeks of gestation

ANSWER: D

A

B

C

D

Feedback

Ankle edema may occur, but normally is not persistent.

This is not an expected finding in pregnancy.

The diaphragm is pushed up by the growing fetus, which decreases the

diaphragmatic excursion, rather than increasing it.

Murmurs, splitting of S 1 and S 2 , and the presence of S 3 may be heard after the

twentieth week of gestation.

DIF: Cognitive Level: Understand REF: 503

TOP: Nursing Process: Assessment

MSC: NCLEX Patient Needs: Health Promotion and Maintenance: Ante/Intra/Postpartum and

Newborn Care

15. A patient at 20 weeks of gestation is concerned about a discharge from her nipples. What is

the nurse’s appropriate response to this patient?

a. “That is expected. It is milk production that begins at the onset of pregnancy.”

b. “A nipple discharge is unusual. I advise you to consult your obstetrician.”

c. “I suggest you decrease your fluid intake for several days to see if this makes a

difference.”

d. “After the first trimester a thin, yellow fluid called colostrum may be secreted from

the nipples.”

ANSWER: D

A

B

C

D

Feedback

Colostrum cannot be expressed until after the first trimester.

This finding is not unusual.

This action is not necessary because this is an expected finding.

This is a correct statement by the nurse.

DIF: Cognitive Level: Understand REF: 504

TOP: Nursing Process: Assessment

MSC: NCLEX Patient Needs: Health Promotion and Maintenance: Ante/Intra/Postpartum and

Newborn Care

16. A woman who is 16 weeks pregnant with her first child is concerned because she has not felt

the fetus move. What is the most appropriate explanation for a nurse to give this patient?

a. Movement of a fetus in the first pregnancy often does not occur until the twentyfourth

week of pregnancy.

b. A referral to an obstetrician should be made for further evaluation of this finding.

c. Movement of the fetus is not expected until the nineteenth week of pregnancy.

d. Movement of the fetus is related to fundal height; the greater the fundal height, the

sooner the fetal movement.

ANSWER: C

Feedback


A

B

C

D

Fetal movement is felt by the nineteenth week and movement observed by

approximately the twenty-eighth week.

A referral is not warranted in this case.

This patient will likely not feel fetal movement for another 3 weeks.

Fetal movement is related to gestational age rather than fundal height.

DIF: Cognitive Level: Understand REF: 506

TOP: Nursing Process: Assessment

MSC: NCLEX Patient Needs: Health Promotion and Maintenance: Ante/Intra/Postpartum and

Newborn Care

17. In measuring fundal height, the nurse documents which finding as abnormal?

a. 29 cm at week 30

b. 28 cm at week 26

c. 34 cm at week 38

d. 26 cm at week 24

ANSWER: C

Feedback

A From the twentieth to thirty-sixth week of gestation, the expected pattern of

uterine growth is an increase in fundal height of about 1 cm per week.

B Measurement of fundal height is an estimate and may vary among examiners by 1

C

to 2 cm.

This is a 4 cm difference; any discrepancy greater than 2 cm between fundal

height and the estimate of gestational age (based on the last menstrual period)

should be evaluated further.

D Measurement of fundal height is an estimate and may vary among examiners by 1

to 2 cm.

DIF: Cognitive Level: Apply REF: 506

TOP: Nursing Process: Assessment

MSC: NCLEX Patient Needs: Health Promotion and Maintenance: Ante/Intra/Postpartum and

Newborn Care

18. A nurse assesses fetal heart rates when patients are examined in the maternity clinic. The

nurse determines which fetus needs further assessment?

a. The fetus with a heart rate of 150 beats/min

b. The fetus with a heart rate of 140 beats/min

c. The fetus with a heart rate of 130 beats/min

d. The fetus with a heart rate of 110 beats/min

ANSWER: D

A

B

C

D

Feedback

This heart rate is within the expected range of 120 to 160 beats/min.

This heart rate is within the expected range of 120 to 160 beats/min.

This heart rate is within the expected range of 120 to 160 beats/min.

This fetal heart rate is too slow and needs further assessment. The expected range

is from 120 to 160 beats/min.


DIF: Cognitive Level: Apply REF: 507

TOP: Nursing Process: Assessment

MSC: NCLEX Patient Needs: Safe and Effective Care Environment: Management of Care:

Establishing Priorities

19. To perform Leopold maneuvers, the nurse uses which assessment technique?

a. Percussing over the symphysis pubis

b. Auscultating all four abdominal quadrants

c. Palpating the fundus

d. Measuring from symphysis pubis to the umbilicus

ANSWER: C

A

B

C

D

Feedback

This technique may be used to determine a full bladder.

This technique determines presence of peristalsis.

This is the procedure for Leopold maneuvers to determine presentation.

This technique is used to determine the size of the fetus.

DIF: Cognitive Level: Understand REF: 508-509

TOP: Nursing Process: Assessment

MSC: NCLEX Patient Needs: Health Promotion and Maintenance: Techniques of Physical

Assessment

20. The nurse correlates which clinical manifestation with the diagnosis of polyhydramnios?

a. Difficulty palpating fetal parts

b. Increased fetal movement

c. Weight gain of less than estimated by gestational age

d. Increase of 2 cm in fundal height in 1 week

ANSWER: A

A

B

C

D

Feedback

Inability to palpate the fetal position could be associated with polyhydramnios, an

excessive quantity of amniotic fluid.

This is not a manifestation of excessive amniotic fluid.

This is not a manifestation of excessive amniotic fluid.

This is not a manifestation of excessive amniotic fluid.

DIF: Cognitive Level: Understand REF: 513

TOP: Nursing Process: Assessment

MSC: NCLEX Patient Needs: Physiologic Integrity: Physiologic Adaptation: Alterations in Body

Systems

21. In reviewing the results of physical examination of a 25-year-old pregnant patient, a nurse

recognizes which finding as expected?

a. Small, round, oval cervix

b. Pale, symmetrical cervix

c. Smooth, bluish-colored cervix

d. Slit-shaped, pink cervix

ANSWER: C


A

B

C

D

Feedback

This describes a nulliparous cervix, but the patient referred to is pregnant, which

would cause a bluish-colored cervix.

The patient referred to is pregnant, which would cause a bluish-colored cervix.

By the second month of pregnancy, the cervix, vagina, and vulva take on a bluish

color (Chadwick sign) due to increased vascularity.

The patient referred to is pregnant, which would cause a bluish-colored cervix.

DIF: Cognitive Level: Apply REF: 508

TOP: Nursing Process: Assessment

MSC: NCLEX Patient Needs: Physiologic Integrity: Reduction of Risk Potential: System Specific

Assessments

22. How does a nurse describe abruptio placentae when teaching a class to pregnant women?

a. A premature separation of the implanted placenta before birth of the fetus

b. An accumulation of amniotic fluid associated with decreased fetal movement

c. An attachment of the placenta in the lower uterine segment near the cervical os

d. Related to premature rupture of membranes with purulent vaginal discharge

ANSWER: A

A

B

C

D

Feedback

A premature separation of the implanted placenta before birth of the fetus is a

description of abruptio placentae.

This is an incorrect description.

This is a description of placenta previa.

Abruptio placentae is premature separation of the implanted placenta rather than a

premature rupture of membranes; there is no purulent vaginal discharge.

DIF: Cognitive Level: Understand REF: 512

TOP: Nursing Process: Assessment

MSC: NCLEX Patient Needs: Physiologic Integrity: Physiologic Adaptation: Alterations in Body

Systems

23. Which complication of pregnancy does a nurse suspect when the patient reports painless

vaginal bleeding at 32 weeks of gestation?

a. Placenta previa

b. Eclampsia

c. Abruptio placentae

d. Premature rupture of membranes

ANSWER: A

A

B

C

D

Feedback

Placenta previa has painless vaginal bleeding as a manifestation.

Eclampsia does not have painless vaginal bleeding as a manifestation.

Abruptio placentae does not have painless vaginal bleeding as a manifestation.

Premature rupture of membranes does not have painless vaginal bleeding as a

manifestation.


DIF: Cognitive Level: Understand REF: 512-513

TOP: Nursing Process: Assessment

MSC: NCLEX Patient Needs: Physiologic Integrity: Physiologic Adaptation: Alterations in Body

Systems

24. In assessing a patient for modifiable risk factors, the nurse correlates which finding with a

high-risk pregnancy?

a. 15-year-old mother

b. Low socioeconomic status

c. Previous birth of infant with isoimmunization

d. Weight less than 100 lb

ANSWER: D

A

B

C

D

Feedback

Age cannot be modified.

Low socioeconomic status cannot be modified easily.

Previous birth of infant with isoimmunization cannot be modified.

This risk factor can be modified by diet.

DIF: Cognitive Level: Understand REF: 512

TOP: Nursing Process: Assessment

MSC: NCLEX Patient Needs: Health Promotion and Maintenance: Ante/Intra/Postpartum and

Newborn Care

MULTIPLE RESPONSE

1. Which physiologic changes are associated with pregnancy? Select all that apply.

a. Increase in brittleness of fingernails

b. 1500 mL increase in blood volume

c. Periodic shortness of breath later in pregnancy

d. Edematous and bleeding gums

e. Painless vaginal bleeding

f. Waddling gait

ANSWER: B, C, D, F

Correct: Blood volume increases by 1500 mL to meet the need of an enlarged uterus and fetal

tissue, causing increased cardiac workload (increased heart rate). Uterine enlargement pushes

up on the diaphragm, causing periodic shortness of breath. Increased estrogen increases

vascularity and tissue proliferation of gums, resulting in edematous and bleeding gums. The

increased size of the uterus and growing fetus results in the center of gravity moving forward,

resulting in a waddling gait.

Incorrect: Increased nail growth is reported by some individuals. Painless vaginal bleeding

indicates a problem with the pregnancy, such as placenta previa.

DIF: Cognitive Level: Understand REF: 496

TOP: Nursing Process: Assessment

MSC: NCLEX Patient Needs: Health Promotion and Maintenance: Ante/Intra/Postpartum and

Newborn Care


2. What does a nurse teach pregnant patients about the effects of smoking while pregnant? Select

all that apply.

a. Fluid retention increases.

b. Prematurity risk increases.

c. Low infant birth weight risk increases.

d. Anemia develops.

e. Vitamin C deficiency develops.

f. Craving of starch, baking soda, or clay develops.

ANSWER: B, C, E

Correct: These three are due to the effects of smoking while pregnant.

Incorrect: Fluid retention increases may occur during pregnancy, but it is not caused by

smoking. Anemia may occur during pregnancy, but it is not caused by smoking. Craving of

starch, baking soda, or clay may occur during pregnancy, but it is not caused by smoking.

DIF: Cognitive Level: Understand REF: 498

TOP: Nursing Process: Assessment

MSC: NCLEX: Patient Needs: Physiologic Integrity: Physiologic Adaptation: Alteration in Body

Systems

3. What manifestations does a nurse anticipate when assessing a patient with preeclampsia?

Select all that apply.

a. Systolic blood pressure greater than 140 mm Hg

b. Increase in systolic blood pressure of 20 mm Hg over baseline

c. Diastolic blood pressure greater than 90 mm Hg

d. Increase in diastolic blood pressure of 15 mm Hg over baseline

e. 2 lb increase in weight in 1 week

f. 9 lb increase in weight in 1 month

ANSWER: A, C, D, F

Correct: Preeclampsia is precipitated by pregnancy-induced hypertension indicated by

systolic blood pressure of 140 mm Hg or higher, diastolic blood pressure more than 90 mm

Hg, and an increase of more than 15 mm Hg of diastolic blood pressure from baseline. A 9-lb

weight gain in this time period contributes to hypertension.

Incorrect: Increase in systolic blood pressure of 20 mm Hg over baseline is an increase of

more than 30 mm Hg of systolic blood pressure from baseline. Two-pound increase in weight

in 1 week is an expected weight gain during pregnancy.

DIF: Cognitive Level: Understand REF: 513

TOP: Nursing Process: Assessment

MSC: NCLEX Patient Needs: Physiologic Integrity: Physiologic Adaptation: Alterations in Body

Systems

COMPLETION

1. If a patient’s last menstrual period was May 13, her estimated date of birth is .

ANSWER:

February 20

May 13 minus 3 months = February 13 plus 7 days = February 20.


DIF: Cognitive Level: Apply REF: 497

TOP: Nursing Process: Assessment

MSC: NCLEX Patient Needs: Health Promotion and Maintenance: Ante/Intra/Postpartum and

Newborn Care

2. A pregnant patient’s weight before pregnancy was 148 lb. Her expected weight during the first

trimester is to lb.

ANSWER:

151

153

151, 153

3- to 5-lb weight gain is expected during the first trimester resulting in a total of 151 to 153 lb.

DIF: Cognitive Level: Apply REF: 501

TOP: Nursing Process: Assessment

MSC: NCLEX Patient Needs: Health Promotion and Maintenance: Ante/Intra/Postpartum and

Newborn Care

3. A pregnant patient’s weight before pregnancy was 163 lb. The nurse expects the patient to

weigh to lb during the second trimester.

ANSWER:

178; 183

178, 178

A 3- to 5-lb weight gain is expected during the first trimester, making the weight 166 to 168.

The second trimester is 12 to 15 lb in addition to the first trimester weight. Thus 166 + 12 lb =

178 lb at the low end and 168 + 15 lb = 183 at the high end.

DIF: Cognitive Level: Apply REF: 501

TOP: Nursing Process: Assessment

MSC: NCLEX Patient Needs: Health Promotion and Maintenance: Ante/Intra/Postpartum and

Newborn Care


Chapter 21: Assessment of the Older Adult

Test Bank

MULTIPLE CHOICE

1. A 75-year old patient tells the nurse, “I just do not enjoy eating as much as I used to because

the food does not have much taste or smell.” Which statement by the nurse is most

appropriate?

a. “You should make an appointment with your health care provider.”

b. “Try eating small, frequent meals.”

c. “The senses of smell and taste decrease as we age.”

d. “Maybe you should use saline drops in your nose.”

ANSWER: C

A

B

C

D

Feedback

This action is not warranted in this case; these changes are expected with aging.

This is practical advice, but will not alter the change in smell and taste.

A decreased sense of smell is caused by a decrease in the number of sensory cells

in the nasal lining. Taste perception may also diminish due to gradual atrophy of

the tongue and a decrease in the number of papillae and taste buds.

This is practical advice, but will not alter the change in smell and taste.

DIF: Cognitive Level: Apply REF: 516

TOP: Nursing Process: Assessment

MSC: NCLEX Patient Needs: Health Promotion and Maintenance: Aging Process

2. During inspection of the mouth of an older adult, a nurse notices which finding as an expected

change associated with aging?

a. Exposed root surfaces of teeth

b. Aphthous ulcers of the mucosa

c. Collection of debris at the gingival margins

d. Leukoplakia of the dorsal and ventral tongue

ANSWER: A

A

B

C

D

Feedback

Root surfaces of the teeth are exposed to caries formation because of gingival

recession.

Ulcers are not an expected finding.

Collection of debris at the gingival margins is not an expected finding.

This is not an expected finding; leukoplakia is a white patch or plaque that cannot

be scraped off and often represents a premalignant lesion.

DIF: Cognitive Level: Understand REF: 516

TOP: Nursing Process: Assessment

MSC: NCLEX Patient Needs: Health Promotion and Maintenance: Aging Process

3. Which assessment finding of older adult patients indicates expected respiratory function?

a. Increased elasticity of the alveoli


b. Flaccidity of the chest wall

c. Reduced inspiratory and expiratory effort

d. Decreased anteroposterior diameter

ANSWER: C

A

B

C

D

Feedback

With aging, alveoli become less elastic and more fibrous.

With aging, the chest wall may become stiffer, possibly because of calcification at

rib articulation points, resulting in decreased chest wall compliance.

Diminished strength of the respiratory muscles results in reduced maximal

inspiratory and expiratory force.

The anteroposterior diameter increases with aging due to kyphoscoliosis.

DIF: Cognitive Level: Understand REF: 516

TOP: Nursing Process: Assessment

MSC: NCLEX Patient Needs: Health Promotion and Maintenance: Aging Process

4. During an office visit, a 78-year-old woman is upset because her height is “2 inches less than

it was when I was 40!” How does the nurse explain this change to the patient?

a. “Reduced height may occur as you age due to shortening of the vertebrae.”

b. “You may be experiencing this height change due to arthritis.”

c. “You need to improve your posture by performing stretching exercises.”

d. “This is a rare occurrence and warrants having a bone density test.”

ANSWER: A

A

B

C

D

Feedback

Decreased bone formation reduces height in most older adults, which may cause

shortening of the vertebrae and thinning of the vertebral disks.

Decreased bone formation reduces height in most older adults and is not due to

arthritis.

This is appropriate advice, but is not related the patient’s height.

This is an expected occurrence and does not warrant concern.

DIF: Cognitive Level: Apply REF: 516

TOP: Nursing Process: Assessment

MSC: NCLEX Patient Needs: Health Promotion and Maintenance: Aging Process

5. In collecting a history from an older adult, which information does the nurse consider least

essential for a patient of this age?

a. Past health history

b. Genogram

c. Functional abilities

d. Mental health

ANSWER: B

A

B

Feedback

Past health history is important to document the patient’s chronic illnesses.

A genogram is not routinely used to document the family history for an older


C

D

adult. The health status and cause of death of the patient’s parents and siblings

lose value as the patient ages.

Assessing functional abilities provides data about how well the patient performs

activities of daily living.

Mental health data are essential to collect about all patients regardless of age.

DIF: Cognitive Level: Understand REF: 517

TOP: Nursing Process: Assessment

MSC: NCLEX Patient Needs: Health Promotion and Maintenance: Aging Process

6. In assessing the mood of older adult patients, a nurse documents which finding as abnormal?

a. Sadness and grief after returning from the funeral of a long-time friend

b. Depression that interferes with the ability to perform activities of daily living

c. Frustration about rearranging the day’s schedule to attend a grandson’s birthday

party

d. Crying about the unexpected death of a pet that had been with the family 12 years

ANSWER: B

A

B

C

D

Feedback

Emotional experiences of sadness, grief, response to loss, and temporary “blue”

moods are normal responses in older adults.

Persistent depression that interferes significantly with the ability to function is not

an expected finding.

This is a normal response for any adult.

Emotional experiences of sadness, grief, response to loss, and temporary “blue”

moods are normal responses in older adults.

DIF: Cognitive Level: Apply REF: 518

TOP: Nursing Process: Assessment

MSC: NCLEX Patient Needs: Physiologic Integrity: Physiologic Adaptation: Alteration in Body

Systems

7. When assessing the pain level of an older adult, a nurse considers which factor?

a. Neural transmission of pain is increased as a part of the aging process.

b. Older adult patients are not reliable in their descriptions of pain and how it affects

them.

c. Physiologic indicators of pain that are unique to older adults are tachycardia and

hypotension.

d. The older adult may believe that pain is a factor of aging and not worth mentioning.

ANSWER: D

A

B

C

D

Feedback

Neural transmission is the same for older and younger adults.

Becoming older does not diminish one’s ability to describe pain.

The physiologic indicators are the same for older and younger adults.

Some older adults may perceive pain as an expected aspect of aging that they

must endure.

DIF: Cognitive Level: Apply REF: 519


TOP: Nursing Process: Assessment

MSC: NCLEX Patient Needs: Health Promotion and Maintenance: Aging Process

8. A nurse is assessing the pain of an 86-year-old man who had hip surgery recently. The patient

has been slightly confused since his surgery, but he responds to simple questions. What is the

best way to assess this patient’s pain?

a. Ask him to rate his pain on a scale of 0 to 10.

b. Ask him to rate his pain using a list of descriptive adjectives.

c. Ask him to rate his pain using a vertical numeric scale.

d. Observe his behavior and measure his vital signs.

ANSWER: C

A

B

C

D

Feedback

This scale is appropriate for adolescents and adults, but older adults are assessed

more accurately with a scale that they can see.

This method is not effective for assessing pain because adjectives have different

meanings to different people. It is best to use a scale or pictures of faces.

Pain assessment in older adults is significantly improved by using a vertical

numeric scale or pain faces.

Patient behavior and vital signs are not accurate ways to assess a patient’s

perception of pain.

DIF: Cognitive Level: Apply REF: 519

TOP: Nursing Process: Assessment

MSC: NCLEX Patient Needs: Health Promotion and Maintenance: Aging Process

9. What finding does a nurse look for when assessing the skin of an older adult with solar

lentigo?

a. Yellowish, thin papules with a central depression

b. Pigmented, raised, wartlike lesions on the face or trunk

c. Small, soft, pigmented tags of skin on the face and neck

d. Irregular, flat, deeply pigmented macules on sun-exposed areas

ANSWER: D

A

B

C

D

Feedback

Yellowish, thin papules with a central depression is a description of sebaceous

hyperplasia.

Pigmented, raised, wart-like lesions on the face or trunk is a description of

seborrheic keratosis.

Small, soft, pigmented tags of skin on the face and neck is a description of

acrochordon (skin tags).

Irregular, flat, deeply pigmented macules on sun-exposed areas is a description of

solar lentigo.

DIF: Cognitive Level: Apply REF: 522

TOP: Nursing Process: Assessment

MSC: NCLEX Patient Needs: Health Promotion and Maintenance: Aging Process


10. When assessing the skin of an older adult, a nurse notices pigmented, raised warty-appearing

lesions on the trunk. How does a nurse document this finding?

a. Solar lentigo

b. Basal cell skin cancer

c. Seborrheic keratosis

d. Sebaceous hyperplasia

ANSWER: C

A

B

C

D

Feedback

Solar lentigo appears as irregularly shaped, flat, deeply pigmented macules that

may appear on body surface areas with repeated exposure to the sun.

Basal cell carcinoma appears as a nodular pigmented lesion with a depressed

center and rolled borders found on sun-exposed areas.

Seborrheic keratosis is described as pigmented, raised warty-appearing lesions on

the trunk.

Sebaceous hyperplasia appears as yellowish, flattened papules that have central

depressions.

DIF: Cognitive Level: Understand REF: 523

TOP: Nursing Process: Assessment

MSC: NCLEX Patient Needs: Health Promotion and Maintenance: Aging Process

11. In assessing the nails of an older adult, which finding does a nurse expect to find?

a. Transverse ridges

b. Thick, brittle, and yellow nails

c. Thin, brittle nails

d. Lateral edges turned upward

ANSWER: B

A

B

C

D

Feedback

These changes occur when the patient has had nail trauma.

Thick, brittle, and yellow nails are expected changes in the nails of older adults.

These changes occur when the patient has anemia.

This change, called spoon nail, occurs when the patient has anemia.

DIF: Cognitive Level: Understand REF: 523

TOP: Nursing Process: Assessment

MSC: NCLEX Patient Needs: Health Promotion and Maintenance: Aging Process

12. In assessing the external eyes of an older adult, a nurse documents which finding as

abnormal?

a. Gray-white circle where the cornea and the sclera merge

b. Brown spots near the limbus in both eyes

c. Lack of luster of the eye and dry bulbar conjunctiva

d. Lower lid drops away from the globe

ANSWER: D

Feedback


A

B

C

D

This is a description of arcus senilis.

This is a description of a normal variation.

This occurs because the lacrimal apparatus may function poorly, producing fewer

tears.

This is a description of ectropion, an abnormal finding.

DIF: Cognitive Level: Apply REF: 523-524

TOP: Nursing Process: Assessment

MSC: NCLEX Patient Needs: Physiologic Integrity: Physiologic Adaptation: Alteration in Body

Systems

13. What is the best color for nurses to select when designing educational materials for older

adults?

a. Blue

b. Yellow

c. Violet

d. Green

ANSWER: B

A

B

C

D

Feedback

Color perception of blue, violet, and green may be impaired for older adults.

Color perception of blue, violet, and green may be impaired for older adults.

Color perception of blue, violet, and green may be impaired for older adults.

Color perception of blue, violet, and green may be impaired for older adults.

DIF: Cognitive Level: Understand REF: 523

TOP: Nursing Process: Assessment

MSC: NCLEX Patient Needs: Health Promotion and Maintenance: Aging Process

14. A 75-year-old man reports he stopped playing cards with his friends because their voices

sounded mumbled. How does the nurse explain the cause of this change?

a. Sudden low-frequency hearing loss

b. Accumulation of earwax in the outer ear

c. Damage to the middle ear from ear infections

d. Gradual high-frequency hearing loss

ANSWER: D

A

B

C

D

Feedback

This does not describe presbycusis and is not an expected change with aging.

This does not describe presbycusis and is not an expected change with aging.

This does not describe presbycusis and is not an expected change with aging.

This is a description of presbycusis, a sensorineural hearing loss, and an expected

change with aging.

DIF: Cognitive Level: Apply REF: 524

TOP: Nursing Process: Assessment

MSC: NCLEX Patient Needs: Health Promotion and Maintenance: Aging Process


15. Which finding on cardiovascular assessment of an older adult patient warrants further

evaluation?

a. Occasional ectopic beats heard on auscultation of the heart

b. Murmur heard over the mitral valve

c. Systolic pressure of 156 in the right arm and 188 in the left arm

d. Persistent S 4 sound in a patient with a history of decreased ventricular function

ANSWER: C

A

B

C

D

Feedback

Occasional ectopic beats are common and may or may not be significant.

Sclerosis of the mitral and aortic valves may cause murmurs.

These systolic pressures are above normal and require further evaluation.

The S 4 heart sound is common in older adults and may be associated with

decreased left ventricular compliance.

DIF: Cognitive Level: Apply REF: 521-522

TOP: Nursing Process: Assessment

MSC: NCLEX Patient Needs: Physiologic Integrity: Physiologic Adaptation: Alteration in Body

Systems

16. The nurse notes which finding as expected during a history and examination of an older adult

patient’s abdomen?

a. Hyperactive bowel sounds in all quadrants

b. Decreased fatty deposits over the abdomen

c. Marked concavity of the abdominal contour

d. Soft abdomen on palpation in all quadrants

ANSWER: D

A

B

C

D

Feedback

The opposite is true; decreased peristalsis causes hypoactive bowel sounds.

The opposite is true; older adults may have increased fat deposits over the

abdominal area.

The opposite is true due to the increased fat deposits over the abdominal area.

The abdomen of older adults may feel soft due to decreased abdominal muscle

tone.

DIF: Cognitive Level: Understand REF: 525

TOP: Nursing Process: Assessment

MSC: NCLEX Patient Needs: Health Promotion and Maintenance: Aging Process

17. Which approach does a nurse use to assess neck range of motion of an older adult patient?

a. Have the patient perform each neck movement separately.

b. Defer range of motion examination if the patient has kyphosis.

c. Ask the patient to turn the head against the resistance of the nurse’s hand.

d. Ask the patient to rotate the head starting with forward flexion and moving

clockwise.

ANSWER: A

Feedback


A

B

C

D

The nurse should assess range of motion of the neck with one movement at a

time, rather than a full rotation of the neck, to avoid causing dizziness on

movement.

Assessing range of motion is important data to gather to determine how limited

the range is due to the kyphosis.

This technique tests muscle strength rather than range of motion.

This technique tests muscle strength rather than range of motion.

DIF: Cognitive Level: Apply REF: 525

TOP: Nursing Process: Assessment

MSC: NCLEX Patient Needs: Health Promotion and Maintenance: Techniques of Physical

Assessment

18. A nurse asks an older adult patient to rise from an arm chair without using the arms, stand

with eyes closed, and turn around in a circle. What is the nurse assessing in this patient?

a. Ability to follow instructions

b. Muscle strength

c. Balance

d. Hearing

ANSWER: C

A

B

C

D

Feedback

Although following instruction is required for this balance assessment, it is not

the primary purpose of the assessment.

Muscle strength is tested by having the patient push or pull against resistance.

These are three of the activities of the Tinetti Balance and Gait Assessment Tool.

Although hearing is required for this balance assessment, it is not the primary

purpose of the assessment.

DIF: Cognitive Level: Analyze REF: 525-526

TOP: Nursing Process: Assessment

MSC: NCLEX Patient Needs: Health Promotion and Maintenance: Techniques of Physical

Assessment

19. An older adult patient reports being able to see her granddaughter play basketball out of the

sides of her eyes, but not in the center of her eyes. Based on this information, what vision

disorder does the nurse suspect?

a. Presbyopia

b. Macular degeneration

c. Pseudoptosis

d. Entropion

ANSWER: B

A

B

C

Feedback

Presbyopia is a decrease in near vision that usually occurs after age 40 and is

treated with corrective lenses.

Gradual loss of central vision may be caused by macular degeneration due to

changes in the retina.

Pseudoptosis is a relaxed upper eyelid.


D

Entropion is a disorder of the eyelid, in which the lower lid turns inward.

DIF: Cognitive Level: Understand REF: 524| 528

TOP: Nursing Process: Assessment

MSC: NCLEX Patient Needs: Physiologic Integrity: Physiologic Adaptation: Alteration in Body

Systems

20. The nurse examining the breasts of an older adult woman recognizes which finding as

normal?

a. Firm and rounded breasts of equal size and shape

b. Relatively large size and number of mammary ducts

c. Loose elasticity and puckering of the suspensory ligaments

d. Flattened breasts with a slightly granular texture on palpation

ANSWER: D

A

B

C

D

Feedback

The breasts in postmenopausal women may appear flattened.

This is not a finding in older women.

The suspensory ligaments in older woman are relaxed, but not puckering.

The breasts in postmenopausal women may appear flattened and elongated or

pendulous secondary to a relaxation of the suspensory ligaments.

DIF: Cognitive Level: Understand REF: 525

TOP: Nursing Process: Assessment

MSC: NCLEX Patient Needs: Health Promotion and Maintenance: Aging Process

MULTIPLE RESPONSE

1. What expected physiologic changes of aging put older adults at risk for respiratory infections?

Select all that apply.

a. Breath sounds are bronchovesicular in the peripheral lung.

b. Alveoli are less elastic.

c. Weak intercostal muscles reduce effective coughing.

d. Fewer cilia make mucociliary clearance less effective.

e. Curvature of the spine limits chest wall expansion.

f. Cough reflex is impaired due to deceased sensitivity of receptors.

ANSWER: B, C, D, E

Correct: These are all expected findings of healthy older adults that impair their ability to

breathe deeply and cough to prevent or recover from a respiratory infection.

Incorrect: Breath sounds are the same as for younger adults—vesicular in the peripheral

lungs. Cough reflex is not changed in the older adult.

DIF: Cognitive Level: Analyze REF: 516| 524

TOP: Nursing Process: Assessment

MSC: NCLEX Patient Needs: Health Promotion and Maintenance: Aging Process

2. What risk factors for falls does a nurse teach a group of older adults? Select all that apply.

a. Being a woman

b. Taking more than six medications


c. Having hypertension

d. Having cataracts

e. Muscle strength 3/5 bilaterally

f. Incontinence

ANSWER: B, D, E, F

Correct: Adverse effects of medications can contribute to falls. Cataracts impair vision,

which is a risk factor for falls. Poor muscle strength is a risk factor for falls. Incontinence of

urine or stool increases risk for falls.

Incorrect: Men have a higher risk for falls than women. Hypertension itself does not

contribute to falls. Dizziness does contribute to falls.

DIF: Cognitive Level: Analyze REF: 528

TOP: Nursing Process: Assessment

MSC: NCLEX Patient Needs: Physiologic Integrity: Reduction of Risk Potential: Potential for

Alteration in Body Systems


Chapter 22: Conducting a Head-to-Toe Examination

Test Bank

MULTIPLE CHOICE

1. When does the health assessment begin?

a. When the nurse first meets the patient

b. When the patient tells the nurse his name and age

c. When the nurse asks the patient the first health-related question

d. When the patient consents to have a health assessment performed

ANSWER: A

A

B

C

D

Feedback

When the nurse and patient first meet, the nurse begins collecting data about the

patient.

Before this, the nurse began collecting data about the patient, such as gait,

posture, and hygiene.

Before this, the nurse began collecting data about the patient, such as gait,

posture, and hygiene.

Before this, the nurse began collecting data about the patient such as gait, posture,

and hygiene.

DIF: Cognitive Level: Understand REF: 531

TOP: Nursing Process: Assessment

MSC: NCLEX Patient Needs: Health Promotion and Maintenance: Techniques of Physical

Assessment

2. Which assessments are routine examination techniques of the upper extremities?

a. Palpating the epitrochlear lymph nodes for size and tenderness

b. Palpating the arms for skin characteristics, symmetry, tenderness, and deformities

c. Testing the range of motion and muscle strength comparing one arm with the other

d. Testing triceps, biceps, and brachioradialis deep tendon reflexes bilaterally

ANSWER: B

A

B

C

D

Feedback

Lymph nodes are not palpated unless indicated.

Palpation of upper extremities is performed in a routine head-to-toe examination.

These data are not routinely assessed unless indicated.

These data are not routinely assessed unless indicated.

DIF: Cognitive Level: Understand REF: 533

TOP: Nursing Process: Assessment

MSC: NCLEX Patient Needs: Health Promotion and Maintenance: Techniques of Physical

Assessment

MULTIPLE RESPONSE

This study source was downloaded by 100000838401522 from CourseHero.com on 03-03-2022 08:58:43 GMT -06:00

https://www.coursehero.com/file/20908986/c22/


1. Which data does a nurse collect during the general survey when meeting a patient for the first

time? Select all that apply.

a. Gait

b. Muscle strength

c. Heart sounds

d. Hearing and speech abilities

e. Mood or affect

f. Position of the trachea

ANSWER: A, D, E

Correct: These data are observed during the general survey as the patient enters the

examination area and greets the nurse.

Incorrect: Although the nurse could detect firmness in a patient’s handshake, muscle strength

testing is performed during the examination if indicated, not during the general survey. Data

about heart sounds are collected during auscultation of the chest. Position of the trachea is

determined by palpating the trachea during the examination.

DIF: Cognitive Level: Apply REF: 532

TOP: Nursing Process: Assessment

MSC: NCLEX Patient Needs: Health Promotion and Maintenance: Techniques of Physical

Assessment

2. Which techniques does a nurse use routinely to collect data when assessing a patient’s

posterior thorax? Select all that apply.

a. Inspection of the thorax for symmetry of shoulders

b. Percussion the costovertebral angle bilaterally

c. Inspection of respiratory movement for symmetry, depth, and rhythm of respiration

d. Percussion of the posterior and lateral thorax for resonance

e. Palpation of vertebrae for alignment and tenderness

f. Inspection of thorax for muscular development and scapular alignment

ANSWER: A, C, E, F

Correct: These techniques are performed in a routine head-to-toe assessment of the posterior

thorax.

Incorrect: Percussion of the costovertebral angle bilaterally is not performed unless indicated.

For example, when the patient has a kidney disorder. Percussion of the posterior and lateral

thorax for resonance is not performed unless indicated.

DIF: Cognitive Level: Apply REF: 534

TOP: Nursing Process: Assessment

MSC: NCLEX Patient Needs: Health Promotion and Maintenance: Techniques of Physical

Assessment

3. Which techniques does a nurse use routinely to collect data when assessing a patient’s anterior

thorax? Select all that apply.

a. Palpation of the thorax for fremitus

b. Inspection of the skin for color, intactness, lesions, and scars

c. Auscultation of breath sounds bilaterally

d. Auscultation of heart sounds for rate, rhythm, frequency, and S 1 and S 2

e. Palpation the anterior chest wall for thoracic expansion

f. Inspection of respiratory movement for symmetry and ease of respiration

This study source was downloaded by 100000838401522 from CourseHero.com on 03-03-2022 08:58:43 GMT -06:00

https://www.coursehero.com/file/20908986/c22/


ANSWER: B, C, D, F

Correct: These techniques are performed in a routine head-to-toe assessment of the anterior

thorax.

Incorrect: Palpation of the thorax for fremitus and palpation of the anterior chest wall for

thoracic expansion are not performed unless indicated.

DIF: Cognitive Level: Apply REF: 534

TOP: Nursing Process: Assessment

MSC: NCLEX Patient Needs: Health Promotion and Maintenance: Techniques of Physical

Assessment

4. Which techniques does a nurse use routinely to collect data when assessing the abdomen of a

patient? Select all that apply.

a. Testing for presence of abdominal reflexes

b. Inspecting skin for contour, scars, lesions, vascularity, and bulges

c. Percussing in all quadrants for tone

d. Lightly palpating for tenderness, guarding, and masses

e. Auscultating for bowel sounds, bruits, and venous hums

f. Deeply palpating for tenderness, guarding, and masses

ANSWER: B, D, E

Correct: These techniques are performed in a routine head-to-toe assessment of the abdomen.

Incorrect: Testing for abdominal reflexes for presence, percussing in all quadrants for tone,

and deeply palpating for tenderness, guarding, and masses are not performed unless indicated.

DIF: Cognitive Level: Apply REF: 535

TOP: Nursing Process: Assessment

MSC: NCLEX Patient Needs: Health Promotion and Maintenance: Techniques of Physical

Assessment

5. Which techniques does a nurse routinely use to collect data when assessing the lower

extremities of a patient? Select all that apply.

a. Inspecting of legs, ankles, and feet for skin characteristics and hair distribution

b. Assessing for knee stability with the drawer test, McMurray test, or Apley test

c. Palpating lower legs and feet for temperature, pulses, and tenderness

d. Assessing for nerve root compression with straight leg raises

e. Palpating hips for stability and tenderness

f. Testing for patellar and Achilles deep tendon reflexes bilaterally

ANSWER: A, C

Correct: These techniques are performed in a routine head-to-toe assessment of the lower

extremities.

Incorrect: Assessing for knee stability with the drawer test, McMurray test, or Apley test is

not performed unless indicated by knee instability. Assessing for nerve root compression with

straight leg raises is not performed unless indicated. Palpating hips for stability and tenderness

is not performed unless the patient has unstable hips. Testing for patellar and Achilles deep

tendon reflexes bilaterally is not performed unless indicated.

DIF: Cognitive Level: Apply REF: 535-536

TOP: Nursing Process: Assessment

MSC: NCLEX Patient Needs: Health Promotion and Maintenance: Techniques of Physical

This study source was downloaded by 100000838401522 from CourseHero.com on 03-03-2022 08:58:43 GMT -06:00

https://www.coursehero.com/file/20908986/c22/


Assessment

This study source was downloaded by 100000838401522 from CourseHero.com on 03-03-2022 08:58:43 GMT -06:00

https://www.coursehero.com/file/20908986/c22/


Chapter 23: Documenting the Head-to-Toe Health Assessment

Test Bank

MULTIPLE CHOICE

1. Which documentation by a nurse is most descriptive?

a. Heart sounds normal.

b. Few ectopic beats heard during auscultation.

c. S 1 murmur is heard at second right sternal border.

d. Pulse within normal limits.

ANSWER: C

A

B

C

D

Feedback

This information is not specific without a description of what is considered

“within normal limits” for hair color, texture, length, etc.

This description is not specific because the number of permanent teeth is not

detailed.

This is the most descriptive data, specifying what is heard and the location.

This description would be more descriptive if the pain scale were used to indicate

the level of tenderness and if the location on the abdomen were documented.

DIF: Cognitive Level: Apply REF: 540

TOP: Nursing Process: Assessment MSC: NCLEX Patient Needs: Management of Care

2. The nurse documents which data under the category of present health status?

a. Heart sounds normal.

b. Few ectopic beats heard during auscultation.

c. Apical pulse palpated at 5th left intercostal space, midclavicular line

d. Pulse within normal limits.

ANSWER: D

A

B

C

D

Feedback

This information is not specific without a description of what is considered

“within normal limits” for heart sounds

This description is not specific

This is the most descriptive data, specifying what is heard and the location.

This description would be more descriptive such as which pulse and the rate

DIF: Cognitive Level: Apply REF: 538

TOP: Nursing Process: Assessment MSC: NCLEX Patient Needs: Management of Care

3. Which data do nurses document under the category of past health history?

a. Chronic diseases

b. Immunizations received

c. Allergies to medications or food

d. Causes of death of the patient’s parents

ANSWER: B

.

.


A

B

C

D

Feedback

Chronic diseases are documented under the present health status category.

Immunizations are documented under the past health history category.

Allergies to medications or food are documented under the present health status

category.

Causes of death of parents are documented under the family history category.

DIF: Cognitive Level: Apply REF: 538

TOP: Nursing Process: Assessment MSC: NCLEX Patient Needs: Management of Care

4. A patient reports she has shortness of breath and peripheral edema. Under which category

does the nurse document these data?

a. Review of systems

b. Present health status

c. Past health history

d. Functional ability

ANSWER: A

A

B

C

D

Feedback

In the review systems part of the history, the nurse asks patients whether they

have had symptoms from specific body systems. Patients either deny or admit to

having the symptoms.

The present health status category contains data on chronic diseases, medications,

and allergies.

The past health history category contains data on childhood diseases, surgeries

and hospitalizations, and immunizations.

The functional ability category contains data on patient’s activities, including

maintaining a home and working full-time.

DIF: Cognitive Level: Understand REF: 539

TOP: Nursing Process: Assessment MSC: NCLEX Patient Needs: Management of Care

5. What data do nurses document under the category general survey?

a. Mental health

b. Functional ability

c. Diet and nutrition

d. Orientation

ANSWER: D

A

B

C

D

Feedback

Mental health is described under the heading of personal and psychosocial

history.

Functional ability data are collected during the history.

Diet and nutrition data are collected during the history.

The general survey is the beginning of the examination when the nurse is

collecting data about the patient.

DIF: Cognitive Level: Understand REF: 540

.

.


TOP: Nursing Process: Assessment

MSC: NCLEX Patient Needs: Management of Care

MULTIPLE RESPONSE

1. Which data do nurses document under the category of personal and psychosocial health

history? Select all that apply.

a. Allergies to medications or food

b. Diet and foods eaten on a regular basis

c. Type of employment

d. Address and date of birth

e. Activities that promote health

f. Use of tobacco and alcohol

ANSWER: B, E, F

Correct: These data are documented under the heading of personal and psychosocial health.

Incorrect: Allergies to medications or food is documented under the present health status

category. Type of employment is documented under the heading of biographic data. Address

and date of birth are documented under the heading of biographic data.

DIF: Cognitive Level: Apply REF: 539

TOP: Nursing Process: Assessment MSC: NCLEX Patient Needs: Management of Care

.

.


Chapter 24: Adapting Health Assessment to the Hospitalized Patient

Test Bank

MULTIPLE CHOICE

1. Development of which complication is considered a never event?

a. Fever

b. Atelectasis

c. Pressure ulcer

d. Thrombophlebitis

ANSWER: C

A

B

C

D

Feedback

Fever is a common occurrence in ill patients that may indicate inflammation or

infection.

Atelectasis is collapse of alveoli that may occur due to the patient’s

hypoventilation, such as after surgery.

Pressure ulcer is termed a never event because it refers to preventable, medical

errors that should never occur.

Thrombophlebitis is inflammation of veins that may occur due to immobility.

DIF: Cognitive Level: Understand REF: 545

TOP: Nursing Process: Assessment

MSC: NCLEX Patient Needs: Physiologic Integrity: Physiologic Adaptation: Alteration in Body

Systems

2. For which patient does the nurse make assessment of the oral mucous membrane a priority?

a. The patient who has an arteriovenous (AV) fistula

b. The patient who has a gastrostomy tube

c. The patient who uses a Ventimask

d. The patient who has a colostomy

ANSWER: B

A

B

C

D

Feedback

The AV fistula is required by patients who need hemodialysis for kidney failure.

They are able to drink fluids by mouth.

Which patient can drink fluids by mouth is the distinguishing fact. This patient

has this gastrostomy tube because he or she has difficulty swallowing. Thus this

patient may not have fluids by mouth, which increases the risk for dry mucous

membranes and makes the assessment most important compared with the other

listed patients.

The Ventimask fits over the nose and mouth to deliver oxygen. This patient is

able to drink fluids by mouth.

This patient has had part or all of the colon removed, but this patient is able to

drink fluids by mouth.

DIF: Cognitive Level: Apply REF: 550

TOP: Nursing Process: Assessment

.

https://www.coursehero.com/file/20908983/c24/


MSC: NCLEX Patient Needs: Physiologic Integrity: Physiologic Adaptation: Alteration in Body

Systems

3. How does a nurse assess perfusion to the foot when a patient has a cast from the left middle

calf to the toes?

a. Palpate the popliteal pulse of the left leg.

b. Palpate the posterior tibial pulse of the left leg.

c. Assess movement and sensation of the left toes.

d. Assess the capillary refill of the left toes.

ANSWER: D

A

B

C

D

Feedback

This pulse is above the foot and does not indicate perfusion of the foot.

The pulse is not palpable because it is covered by the cast.

This assessment is important for this patient but assesses neurologic function

rather than perfusion.

The presence of capillary refill in less than 2 seconds indicates perfusion of the

left foot when the dorsalis pedis pulse cannot be palpated.

DIF: Cognitive Level: Understand REF: 553

TOP: Nursing Process: Assessment

MSC: NCLEX Patient Needs: Physiologic Integrity: Physiologic Adaptation: Alteration in Body

Systems

4. A nurse uses the Glasgow Coma Scale to assess which patient?

a. The patient who has a new onset of quadriplegia

b. The patient who has tonic-clonic seizures

c. The patient who requires stimuli for responses

d. The patient who has dementia

ANSWER: C

A

B

C

D

Feedback

Although this patient is paralyzed, he or she is conscious. The Glasgow Coma

Scale would not yield useful data about this patient.

Although this patient may be unconscious during seizures, consciousness will

return. The Glasgow Coma Scale would not yield useful data about this patient.

The Glasgow Coma Scale is applicable only to patients who are unconscious,

meaning they do not respond unless stimulated in some way from touch to pain.

This patient is not unconscious. The Glasgow Coma Scale would not yield useful

data about this patient.

DIF: Cognitive Level: Apply REF: 555-556

TOP: Nursing Process: Assessment

MSC: NCLEX Patient Needs: Physiologic Integrity: Physiologic Adaptation: Alteration in Body

Systems

5. During the assessment, the nurse determines that the patient’s Glasgow Coma Scale score is

15. What is the meaning of this number for this patient?

a. This patient is fully conscious.

.

https://www.coursehero.com/file/20908983/c24/


b. This patient has movement but does not open the eyes or speak.

c. This patient is unable to respond to any stimuli.

d. This patient opens the eyes but does not speak or move.

ANSWER: A

A

B

C

D

Feedback

A score of 15 is the expected value for the Glasgow Coma Scale.

This patient would score a 9 on the Glasgow Coma Scale.

This patient would score a 3 on the Glasgow Coma Scale.

This patient would score a 4 on the Glasgow Coma Scale.

DIF: Cognitive Level: Understand REF: 556

TOP: Nursing Process: Assessment

MSC: NCLEX Patient Needs: Physiologic Integrity: Reduction of Risk Potential: System Specific

Assessments

6. When performing a neurologic assessment of a male patient, a nurse discovers that shouting

and shaking are necessary to arouse the patient enough to assess his neurologic status. After

the patient answers questions about who he is and squeezes the nurse’s hand as requested, he

returns to “sleep.” How does the nurse document this patient’s level of consciousness?

a. Lethargic

b. Obtunded

c. Stuporous

d. Semicomatose

ANSWER: B

A

B

C

D

Feedback

Lethargic patients can be aroused by saying their names and touching them.

Obtunded patients require shouting and vigorous shaking to arouse them; they

carry out requests while awake, but return to “sleep” when stimuli stops.

Stuporous patients require painful stimuli to respond and the response usually is a

withdrawal from the source of pain.

Semicomatose patients require painful stimuli and respond with abnormal flexion

or extension.

DIF: Cognitive Level: Understand REF: 555

TOP: Nursing Process: Assessment

MSC: NCLEX Patient Needs: Physiologic Integrity: Physiologic Adaptation: Alteration in Body

Systems

MULTIPLE RESPONSE

1. What data do nurses collect when assessing a patient’s wound? Select all that apply.

a. Skin turgor

b. Width, length, and depth

c. Presence of pulsations

d. Wound color

e. Presence of edema


f. Drainage color

ANSWER: B, D, E, F

Correct: These data are collected when assessing a wound.

Incorrect: Skin turgor is assessed in intact skin rather than wounds. Presence of pulsations is

not indicated when assessing a wound.

DIF: Cognitive Level: Understand REF: 546

TOP: Nursing Process: Assessment

MSC: NCLEX Patient Needs: Physiologic Integrity: Physiologic Adaptation: Alteration in Body

Systems

2. Which patient using respiratory equipment requires skin assessment? Select all that apply.

a. A patient using a nasal cannula

b. A patient with a tracheostomy

c. A patient using an incentive spirometer

d. A patient using a Ventimask

e. A patient with an IV

ANSWER: A, B, D

Correct: Patients using a nasal cannula need inspection of the nares and behind the helix of

the ears. Patients with a tracheostomy need inspection of skin around the stoma where the

tracheostomy tube enters the trachea. Patients using a Ventimask need inspection of skin

where the mask comes in contact with the face and behind the helix of the ears. Patients with

IVs need inspection of the skin to verify the catheter is secured and to assess for redness or

edema.

Incorrect: Using an incentive spirometer requires the patient to take deep breaths, thus a skin

assessment is not indicated.

DIF: Cognitive Level: Understand REF: 547-548

TOP: Nursing Process: Assessment

MSC: NCLEX Patient Needs: Physiologic Integrity: Physiologic Adaptation: Alteration in Body

Systems

3. Which tube interferes with hearing lung sounds during auscultation? Select all that apply.

a. Gastrostomy tube

b. Chest tube

c. Nasogastric tube

d. Tracheostomy tube

e. Oral endotracheal tube

ANSWER: B, C

Correct: When attached to suction, chest and nasogastric tubes can create sounds that may

mimic lung sounds.

Incorrect: Gastrostomy tube is in the stomach and not attached to suction that might create a

false sound similar to lung sounds. Tracheostomy tube is not attached to suction that might

create a false sound similar to lung sounds. Oral endotracheal tubes are not attached to suction

that might create a false sound similar to lung sounds.

DIF: Cognitive Level: Understand REF: 548| 551

TOP: Nursing Process: Assessment

MSC: NCLEX Patient Needs: Physiologic Integrity: Physiologic Adaptation: Alteration in Body


Systems


Test Bank for Health Assessment for

Nursing Practice 7th Edition by Wilson

This is a bank of tests (study questions) to help you

prepare for the tests.

To clarify, this is a test bank, not a textbook.

You have immediate access to download your test

bank. No

delays, loading is fast and instant immediately after

Purchase!

You will receive a full bank of tests; in other words, all

chapters will be there.

Test banks are presented in PDF format; therefore,

no special software is required to open them


Hooray! Your file is uploaded and ready to be published.

Saved successfully!

Ooh no, something went wrong!